You are on page 1of 235

Hội thảo khoa học, Ninh Bình 15-16/09/2018

MỘT SỐ PHƯƠNG PHÁP GIẢI BÀI TOÁN TRÒ CHƠI


BỐC VẬT

Đặng Huy Ruận


Trường Đại học Khoa học tự nhiên, ĐHQG Hà Nội

Tóm tắt nội dung


Trên bàn có một hay nhiều đống vật với số lượng hữu hạn. Hai đấu thủ thực hiện trò
chơi bốc các vật với người đi đầu được xác định ngẫu nhiên và bằng cách mỗi người đến
lượt phải bốc ít nhất một vật và không được bốc quá số lượng quy định. Trong trường
hợp có nhiều đống vật cũng chỉ được bốc ở một trong những đống còn vật.
Về thắng thua có hai cách qui định: Hoặc người bốc được vật cuối cùng thắng cuộc,
hoặc người không phải bốc vật cuối cùng thắng cuộc. Trong bài viết này chỉ xi n trình bầy
thuật toán bốc các vật đảm bảo cho người bốc được vật cuối cùng thắng cuộc.
Trường hợp có một đống vật trò chơi được gọi là Trò chơi đơn, còn trường hợp có nhiều
đống vật trò chơi được gọi là Trò chơi hợp.
Cả hai dạng trò chơi đều có hai cách giải quyết. Cách thứ nhất sử dụng tính chất đồng
dư được gọi là phương pháp đồng dư. Cách thứ hai sử dụng nhân của đồ thị được gọi là
phương pháp đồ thị.

1 Một số khái niệm và kết quả cần dùng


1.1 Đồ thị có hướng
Định nghĩa 1. Trên mặt phẳng hoặc trong không gian lấy n điểm tùy ý khác nhau và ký
hiệu bằng x1 , x2 , . . . , xn .
Giữa một số cặp điểm được nối bằng những đoạn thẳng hoặc đoạn cong được định
hướng.
Người ta gọi hình nhận được là một đồ thị có hướng đồng thời ký hiệu bằng G.
Các điểm đã chọn xi (1 ≤ i ≤ n) được gọi là các đỉnh, còn các đoạn thẳng hoặc đoạn
cong được định hướng đã nối được gọi là các cung của đồ thị G.
Nếu ký hiệu tập đỉnh bằng X, còn tập cung bằng E, thì đồ thị G còn được ký hiệu
bằng G(X, E).
Giả sử cung u đi từ đỉnh xi sang đỉnh x j . Khi đó xi được gọi là đỉnh đầu, còn x j được
gọi là đỉnh cuối của cung u.
Cặp đỉnh xi , x j được gọi là hai đỉnh kề nhau, nếu chúng là hai đầu của cùng một
cung.
Định nghĩa 2 (Nhân của đồ thị). Tập con A ⊆các đỉnh của đồ thị G = (X, E) được gọi là
nhân của đồ thị G, nếu

5
Hội thảo khoa học, Ninh Bình 15-16/09/2018

1) Hai đỉnh tùy ý x, y∈A đều không kề nhau.


2) Đỉnh tuỳ ý u không thuộc A đều tồn tại đỉnh v∈A, để từ u vào v có cung.

1.2 Đồ thị hợp


Giả sử có n đa đồ thị có hướng với các tập đỉnh không giao nhau từng đôi một G1 (X1 ,
E1 ), G2 (X2 , E2 ),. . . , Gn (xn , En ).
Xét tích đề các của các tập đỉnh X1 , X2 , . . . , Xn
X = X1 × X2 × · · · × Xn = {( x1 , x2 , . . . , xn )| xi ∈ Xi (1 ≤ i ≤ n)}
và tập
0 0 0 0
E = {( x1 , x2 , . . . , xn ), ( x1 , x2 , . . . , xn )|∃i!(1 ≤ i ≤ n) (( xi , xi ) ∈ Ei )}.
Định nghĩa 3. Đồ thị G = (X, E) được gọi là đồ thị hợp của các đồ thị G1 , G2 , . . . , Gn ,
đồng thời còn được ký hiệu bằng G1 , G2 , . . . Gn .

1.3 Tổng digit


Giả sử có n số nguyên không âm C1 , C2 , . . . , Cn với các dạng khai triển nhị phân
Ck = (Ck0 , Ck1 , Ck2 , . . . ) (1 ≤ k ≤ n)
Trong số học [m](2) hay ”m theo modul 2” là số dư (bằng 0 hoặc bằng 1) nhận được
khi chia m cho 2.
Định nghĩa 4. Véctơ:
      !
n n n
C= ∑ Ck0 , ∑ Ck1 , ∑ Ck2 ,....
k =1 (2) k =1 (2) k =1 (2)

được gọi là tổng digit của các số nguyên C1 , C1 , . . . , Cn , đồng thời ký hiệu bằng:
C = C1 +̇C2 +̇ . . . +̇Cn−1 +̇Cn
Ví dụ 1. 3+̇7 = [(1, 1) + (1, 1, 1)](2) = [(1, 1, 0) + (1, 1, 1)](2) = (0, 0, 1)
3+̇7+̇13 = [(1, 1) + (1, 1, 1) + (1, 0, 1, 1)](2)
= [(1, 1, 1, 0) + (1, 1, 1, 0) + (1, 0, 1, 1)](2) = (1, 0, 0, 1)

2 Trò chơi đơn


Bài toán 1. Giả sử m, n là hai số tự nhiên m < n và n không chia hết cho m + 1.
Trên bàn có một đống gồm n vật. Hai em A, B thực hiện trò chơi bốc các vật theo các
nguyên tắc sau:
1) Người đi đầu được xác định bằng gieo đồng tiền.
2) Mỗi người đến lượt phải bốc ít nhất một vật và không được bốc quá m vật.
3) Người bốc được vật cuối cùng sẽ thắng cuộc.
Nếu em A được đi đầu, thì em phải có cách bốc các vật như thế nào để đảm bảo thắng
cuộc, tức bốc được vật cuối cùng.
Có hai cách đưa ra thuật toán bốc các vật để em A chiến thắng. Đó là phương pháp
đồng dư và phương pháp đồ thị.

6
Hội thảo khoa học, Ninh Bình 15-16/09/2018

2.1 Phương pháp đồng dư


Thuật toán được hình thành dựa trên cơ sở tính đồng dư theo modul (m+1)
1) Vì n không chia hết cho m+1, nên khi chia n cho m+1 nhận được số dư r (0 < r ≤
m). Bởi vậy tại bước xuất phát em A có thể bốc r vật, để trên bàn còn lại số lượng vật
bằng nguyên lần của m+1.
2) Tại các bước tiếp theo, nếu đến lượt mình em B bốc s (1 ≤ s ≤ m) vật, thì ngay sau
đó em A bốc (m+1-s) vật, nên trên bàn số lượng vật còn lại vẫn là nguyên lần của m+1.
3) Trước khi em B bốc lần cuối cùng trên bàn còn đúng m+1 vật, nhưng em B phải
bốc ít nhất 1 vật và không được bốc quá m vật. Bởi vậy, sau khi em B bốc lần cuối cùng
trên bàn còn ít nhất 1 vật và không vượt quá m vật, nên em A có quyền bốc hết số vật
còn lại này và thắng cuộc.

Ví dụ 2. Trên bàn có một đống bi gồm 14 viên. Hai em Anh, Việt thực hiện trò chơi bốc
bi theo các nguyên tắc sau:
1) Người đi đầu được xác định bằng gieo đồng tiền.
2) Mỗi người đến lượt phải bốc ít nhất một viên bi và không được bốc quá 3 viên bi.
3) Người bốc được viên bi cuối cùng sẽ thắng cuộc.
Hỏi: Nếu em Anh đi đầu, thì em phải có cách bốc bi như thế nào để thắng cuộc, tức
bốc được viên bi cuối cùng?

Giải.
Trò chơi trong ví dụ tương ứng với trường hợp n = 14, còn m = 3.
Vì 14 chia cho 4 còn dư 2, nên với tư cách người đi đầu em Anh bốc 2 viên bi, để số bi
còn lại là 12 viên.
Tiếp theo giả sử em Việt bốc 3 viên, nên trên bàn còn 9 viên bi.
Đến lượt mình em Anh bốc 1 viên bi, để số bi còn lại là 8 viên.
Đến lượt mình giả sử em Việt bốc 2 viên bi, nên trên bàn còn 6 viên bi.
Đến lượt mình em Anh phải bốc 2 viên bi, để số bi còn lại là 4 viên.
Lần cuối cùng giả sử em Việt bốc 1 viên, nên số bi còn lại 3 viên.
Đến lượt mình đồng thời là người bốc cuối cùng em Anh bốc tất cả 3 viên bi còn lại
và thắng cuộc.

2.2 Phương pháp đồ thị


Để vận dụng phương pháp này đầu tiên phải xây dựng đồ thị mô tả diễn biến cuộc
chơi, sau đó dựa vào nhân của đồ thị mà đưa ra thuật toán.

2.3 Xây dựng đồ thị G mô tả diễn biến cuộc chơi


a. Đỉnh: Lấy n+1 điểm trên mặt phẳng hoặc trong không gian và ký hiệu một cách
tương ứng bằng các số

0, 1, 2, . . . , n − 1, n
Đỉnh n được thừa nhận là đỉnh xuất, nên được đặt trong khuyên tròn có mũi tên đi
vào.
Các đỉnh

7
Hội thảo khoa học, Ninh Bình 15-16/09/2018

0, m + 1, 2(m + 1), . . . ., s(m + 1)(s(m + 1) < n)


được đặt trong các ô chữ nhật. Các đỉnh còn lại được đặt trong khuyên tròn.
b. Cung:
α) Từ mỗi đỉnh s (s ≥ m) xuất phát m cung với các nhãn tương ứng là
1, 2, . . . , m - 1, m
β). Từ mỗi đỉnh t (1 ≤ t ≤ m) xuất phát t cung với các nhãn tương ứng là
1, 2, . . . , t - 1, t
Cung nhãn c xuất phát từ đỉnh a sẽ đi tới đỉnh a - c

2.4 Nhân N của đồ thị G


Tập con N = {0, m + 1, 2(m + 1), . . . , s(m + 1)}với s(m+1) < n là nhân của đồ thị G.
Bởi vì
α) Từng cặp đỉnh thuộc N không có cung  nối
 với nhau;
β) Với các số nguyên l, r tùy ý (0 ≤ k ≤ mn+1 , 1 ≤ r ≤ m) đỉnh k(m+1)+r không thuộc
N và từ đỉnh k(m+1)+r vào đỉnh k(m+1) ∈N có cung.

2.5 Thuật toán


Giả sử n = s(m+1) + r (1 ≤ r ≤ m). Để chiến thắng em A phải xuất phát từ đỉnh n, đi
theo cung nhãn r (tức bốc r vật) để đạt được đỉnh s(m + 1) ∈N.
Đến lượt mình em B phải xuất phát từ đỉnh s(m+1) và đi theo cung nhãn r (1 ≤ r ≤
m) (tức bốc r vật) và đi tới đỉnh s(m+1) - r không thuộc nhân N.
Đến lượt mình em A xuất phát từ đỉnh s(m+1) - r đi theo cung nhãn m+1- r (1 ≤ m +
1 - r ≤ m) (tức bốc m + 1- r vật) để đạt đỉnh (s - 1)(m + 1) thuộc nhân N.
Giả sử sau một số hữu hạn bước thay nhau bốc các vật em A đạt được đỉnh k(m+1)
∈N (tức trên bàn còn k(m+1) vật. Khi đó đến lượt mình em B phải xuất phát từ đỉnh
k(m+1) đi theo cung nhãn t (1 ≤ t ≤ m) (tức bốc t vật) để tới đỉnh k(m+1)-t không thuộc
nhân N. Tiếp theo đó đến lượt A, em sẽ bốc m+1-t (1 ≤ m+1-t ≤ m) vật và lại đạt được
đỉnh (k-1)(m+1) thuộc nhân N.
Sau khi bốc lần cận cuối em A đạt được đỉnh m+1 thuộc nhân N. Khi đó đến lượt
mình em B phải xuất phát từ đỉnh m+1 và đi theo cung với nhãn p(1 ≤ p ≤ m) để tới
đỉnh m+1-p (1 ≤ m+1-p ≤ m) không thuộc nhân N. Đến lượt mình em A xuất phát từ
đỉnh m+1-p đi theo cung m+1-p (tức bốc m+1-p vật) để đạt đỉnh O∈N và thắng cuộc.

8
Hội thảo khoa học, Ninh Bình 15-16/09/2018

Ví dụ 3. Trên bàn có một đống diêm gồm 9 que. Hai em Dũng, Hằng thực hiện trò chơi
bốc diêm theo nguyên tắc sau:
1) Người đi đầu được xác định bằng gắp thăm;
2) Mỗi người đến lượt mình phải bốc ít nhất 1 que diêm và không được bốc quá 3 que
diêm.
3) Người nào bốc được que diêm cuối cùng sẽ thắng cuộc.
Hỏi: Giả sử em Dũng được đi đầu, thì em phải có cách bốc diêm như thế nào để
thắng cuộc, tức bốc được que diêm cuối cùng?

Giải.
Để giải bài toán này ta dùng phương pháp đồ thị.
Trò chơi trong ví dụ này tương ứng với trường hợp n = 9, còn m = 3.
Đầu tiên ta xây dựng đồ thị G mô tả diễn biến cuộc chơi. Sau đó dựa vào nhân của
đồ thị G, mà đưa ra thuật toán đảm bảo người đi đầu (em Dũng) thắng cuộc.
1. Xây dựng đồ thị G
Để có đồ thị G cần xác định đỉnh và cung.
Đỉnh: Lấy 10 điểm trên mặt phẳng hoặc trong không gian và ký hiệu một cách tương
ứng bằng các số
0, 1, 2, 3, 4, 5, 6, 7, 8, 9.
Đỉnh ký hiệu bằng số 9 được thừa nhận là đỉnh xuất phát, nên được đặt trong ô tròn
có mũi tên đi vào.
Các đỉnh 1, 2, 3, 5, 6, 7, được đạt trong các ô tròn.
Các đỉnh 0, 4, 8 được đặt trong các ô chữ nhật.
Cung:
Với mỗi k∈3, 4, 5, 6, 7, 8, 9 từ đỉnh k xuất phát ba cung với nhãn 1,2,3 đi tới ba đỉnh
tương ứng là (k-1), (k-2) và (k-3).
Từ đỉnh 2 có cung nhãn 1 đi tới đỉnh 1 và cung nhãn 2 đi tới đỉnh 0.
Từ đỉnh 1 xuất phát cung duy nhất đi tới đỉnh 0.

9
Hội thảo khoa học, Ninh Bình 15-16/09/2018

Đồ thị G

2.Nhân của đồ thị G


Vì từng cặp đỉnh thuộc tập N = 0, 4, 8,không có cung nối với nhau.
Các đỉnh nằm ngoài N đều có cung đi tới một đỉnh thuộc N, nên tập N là nhân của
đồ thị G.
3. Thuật toán giúp người đi đầu (em Dũng) chiến thắng.
Với tư cách người đi đầu em Dũng phải xuất phát từ đỉnh 9, đi theo cung nhãn 1, tức
bốc 1 que diêm, để đạt được đỉnh 8 thuộc nhãn N.
Tiếp theo, đến lượt em Hằng, em phải xuất phát từ đỉnh 8 và chỉ có thể đi theo một
trong ba cung thuộc đỉnh này.
Chẳng hạn, em Hằng đi theo cung nhãn 2, tức bốc 2 que diêm, để tới đỉnh 6 không
thuộc nhân N.
Vì từ đỉnh 6 có cung nhãn 2 đi tới đỉnh 4 thuộc nhân N, nên đến lượt mình em Dũng
xuất phát từ đỉnh 6 và đi theo cung nhãn 2, tức bốc 2 que diêm, để đạt đỉnh 4 thuộc nhân
N.
Lần thứ ba đến lượt mình em Hằng phải xuất phát từ đỉnh 4 và cũng chỉ được phép
đi theo một trong ba cung xuất phát từ đỉnh này. Chẳng hạn em Hằng đi theo cung nhãn
3, tức bốc tối đa là 3 que diêm, và chỉ đạt cực đại tại đỉnh 1 không thuộc nhân N. Đến
lượt mình em Dũng xuất phát từ đỉnh 1 và đi theo cung duy nhất xuất phát từ đỉnh 1 và
cung này có nhãn là 1, tức bốc nốt que diêm cuối cùng, để đạt được đỉnh 0 thuộc nhân N
và thắng cuộc.

3 Trò chơi hợp


Giả sử mi , ni (1 ≤ i ≤ k) là các số nguyên dương và ni chia cho mi +1 có dư là ri và
tổng digit các số dư r1 + r2 + · · · + rk 6= 0.

Bài toán 2. Trên bàn có k đống vật M1 , M2 , . . . Mk với các số lượng tương ứng n1 , n2 , . . . ,
nk .
Hai em A, B thực hiện trò chơi bốc các vật theo nguyên tắc sau:
1) Người đi đầu được xác định bằng gieo đồng tiền;
2) Mỗi người đến lượt chỉ được bốc ở một trong những đống còn vật, phải bốc ít nhất
một vật và nếu bốc đống thứ i (1 ≤ i ≤ k), thì không được bốc quá mi vật;
3) Người bốc được vật cuối cùng sẽ thắng cuộc.
Hỏi: Nếu em A được đi đầu, thì em phải có cách bốc các vật như thế nào để đảm bảo
thắng cuộc, tức bốc được vật cuối cùng?
Có hai cách đưa ra thuật toán bốc các vật, để em A chiến thắng. Đó là phương pháp
đồng dư và phương pháp đồ thị.

3.1 Phương pháp đồng dư


Vì ri (1 ≤ i ≤ k) là số dư nhận được khi chia ni cho mi +1 và có tổng digit
To = r1 +̇r2 +̇ . . . +̇rk 6= 0
nên với tư cách người được đi đầu em A phải chọn đống nào và phải bốc số lượng
vật bằng bao nhiêu trong phạm vi cho phép, để sau khi em bốc tổng digit của số dư tất
cả các đống bằng 0.

10
Hội thảo khoa học, Ninh Bình 15-16/09/2018

Giả sử em A bốc vật thuộc đống thứ t (1 ≤ t ≤ k) và số lượng vật còn lại tại đống này
0
sau khi em bốc chia cho mt+1 có dư là rt . Khi đó tổng digit
0
T1 = r1 +̇r2 +̇ . . . +̇rt−1 +̇rt +̇rt+1 +̇ . . . .+̇rk = 0
Giả sử hai em A, B cứ thay nhau bốc các vật theo quy tắc đã định và đến một thời
điểm nào đó sau khi em B bốc số lượng vật còn lại tại đống thứ i (1≤ i ≤ k) đem chia cho
mi +1 có dư là si và tổng digit
T2 = s1 +̇s2 +̇ . . . +̇si−1 +̇si +̇si+1 +̇ . . . .+̇sk 6= 0
Sau đó đến lượt mình em A phải chọn đống nào và bốc với số lượng vật bằng bao
nhiêu trong phạm vi cho phép, để sau khi em bốc tổng digit của số dư tất cả các đống
bằng 0.
Giả sử em A bốc vật thuộc thứ l (1 ≤ l ≤ k) và số lượng vật còn lại tại đống này sau
0
khi em bốc chia cho ml+1 có dư là sl . Khi đó tổng digit
0
T3 = s1 +̇s2 +̇ . . . +̇sl −1 +̇sl +̇sl +1 +̇ . . . .+̇sk = 0
Cứ tiếp tục quá trình thay nhau bốc các vật như trên thì em A sẽ là người bốc được
vật cuối cùng và thắng cuộc.

Ví dụ 4. Trên bàn có ba đống bi: Đống M1 gồm 14 viên, đống M2 gồm 9 viên và đống
M3 gồm 5 viên.
Hai em Tuấn, Bình thực hiện trò chơi bốc bi theo nguyên tắc sau:
1) Người đi đầu được xác định bằng gắp thăm;
2) Mỗi người đến lượt chỉ được bốc ở một trong những đống còn bi, phải bốc ít nhất
1 viên và
- Nếu bốc đống M1 , thì không được bốc quá 4 viên,
- Nếu bốc ở đống M2 , thì không được bốc quá 3 viên,
- Nếu bốc ở đống M3, thì không được bốc quá 2 viên.
3) Em bốc được viên bi cuối cùng sẽ thắng cuộc.
Hỏi: Nếu em Bình được đi đầu, thì em phải có cách bốc bi như thế nào để thắng
cuộc, tức bốc được viên bi cuối cùng?

Giải.
Do khi chia 14 cho 5 có dư là 4, chia 9 cho 4 có dư là 1, chia 5 cho 3 có dư là 2 và
T0 = 4+̇1+̇2 = (0, 0, 1) + (1, 0, 0) + (0, 1, 0) = (1, 1, 1) 6= (0, 0, 0)
nên tại bước xuất phát em Bình phải bốc 1 viên thuộc đống M1 , để số lượng còn lại
tại các đống là: M1 có 13 viên và chia 13 cho 5 có dư là 3, M2 có 9 viên và chia 9 cho 4 có
dư là 1, M3 có 5 viên và chia 5 cho 3 có dư là 2. Bởi vậy có tổng digit các số dư
T1 = 3+̇1+̇2 = (1, 1, 0) + (1, 0, 0) + (0, 1, 0) = (0, 0, 0)
Đến lượt mình, giả sử em Tuấn bốc 3 viên tại đống M2 , nên số lượng sau khi em
Tuấn bốc là: M1 có 13 viên và chia 13 cho 5 có dư 3, M2 còn 6 viên và chia 6 cho 4 có
dư 2, M3 có 5 viên và chia 5 cho 3 có dư 2. Bởi vậy tổng digit các số dư T2 = 3+̇2+̇2 =
(1, 1, 0) + (0, 1, 0) + (0, 1, 0) = (1, 1, 0) 6= (0, 0, 0)
Đến lượt mình em Bình bốc 3 viên ở đống M1 . Khi đó số lượng tại các đống là: M1
còn 10 viên và 10 chia hết cho 5, M2 có 6 viên và 6 chia cho 4 còn dư 2, M3 có 5 viên và 5
chia cho 3 còn dư 2. Bởi vậy tổng digit của các số dư
T3 = 0+̇2+̇2 = (0, 0, 0) + (0, 1, 0) + (0, 1, 0) = (0, 0, 0)
Đến lượt mình, chẳng hạn em Tuấn bốc 4 viên tại đống M1 , nên số lượng bi còn lại
tại các đống là: M1 còn 6 viên và 6 chia cho 5 còn dư 1, M2 có 6 viên và 6 chia cho 4 còn
dư 2, M3 có 5 viên và 5 chia cho 3 còn dư 2. Bởi vậy tổng digit của các số dư
T4 = 1+̇2+̇2 = (1, 0, 0) + (0, 1, 0) + (0, 1, 0) = (1, 0, 0) 6= (0, 0, 0)

11
Hội thảo khoa học, Ninh Bình 15-16/09/2018

Đến lượt mình, em Bình chỉ còn cách duy nhất là bốc 1 viên thuộc M1 , để số lượng
còn lại tại các đống là: Đống M1 còn 5 viên và 5 chia hết cho 5, M2 có 6 viên và 6 chia cho
4 còn dư 2, M3 có 5 viên và 5 chia cho 3 còn dư 2. Bởi vậy tổng digit của các số dư
T5 = 0+̇2+̇2 = (0, 0, 0) + (0, 1, 0) + (0, 1, 0) = (0, 0, 0)
Đến lượt mình, chẳng hạn em Tuấn bốc 3 viên tại đống M2 , nên số lượng bi còn lại
tại các đống là: M1 có 5 viên và 5 chia hết cho 5, M2 còn 3 viên và 3 chia cho 4 còn dư 3,
M3 có 5 viên và 5 chia cho 3 còn dư 2. Bởi vậy tổng digit của các số dư
T6 = 0+̇3+̇2 = (0, 0, 0) + (1, 1, 0) + (0, 1, 0) = (1, 0, 0) 6= (0, 0, 0).
Đến lượt mình, em Bình bốc 4 viên tại đống M1 , nên số lượng bi còn lại tại các đống
là: M1 còn 1 viên và 1 chia cho 5 còn dư 1, M2 có 3 viên và 3 chia cho 4 còn dư 3, M3
có 5 viên và 5 chia cho 3 còn dư 2. Bởi vậy tổng digit của các số dư T7 = 1+̇3+̇2 =
(1, 0, 0) + (1, 1, 0) + (0, 1, 0) = (0, 0, 0).
Đến lượt mình, chẳng hạn em Tuấn bốc nốt 3 viên ở đống M2 , nên số lượng bi còn lại
tại các đống là: M1 có 1 viên và 1 chia cho 5 còn dư 1, M2 không còn viên bi nào và 0 chia
hết cho 4, M3 có 5 viên và 5 chia cho 3 còn dư 2. Bởi vậy tổng digit của các số dư
T8 = 1+̇0+̇2 = (1, 0, 0) + (0, 0, 0) + (0, 1, 0) = (1, 1, 0) 6= (0, 0, 0).
Đến lượt mình em Bình phải bốc 1 viên ở đống M3, nên số lượng bi còn lại tại các
đống là: M1 có 1 viên vì 1 chia cho 5 còn dư 1, M2 không còn viên nào và 0 chia hết cho
4, M3 còn 4 viên và 4 chia cho 3 dư 1. Bởi vậy tổng digit của các số dư
T9 = 1+̇0+̇1 = (1, 0, 0) + (0, 0, 0) + (1, 0, 0) = (0, 0, 0)
Đến lượt mình, giả sử em Tuấn bốc nốt 1 viên bi còn lại của đống M1 . Khi đó chỉ đống
M3 còn 4 viên bi và 4 chia cho 3 còn dư 1, nên tổng digit của các số dư
T10 = 0+̇0+̇1 = (0, 0, 0) + (0, 0, 0) + (1, 0, 0) = (1, 0, 0) 6= (0, 0, 0).
Đến lượt mình em Bình phải bốc 1 viên ở đống M3, nên đống M3 còn lại 3 viên và 3
chia hết cho 3, nên tổng digit của các số dư T11 = 0+̇0+̇0 = 0.
Đến lượt mình em Tuấn không thể bốc hết 3 viên bi còn lại tại đống M3, và phải bốc
ít nhất 1 viên, nên:
- Nếu em Tuấn bốc 1 viên, thì em Bình có quyền bốc cả 2 viên bi còn lại và thắng cuộc.
- Nếu em Tuấn bốc 2 viên, thì vẫn còn lại 1 viên để em Bình bốc và thắng cuộc.

3.2 Phương pháp đồ thị


Đầu tiên đối với mỗi đống vật mi (1 ≤ i ≤ k) xây dựng đồ thị Gi mô tả diễn biến cuộc
chơi tại đống mi . Sau đó xây dựng đồ thị hợp G của các đồ thị G1, G2, . . . , Gk, để mô tả
diễn biến trò chơi hợp trên các đống M1 , M2 , . . . , Mk . Cuối cùng xác định nhân N của đồ
thị G và đưa ra thuật toán đảm bảo cho người đi đầu chiến thắng.
1. Đồ thị thành phần Gi (1 ≤ i ≤ k).
Để có đồ thị Gi mô tả cuộc chơi tại đống mi cần xác định đỉnh và cung.
a. Đỉnh
Lấy ni +1 điểm trên mặt phẳng hoặc trong không gian và ký hiệu một cách tương ứng
bằng các số
0, 1, 2, . . . ., ni - 1, ni làm đỉnh của đồ thị.
Đỉnh ni là đỉnh xuất phát, nên được đặt trong ô tròn có mũi tên đi vào. Các đỉnh còn
lại đều được đặt trong khuyên tròn.
b. Cung
α ) Từ mỗi đỉnh s(s ≥ mi ) xuất phát mi cung với các nhãn tương ứng
1, 2, . . . , mi - 1, mi
β) Từ đỉnh t (1 ≤ t ≤ mi ) xuất phát t cung với các nhãn tương ứng

12
Hội thảo khoa học, Ninh Bình 15-16/09/2018

1, 2, . . . , t - 1, t
Cung nhãn c xuất phát từ đỉnh e sẽ đi tới đỉnh e - c

3.3 Đồ thị hợp


Từ các đồ thị thành phần G1 , G2 , . . . , Gk ta xây dựng đồ thị hợp G.

3.4 Nhân của đồ thị G


Tập N gồm các đỉnh x = (x1 , x2 , . . . , xk ) của đồ thị G, mà xi ≡ ri (mod(mi+1)) và có
tổng digit các số dư
r1 +̇r2 +̇ . . . .+̇rk = 0
Vì các đỉnh có tổng digit các số dư bằng 0 đều không có cung nối với nhau, nên hai
đỉnh tùy ý thuộc tập N đều không có cung nối với nhau, đồng thời mỗi đỉnh có tổng
digit các số dư khác 0 (không thuộc tập N) đều có cung đi tới đỉnh có tổng digit các số
dư bằng 0, tức đi tới một đỉnh thuộc N. Bởi vậy N là nhân của đồ thị G.

3.5 Thuật toán


Dựa vào nhân N của đồ thị G ta đưa ra thuật toán cho người đi đầu chiến thắng, tức
bốc được vật cuối cùng.
Vì đỉnh x = (n1 , n2 , . . . , nk ) có ni ≡ ri (mod mi + 1)

r1 +̇r2 +̇ . . . .+̇rk 6= 0,
nên x ∈ / N và từ x có cung với nhãn at (1 ≤ at ≤ mt )(1 ≤ t ≤ k ) đi vào đỉnh y ∈ N.
Bởi vậy em A có quyền bốc at vật thuộc đống Mt .
Đến lượt mình, em B phải xuất phát từ đỉnh y, nhưng xuất phát từ đỉnh y chỉ có cung
đi tới các đỉnh nằm ngoài nhân N. Bởi vậy em B chỉ có thể bốc số lượng vật là nhãn của
cung đi từ nhân ra ngoài, nên em B chỉ có thể đạt đỉnh mà tổng digit của các số dư khác
0.
Sau đó, đến lượt mình em A lại có thể xuất phát từ đỉnh ngoài nhân N, mà em B vừa
đạt được để đi vào một đỉnh thuộc nhân N.
Cứ tiếp tục như vậy hai em A, B lần lượt thay nhau đi vào nhân N rồi lại ra khỏi nhân
N và bốc số lượng vật theo nhãn của các cung đi qua và đến cuối cùng em A đạt được
đỉnh 0, tức là bốc được vật cuối cùng và thắng cuộc.

13
Hội thảo khoa học, Ninh Bình 15-16/09/2018

Ví dụ 5. Trên bàn có hai đống bi với số lượng tương ứng: Đống M1 gồm 5 viên, đống M2
gồm 7 viên.
Hai em Hoè, Liên thay nhau bốc bi theo nguyên tắc:
1) Người đi đầu được xác định bằng gieo đồng tiền;
2) Mỗi người đến lượt chỉ được bốc tại một trong hai đống, phải bốc ít nhất 1 viên và
- Nếu bốc ở đống M1 , thì không được bốc quá 2 viên,
- Nếu bốc ở đống M2 , thì không được bốc quá 3 viên.
3) Người nào bốc được viên cuối cùng sẽ thắng cuộc.
Hỏi: Nếu em Hoè được đi đầu, thì em phải co cách bốc bi như thế nào để thắng cuộc,
tức bốc được viên bi cuối cùng?

Giải. Ta sẽ giải bài toán này bằng phương pháp đồ thị.


1) Xây dựng các đồ thị G1 , G2 để mô tả diễn biến cuộc chơi tại các đống M1 và M2

2. Xây dựng đồ thị hợp G.

14
Hội thảo khoa học, Ninh Bình 15-16/09/2018

15
Hội thảo khoa học, Ninh Bình 15-16/09/2018

3. Thuật toán cho em Hoè là người đi đầu chiến thắng.


Đồ thị hợp G có nhân N gồm các đỉnh (5,6), (4,5), (3,4), (5,2), (2,6), (4,1), (1,5), (0,4), (2,
2), (1,1), (3,0) và (0,0) nên tại bước xuất phát với tư cách người được đi đầu em Hoè xuất
từ đỉnh (5,7) đi theo cung nhãn 1 để đạt đỉnh (5,6) thuộc N, tức em Hoè phải bốc tại đống
M2 1 viên, để đống M2 còn 6 viên, còn đống M1 vẫn giữ nguyên số lượng cũ là 5 viên.
Đến lượt mình, em Liên phải xuất phát từ đỉnh (5,6) và đi theo một trong những cung
thuộc đỉnh này. Chẳng hạn, em Liên đi theo cung nhãn 2 đi tới đỉnh (3,6), tức là bốc 2
viên ở đống thứ M1 , để đống M1 còn 3 viên và đống M2 vẫn giữ nguyên số lượng cũ là
6 viên.
Đến lượt mình, em Hoè xuất phát từ đỉnh (3,6) đi theo cung nhãn 1 sang đỉnh (2,6)
thuộc N, tức bốc 1 viên ở đống M1 , để đống M1 còn 2 viên và đống M2 vẫn giữ số lượng
cũ là 6 viên.
Đến lượt mình, em Liên phải xuất phát từ đỉnh (2,6) và đi theo một trong những cung
xuất phát từ đỉnh này. Chẳng hạn, em Liên đi theo cung nhãn 3 để về đỉnh (2,3) tức bốc
3 viên thuộc đống M2 , để đống M2 còn 3 viên và đống M1 vẫn giữ nguyên số lượng cũ
là 2 viên.
Đến lượt mình, em Hoè xuất phát từ đỉnh (2,3) đi theo cung nhãn 1 để về đỉnh (2,2)
thuộc N, tức bốc 1 viên ở đống M2 , để M2 còn 2 viên và M1 giữ nguyên số lượng cũ là 2
viên.
Đến lượt mình, em Liên phải xuất phát từ đỉnh (2,2) và đi theo một trong những cung
xuất phát tại đỉnh này. Chẳng hạn, em Liên đi theo cung nhãn 2 để về đỉnh (0,2), tức bốc
2 viên tại đống M1 . Khi đó đống M1 hết bi, còn đống M2 vẫn giữ nguyên số lượng cũ là
2 viên.
Đến lượt mình, em Hoè xuất phát từ đỉnh (0,2) đi theo cung nhãn 2 để về đỉnh (0,0)
thuộc N, tức bốc nốt 2 viên bi cuối cùng và thắng cuộc.

Tài liệu
[1] Đặng Huy Ruận, Lý thuyết đồ thị và ứng dụng
Nhà xuất bản Khoa học kỹ thuật, 2002.
[2] Claude Berge. Theorie des graphes et ses applications. Dunod. Paris 1967
[3] Đặng Huy Ruận. Trò chơi và đồ thị Nhà xuất bản Khoa học kỹ thuật, 2004
[4] Đặng Huy Ruận. Phương pháp giải bài toán chia hết Nhà xuất bản Khoa học kỹ thuật,
2005

16
Hội thảo khoa học, Ninh Bình 15-16/09/2018

PHÂN TÍCH MỘT SỐ BÀI TOÁN HÌNH HỌC


TỪ CÁC ĐỀ THI TUYỂN SINH THPT
CHUYÊN NĂM HỌC 2018

Nguyễn Văn Lợi


Hội toán học Hà Nội

Tóm tắt nội dung

Trào lưu thi vào các lớp 10 chuyên và đặc biệt là chuyên toán đã từ lâu là các cuộc
cạnh tranh nổi bật. Sau khi phong trào BĐT được giữ ở mức độ thân thiện hơn, thì các
bài toán hình học lập tức chiếm ngôi trong cuộc đua kiếm điểm. Hình học phẳng Việt
Nam trong phạm vi toán cấp trung học cơ sở nổi tiếng là khó, đây là niềm tự hào của
giới chuyên môn, nhưng cũng là gánh nặng của các học sinh lớp 9 khi phải đương đầu
với các bài toán hình nhiều khi ngang tầm thi quốc tế. Trước lượng bài ôn luyện khổng
lồ, nhiệm vụ cấp bách được đặt ra là: Hãy tìm phương pháp hiệu quả cho cả người học
và người dạy môn hình học.
Thông qua việc phân tích và nêu hướng giải quyết một số bài tiêu biểu của kì thi 2018
vừa qua, chúng tôi cũng xin đóng góp một cách tiếp cận các bài toán hình học với phong
cách tự tin, ngoài giải xong bài toán chúng ta tự phân tích sâu, tìm nguồn bài toán. Tạo
tác phong chủ động học toán hình.

1 Cấu hình đồng dạng


Bài toán 1 (Đề thi vào THPT chuyên, Sở GDĐT Hà Nội 2018). Cho tứ giác ABCD (không
có cạnh nào song song) nội tiếp đường tròn tâm (O). Các tia BA và CD cắt nhau tại điểm F. Gọi
E là giao điểm của hai đường chéo AC và BD. Vẽ hình bình hành AEDK.
a) Chứng minh rằng tam giác FKD đồng dạng với tam giác FEB.
b) Gọi M, N tương ứng là trung điểm các cạnh AD, BC. Chứng minh rằng đường thẳng
MN đi qua trung điểm đoạn thẳng EF.
c) Chứng minh rằng đường thẳng EF tiếp xúc với đường tròn ngoại tiếp tam giác EMN.

17
Hội thảo khoa học, Ninh Bình 15-16/09/2018

Lời giải và bình luận.


− Gọi d là phép đối xứng qua trục đối xứng là đường phân giác góc BFC.
d
FD FA
− Gọi f là phép chiếu (phóng đại) tâm F tỉ lệ k = (= vì FA · FB = FD · FC).
FB FC
− Tích của hai phép biến hình này là f ◦ d =⇒ Đây là phép biến hình đồng dạng.
Qua phép biến hình này D → B, A → C. Vậy ∆FAD → ∆FCB do đó các thành phần
được vẽ trong tam giác này có ảnh đồng dạng trong tam giác kia. Vì ACB
[ = ADB [ =
[ nên ảnh của đường thẳng AK là đường thẳng CA. Tương tự ta có ảnh của đường
DAK
thẳng DK là đường thẳng BD. Suy ra K → E.
Vì đoạn thẳng AD có ảnh là CB nên trung điểm M có ảnh là N của mỗi đoạn thẳng tương
ứng. Ta quay lại bài toán.
1. ∆FKD v ∆FEB vì là ảnh của nhau qua f ◦ d.
2. Đường thẳng Gaus của tứ giác toàn phần AFDE − BC.

3. Ta chứng minh phần c) không cần dựa trên kết quả của phần b). ∆FKE
có ảnh qua phép biến hình f ◦ d là ∆FET nên đồng dạng. Suy ra các
góc FEK
d = FTE d = \ MNE. Vậy FE tiếp xúc với đường tròn (EMN).


18
Hội thảo khoa học, Ninh Bình 15-16/09/2018

2 Tiếp cận cấu hình từ cách nhìn nhận khác


Trong phần này chúng ta xây dựng lại bài toán bằng việc chuyển các cấu hình từ dạng
đường tròn về dạng các đường thẳng song song để sử dụng nhóm định lý Thalet.

Bài toán 2 (Đề thi vào THPT chuyên ĐHSP Vinh 2018). Cho hai đường tròn (O; R) và
(O0 ; r ) cắt nhau tại hai điểm A và B (R > r) sao cho O và O0 ở hai phía đối với đường thẳng AB.
Gọi K là điểm sao cho OAO0 K là hình bình hành.
a) Chứng minh rằng tam giác ABK là tam giác vuông.
b) Đường tròn tâm K bán kính KA cắt các đường tròn (O; R) và (O0 ; r ) theo thứ tự tại M và
N (M, N khác A). Chứng minh rằng \ABM = ABN.
[
c) Trên đường tròn (O; R) lấy điểm C thuộc cung AM không chứa B (C khác A, M). Đường
thẳng CA cắt đường tròn (O0 , r ) tại D. Chứng minh rằng KC = KD.

Lời giải bình luận.

Gọi XY là ảnh của OO0 qua phép chiếu tâm A tỉ lệ 1 : 2. Vì OO0 cắt AK và AB tại điểm
giữa của mỗi đoạn, nên KB nằm trên XY. Do đó:
1. KB ⊥ AB do đó tam giác AKB là tam giác vuông.
2. Cách xác định vị trí của M và N: Vì M là giao của đường tròn (O, R) và đương tròn
(K, KA) nên nó là điểm đối xứng trục của A qua trục OK. Tương tự như vậy N và
A đối xứng qua trục O0 K.

19
Hội thảo khoa học, Ninh Bình 15-16/09/2018

\ = 900 ⇐⇒ BAMX nội tiếp, MAX


Vì M và A đối xứng qua trục KO, AMX \=\ MBX.
Tương tự ta có NBY = N AY. Mà MAX = N AY vì có các cặp cạnh tương ứng vuông
[ [ \ [
góc và là các góc nhọn. Do đó \ [ ⇐⇒ \
XBM = YBN MBA = NBA.
[
3. Cách xác định các điểm C và D từ một dây cung bất kì đi qua A. Vì AX và
AY là các đường kính của đường tron (O) và (O0 ) tương ứng, nên các góc
[ = 900 = YDA.
XCA [ Vậy C và D là chân các đường vuông góc hạ từ X và Y xuống
dây cung qua A đã nói. Việc hoàn tất chứng minh đã không còn gì phức tạp.

Vì XC và YD ⊥ CD, XCDY là hình thang vuông


có ZK là đường trung bình nên KC = KD.


3 Sử dụng sự giúp đỡ bất ngờ của các bài toán đơn


giản
Một bài toán quen thuộc nêu lên mối liên hệ của các cát tuyến đến hai đường tròn cắt
nhau vô cùng thú vị và có ứng dụng không ngờ khi ta xét các vị trí khác nhau của nó.

Bổ đề 1. Hai đường tròn (O) và (U ) cắt nhau tại hai điểm A và B. Cát tuyến x qua A cắt (O)
tại P, cắt (U ) tại Q. Cát tuyến y qua B cắt (O) tại R, cắt (U ) tại S. Chứng minh rằng PR//QS.

Mệnh đề ngược lại cũng đúng.


Hai đường tròn (O) và (U ) cắt nhau tại hai điểm A và B. Cát tuyến x qua A cắt (O) tại P, cắt
(U ) tại Q, qua P kẻ PR (R thuộc (O)), qua P kẻ S (S ∈ (U )) sao cho PR//QS khi đó R, B, S
thẳng hàng.

20
Hội thảo khoa học, Ninh Bình 15-16/09/2018

Hệ quả 1. Các dạng ứng dụng của bổ đề khi hai cát tuyến ở những vị trí đặc biệt.

Bài toán 3 (Đề thi vào THPT chuyên ĐHSP Hà Nội 2018). Cho tam giác ABC có 3 góc nhọn,
AB < AC và nội tiếp đường tròn (O). Đường tròn ngoại tiếp tam giác BOC cắt các đường thẳng
AB và AC theo thứ tự D và E. Trên đường tròn ngoại tiếp tam giác BOC lấy điểm P sao cho AP
vuông góc với PC. Đường thẳng qua B song song với OP cắt PC tại Q. Chứng minh rằng:
a) PB = PQ.
b) O là trực tâm của tam giác ADE.
[ = QAC.
c) PAO [

Lời giải và bình luận


_ _
b = 1 BOC
Ta có A [ và CO=OB do đó A [ = 900 . Vậy DO ⊥ AC và là phân giác của
b + BDO
2
[ ⇐⇒ ∆ADC cân tại D và DM ⊥ AC.
ADC
Ta đảo trật tự trả lời các câu hỏi của bài toán.

21
Hội thảo khoa học, Ninh Bình 15-16/09/2018

b) Xét hai đường tròn ( ABC ) và ( BCD ) và hai cát tuyến là các đường thẳng BD và AC.
Theo bổ đề 1.c OA ⊥ DE ⇐⇒ O là trực tâm của tam giác ADE.
a) Đường thẳng AP cắt (O) tại K (khác A). Đường thẳng kẻ từ K vuông góc với AC cắt
CP tại G 0 . Ta sẽ chứng minh G 0 chính là điểm G trong đầu bài.

Vì CP ⊥ AK và KG 0 ⊥ AC nên G 0 là trực tâm tam giác AKC. Gọi N là giao của đường
thẳng CP với (O). Theo tính chất trực tâm tam giác NP = PG 0 . Mặt khác:

[ = COB
CPB [ = 2 BAC
[ = 2 BNC
[ = BNC
[ + NBP
[

Do đó BAC
[ = BNC [ = NBP [ =⇒ NPB là tam giác cân ⇐⇒ BP = PG 0 và NPG 0 là tam
giác vuông ⇐⇒ BGP[ = OPC [ (= 900 − BAC [ ). Do đó BG 0 //PO. Điểm G 0 chính là điểm
G. ⇐⇒ BP = PG.
c) Theo tính chất trực tâm ta có PAG [ = OAC [ (= 900 − ABC [ ) ⇐⇒ PAO [ = GAC. [

Ghi chú. Trong phần đầu của lời giải chúng ta chứng minh tam giác DAC là tam giác cân
tại D. Kết quả này giúp cho lời giải phần b) trở nên độc lập. Kết quả này nên tiếp tục khai
thác thêm.

22
Hội thảo khoa học, Ninh Bình 15-16/09/2018

Bài toán 4 (Đề thi vào THPT chuyên KHTN Hà Nội 2018). Cho ngũ giác lồi ABCDE nội
tiếp đường tròn (O) có CD song song với BE. Hai đường chéo BD và CE cắt nhau tại P. Điểm
M thuộc đoạn thẳng BE sao cho \ MAB = PAE.d Điểm K thuộc đường thẳng AC sao cho MK
song song với AD, điểm L thuộc đường thẳng AD sao cho ML song song với AC. Đường tròn
ngoại tiếp tam giác KBC lần lượt cắt BD, CE tại Q, S (Q khác B, S khác C).
a) Chứng minh rằng ba điểm K, M, Q thẳng hàng.
b) Đường tròn ngoại tiếp tam giác LDE lần lượt cắt BD, CE tại T, R (T khác D, R khác
E).Chứng minh rằng năm điểm M, S, Q, R, T cùng thuộc một đường tròn.
c) Chứng minh rằng đường tròn ngoại tiếp tam giác PQR tiếp xúc với đường tròn (O).

Lời giải và bình luận.


− Phân tích đầu bài.
Kéo dài AM và AP cắt (O) tại X và Y. Ta sẽ tạo một bài toán tương tự nhưng vai trò các
điểm P và M từ vai trò là các chốt thụ động thành các điểm chủ động (hình a). Bài toán
bắt đầu lại như sau:

Cho tam giác AXY nội tiếp (O). Tia AB cắt (O) tại B. Tia AE là ảnh của tia AB qua
đường phân giác XAY
[ và AE cắt (O) tại E (hình b).

23
Hội thảo khoa học, Ninh Bình 15-16/09/2018

Như vậy điểm P xác định được. Vì P nằm trên đường trung trực BE do đó nằm trên
trung trực của XY. Mặt khác P nằm trên AY, do đó là giao của 2 đường này.
(*) Từ P kẻ đường thẳng song song với XY cắt AX tại I. đường thẳng PX cắt (O) tại A0
XP XI
(khác X). Tam giác ∆PIX v ∆AA0 X qua phép chiếu đồng dạng tâm X tỉ lệ = .
XA0 XA
Vì thế đường tròn (O) và ( PIX ) cũng là ảnh của nhau qua phép chiếu này. X ∈ (O) nên
chúng tiếp xúc nhau. Nhận xét này làm chứng minh phần c) trở thành đơn giản hơn
nhiều.
Ta lấy M là một điểm trên AX và từ đó dựng lại toàn bộ các điểm B, C, D, E của bài toán
đầu tiên (M ∈ BE//XY, BP cắt (O) tại P, EP cắt (O) tại C.

1) Sử dụng Bổ đề 1.b cho hai đường tròn ( DEL) và (O) với 2 cát tuyến ADL và CEQ ta
thu được trực tiếp đpcm. Từ đây ta cũng có D, E, L, R nội tiếp một đường tròn.
2) Tiếp tục sử dụng bổ đề trên ta có SQ//BC, RT//DE. Từ đó suy ra:

[ = DEC
QSR [ = DAB
[ = QMR
\ = DBC [ Vậy ta có RQSMT nằm trên một đường
[ = QTR.

24
Hội thảo khoa học, Ninh Bình 15-16/09/2018

tròn.
3) Ta phải chứng minh đường tròn ( PRQ) tiếp xúc với (O).
Ta có X, R, M, E thuộc một đường tròn vì XAC [ = XEC [ cùng chắn cung
\ = XAC
XC; XMR [ (đồng vị). Từ đó suy ra XRE [ = \XME = PIX. d
Vậy X, P, I, R cùng thuộc đường tròn ( PXI ). Trong phần xây dựng các
điểm (*) ở trên chúng ta đã chỉ ra đường tròn ( PIX ) tiếp xúc với (O).


4 Một hướng ra đề chân phương


Đã nhiều năm nay đề thi của các trường phía Nam có phong cách rất trong sáng và
vừa sức với các bạn có năng khiếu hình học hoặc không có điều kiện luyện lò, hoặc tự
học cũng có thể làm tốt bài thi.
Quan điểm “cũ” với người đã qua và “mới” với người vừa đến là hướng đi thực tế và
phù hợp. Các bài toán với kết quả hay có thể tìm các lời giải đẹp tạo được niềm say mê
học tập cho học sinh. Hầu hết các trường chuyên phía Nam thoát được cái bẫy dồn học
sinh thành thợ giải toán với phương châm ra đề rất nhân văn này.
Chúng tôi xin lấy một ví dụ của một trường có tầm quan trọng tiên phong.

Bài toán 5 (Đề thi vào THPT chuyên ĐHSP tp HCM 2018). Cho đường tròn tâm O và
một điểm S nằm ngoài (O). Kẻ các cát tuyến SAB và SCD đến (O) (A nằm giữa S và B,
C nằm giữa S và D). Đường thẳng (d) vuông góc với OS tại S cắt các đường thẳng BC
và AD tại E và F. Chứng minh rằng OE = OF.

Lời giải và bình luận.

25
Hội thảo khoa học, Ninh Bình 15-16/09/2018

Kí hiệu I và J là trung điểm của BC và AD.


Ta có ∆SBC v ∆SDA (g.g) ⇐⇒ ∆SBI v ∆SDJ (c. g. c.) ⇐⇒
SIE
d = SJF. Tứ giác SFOJ nội tiếp (các góc tại S và J là góc
d
vuông) ⇐⇒ SOF d = SJF
d = SIE d [ Vậy tam giác FOE cân.
= SOE.


Các đề bài được lấy từ các đề thi toán lên lớp 10 năm 2018.

26
Hội thảo khoa học, Ninh Bình 15-16/09/2018

MỘT SỐ ỨNG DỤNG CỦA VÀNH VÀ ĐỒNG CẤU


VÀNH TRONG TOÁN PHỔ THÔNG

Đàm Văn Nhỉ


Trường ĐHSP Hà Nội

Tóm tắt nội dung



Trong bài này chúng tôi sử dụng một số tính chất của vành Z[ d] để khảo sát các
dạng toán liên quan đến số học và đa thức nguyên.

1 Phương pháp vành và đồng cấu


Nhiều khi để giải một √
bài toán ta phải sử dụng một vành nào đó. Trong mục này
chúng tôi sử dụng vành Z[ d].
Mệnh đề 1. Cho số nguyên d > 1 không là số chính phương, Khi đó
√ √
(1) Tập Z[ d] = { a + b d| a,√b ∈ Z} cùng √ phép cộng √ và nhân lập
√ thành một vành giao hoán
với đơn vị và ánh xạ f : Z[ d] → Z[ d], a + b d 7→ a − b d, là một tự đẳng cấu (liên
hợp).
√ √
(2) Tập Z[ −d] = { a + ib d| a, b ∈√Z} cùng phép √ cộng và nhân
√ lập thành một
√ vành giao
hoán với đơn vị và ánh xạ f : Z[ −d] → Z[ −d], a + ib d 7→ a − ib d, là một tự
đẳng cấu (liên hợp).
√ √ √ √
Với z = a + b d ∈ Z[ d] và u = a + ib d ∈ Z[ −d] ta ký hiệu N (z) = a2 −
db2 , N (u) = a2 + db2 và gọi là chuẩn của z hay u. Khi đó
√ √
Hệ quả 1. Với z1 , z2 , . . . , zn ∈ Z[ d] và u1 , u2 , . . . , un ∈ Z[ −d] ta luôn có hệ thức
N ( z1 z2 . . . z n ) = N ( z1 ) N ( z2 ) . . . N ( z n )
N ( u1 u2 . . . u n ) = N ( u1 ) N ( u2 ) . . . N ( u n ).
√ √ n
Chứng minh. Giả sử zk = ak + bk d ∈ Z[ d] với k = 1, . . . , n, và viết tích ∏ ( ak +
k =1
√ √ n √ √
bk d) = a + b d. Qua tự đẳng cấu liên hợp ta có ngay ∏ ( ak − bk d) = a − b d. Từ
k =1
đây suy ra N (z1 z2 . . . zn ) = N (z1 ) N (z2 ) . . . N (zn ) vì
n √ n √ n
a2 − b2 d = ∏ ( a k + b k d ) ∏ ( a k − b k d) = ∏ (a2k − bk2 d).
k =1 k =1 k =1

Tương tự, ta cũng có N (u1 u2 . . . un ) = N (u1 ) N (u2 ) . . . N (un ).

27
Hội thảo khoa học, Ninh Bình 15-16/09/2018

Mệnh đề 2. Giả thiết p, q là hai số nguyên dương không có nhân tử là số chính phương với
√ √ √ √ √
( p, q) = 1. Khi đó tập Z[ p, q] = { a + b p + c q + d pq| a, b, c, d ∈ Z} cùng phép cộng
√ √ √ √
và nhân lập thành một vành giao hoán với đơn vị và các ánh xạ φi : Z[ p, q] → Z[ p, q] :
√ √ √
z = a + b p + c q + d pq 7→
 √ √ √
 φ1 (z) = a + b p + c q + d pq
φ (z) = a − b√ p + c√q − d√ pq



2
√ √ √
 φ3 (z) = a + b p − c q − d pq
φ (z) = a − b√ p − c√q + d√ pq



4

là những tự đẳng cấu.


√ √
Với z ∈ Z[ p, q], đặt N (z) = φ1 (z)φ2 (z)φ3 (z)φ4 (z). Khi đó ta có:
√ √
Hệ quả 2. Với z1 , z2 ∈ Z[ p, q] có hệ thức N (z1 z2 ) = N (z1 ) N (z2 ).
√ √ √ √
Chứng minh. Với z1 , z2 ∈ Z[ p, q] có z1 z2 ∈ Z[ p, q] và φi (z1 z2 ) = φi (z1 )φi (z2 ).
4 4
Vậy N (z1 z2 ) = ∏ φi (z1 ) ∏ φi (z2 ) = N (z1 ) N (z2 ).
i =1 i =1

Mệnh đề 3. p, q là hai số nguyên dương không có nhân tử là số chính phương với ( p, q) = 1.


√ √  √ √  √ √  √ √
Khi đó Q p, q = Q p + q . Đặc biệt ta còn có Q p, q = Q p, q .
√ √ √ √  √ √  √ √  √
Chứng minh. Vì p + q ∈ Q p, q nên Q p + q ⊂ Q p, q . Đặt u = p +

q. Khi đó ta có hệ
(√ √
p+ q = u
√ √
( p + 3q) p + (q + 3p) q = u3 .
√ √ √ √  √ √  √ √ 
Vậy p, q ∈ Q(u) và như vậy Q p + q ⊃ Q p, q . Tóm lại Q p, q =
√ √ √ √ √ √
Q p + q . Dễ dàng chỉ ra Q p, q = Q p, q .
   

√ √
Bài toán 1. Số α = 2 + 3 là số đại số nguyên vì nó là một nghiệm của đa thức f ( x ) =
x4 − 10x2 + 1 ∈ Z[ x ].

Bài toán 2. Số sin 10◦ là một số đại số trên Z vì nó là một nghiệm của đa thức f ( x ) = 8x3 −
6x + 1 ∈ Z[ x ].
√ √
Bài toán
√ 3. Có hay
√ không một đa thức p ( x ) ∈ Z [ x ] thỏa mãn p ( 1 + 3 ) = 2 + 3 và
p(3 + 5) = 3 + 5.

Lời giải. Hiển nhiên p( x ) − √ x có nghiệm√ là 3 + 5.√Do vậy p( x )√= ( x2 − 6x√+ 4)q( x ) +
√x
với q( x ) ∈√Z[ x ]. Vì √
p(1 + 3) = 2 + 3 nên 2 + 3 = q(1 + 3)( √ 2 − 4 3 ) + 1
√ + 3
hay q(1 + 3)(2 − 4 3) = 1. Lấy chuẩn hai vế, ta có 1 = 4N (q(1 + 3)) N (1 − 2 3), vô
lý. Từ đây suy ra rằng, đa thức p( x ) không tồn tại.

28
Hội thảo khoa học, Ninh Bình 15-16/09/2018

2 Phương trình Pell


Ta chứng minh sự tồn tại nghiệm của phương trình Pell.

Bài toán 4. Với số nguyên dương n, giả sử các số a1 , b1 , . . . , an , bn , a, b ∈ Z thỏa mãn quan hệ
n √ √
∏ ( ai + bi 5) = a + b 5. Khi đó
i =1
n
(1) ∏ ( a2i − 5bi2 ) = a2 − 5b2 .
i =1
√ n √
(2) Giả sử 9 + 4 5 = an + bn 5. Ta có a2n − 5bn2 = 1. Từ đó suy ra phương trình
x2 − 5y2 = 4 có nhiều vô hạn nghiệm nguyên dương x, y.
√ √ √
Lời giải. (1) Xét vành giao hoán Z[ 5] = {r + s 5|r, s ∈ Z}. Đồng cấu f : Z[ 5] →
√ √ √ n √
Z[ 5], r + s 5 7→ r − s 5, là một đẳng cấu. Tác động f lên hai vế f ( ∏ ( ai + bi 5)) =
i =1
√ n √ √ n √ n √
f ( a + b 5) hay ∏ ( ai − bi 5) = a − b 5. Do đó ∏ ( ai + bi 5) ∏ ( ai − bi 5) = ( a +
√ √ i =1 i =1 i =1
b 5)( a − b 5) = a2 − 5b2 . √  √ n √
n
(2) Ta có a2n − 5bn2 = 9 + 4 5 9 − 4 5 = 1 theo (1). Đặt xn + yn 5 = an +
√  √  √
bn 5 3 + 5 = (3an + 5bn ) + ( an + 3bn ) 5. Khi đó xn2 − 5y2n = ( a2n − 5bn2 )(32 − 5.12 ) =
4 với mọi n. Như thế phương trình x2 − 5y2 = 4 có nhiều vô hạn nghiệm nguyên x, y.
Khi x, y là nghiệm của x2 − 5y2 = 4 thì ± x, ±y cũng là nghiệm. Do vậy x2 − 5y2 = 4 có
nhiều vô hạn nghiệm nguyên dương x, y.

Bài toán 5. Với bất kỳ số nguyên n > 3 đều có hai số nguyên lẻ x và y để 2n = 7x2 + y2 .
√ √ √
Lời giải. Xét vành giao hoán
√ Z[ −√7] = {r + is 7|r, s ∈ Z}. Chuẩn của z = r + is 7 là
N (z) = r2 + 7s2 = (r + is 7)(r − is 7). Hiển nhiên N (z1 z2 ) = N (z1 ) N (z2 ), chứng minh
như trong Ví dụ 4. Bằng phương pháp quy nạp theo n để chỉ ra, với bất kỳ số nguyên
n > 3 đều có hai số nguyên lẻ x và y để 2n = 7x2 + y2 .
Với n = 3 có 23 = 8 = 1 + 7 = 12 + 7.12 .
Giả sử đã có hai số nguyên lẻ x, y để y2 + 7x2 = 2n với số nguyên n > 3. Với số nguyên
n + 1 ta xét
√ √ √
z1 = (y + ix 7)(1 + i 7) = (y + x − 8x ) + i ( x + y) 7
√ √ √
z2 = (y + ix 7)(1 − i 7) = (y − x + 8x ) + i ( x − y) 7.

Vì x, y là số lẻ nên x = 2k + 1, y = 2h + 1. Khi đó x + y = 2(k + h + 1), y − x = 2(h − k )


và ( x + y) − (y − x ) = 4k + 2. Do đó trong hai số x + y và y − x chỉ có đúng một số chia
hết cho 4.
y−x y−x x − y√
Nếu x + y chia hết cho 4 thì là số nguyên lẻ. Khi đó z3 = ( + 4x ) + i 7
√ 2 √ 2 2
N (y + ix 7) N (1 − i 7)
với N (z3 ) = = 2n .2 = 2n+1 .
4
y+x x+y x + y√
Nếu y − x chia hết cho 4 thì là số lẻ. Khi đó z4 = ( − 4x ) + i 7 với
√ √2 2 2
N (y + ix 7) N (1 + i 7)
N ( z4 ) = = 2n .2 = 2n+1 .
4
Bài toán 6. Với bất kỳ số nguyên n > 1 đều có hai số nguyên lẻ x và y để 4.3n = 11x2 + y2 .

29
Hội thảo khoa học, Ninh Bình 15-16/09/2018
√ √ √
Lời giải. Xét vành giao hoán √ Z[ −11] =√{r + is 11|r, s ∈ Z}. Chuẩn của z = r + is 11
là N (z) = r2 + 11s2 = (r + is 11)(r − is 11). Hiển nhiên N (z1 z2 ) = N (z1 ) N (z2 ). Ta có
một vài số cụ thể: 4.3 = 12 = 11.12 + 12 , 4.32 = 36 = 11.12 + 52 , 4.33 = 108 = 11.32 + 32 .
Bằng phương pháp quy nạp theo n để chỉ ra, với bất kỳ số nguyên n > 1 đều có hai số
nguyên lẻ x và y để 4.3n = 11x2 + y2 .
Với n = 1 có 4.3 = 11.12 + 12 .
Giả sử đã có hai số nguyên lẻ x, y để 11x2 + y2 = 4.3n với số nguyên n > 1. Với số nguyên
n + 1 ta xét
√ √ √
z1 = (y + ix 11)(1 + i 11) = (y + x − 12x ) + i ( x + y) 11
√ √ √
z2 = (y + ix 11)(1 − i 11) = (y − x + 12x ) + i ( x − y) 11.

Vì x, y là số lẻ nên x = 2k + 1, y = 2h + 1. Khi đó x + y = 2(k + h + 1), y − x = 2(h − k )


và ( x + y) − (y − x ) = 4k + 2. Do đó trong hai số x + y và y − x chỉ có đúng một số chia
hết cho 4.
y−x y−x x − y√
Nếu x + y chia hết cho 4 thì là số lẻ. Khi đó z3 = ( + 6x ) + i 11 với
√ 2
√ 2 2
N (y + ix 11) N (1 − i 11)
N ( z3 ) = = 4.3n .3 = 4.3n+1 .
4
y+x x+y x + y√
Nếu y − x chia hết cho 4 thì là số lẻ. Khi đó z4 = ( − 6x ) + i 11 với
√ 2
√ 2 2
N (y + ix 11) N (1 + i 11)
N ( z4 ) = = 4.3n .3 = 4.3n+1 . Tóm lại, đã chứng minh xong
4
bài toán.
n 2
Bài toán ( 7. Chứng minh rằng luôn luôn tồn tại hai số nguyên x và y để 19 = x −
x lẻ và y lẻ khi n lẻ
6y2 với
x lẻ và y chẵn khi n chẵn.
√ √ √
Lời√giải. Xét√vành giao√hoán Z[ 6] = {r + s 6|r, s ∈ Z}. Đồng cấu √ f : Z[ √6] →
Z[ 6√ ], r + s 6 7→ r − s 6, là một đẳng cấu. √ Do như vậy √ mà khi ( a√+ b 6)(c + d 6) =
u + v 6 thì tác động √ f lên hai vế được ( a − b 6)(c − d 6) = u − v 6. Sử dụng kết quả
này, với z = a + b 6 và đặt N (z) = a2 − 6b2 có ngay N (z1 z2 ) = N (z1 ) N (z2 ). Với n = 1
có 19 = 52 − 6.12 và 192 = 312 − 6.102 . Giả sử số nguyên n > 1 và hai số nguyên a, b để
19n = a2 − 6.b2 , trong đó a và b đều lẻ khi n lẻ; còn a lẻ và b chẵn khi n chẵn. Với n + 1 ta
xét:
(1) Nếu n lẻ thì có số a lẻ và số b lẻ để a2 − 6b2 = 19n . Khi đó n + 1 là số chẵn và
19n+1 = (5a + 6b)2 − 6( a + 5b)2 với 5a + 6b lẻ và a + 5b chẵn.
(2) Nếu n chẵn thì có số a lẻ và số b chẵn để a2 − 6b2 = 19n . Khi đó n + 1 là số lẻ và
19n+1 = (5a + 6b)2 − 6( a + 5b)2 với 5a + 6b lẻ và a + 5b cũng lẻ.
Tóm lại ta đã có điều cần chứng minh.
n 2
Bài toán( 8. Chứng minh rằng luôn luôn tồn tại hai số nguyên x và y để 31 = x +
x lẻ và y lẻ khi n lẻ
6y2 với
x lẻ và y chẵn khi n chẵn.
√ √ √
√ giao hoán√Z[ −6] = {r + is 6|r, s ∈ Z}. Đồng cấu f : Z[ √−6] →
Lời√giải. Xét vành
Z[√ −6], r + is √ 6 7→ r − is 6, là một đẳng cấu. Do như√vậy mà khi
√ ( a + ib 6)(√
c+
id 6) = u + iv 6 thì tác động f lên hai vế được ( a − ib 6)(c − id 6) = u − iv 6.

30
Hội thảo khoa học, Ninh Bình 15-16/09/2018

Sử dụng kết quả này, với z = a + ib 6 và đặt N (z) = a2 + 6b2 có ngay N (z1 z2 ) =
N (z1 ) N (z2 ). Với n = 1 có 31 = 52 + 6.12 và 312 = 192 + 6.102 . Giả sử số nguyên n > 1 và
hai số nguyên a, b để 31n = a2 + 6b2 , trong đó a và b đều lẻ khi n lẻ; còn a lẻ và b chẵn khi
n chẵn. Với n + 1 ta xét:
(1) Nếu n lẻ thì có số a lẻ và số b lẻ để a2 + 6b2 = 31n . Khi đó n + 1 là số chẵn và
31n+1 = (5a − 6b)2 + 6( a + 5b)2 với 5a − 6b lẻ và a + 5b chẵn.
(2) Nếu n chẵn thì có số a lẻ và số b chẵn để a2 + 6b2 = 31n . Khi đó n + 1 là số lẻ và
31n+1 = (5a − 6b)2 + 6( a + 5b)2 với 5a − 6b lẻ và a + 5b cũng lẻ.
Tóm lại ta đã có điều cần chứng minh.

Bài toán 9. Tìm các số x, y, z, t ∈ Z thỏa mãn phương trình


√ √ √
( x + y 3)4 + (z + t 3)4 = 20 + 12 3.
√ 4 √ 4 √
Lời giải. Giả sử có x, y, z, t ∈ Z thỏa mãn √ ( x + y 3) + (z + t 3) = 20 + 12 3. Sử
dụng
√ 4tự đẳng cấu √ liên hợp của√vành Z[ 3] cho đồng nhất thức này có mâu thuẫn ( x −
4
y 3) + (z − t 3) = 20 − 12 3 < 0.
√ √ √ √
Bài toán 10. Với mỗi số nguyên dương n ta biểu diễn (1 + 2 + 3)n = an + bn 2 + cn 3 +
√ bn dn
dn 6 với an , bn , cn , dn ∈ N. Tìm lim , lim .
n→∞ an n→∞ an
√ √ √
Lời
√ n giải. Sử dụng√ các tự
√ đẳng cấu
√ của vành Z [ 2, 3 ] cho đồng nhất thức ( 1 + 2+
3) = an + bn 2 + cn 3 + dn 6, ta được
√ √ √ √ √
x1n = (1 + 2 + 3)n = an + bn 2 + cn 3 + dn 6
√ √ √ √ √
x2n = (1 − 2 + 3)n = an − bn 2 + cn 3 − dn 6
√ √ √ √ √
x3n = (1 + 2 − 3)n = an + bn 2 − cn 3 − dn 6
√ √ √ √ √
x4n = (1 − 2 − 3)n = an − bn 2 − cn 3 + dn 6.
√ bn
Vậy 4an = x1n + x2n + x3n + x4n , 4 2bn = x1n − x2n + x3n − x4n . Từ đó ta có lim =
n→∞ an
1 x n − x2n + x3n − x4n 1 dn 1
√ lim 1n n n n = √ , lim =√ .
2 n → ∞ x1 + x2 + x3 + x4 2 n → ∞ an 6
Bài toán 11. Giả sử dãy số ( an ), (bn ), (cn ), (dn ) được xác định như sau:

 a1 = 1, b1 = 1, c1 = 1, d1 = 0


 an+1 = an + 2bn + 5cn



bn+1 = an + bn + 5dn

cn+1 = an + cn + 2dn





d n + 1 = bn + c n + d n .

√ √ √ √ √
Khi đó, với mỗi số nguyên dương n có (1 + 2+ 5)n = an + bn 2 + cn 5 + dn 10 và
an
a2n − 10d2n chia hết cho 2n−1 . Tìm lim .
n→+∞ dn

31
Hội thảo khoa học, Ninh Bình 15-16/09/2018
√ √ n √
Lời√giải. Với
√ mỗi số nguyên dương n ta biểu diễn (1 + 2 + 5) = An + Bn 2 +
Cn 5 + Dn 10, trong đó An , Bn , Cn , D√n ∈ N. Hiển
√ nhiên A1√= 1 = a1 , B1√= 1 =
√b1 ,nC 1 =
1 = c1 , D1 = 0 = d1 . Từ An+1 + Bn+1 2 + Cn+1 5 + Dn+1 10 = (1 + 2 + 5) + 1 ta
suy ra
An+1 = an+1 , Bn+1 = bn+1 , Cn+1 = cn+1 , Dn+1 = dn+1 .
√ √ √ √ n
Sử dụng √ của vành Z[ 2, 5] cho đồng nhất thức (1 + 2 + 5) =
√ các tự√đẳng cấu
an + bn 2 + cn 5 + dn 10, ta nhận được
√ √ √ √ √
(1 + 2 + 5)n = an + bn 2 + cn 5 + dn 10
√ √ √ √ √
(1 − 2 + 5)n = an − bn 2 + cn 5 − dn 10
√ √ √ √ √
(1 + 2 − 5)n = an + bn 2 − cn 5 − dn 10
√ √ √ √ √
(1 − 2 − 5)n = an − bn 2 − cn 5 + dn 10. Do đó có hai quan hệ
√ √ √ √ √
2an + 2dn 10 = (1 + 2 + 5)n + (1 − 2 − 5)n
√ √ √ √ √
2an − 2dn 10 = (1 − 2 + 5)n + (1 + 2 − 5)n .
h √ √ √ √ i
Vậy 4a2n − 40d2n = 2n (2 + 5)n + (2 − 5)n + (−1 + 2)n + (−1 − 2)n hay
h
a2n − 10d2n = 2n−1 (2n + (−1)n )(n0 ) + (2n−2 .5 + (−1)n .2)(n2 ) + · · · + +(2n−2k .5k +
n
i an √
(−1)n .2k )(2k ) + · · · và lim = 10.
n→+∞ dn

Bài toán 12. Có hay không một số nguyên dương n thỏa mãn hệ thức
√ √ 2010 √ √
P= 2009 − 2008 = n − n − 1.
√ √ 2010
Lời giải. Đặt r = 2008. Ta có P = r+1− r . Khai triển được:
q 1005 q
P = 2r + 1 − 2 r (r + 1) = a − b r (r + 1)
1005
với a, b ∈ Z. Khi đó Q = 2r + 1 + 2 r (r + 1)
p p
= a + b r (r + 1). Vậy a2 − b2 r (r +
1005 1005
= 1. Với n = a2 ∈ N+ có
p p
1) = 2r + 1 − 2 r (r + 1) 2r + 1 + 2 r (r + 1)
√ √ 2010 p √ √
b2 r (r + 1) = n − 1. Như thế

2009 − 2008 = a − b r (r + 1) = n − n − 1.
Vậy, có số tự nhiên n thỏa mãn đề bài.
√ √
√ Q. √
4 2
√ α =√ 2 +√ 3 trên
Bài toán√13. Xét Đa thức √ của nó là x − 10x + 1. 4 số liên hợp
√tối tiểu
của α là 2 + 3, − 2 − 3, − 2 + 3 và 2 − 3. Ta có N (α) = 1, Sp(α) = 0.
1
Bài toán 14. Chứng minh rằng số thực √ √ là đại số trên Z. Tìm đa thức bậc thấp
1+ 3
2+2 3 4
nhất thuộc Z[ x ] nhận nó làm nghiệm.
√ √ √ √
Lời giải. Ta chỉ cần chỉ ra số α = 1 + 3 2 + 2 3 4 là đại số trên Z. Vì số 3 2 và√
2 3 4 là đại
√ số
trên Z nên dễ dàng suy ra α là đại số trên Z theo Bổ đề ??. Đặt x = α − 1 = 3 2(1 + 2 3 2).
Ta nhận được phương trình
√ √
x3 = 2[17 + 6 2(1 + 2 2)] = 34 + 12x.
3 3

Đa thức f ( x ) = ( x − 1)3 − 12( x − 1) − 34 = x3 − 3x2 − 9x − 23 nhận α làm nghiệm. Vậy


g( x ) = 23x3 + 9x2 + 3x − 1 ∈ Z[ x ] là đa thức cần tìm.

32
Hội thảo khoa học, Ninh Bình 15-16/09/2018
 3 + √17 n  3 − √17 n
Bài toán 15. Chứng minh rằng, số + là số nguyên lẻ và phần
√ 2 2
h 3 + 17 2n i
nguyên là số nguyên chẵn với mọi số tự nhiên dương n.
2
√ √
√ 3 + 17 3 − 17
Lời giải. Vì số 17 là đại số trên Z nên dễ dàng suy ra x1 = và x2 =
2 2
2 n n
là hai nghiệm của x − 3x − 2 = 0. Đặt an = x1 + x2 với n = 0, 1, 2, . . . . Khi đó an+2 =
3an+1 + 2an . Vì a1 = 3 nên a2 là số nguyên lẻ. Quy nạp theo n, khi an là số nguyên √ lẻ thì
3 − 17
an+1 cũng là số nguyên lẻ. Vậy an là số nguyên lẻ với mọi n. Ta lại có 0 < | |<1
√ √ 2
 3 + 17 2n h 3 + 17 2n i
nên a2n − 1 < < a2n . Vậy = a2n − 1 là số nguyên chẵn.
2 2
√ p √ a3 − 6an − 4
Bài toán 16. Xét dãy ( an ) với a0 = 5 − 5 + 2 5, an+1 = n2 với số nguyên
3an + 6an + 2
n > 0. Chứng minh an ∈ / Q và đều là nguyên trên Z với mọi số tự nhiên n.
√ p √ √ p √
Lời giải. Vì p 5, 5 + 2 5 là nguyên trên Z nên 5 − 5 + 2 5 là nguyên trên Z. Xét
√ √ √ √
x − 5 = − 5 + 2 5. Khi đópx2 − 2 5x = 2 5. Vậy ta có f ( x ) = x4 − 20x2 − 40x − 20
√ √
là đa thức tối tiểu của 5 − 5 + 2 5. Vì đa thức này không có nghiệm hữu tỷ nên
a0 ∈/ Q. √
π 5−1 π  3n π 
Từ sin = có a0 = tan − 1. Nếu an = tan − 1 = t − 1 thì an+1 =
10 4 20 20
a3n − 6an − 4 ( t − 1)3 − 6( t − 1) − 4 t3 − 3t2 − 3t + 1  3n +1 π 
= = = tan − 1. Tóm
3a2n + 6an + 2 3( t − 1)2 + 6( t − 1) + 2 3t2 − 1 20
 3n π 
lại an = tan − 1 với mọi số tự nhiên n. Vì 3n là số lẻ với mọi số tự nhiên n nên
20
 3n π 3n π 20
−1 = cos 3n π + i sin 3n π = cos + i sin
20 20
 3n π 
và ta suy ra được đa thức f ( x ) ∈ Z[ x ] nhận tan làm nghiệm. Do đó an là nguyên
20 π  3π   9π 
trên Z. Vì 3 ≡ 1( mod 20) nên ta chỉ cần kiểm tra tan
4 , tan , tan =
π  7π   3π  20 20 20
cot và tan = cot có là số vô tỷ hay không. Dễ dàng kiểm tra trực
20 20 20
tiếp.

Bài toán 17. Tìm tất cả các số nguyên dương n sao cho 1999n + 2011n chia hết cho 1999n−1 +
2011n−1 .

Lời giải. Đặt am = 1999m + 2011m với m = 0, 1, 2, . . . Vì 1999 và 2011 là hai nghiệm
của x2 − (1999 + 2011) x + 1999.2011 = 0 nên ta có hệ thức am = (1999 + 2011) am−1 −
1999.2011am−2 với mọi số nguyên m > 2. Với n = 1 có a1 chia hết cho a0 . Nếu n > 1 thì
do an−1 nguyên tố cùng nhau dối với 1999 và 2011 nên khi an mà chia hết cho an−1 thì
an−2 cũng phải chia hết cho an−1 : nhưng điều này không thể vì an−2 < an−1 .

33
Hội thảo khoa học, Ninh Bình 15-16/09/2018

Tài liệu
[1] Nuyên Văn Mậu, Đa thức đại số và phân thức hữu tỉ, Nhà xuất bản Giáo dục 2002.
[2] T. T. Nam, Đ. V. Nhỉ, L. B. Thắng, Một số ứng dụng của Đại số hiện đại vào giải Toán sơ
cấp, NXB Đại học Sư phạm TP Hồ Chí Minh, 2015.
[3] Đàm Văn Nhỉ,..., Đa thức - Chuỗi và Chuyên đề nâng cao, Nhà xuất bản Thông tin và
Truyền thông 2017.

34
Hội thảo khoa học, Ninh Bình 15-16/09/2018

VỀ MỘT CÔNG THỨC TÍNH DIỆN TÍCH TỨ GIÁC


TRONG CÁC SÁCH CỔ

Tạ Duy Phượng, Viện Toán học,


Mai Văn Thu, Trường ĐH Hồng Đức, Thanh Hóa

Tóm tắt nội dung

Bài báo cáo này nhằm trình bày về một công thức tính diện tích tứ giác có trong các
sách cổ.

1 Mở đầu
Một tam giác hoàn toàn được xác định (dựng được) nếu biết ba yếu tố độc lập (thí dụ,
ba cạnh a, b, c thỏa mãn tính chất tổng độ dài hai cạnh lớn hơn độ dài cạnh thứ ba; hoặc
hai cạnh và một góc xen giữa; hoặc một cạnh và hai góc kề cạnh ấy). Để một tứ giác được
hoàn toàn xác định, ta phải biết năm yếu tố. Thí dụ, biết độ dài bốn cạnh và một đường
chéo, hoặc độ dài bốn cạnh và tổng hai góc đối diện.
Để tínhp diện tích tam giác khi biết ba cạnh, ta có Hệ thức Heron
S = p( p − a)( p − b)( p − c). Để tính diện tích tứ giác, ta có công
thức Bretschneider (1842, xem chứng minh trong Bretschneider’s formula
- Wikipedia
s hoặc Bretschneider’s Formula – from Wolfram MathWorld):
 
2
A+C
S = ( p − a)( p − b)( p − c)( p − d) − abcdcos , trong đó a, b, c, d là độ
2
a+b+c+d
dài các cạnh, p = là nửa chu vi, S là diện tích, A và C là số đo hai góc A và
2
C.

Nhận xét 1. Vì tổng các góc trong một tứ giác bằng 360◦ nên công thức trên không thay
A+C B+D
đổi nếu thay bằng .
2 2
A+C
Nhận xét 2. Nếu ABCD là tứ giác nội tiếp ( = 90◦ ) thì công thức trên có dạng mở
p 2
rộng của hệ thức Heron: S = ( p − a)( p − b)( p − c)( p − d).

Nhận xét 3. Nếu coi tam giác là tứ giác đặc biệt khi cạnh d = 0 thì công thức Bretschnei-
der trở về hệ thức Heron.

Nhận xét 4. Công thức Bretschneider khá cồng kềnh. Liệu ta có công thức đánh giá trên
(đánh giá theo bất đẳng thức) diện tích tứ giác qua bốn cạnh không?-Xét bài toán sau
đây.

35
Hội thảo khoa học, Ninh Bình 15-16/09/2018

Bài toán 1 (Olympic Moscow lần thứ 16, 1953, Lớp 8, vòng II). Cho a, b, c, d là các cạnh
a+c b+d
liên tiếp và S là diện tích của một tứ giác. Chứng minh rằng S ≤ . .
2 2
Chứng minh. Có thể giả thiết ABCDlà tứ giác lồi vì nếu ABCD là tứ giác lõm thì ta có
(Hình 1) S ABCD ≤ S ABCD0 . Gọi các cạnh AB, BC, CD, DA tương ứng là

a, b, c, d;

các đường cao hạ từ đỉnh A xuống các cạnh b, c là hb , hc . Ta có (Hình 2): S = S ABC +
1 1 1
S ACD = bhb + chc ≤ (ba + cd) .
2 2 2

Hình 1

Hình 2

Kí hiệu các đường cao hạ từ đỉnh B xuống các cạnh c, d là hb , hc0 . Ta có (Hình 3):
1 1 1 1 1
S = S ABD + SCBD = dhd + chc0 ≤ (da + cb) . Suy ra 2S ≤ (ba + cd) + (da + cb)
2 2 2 2 2
a+c b+d
hay S ≤ . .
2 2

36
Hội thảo khoa học, Ninh Bình 15-16/09/2018

Hình 3

Nhận xét 5. Dấu bằng trong công thức (1) xảy ra khi và chỉ khi tất cả bốn đường cao
hb , hc , hb , hc0 trùng với các cạnh tứ giác hay ABCD là hình chữ nhật. Một điều đáng ngạc
a+c b+d
nhiên là đẳng thức (công thức) sai S = . (1) đã được sử dụng ở khá nhiều nơi
2 2
trên thế giới trong suốt hơn 3000 năm (từ thời Babilon, 1600 năm trước công nguyên đến
tận thế kỉ thứ 19).
Công thức (1) có thể tìm thấy trong Rind Papyrus (giấy cói) vào khoảng 1550 trước
công nguyên và trong các tứ giác trong ngôi đền thần Horus tại Edfu (Ai Cập) vào
khoảng 100 năm trước công nguyên (xem [1], trang 54). Xét bài toán sau đây.

Bài toán 2. Một hình bình hành có các cạnh với đội dài 20, 5, 20, 5. Khoảng cách giữa hai
cạnh có độ dài 20 là 4. Tính sai số giữa diện tích thực của hình bình hành và diện tích
tính theo công thức (1).

Giải. Diện tích hình bình hành bằng S = 20 × 4 = 80 (đvdt).


20 + 20 5 + 5
Diện tích tính theo công thức (1) là S0 = × = 100 (đvdt).
2 2
100 − 80 1
Như vậy, sai số lên đến % = 20%, bằng diện tích chính xác. Một con số
100 5
không nhỏ!
Công thức (1) cũng đã được sử dụng trong các sách toán Hán Nôm, từ Toán pháp đại
thành của Lương Thế Vinh cuối thế kỉ XV đến Ý Trai toán pháp nhất đắc lục của Nguyễn
Hữu Thận hoàn thành năm 1829 và một số sách khác. Dưới đây là các bài toán minh họa.

Bài toán 3 (Toán pháp đại thành, trang 44a). Có một thửa ruộng đông-tây cộng lại được
8 trượng 4 xích, nam-bắc cộng lại được 27 trượng 6 xích. Hỏi diện tích thửa ruộng là bao
nhiêu?
Đáp số: ruộng có diện tích 2 sào 8 xích 6 thốn 4 phân.

Giải. Đổi đơn vị:


8 trượng 4 xích = 84 xích = 33,6 m;
27 trượng 6 xích = 276 xích = 110,4 m.
Theo công thức (1) tính diện tích tứ giác ta có
a+c b+d 33, 6 110, 4
S= × = × = 927, 36 (m2 )
2 2 2 2
= 2 × 360 + 8 × 24 + 6 × 2, 4 + 4 × 0, 24 (m2 ) = 2 sào 8 xích 6 thốn 4 phân.

37
Hội thảo khoa học, Ninh Bình 15-16/09/2018

Bài toán 4 (Toán pháp đại thành, trang 45a). Có một thửa ruộng đông-tây cộng lại được
100 trượng 5 xích, nam-bắc cộng lại được 87 trượng 8 xích. Hỏi ruộng bao nhiêu ly, bao
nhiêu mẫu? Đáp số: ruộng 9 mẫu 8 sào 6 thốn 5 phân.

Giải. Đổi đơn vị:


100 trượng 5 xích = 1005 xích = 402 m;
87 trượng 8 xích = 878 xích = 351,2 m.
Theo công thức tính diện tích tứ giác ta có
a+c b+d 402 351, 2
S= × = × = 35295, 6 (m2 )
2 2 2 2
= 9 × 3600 + 8 × 360 + 6 × 2, 4 + 5 × 0, 24 (m2 ) = 9 mẫu 8 sào 6 thốn 5 phân = 882390
ly.
Nhận xét 6. A. Volkov trong [2] có nhắc đến Lương Thế Vinh đã sử dụng công thức (1)
để tính diện tích tứ giác, nhưng Ông không trình bày tính toán cụ thể như trên.
Bài toán 5 (Ý Trai toán pháp nhất đắc lục, Quyển 2). Cho một thửa ruộng, đông dài 17
sào 2 xích, tây dài 16 sào 2 xích, nam rộng 15 sào 7 xích, bắc rộng 14 sào 7 xích, hỏi thực
điền là bao nhiêu?

Giải. Phép tính chiều dài đông tây: cộng (được) 33 sào 4 xích. Gia ngũ (đổi đơn vị), từ
sào dĩ nội, được 499 xích, chia đôi, được 249 xích 5 thốn.
Phép tính chiều rộng nam bắc: cộng (được) 29 sào 14 xích. Gia ngũ, từ sào dĩ nội,
được 449 xích, chia đôi, được 224 xích 5 thốn.
Lấy kết quả hai phép chia ấy nhân với nhau được diện tích: 5 vạn 6012 xích 1 thốn 5
phân. Đơn vị hàng trăm định theo xích, giảm ngũ hai lần, lần thứ nhất giảm đến đơn vị
cuối cùng, được thực điền theo xích: 373 xích, 4 thốn, 1 phân lẻ 8 ly. Lần thứ hai giảm
ngũ, đến thành sào thì dừng, được thực điền theo mẫu sào: 2 mẫu 4 sào 13 xích 4 thốn 1
phân lẻ 8 ly.

Giải thích. Đổi đơn vị:


33 sào 4 xích = 33 × 15 + 4 = 499 xích = 499 × 0, 4 =199,6 m;
29 sào 14 xích = 29 × 15 + 14 = 449 xích = 179,6 m.
Nguyễn Hữu Thận đã sử dụng công thức tính diện tích (1) để tính
a+c b+d 199, 6 179, 6
S= × = × = 8962, 04 (m2 )
2 2 2 2
= 2 × 3600 + 4 × 360 + 13 × 24 + 4 × 2, 4 + 1 × 0, 24 + 8 × 0, 024 + 0, 008 (m2 ) = 2 mẫu
4 sào 13 xích 4 thốn 1 phân 8 ly.
Bài toán 6 (Toán điền trừ cửu pháp, trang 5a). Tìm diện tích ruộng biết đông dài 15 cao
8 xích 7 thốn, nam dài 14 cao 3 xích 1 thốn, tây dài 14 cao 5 xích 3 thốn, bốc dài 14 cao 4
xích 2 thốn.
Đáp số: Thực điền 2 mều 1 cao 4 xích 7 thốn 6 phân.

Cách làm Trên cộng đông tây làm một. Lấy ngũ gia từ cao vị đến cuối (tức là phép
ngũ gia, đổi đơn vị từ sào về thốn). Lấi chia đôi từ thốn vị đế đấu.
Dưới cộng nam bốc làm một. Lấy ngũ gia từ cao vị đến cuối. Lấi chia đôi từ mất vị (vị
cuối) đến đầu. Hai vị nhân nhau thành phàm xích. 100 vị định xích. Lấy phép giảm ngũ
giảm kết quả hai lần. Lần đầu giảm hết, lần sau giảm đến xích thì dừng thành số ruộng
vậy.

38
Hội thảo khoa học, Ninh Bình 15-16/09/2018

Nếu muốn thử lấi, lấy phép tứ phá kiểm tra kết quả thành số thực điền vậy.
Giải thích. Đổi đơn vị:
Đông: 15 cao 8 xích 7 thốn = 15 × 15 + 8 xích 7 thốn= 233 xích 7 thốn =2337 ×
0, 04 =93,48 m;
Tây: 14 cao 5 xích 3 thốn=14 × 15 + 5 xích 3 thốn=215 xích 3 thốn =2153 × 0, 04 = 86,12
m;
Nam: 14 cao 3 xích 1 thốn=14 × 15 + 3 xích 1 thốn=213 xích 1 thốn =2131 × 0, 04 =
85,24 m;
Bắc: 14 cao 4 xích 2 thốn=14 × 15 + 4 xích 2 thốn=214 xích 2 thốn =2142 × 0, 04 = 85,68
m.
a+c b+d 93, 48 + 86, 12 85, 32 + 85, 68
S= × = × = 7674,308 (m2 )
2 2 2 2
= 2 × 3600 + 1 × 360 + 4 × 24 + 7 × 2, 4 + 6 × 0, 24 + 0, 068 (m2 ) = 2 mều 1 cao 4 xích
7 thốn 6 phân.

Bài toán 7 (Toán điền trừ cửu pháp, trang 5b). Đông 35 trượng 2 xích, nam 12 trượng 4
xích, tây 36 trượng 3 xích, Bốc 12 trượng 4 xích. Dùng phép Tứ phá được: diện tích được
117320 xích. Dùng phép Đấi thành: diện tích được 44330 xích. Thị điền 1 mều 9 cao 10
xích 5 thốn 3 phân 3 li 3 hào.
Giải thích. Tính theo m2 :
Đông+Tây= 35 trượng 2 xích+ Tây 36 trượng 3 xích=715 xích= 286m;
Nam+Bắc=12 trượng 4 xích+12 trượng 4 xích=248 xích=99,2m.
Do đó:
a+c b+d
S= × = (286 + 99, 2) : 4 = 7092,8 (m2 ) =1 × 3600 + 9 × 360 + 10 × 24 +
2 2
5 × 2, 4 + 3 × 0, 24 + 3 × 0, 024 + 3 × 0, 0024 + 0, 0008.
Vậy diện tích hình tứ giác là 1 mều 9 cao 10 xích 5 thốn 3 phân 3 li 3 hào. Sử dụng
công thức (1) tính diện tích sai, dẫn đến tính thể tích sai. Bài toán dưới đây trong Đại
thành toán học chí minh của Phạm Gia Kỉ viết năm 1820 là ví dụ.

Bài toán 8 (Đại thành toán học chí minh, trang 7b). Nay có đất ruộng, Đông dài 5 trượng
3 thước, Tây dài 6 trượng 3 thước, Nam dài 7 trượng, Bắc dài 8 trượng, khơi sâu 2 thước.
Tính đất bao nhiêu phương?
Đáp số: 87 khối.

Giải. Cộng Đông Tây làm một, chia đôi, còn 5 trượng 8 thước. Cộng Nam Bắc làm một,
chia đôi, còn 7 trượng 5 thước. Nhân hai cái với nhau, được 43 trượng 5 thước. Lại nhân
với chiều sâu 2 thước, được 870 thước= 87 khổi.

Giải thích. Sử dụng công thức (1) ta được


53 + 63 70 + 80
V = Sd × h = × × 2 = 58 × 75 × 2 = 4350 × 2 = 8700 thước khổi=87
2 2
khối.
Lời bình: Toán học thời cổ (từ Babilon, Ai Cập đến Trung Hoa và Việt Nam) thường
được gọi là toán học kinh nghiệm (empirical mathematics). Nhiều công thức (như công
thức (1)) hoặc phương pháp nhận được từ kinh nghiệm đo đạc thực tế mà không hề có
chứng minh. Mặc dù các công thức này chỉ là gần đúng, thậm chí có sai số lớn, nhưng
các kiến thức toán học này cũng đã đáp ứng được các yêu cầu của thực tế cuộc sống. Tuy
nhiên, với sự phát triển của toán học, không thể áp dụng một công thức mà không có
chứng minh chặt chẽ. Dạy toán theo con đường lôgich chặt chẽ (xây dựng hệ tiên đề, các

39
Hội thảo khoa học, Ninh Bình 15-16/09/2018

định nghĩa chính xác, các định lí phải được chứng minh chặt chẽ...) thường đòi hỏi nhiều
thời gian và công sức, vì vậy hiện nay có xu hướng dạy toán theo con đường trực quan
và kinh nghiệm. Sử dụng lâu dài và phổ biến một công thức sai (công thức (1)) cho thấy
việc giải quyết mối quan hệ và tỉ lệ thời lượng giữa toán học kinh nghiệm và toán học
lôgich trong giảng dạy toán học có lẽ cũng cần được quan tâm và giải quyết thỏa đáng
trong chương trình và nội dung giảng dạy mới đang được biên soạn và triển khai.

Tài liệu
[1] David M. Burton, The History of Mathematics: Introduction, Seventh Edition,
Mc Graw Hill, 2011.
[2] Alexei Volkov, On the origins of the "Toan phap dai thanh (Great Com-
pendium of Mathematical Methods)”, in Y. Dold-Samplonius, J. W. Dauben,
M. Folkerts, B. van Dallen (eds.), Proceedings of International Conference
From China to Paris: 2000 years transmission of mathematical ideas, Franz
Steiner Verlag, Stuttgart, 2002, pp. 369-410.

40
Hội thảo khoa học, Ninh Bình 15-16/09/2018

C ÁC ĐỊNH LÝ GIÁ TRỊ TRUNG BÌNH


TRONG TÍCH PHÂN

Vũ Tiến Việt
Học viện An ninh nhân dân

Tóm tắt nội dung

Các định lý giá trị trung bình đóng một vai trò quan trọng trong Giải tích Toán học.
Chúng tôi xin giới thiệu một số phát triển của các định lý đó trong khoảng thời gian 50
năm trở lại đây. Chúng tôi cũng nêu ra một số áp dụng các định lý giá trị trung bình cho
các bài toán tích phân, trong đó có những bài thi Olympic sinh viên Việt Nam.

1 Các định lý kinh điển


Trước hết ta nhắc lại các định lý giá trị trung bình cho hàm khả vi kinh điển là Fermat,
Rolle, Lagrange, Cauchy.
1. Định lý Fermat1 :
Giả sử hàm f : [ a, b] → R liên tục trên [ a, b], đạt cực trị (địa phương) tại điểm
x0 ∈ ( a, b) và khả vi tại x0 . Khi đó f 0 ( x0 ) = 0.
2. Định lý Rolle2 : Năm 1691 Rolle đưa ra định lý sau mang tên ông:
Giả sử hàm f : [ a, b] → R liên tục trên [ a, b], khả vi trong ( a, b)
và f ( a) = f (b). Khi đó tồn tại điểm c ∈ ( a, b) để f 0 (c) = 0.
3. Định lý Lagrange3 :
Giả sử hàm f : [ a, b] → R liên tục trên [ a, b], khả vi trong ( a, b). Khi đó tồn tại điểm
c ∈ ( a, b) sao cho

f (b) − f ( a)
f 0 (c) = .
b−a

4. Định lý Cauchy4 :
1 Pierrede Fermat (1601-1665), nhà toán học người Pháp.
2 Michel Rolle (1652-1719), nhà toán học người Pháp.
3 Joseph-Louis Lagrange (1736-1813), nhà toán học người Pháp.
4 Augustin-Louis Cauchy (1789-1857), nhà toán học người Pháp.

41
Hội thảo khoa học, Ninh Bình 15-16/09/2018

Giả sử các hàm f , g : [ a, b] → R liên tục trên [ a, b], khả vi trong ( a, b), đồng thời
g0 ( x ) 6= 0, ∀ x ∈ ( a, b). Khi đó tồn tại điểm c ∈ ( a, b) sao cho

f 0 (c) f (b) − f ( a)
0
= .
g (c) g(b) − g( a)

5. Định lý giá trị trung bình tích phân thứ nhất:


Xét các hàm f , g khả tích trên [ a, b] và gọi m = inf f ( x ), M = sup f ( x ). Nếu g là
x ∈[ a,b] x ∈[ a,b]
hàm không âm (hoặc không dương) trên [ a, b] thì
Z b Z b
f ( x ) g( x )dx = µ g( x )dx với µ ∈ [m, M ].
a a

Hơn nữa, nếu f ∈ C [ a, b] thì ∃ξ ∈ [ a, b] sao cho


Z b Z b
f ( x ) g( x )dx = f (ξ ) g( x )dx.
a a

6. Định lý giá trị trung bình tích phân thứ hai:


Xét các hàm f , g khả tích và g là hàm đơn điệu trên [ a, b].
Khi đó ∃ξ ∈ [ a, b] sao cho
Z b Z ξ Z b
f ( x ) g( x )dx = g( a) f ( x )dx + g(b) f ( x )dx.
a a ξ

Nhận xét 1. Các định lý trên đây có ý nghĩa hình học là "tồn tại một hình chữ nhật
có diện tích bằng một hình phẳng cho trước".

2 Một áp dụng
Z b
Với hàm f ( x ) liên tục trên [ a, b] thì tồn tại f ( x )dx.
a
Theo định lý giá trị trung bình tích phân thứ nhất ∃c ∈ [ a, b] sao cho
Z b
1
f (c) = f ( x )dx.
b−a a

Với hàm f ( x ) liên tục trên [ a, b], khả vi trong khoảng ( a, b) thì theo định lý Lagrange
tồn tại c ∈ ( a, b) sao cho

f (b) − f ( a)
f 0 (c) = hay f 0 (c)(b − a) = f (b) − f ( a).
b−a

Điều này hình như dẫn tới quan hệ


h 1 Z b i
f ( x )dx f 0 (c)(b − a) = f (c)[ f (b) − f ( a)].
b−a a

42
Hội thảo khoa học, Ninh Bình 15-16/09/2018

Do đó suy ra
Z b
0
f (c) f ( x )dx = f (c)[ f (b) − f ( a)]. (2.1)
a

Tuy nhiên, ta nghi ngờ tính đúng đắn của (2.1) vì điểm c của định lý giá trị trung
bình tích phân và điểm c của định lý Lagrange không chắc là trùng nhau.
Lấy ví dụ sau: Xét f ( x ) = x2 trên [0, 1].
Theo định lý giá trị trung bình tích phân thứ nhất
Z b Z 1
1 1 1 1
f ( x )dx = x2 dx = = f ( c ) = c2 ⇒ c= √ ·
b−a a 1−0 0 3 3
Mặt khác, theo định lý Lagrange

f (b) − f ( a) 12 − 02 1
= = 1 = f 0 (c) = 2c ⇒ c= ·
b−a 1−0 2

• Chúng ta sẽ chứng tỏ rằng hệ thức (2.1) là đúng đắn bằng mệnh đề sau đây:
Mệnh đề 1. Giả sử f ( x ) là hàm liên tục trên đoạn [ a, b], khả vi trong khoảng ( a, b).
Khi đó tồn tại điểm c ∈ ( a, b) thỏa mãn hệ thức (2.1).
Chứng minh. Xét hàm
Z x Z b
h( x ) = [ f ( x ) − f (b)] f (t)dt + [ f ( x ) − f ( a)] f (t)dt.
a x

Ta thấy h( x ) liên tục trên [ a, b], khả vi trong ( a, b) và h( a) = h(b) = 0. Theo định lý
Rolle tồn tại điểm c ∈ ( a, b) sao cho h0 (c) = 0. Thế mà
Z x
h0 ( x ) = f ( x )[ f ( x ) − f (b)] + f 0 ( x ) f (t)dt − f ( x )[ f ( x ) − f ( a)]
a
Z b
+ f 0 (x) f (t)dt
x
Z b
0
= f (x) f (t)dt − f ( x )[ f (b) − f ( a)].
a

Từ đây suy ra hệ thức (2.1).


Trở lại ví dụ ở trên ta có
Z 1
2 2
2c t2 dt = c2 (12 − 02 ) hay c = c2 ⇒ c= ·
0 3 3

• Ta mở rộng mệnh đề trên cho 2 hàm.


Mệnh đề 2. Giả sử f ( x ) và g( x ) là các hàm liên tục trên đoạn [ a, b], khả vi trong
khoảng ( a, b). Khi đó tồn tại điểm c ∈ ( a, b) sao cho
Z b Z b
f 0 (c) g(t)dt + g0 (c) f (t)dt = f (c)[ g(b) − g( a)] + g(c)[ f (b) − f ( a)]. (2.2)
a a

43
Hội thảo khoa học, Ninh Bình 15-16/09/2018

Chứng minh. Xét hàm


Z x Z b
H ( x ) = [ f ( x ) − f (b)] g(t)dt + [ f ( x ) − f ( a)] g(t)dt
a x
Z x Z b
+ [ g( x ) − g(b)] f (t)dt + [ g( x ) − g( a)] f (t)dt.
a x

Ta thấy H ( x ) liên tục trên [ a, b], khả vi trong ( a, b) và H ( a) = H (b) = 0.


Theo định lý Rolle tồn tại điểm c ∈ ( a, b) sao cho H 0 (c) = 0. Thế mà
Z x Z b
H 0 (x) = f 0 (x) g(t)dt + g( x )[ f ( x ) − f (b)] + f 0 ( x ) g(t)dt − g( x )[ f ( x ) − f ( a)]
a x
Z x
+ g0 ( x ) f (t)dt + f ( x )[ g( x ) − g(b)] + g0 ( x )[ g( x ) − g(b)] − f ( x )[ g( x ) − g( a)]
a
Z b Z b
= f 0 (x) g(t)dt + g0 ( x ) f (t)dt − f ( x )[ g(b) − g( a)] − g( x )[ f (b) − f ( a)].
a a

Từ đây suy ra hệ thức (2.2).

3 Các định lý mở rộng


• Định lý Flett5 . Năm 1958 Flett đưa ra định lý sau:
Cho hàm f ( x ) khả vi trên [ a, b] và thỏa mãn f 0 ( a) = f 0 (b) (hai đạo hàm này được
hiểu là đạo hàm một phía). Khi đó tồn tại điểm c ∈ ( a, b) sao cho

f (c) − f ( a)
f 0 (c) = .
c−a
Chứng minh. (Xem [2] hoặc [6]).
Ta có thể giả thiết f 0 ( a) = f 0 (b) = 0, vì có thể thay f ( x ) bởi h( x ) = f ( x ) − x f 0 ( a), lúc
đó h0 ( x ) = f 0 ( x ) − f 0 ( a), h0 ( a) = h0 (b) = 0 và nếu kết luận đúng với h( x ) thì

h(c) − h( a)
h0 (c) = ,
c−a
f (c) − c f 0 ( a) − f ( a) + a f 0 ( a)
hay f 0 (c) − f 0 ( a) = ,
c−a
f (c) − c f 0 ( a) − f ( a) + a f 0 ( a) f (c) − f ( a)
nên f 0 (c) = + f 0 ( a) = ,
c−a c−a

tức là kết luận đúng cho f ( x ).


Ta xét hàm

 f 0 ( a) = 0 khi x=a
g( x ) =
 f ( x ) − f ( a) khi a<x6b
x−a
5 T.M. Flett, giáo sư đại học Liverpool, UK.

44
Hội thảo khoa học, Ninh Bình 15-16/09/2018

f ( x ) − f ( a)
Khi đó lim = f 0 ( a) = 0 = g( a), nên g( x ) liên tục trên [ a, b]. Rõ ràng g( x )
x → a +0 x−a
khả vi trong ( a, b] và

f 0 (x) f ( x ) − f ( a)
g0 ( x ) = − .
x−a ( x − a )2

Ta thấy: +) Nếu g(b) = 0, thì theo định lý Rolle ∃c ∈ ( a, b) sao cho g0 (c) = 0, tức là

f 0 (c) f (c) − f ( a) f (c) − f ( a)


− =0 hay f 0 (c) = .
c−a ( c − a )2 c−a

f (b) − f ( a) g(b)
+) Nếu g(b) > 0, thì g0 (b) = − 2
=− < 0.
(b − a) b−a
Suy ra ∃ x1 ∈ ( a, b) để g( x1 ) > g(b) (bởi vì nếu g( x ) 6 g(b), ∀ x ∈ ( a, b) thì sẽ dẫn tới
g0 (b) > 0, mâu thuẫn với g0 (b) < 0).
Lúc này g( a) = 0 < g(b) < g( x1 ) và do g( x ) liên tục, nên phải tồn tại x2 ∈ ( a, b)
để g( x2 ) = g(b). Sử dụng định lý Rolle cho hàm g( x ) trên [ x2 , b] thì tồn tại điểm c ∈
( x2 , b) ⊂ ( a, b) sao cho g0 (c) = 0, dẫn tới kết luận như trên.
+) Nếu g(b) < 0 ta chứng minh tương tự.
• Định lý Meyer6 . Năm 1977 Meyer đưa ra định lý sau:
Giả sử f : [ a, b] → R là hàm khả vi trên [ a, b] và thỏa mãn f 0 ( a) = f 0 (b). Khi đó tồn
tại điểm c ∈ ( a, b) sao cho

f (b) − f (c)
f 0 (c) = .
b−c
Chứng minh. (Xem [2] hoặc [6]).
Ta xét hàm

 f (b) − f ( x )

khi a6x<b
m( x ) = b−x
f 0 (b) khi x=b

Dễ thấy m( x ) liên tục và khả vi. Nếu m( x ) đạt cực trị tại c ∈ ( a, b) thì theo định lý
Fermat m0 (c) = 0, dẫn đến kết luận của định lý.
Nếu m( x ) đạt cực trị tại a hoặc b thì không giảm tổng quát ta giả thiết m( a) 6 m( x ) 6
m(b), ∀ x ∈ [ a, b]. Khi đó f ( x ) 6 f (b) − (b − x )m( a), ∀ x ∈ [ a, b] và

f ( x ) − f ( a) f (b) − f ( a) − (b − x )m( a)
6
x−a x−a
f (b) − f ( a)
= = m ( a ).
b−a

Lấy giới hạn khi x → a + 0 ta được f 0 ( a) 6 m( a), nên m(b) = f 0 (b) = f 0 ( a) 6 m( a).
Dẫn tới m( x ) là hàm hằng, do đó m0 ( x ) ≡ 0 và ta có kết luận của định lý.
• Định lý Sahoo và Riedel7 . Năm 1998 Sahoo và Riedel đưa ra định lý:
6 R.E. Meyer (1919-2008), giáo sư đại học Wisconsin–Madison, USA.
7 P.K. Sahoo và T. Riedel, các giáo sư đại học Louisville, USA.

45
Hội thảo khoa học, Ninh Bình 15-16/09/2018

Giả sử f : [ a, b] → R là hàm khả vi trên [ a, b]. Khi đó tồn tại điểm c ∈ ( a, b) sao cho
1 f 0 (b) − f 0 ( a)
f (c) − f ( a) = f 0 (c)(c − a) − · ( c − a )2 .
2 b−a

• Định lý Pawlikowska8 . Năm 1999 Pawlikowska đưa ra định lý sau:


Cho f : [ a, b] → R là hàm khả vi cấp n trên [ a, b] và f (n) ( a) = f (n) (b). Khi đó tồn tại
ξ ∈ ( a, b) sao cho
n
f (ξ ) − f ( a) (−1)i+1
=∑ ( ξ − a ) i −1 f ( i ) ( ξ ).
ξ−a i =1
i!

• Định lý Riedel và Sablik9 . Năm 2004 Riedel và Sablik đưa ra định lý:
Giả sử f : [ a, b] → R là hàm khả vi trên [ a, b]. Khi đó tồn tại c ∈ ( a, b) sao cho
1 h 0 f (c) − f ( a) i 1 h 0 f (c) − f (b) i f 0 (b) − f 0 ( a)
f (c) − + f (c) − = .
c−a c−a c−b c−b b−a

• Định lý Çakmak và Tiryaki10 .


Năm 2011 Devrim Çakmak và Tiryaki đưa ra định lý:
Giả sử f : [ a, b] → R là hàm khả vi. Khi đó tồn tại c ∈ ( a, b) sao cho
1 f 0 (b) − f 0 ( a)
f (b) − f (c) = f 0 (c)(b − c) − · ( b − c )2 .
2 b−a
Ghi chú. Chứng minh của các định lý này có thể xem trong [2] hoặc [6].
Nhận xét 2. Định lý Meyer là một cách bổ sung cho đầy đủ của định lý Flett và chứng
minh của định lý Meyer dựa theo cách chứng minh thứ hai của định lý Flett.
Khi f 0 ( a) = f 0 (b) thì định lý Sahoo và Riedel trở thành định lý Flett, còn định lý
Çakmak và Tiryaki trở thành định lý Meyer.
Định lý Lagrange nói rằng tồn tại tiếp tuyến của đồ thị hàm f ( x ) song song với đường
thẳng AB.
Định lý Flett nói rằng tồn tại tiếp tuyến của đồ thị hàm f ( x ) đi qua điểm A, định lý
Meyer nói rằng tồn tại tiếp tuyến của đồ thị hàm f ( x ) đi qua điểm B.

4 Một số bài toán áp dụng


Z 1 Z 1
Bài toán 1. Cho hàm f : [0, 1] → R liên tục thỏa mãn f ( x )dx = x f ( x )dx.
Z α0 0
Chứng minh rằng tồn tại điểm α ∈ (0, 1) sao cho f ( x )dx = 0.
Z t Z t 0

Lời giải. Xét hàm h(t) = t f ( x )dx − x f ( x )dx, t ∈ [0, 1].


0 Z t 0

Ta có h(0) = h(1) = 0 và h0 (t) = f ( x )dx.


0
Theo định lý Rolle tồn tại α ∈ (0, 1) sao cho h0 (α) = 0.
Suy ra điều cần chứng minh.
8 Iwona Pawlikowska, PhD Mathematics, Memphis, TN, United States.
9 M. Sablik, giáo sư đại học Silesia, Katowice, Polska.
10 D. Çakmak và A. Tiryaki, các giáo sư đại học Gazi, Türkiye.

46
Hội thảo khoa học, Ninh Bình 15-16/09/2018
Z 1 Z 1
Bài toán 2. Cho hàm f : [0, 1] → R liên tục thỏa mãn f ( x )dx = x f ( x )dx.
0 0
Chứng minh rằng với mỗi số a ∈ R tồn tại điểm c ∈ (0, 1) sao cho
Z c
f (c) = a f ( x )dx.
0
Z t
Lời giải. Với t ∈ [0, 1] ta xét hàm g(t) = e−at f ( x )dx.
Z α 0
Theo bài 1 tồn tại α ∈ (0, 1) sao cho f ( x )dx = 0.
0
Như vậy g(0) = g(α) = 0 và ta có
h Z t i
0 − at
g (t) = e f (t) − a f ( x )dx .
0

Theo định lý Rolle tồn tại c ∈ (0, α) ⊂ (0, 1) sao cho g0 (c) = 0.
Suy ra điều cần chứng minh.

Nhận xét 3. Lấy a = 2018 ta được bài B.3 trong đề thi Giải tích OLP-2018.
Z 1 Z 1
Bài toán 3. Cho hàm f : [0, 1] → R liên tục thỏa mãn f ( x )dx = x f ( x )dx.
Z α0 0
Chứng minh rằng tồn tại điểm α ∈ (0, 1) sao cho x f ( x )dx = 0.
Z t Z t 0

Lời giải. Xét hàm h(t) = t f ( x )dx − x f ( x )dx, t ∈ [0, 1].


0 Z t0
Ta thấy h(0) = h(1) = 0 và h0 (t) = f ( x )dx.
0
Theo định lý Rolle tồn tại b ∈ (0, 1) sao cho h0 (b) = 0.
Do h0 (0) = h0 (b) = 0, nên theo định lý Flett tồn tại α ∈ (0, b) ⊂ (0, 1) sao cho

h ( α ) − h (0) h(α)
h0 (α) = = , hay αh0 (α) = h(α).
α−0 α
Tức là
Z α Z α Z α
α f ( x )dx = α f ( x )dx − x f ( x )dx,
0 0 0

suy ra điều phải chứng minh.


Z 1 Z 1
Bài toán 4. Cho hàm f : [0, 1] → R liên tục thỏa mãn f ( x )dx = x f ( x )dx.
0 0
Chứng minh rằng với mỗi số a ∈ R tồn tại điểm c ∈ (0, 1) sao cho
Z c
c f (c) = a x f ( x )dx.
0
Z t
− at
Lời giải. Với t ∈ [0, 1] ta xét hàm g(t) = e x f ( x )dx.
Z α 0
Theo bài 3 tồn tại α ∈ (0, 1) sao cho x f ( x )dx = 0.
0

47
Hội thảo khoa học, Ninh Bình 15-16/09/2018

Như thế ta có g(0) = g(α) = 0 và


h Z t i
g0 (t) = e−at t f (t) − a x f ( x )dx .
0

Theo định lý Rolle tồn tại c ∈ (0, α) ⊂ (0, 1) sao cho g0 (c) = 0.
Suy ra điều cần chứng minh.
Z 1 Z 1
Bài toán 5. Cho hàm f : [0, 1] → R khả vi thỏa mãn f ( x )dx = x f ( x )dx.
0 0
Chứng minh rằng với mỗi số a ∈ R tồn tại điểm c ∈ (0, 1) sao cho
Z c
f (c) = a f 0 (c) f ( x )dx.
0

Lời giải. Với t ∈ [0, 1] ta xét hàm


Z t
− a f (t)
h(t) = e f ( x )dx.
0
Z α
Theo bài 1 tồn tại α ∈ (0, 1) để f ( x )dx = 0. Ta có h(0) = h(α) = 0 và
0
h Z t i
h0 ( t ) = e− a f (t) f ( t ) − a f 0 ( t ) f ( x )dx .
0

Theo định lý Rolle tồn tại c ∈ (0, α) ⊂ (0, 1) sao cho h0 (c) = 0.
Suy ra điều cần chứng minh.
Nhận xét 4. Lấy a = 2018 ta được bài A.2 trong đề thi Giải tích OLP-2018.
Z 1 Z 1
Bài toán 6. Cho hàm f : [0, 1] → R khả vi thỏa mãn f ( x )dx = x f ( x )dx.
0 0
Chứng minh rằng với mỗi số a ∈ R tồn tại điểm c ∈ (0, 1) sao cho
Z c
c f (c) = a f 0 (c) x f ( x )dx.
0
Z t
Lời giải. Với t ∈ [0, 1] ta xét hàm h(t) = e−a f (t) x f ( x )dx.
Z α 0
Theo bài 3 tồn tại α ∈ (0, 1) để x f ( x )dx = 0. Ta có h(0) = h(α) = 0 và
0
h Z t i
h0 ( t ) = e− a f (t) t f ( t ) − a f 0 ( t ) x f ( x )dx .
0

Theo định lý Rolle tồn tại c ∈ (0, α) ⊂ (0, 1) sao cho h0 ( β) = 0.


Suy ra điều cần chứng minh.
Z 1 Z 1
Bài toán 7. Cho hàm f : [0, 1] → R liên tục sao cho f ( x )dx = x f ( x )dx.
0 0
Chứng minh rằng với mỗi số a ∈ R tồn tại điểm c ∈ (0, 1) sao cho
Z c
c f (c) + a f ( x )dx = 0.
0

48
Hội thảo khoa học, Ninh Bình 15-16/09/2018
Z x
Lời giải. Xét hàm F ( x ) = f (t)dt. Ta có F (0) = 0, F 0 ( x ) = f ( x ) và
0
Z 1 Z 1 Z 1
F (1) = f ( x )dx = x f (x) = xF 0 ( x )dx
0 0 0
1 Z 1 Z 1
= xF ( x ) − F ( x )dx = F (1) − F ( x )dx.

0 0 0
Z 1
Do đó F ( x )dx = 0 (∗).
0
Hàm F ( x ) liên tục trên [0, 1]. Ta nhận thấy rằng nếu F ( x ) > 0, ∀ x ∈ (0, 1) thì
Z 1 Z 1
F ( x )dx > 0, nếu F ( x ) < 0, ∀ x ∈ (0, 1) thì F ( x )dx < 0, đều trái với (∗).
0 0
Vậy phải tồn tại b ∈ (0, 1) để F (b) = 0.
Lại xét hàm g( x ) = x a F ( x ). Ta có g0 ( x ) = ax a−1 F ( x ) + x a f ( x ), g(0) = g(b) = 0.
Theo định lý Rolle tồn tại c ∈ (0, b) ⊂ (0, 1) sao g0 (c) = 0. Từ đó
Z c Z c
ac a−1 f ( x )dx + c a f (c) = 0 hay c f (c) + a f ( x )dx = 0.
0 0

Bài toán 8. Cho hàm f ( x ) khả vi trên [0, 1] và thỏa mãn


Z 1 Z 1
f ( x )dx = x f ( x )dx = 0.
0 0

Chứng minh rằng tồn tại c ∈ (0, 1) sao cho f 0 (c) = 0.


Z x
Lời giải. Xét hàm g( x ) = f (t)dt, t ∈ [0, 1] thì g0 ( x ) = f ( x ).
0 Z 1
Ta có g(0) = 0 và g(1) = f (t)dt = 0. Theo định lý Rolle tồn tại c1 ∈ (0, 1) sao cho
0
g0 (c1 ) = 0, tức là f (cZ1 ) = 0.
x
Xét hàm h( x ) = t f (t)dt, t ∈ [0, 1] thì h0 ( x ) = x f ( x ).
0 Z 1
Ta có h(0) = 0 và h(1) = t f (t)dt = 0. Theo định lý Rolle tồn tại c2 ∈ (0, 1) sao cho
0
h0 (c 2)= 0, tức là c2 f (c2 ) = 0, hay f (c2 ) = 0.
Lại sử dụng định lý Rolle cho hàm f ( x ) trên đoạn [c1 , c2 ] (hoặc đoạn [c2 , c1 ]) thì tồn
tại c ∈ (c1 , c2 ) ⊂ (0, 1) (hoặc c ∈ (c2 , c1 ) ⊂ (0, 1)) sao cho f 0 (c) = 0.
Lưu ý. Nếu xảy ra trường hợp c1 = c2 = α ∈ (0, 1) thì có 2 khả năng:
+) Khả năng 1: Tồn tại β ∈ (0, 1), β 6= α, f ( β) = 0 thì ta áp dụng định lý Rolle cho
f ( x ) trên [α, β] ⊂ (0, 1) (hoặc ] β, α] ⊂ (0, 1)). Kết luận của bài toán vẫn đúng.
+) Khả năng 2: f ( x ) 6= 0, ∀ x ∈ [0, 1], x 6= α thì do tính liên tục của f ( x ) ta có thể giả
thiết f ( x ) < 0, ∀ x ∈ [0, α) và f ( x ) > 0, ∀ x ∈ (α, 1]. Từ đó
Z α Z α
x < α, x f ( x ) > α f ( x ), ∀ x ∈ [0, α) ⇒ x f ( x )dx > α f ( x )dx,
0 0
Z 1 Z 1
x > α, x f ( x ) > α f ( x ), ∀ x ∈ (α, 1] ⇒ x f ( x )dx > α f ( x )dx,
α α
Z 1 Z 1
⇒ x f ( x )dx > α f ( x )dx = 0, trái giả thiết của đề bài!
0 0

Vậy không thể xáy ra khả năng này!

49
Hội thảo khoa học, Ninh Bình 15-16/09/2018

Bài toán 9 (OLP-2010, xem [1]). Cho hàm f ( x ) khả vi trên [0, 1] và thỏa mãn
Z 1 Z 1
f ( x )dx = x f ( x )dx = 1.
0 0

Chứng minh rằng tồn tại điểm c ∈ (0, 1) sao cho f 0 (c) = 6.
Lời giải. Xét hàm g( x ) = 6x − 2. Dễ dàng thấy rằng
Z 1 Z 1
g( x )dx = xg( x )dx = 1.
0 0
Z 1
Suy ra [ f ( x ) − g( x )]dx = 0. Hàm h( x ) = f ( x ) − g( x ) liên tục trên [0, 1] và có tích
Z 1 0
phân h( x )dx = 0, nên không thể xảy ra trường hợp h( x ) > 0, ∀ x ∈ (0, 1) hoặc trường
0
hợp h( x ) < 0, ∀ x ∈ (0, 1).
Như thế phương trình h( x ) = 0 phải có ít nhất một nghiệm trong (0, 1).
Giả sử rằng h( x ) = 0 chỉ có một nghiệm x = a ∈ (0, 1).
Xảy ra hai khả năng sau:
+) Nếu h( x ) < 0, ∀ x ∈ (0, a), thì h( x ) > 0, ∀ x ∈ ( a, 1). Khi đó
Z 1 Z 1 Z 1
x f ( x )dx − 1 = x f ( x )dx − xg( x )dx
0 0 0
Z 1 Z 1
= x [ f ( x ) − g( x )]dx = xh( x )dx
0 0
Z a Z 1 Z a Z 1
= xh( x )dx + xh( x )dx > ah( x )dx + ah( x )dx
0 a 0 a
hZ a Z 1 i Z 1
=a h( x )dx + h( x )dx = a h( x )dx = 0.
0 a 0
Z 1
Suy ra x f ( x )dx > 1, mâu thuẫn với giả thiết của đề bài!
0
+) Nếu h( x ) > 0, ∀ x ∈ (0, a), thì h( x ) < 0, ∀ x ∈ ( a, 1). Khi đó
Z 1 Z 1 Z 1
x f ( x )dx − 1 = x f ( x )dx − xg( x )dx
0 0 0
Z 1 Z 1
= x [ f ( x ) − g( x )]dx = xh( x )dx
0 0
Z a Z 1 Z a Z 1
= xh( x )dx + xh( x )dx < ah( x )dx + ah( x )dx
0 a 0 a
hZ a Z 1 i Z 1
=a h( x )dx + h( x )dx = a h( x )dx = 0.
0 a 0
Z 1
Suy ra x f ( x )dx < 1, mâu thuẫn với giả thiết của đề bài!
0
Vậy h( x ) = 0 phải có ít nhất hai nghiệm trong (0, 1).
Giả sử hai nghiệm đó là a, b ∈ (0, 1) và a < b. Ta có h( a) = h(b) = 0, nên f (b) −
f ( a) = g(b) − g( a). Theo định lý Lagrange tồn tại c ∈ ( a, b) ⊂ (0, 1) sao cho

f (b) − f ( a) g(b) − g( a)
f 0 (c) = = = 6.
b−a b−a

50
Hội thảo khoa học, Ninh Bình 15-16/09/2018

Nhận xét 5. Bài 8 được đưa về bài 9 nếu thay hàm f ( x ) bởi hàm f ( x ) + 6x − 2 và bài 9
được đưa về bài 8 nếu thay hàm f ( x ) bởi hàm f ( x ) − 6x + 2.
Z 1
Bài toán 10. Cho hàm f : [0, 1] → R liên tục và thỏa mãn f ( x )dx = 0.
0 Z c
Chứng minh với mỗi số a ∈ R tồn tại c ∈ (0, 1) sao cho f (c) = a f ( x )dx.
Z t 0

Lời giải. Đặt g(t) = e−at f ( x )dx, t ∈ [0, 1].


0
Ta có g(0) = g(1) = 0 và g(t) khả vi với
h Z t i
g0 (t) = e−at f (t) − a f ( x )dx .
0

Theo định lý Rolle tồn tại c ∈ (0, 1) để g0 (c) = 0.


Suy ra điều phải chứng minh.
Z 1
Bài toán 11. Cho hàm f : [0, 1] → R liên tục và thỏa mãn f ( x )dx = 0.
0 Z c
Chứng minh rằng tồn tại c ∈ (0, 1) sao cho (1 − c) f (c) = c f ( x )dx.
Z t 0

Lời giải. Đặt g(t) = et (1 − t) f ( x )dx, t ∈ [0, 1].


0
Ta có g(0) = g(1) = 0 và g(t) khả vi với
h Z t i
g 0 ( t ) = e t (1 − t ) f ( t ) − t f ( x )dx .
0

Theo định lý Rolle tồn tại c ∈ (0, 1) để g0 (c) = 0.


Suy ra điều phải chứng minh.
Z 1
Bài toán 12. Cho hàm f ( x ) khả vi trên [0, 1] và thỏa mãn f ( x )dx = 0.
0 Z c
Chứng minh với mỗi số a ∈ R tồn tại c ∈ (0, 1) sao cho f (c) = a f 0 (c) f ( x )dx.
0
Z t
− a f (t)
Lời giải. Đặt g(t) = e f ( x )dx, t ∈ [ a, b]. Ta có g(0) = g(1) = 0 và
0
h Z t i
0 − a f (t) 0
g (t) = e f (t) − a f (t) f ( x )dx .
0

Theo định lý Rolle tồn tại c ∈ (0, 1) sao cho g0 (c) = 0, tức là
h Z c i
g0 ( c ) = e− a f (c) f ( c ) − a f 0 ( c ) f ( x )dx = 0,
0

suy ra điều phải chứng minh.

Bài toán 13 (OLP-Romania-2006). Cho hàm f : [0, 1] → R liên tục và thỏa mãn
Z 1
f ( x )dx = 0.
0 Z ξ
Chứng minh rằng tồn tại ξ ∈ (0, 1) sao cho x f ( x )dx = 0.
0

51
Hội thảo khoa học, Ninh Bình 15-16/09/2018
Z t Z t
Lời giải. Xét hàm g(t) = t f ( x )dx − x f ( x )dx khả vi trên [0, 1].
0 Z t 0

Dễ dàng tính được g0 (t) = f ( x )dx và ta có g0 (0) = g0 (1) = 0.


0
Theo định lý Flett tồn tại ξ ∈ (0, 1) sao cho

g ( ξ ) − g (0) g(ξ )
g0 (ξ ) = = .
ξ −0 ξ

Như vậy ta được ξg0 (ξ ) = g(ξ ), hay là


Z ξ Z ξ Z ξ
ξ f ( x )dx = ξ f ( x )dx − x f ( x )dx,
0 0 0
Z ξ
do đó x f ( x )dx = 0.
0
Z 1
Bài toán 14. Cho f là hàm liên tục trên [0, 1] và thỏa mãn f ( x )dx = 0.
0
Chứng minh với mỗi số a ∈ R tồn tại c ∈ (0, 1) sao cho
Z c
c f (c) = a x f ( x )dx.
0
Z t
Lời giải. Với t ∈ [0, 1] ta xét hàm g(t) = e−at x f ( x )dx. Ta có
0
h Z t i
g0 (t) = e−at t f (t) − a x f ( x )dx .
0
Z ξ
Ta thấy g(0) = 0 và theo bài 13 thì tồn tại ξ ∈ (0, 1) sao cho x f ( x )dx = 0, nên
Z ξ 0

g ( ξ ) = e−ξ x f ( x )dx = 0.
0
Áp dụng định lý Rolle cho hàm g(t) thì tồn tại c ∈ (0, ξ ) ⊂ (0, 1) sao cho g0 (c) = 0.
Từ đây suy ra điều cần chứng minh.
Z 1
Bài toán 15. Cho f là hàm liên tục trên [0, 1] và thỏa mãn f ( x )dx = 0.
0
Chứng minh rằng tồn tại c ∈ (0, 1) sao cho
Z c
c2 f ( c ) = ( x + x2 ) f ( x )dx.
0
Z x Z x
Lời giải. Đặt F ( x ) = x f (t)dt − t f (t)dt, x ∈ [0, 1].
Z x 0 0 Z 1
0
Ta có F ( x ) = f (t)dt và F 0 (0) = 0. Do f (t)dt = 0, nên F 0 (1) = 0.
0 0
F ( a ) − F (0)
Theo định lý Flett tồn tại a ∈ (0, 1) sao cho = F 0 ( a ).
a−0
Do đó
Z a
t f (t)dt = 0, a ∈ (0, 1). (1)
0

52
Hội thảo khoa học, Ninh Bình 15-16/09/2018
Z x
−x
Đặt G ( x ) = e t f (t)dt, x ∈ [0, 1]. Từ (1) ta có G (0) = G ( a) = 0.
0
Theo định lý Rolle tồn tại b ∈ (0, a) sao cho
Z b
0 = G 0 ( b ) = − e−b t f (t)dt + e−b b f (b).
0

Do đó
Z b
t f (t)dt = b f (b), b ∈ (0, a). (2)
0
Z x Z x
Đặt H ( x ) = x t f (t)dt − (t2 + t) f (t)dt, x ∈ [0.1]. Ta có
0 0
Z x
H 0 (x) = t f (t)dt − x f ( x ).
0

Từ (2) ta có H 0 (0) = H 0 (b) = 0.


Theo định lý Flett tồn tại c ∈ (0, b) ⊂ (0, 1) sao cho

H ( c ) − H (0)
= H 0 ( c ).
c−0
Do đó
Z c Z c Z x
c x f ( x )dx − ( x2 + x ) f ( x )dx = c x f ( x )dx − c2 f (c).
0 0 0
Z c
2
Điều này kéo theo c f (c) = ( x2 + x ) f ( x )dx.
0
Z b
Bài toán 16. Cho hàm f ( x ) liên tục trên [ a, b] thỏa mãn f ( x )dx = 0.
Z ab
Chứng minh rằng tồn tại c ∈ ( a, b) sao cho c f (c) = f ( x )dx.
c
Z b
Lời giải. Đặt g( x ) = x f (t)dt, x ∈ [ a, b]. Ta thấy g( x ) khả vi trên [ a, b] và
x
Z b
g( a) = g(b) = 0, g0 ( x ) = f (t)dt − x f ( x ).
x

Từ đây theo định lý Rolle suy ra điều phải chứng minh.


Z b
Bài toán 17. Cho hàm f : [ a, b] → R liên tục và thỏa mãn f ( x )dx = 0.
a
b−a
Chứng minh rằng tồn tại α ∈ (0, ) sao cho
2
a+b a+b
f( − α) + f ( + α) = 0.
2 2
Lời giải. Xét hàm g : [0, 1] → R như sau
Z ta+(1−t)b
g(t) = f ( x )dx.
(1−t) a+tb

53
Hội thảo khoa học, Ninh Bình 15-16/09/2018

1
Ta thấy g(0) = g( ) = g(1) = 0 và g(t) là hàm khả vi. Theo định lý Rolle tồn tại
2
1 1
c1 ∈ (0, ) và c2 ∈ ( , 1) để g0 (c1 ) = g0 (c2 ) = 0. Mặt khác
2 2
g0 (t) = f (ta + (1 − t)b)( a − b) − f (1 − t) a + tb)(b − a)
= ( a − b)[ f (ta + (1 − t)b) + f ((1 − t) a + tb)]
Do đó f (ck a + (1 − ck )b) + f ((1 − ck ) a + ck b) = 0, (k = 1, 2).
1 a+b
Rõ ràng ck 6= nên ck a + (1 − ck )b 6= , (k = 1, 2).
2 2
1
Gọi c1 = c ∈ (0, ) ta có f (ca + (1 − c)b) + f ((1 − c) a + cb) = 0, đồng thời ta thấy
2
(1 − c) a + cb < ca + (1 − c)b ⇔ 0 < (b − a)(1 − 2c).
1 1 b−a
Đặt α = (b − a)(1 − 2c) = (b − a) − c(b − a) thì 0 < α < và ta được
2 2 2
a+b a+b
− α = (1 − c) a + cb, + α = ca + (1 − c)b, nên
2 2
a+b a+b
f( − α) + f ( + α) = 0.
2 2
b−a
Bài toán 18. Cho hàm f : [ a, b] → R liên tục. Chứng minh rằng tồn tại α ∈ (0, ) sao
2
cho
Z b
a+b a+b 2
f( − α) + f ( + α) = f ( x )dx.
2 2 b−a a

Lời giải. Xét hàm g : [ a, b] → R như sau


Z b
1
g( x ) = f ( x ) − f (t)dt.
b−a a
Z b Z b Z b
Ta thấy g( x )dx = f ( x )dx − f (t)dt = 0.
a a a
b−a
Áp dụng bài 18 thì tồn tại α ∈ (0, ) sao cho
2
a+b a+b
− α) + g(
g( + α) = 0,
2 2
Z b Z b
a+b 1 a+b 1
f( − α) − f (t)dt + f ( + α) − f (t)dt = 0.
2 b−a a 2 b−a a
Suy ra điều cần chứng minh.
Bài toán 19. Cho f : [0, 1] → R là hàm khả vi thỏa mãn f (1) = 0, f 0 (1) = 1. Chứng tỏ
rằng tồn tại c ∈ (0, 1) sao cho
Z c
0
f (c) = f (c) f ( x )dx.
0
Z x
− f (x)
Lời giải. Xét hàm g( x ) = xe f (t)dt, ta có
0
hZ x Z x i
0 − f (x) 0
g (x) = e f (t)dt − x f ( x ) f (t)dt + x f ( x ) .
0 0

54
Hội thảo khoa học, Ninh Bình 15-16/09/2018

Ta thấy g0 (0) = g0 (1) = 0, nên theo định lý Flett tồn tại c ∈ (0, 1) sao cho
Z c
0 g ( c ) − g (0) g(c) 0
g (c) = = , tức là f (c) = f (c) f ( x )dx.
c−0 c 0

Bài toán 20. Cho 0 < a < b và hàm f : [ a, b] → R liên tục.


Chứng minh rằng tồn tại c ∈ ( a, b) sao cho
√ √ √ √ Z
1 h a+ c b + ci c
2 f (c) = √ + f ( x )dx.
c a−c b−c a

Lời giải. Xét hàm


√ √ √ √ Z x
g( x ) = ( x − a)( x − b) f (t)dt, x ∈ [ a, b].
a

Ta thấy g( a) = g(b) = 0 và
1 h √ √ √ √ iZ x √ √ √ √
g0 ( x ) = √ ( x − a) + x − b) f (t)dy + ( x − a)( x − b) f ( x ).
2 x a

Theo định lý Rolle tồn tại c ∈ ( a, b) sao cho g0 (c) = 0, tức là


1 h √ √ √ √ iZ c √ √ √ √
√ ( c − a) + ( c − b) f ( x )dx + ( c − a)( c − b) f (c) = 0.
2 c a

Suy ra
√ √ √ √ Z
1 ( a − c) + ( b − c) c
2 f (c) = √ · √ √ √ √ f ( x )dx
c ( a − c)( b − c) a
√ √ √ √ Z
1 h a+ c b + ci c
=√ + f ( x )dx.
c a−c b−c a

Bài toán 21. Cho f ( x ), g( x ) là các hàm dương, liên tục trên [ a, b] và cho số thực α.
Chứng minh rằng tồn tại c ∈ ( a, b) sao cho
f (c) g(c)
Rc − Rb = α.
a
f ( x )dx g( x )dx
c
Z x Z b
Lời giải. Xét h( x ) = e−αx f (t)dt g(t)dt, x ∈ [ a, b]. Ta có h( a) = h(b) = 0 và
a x
Z x Z b Z b Z x
0 −αx −αx −αx
h ( x ) = −αe f (t)dt g(t)dt + e f (x) g(t)dt − e g( x ) f (t)dt
a x x a
h Z x Z b Z b Z x i
= −e−αx α f (t)dt g(t)dt − f ( x ) g(t)dt + g( x ) f (t)dt
a x x a

Sử dụng định lý Rolle thì tồn tại c ∈ ( a, b) để = 0. h0 (c)



Chú ý e , f ( x ), g( x ) là các hàm dương, ta suy ra
αx

Z c Z b Z c Z c
α f (t)dt g(t)dt − f ( x ) g(t)dt + g( x ) f (t)dt = 0,
a c x a
hay là
f (c) g(c)
Rc − Rb = α.
a
f ( x )dx g ( x ) dx
c

55
Hội thảo khoa học, Ninh Bình 15-16/09/2018

Bài toán 22 (OLP-2001, xem [1]). Chứng minh rằng tồn tại số thực x ∈ (0, 1) sao cho

t2000 x2001
Z 1
dt = .
x (1 + t)(1 + t2 )...(1 + t2001 ) (1 + x )(1 + x2 )...(1 + x2001 )

Lời giải. Xét các hàm số

t2000
f (t) = , t ∈ [0, 1],
(1 + t)(1 + t2 )...(1 + t2001 )
Z 1
F(x) = x f (t)dt, x ∈ [0, 1].
x

Ta có F (0) = F (1) = 0 và do f (t) liên tục trên [0, 1] nên F ( x ) khả vi trong (0, 1). Theo
định lý Rolle tồn tại x ∈ (0, 1) để F 0 ( x ) = 0. Thế mà
Z 1
F0 (x) = f (t)dt − x f ( x )
x
t2000 x2001
Z 1
= dt − .
x (1 + t)(1 + t2 )...(1 + t2001 ) (1 + x )(1 + x2 )...(1 + x2001 )

Vậy có điều phải chứng minh.


Bài toán 23. Cho hàm f ( x ) khả vi cấp 2 trên [0, 1]
và f 00 ( x ) > 0, ∀ x ∈ [0, 1]. Chứng minh rằng
Z 1 Z 1
2 (1 − x ) f ( x )dt 6 f ( x2 )dx.
0 0

Lời giải. Vì f 00 ( x ) > 0, ∀ x ∈ [0, 1], nên f 0 ( x ) đơn điệu tăng trên [0, 1].
Với mỗi x ∈ (0, 1) thì x2 ∈ (0, 1) và x > x2 .
Áp dụng định lý Lagrange cho hàm f trên đoạn [ x2 , x ] ta được

f ( x ) − f ( x2 )
= f 0 ( t ), t ∈ ( x 2 , x )
x − x2
Rõ ràng f 0 (t) 6 f 0 ( x ) và x − x2 > 0 nên suy ra f ( x ) − f ( x2 ) 6 ( x − x2 ) f 0 ( x ).
Từ đó dẫn đến
Z 1 Z 1
[ f ( x ) − f ( x2 )]dx 6 ( x − x2 ) f 0 ( x )dx,
0 0
Z 1 Z 1 1 Z 1
f ( x )dx − f ( x2 )dx 6 ( x − x2 ) f ( x ) − (1 − 2x ) f ( x )dx,

0 0 0 0
Z 1 Z 1 Z 1
f ( x )dx + (1 − 2x ) f ( x )dx 6 f ( x2 )dx,
0 0 0
Z 1 Z 1
2 (1 − x ) f ( x )dx 6 f ( x2 )dx (điều phải chứng minh!)
0 0

Bài toán 24 (OLP-2009, xem [1]). Cho hàm f ( x ) khả vi cấp 2 trên [0, 1] và f 00 ( x ) > 0, ∀ x ∈
[0, 1]. Chứng minh rằng
Z 1 Z 1
2 f ( x )dx > 3 f ( x2 )dx − f (0).
0 0

56
Hội thảo khoa học, Ninh Bình 15-16/09/2018

Lời giải. Vì f 00 ( x ) > 0, ∀ x ∈ [0, 1], nên f 0 ( x ) đơn điệu tăng trên [0, 1].
Với mỗi x ∈ (0, 1) thì x2 ∈ (0, 1) và x > x2 .
Áp dụng định lý Lagrange cho hàm f trên đoạn [ x2 , x ] ta được

f ( x ) − f ( x2 )
= f 0 ( t ), t ∈ ( x 2 , x ).
x − x2

Vì f 0 (t) > f 0 ( x2 ) và x − x2 > 0 nên f ( x ) − f ( x2 ) > ( x − x2 ) f 0 ( x2 ).


Từ đó dẫn đến
Z 1 Z 1
[ f ( x ) − f ( x )]dx > 2
( x − x2 ) f 0 ( x2 )dx,
0 0
Z 1 Z 1 Z 1
1
f ( x )dx − f ( x2 )dx > (1 − x )2x f 0 ( x2 )dx.
0 0 2 0

Dùng tích phân từng phần với u = 1 − x, du = −dx


và dv = 2x f 0 ( x2 )dx, v = f ( x2 ) thì tích phân ở vế phải trở thành
Z 1 1 Z 1 Z 1
0 2 2 2
(1 − x )2x f ( x )dx = (1 − x ) f ( x ) + f ( x )dx = − f (0) + f ( x2 )dx.

0 0 0 0

Vì thế ta suy ra
Z 1 Z 1 Z 1
2 1h i
f ( x )dx − f ( x )dx > − f (0) + f ( x2 )dx ,
0 0 2 0
Z 1 Z 1
2 f ( x )dx > 3 f ( x2 )dx − f (0) (điều phải chứng minh!)
0 0

Bài toán 25 (OLP-2012, xem [1]). Cho hàm f ( x ) liên tục trên [0, 2012] và thỏa mãn f ( x ) +
f (2012 − x ) = 0, ∀ x ∈ [0, 2012].
Z 2012
Chứng minh f ( x )dx = 0 và phương trình
0 Z 2012− x
( x − 2012) f ( x ) = 2012 f (t)dt có nghiệm trong (0, 2012).
0

Lời giải. Từ giả thiết ta có f ( x ) = − f (2012 − x ), ∀ x ∈ [0, 2012], nên


Z 2012 Z 2012
f ( x )dx = − f (2012 − x )dx
0 0
Z 0
=− f (t)d(2012 − t)
2012
Z 0 Z 2012
= f (t)dt = − f (t)dt,
2012 0
Z 2012
suy ra f ( x )dx = 0.
0 Z 2012− x
2012
Đặt g( x ) = (2012 − x ) f (t)dt, x ∈ [0, 2012].
0

57
Hội thảo khoa học, Ninh Bình 15-16/09/2018

Ta có g(0) = g(2012) = 0 và
Z 2012− x
0 2011
g ( x ) = −2012(2012 − x ) f (t)dt
0
− (2012 − x )2012 f (2012 − x )
h Z 2012− x
= −(2012 − x )2011 2012 f (t)dt
0
i
+ (2012 − x ) f (2012 − x )
h Z 2012− x i
2011
= −(2012 − x ) 2012 f (t)dt − (2012 − x ) f ( x ) .
0

Theo định lý Rolle tồn tại c ∈ (0, 2012) để g0 (c) = 0, suy ra


Z 2012−c
(2012 − c) f (c) = 2012 f (t)dt
0

và ta được điều cần chứng minh.


Bài toán 26. Cho hàm f : [ a, b] → R khả vi liên tục. Chứng minh rằng tồn tại c ∈ [ a, b]
sao cho
f 0 (c)
Z b
f ( x )dx = f (b)(b − a) − ( b − a )2 .
a 2
Lời giải. Xét x ∈ [ a, b], theo định lý Lagrange tồn tại ξ x ∈ ( x, b) ⊂ ( a, b) sao cho f (b) −
f ( x ) = f 0 (ξ x )(b − x ), dẫn tới
Z b Z b
f (b)(b − a) − f ( x )dx = f 0 (ξ x )(b − x )dx.
a a

Theo định lý giá trị trung bình tích phân tồn tại c ∈ [ a, b] sao cho
f 0 (c)
Z b Z b
0 0
f (ξ x )(b − x )dx = f (c) (b − x )dx = − ( b − a )2 ,
a a 2
từ đây ta suy ra điều cần chứng minh.

Tài liệu
[1] Hội Toán học Việt Nam. Tuyển tập các đề thi Olympic Toán học sinh viên từ năm
1993 đến năm 2018.
[2] Vũ Tiến Việt (chủ biên), Phạm Thị Hằng, Nguyễn Thị Lê, Giáo trình Toán Cao cấp - Học
phần A2. Nhà xuất bản Đại học Quốc gia Hà Nội, 2016.
[3] Peter R. Mercer, More Calculus of a Single Variable, Springer Science and Business Me-
dia, New York, 2014.
[4] Radulescu T. L., Radulescu V. D., Andreescu T., Problems in Real Analysis: Advanced
Calculus on the Real Axis, Springer Verlag, 2009.
[5] József Sándor, Selected Chapters of Geometry, Analysis and Number Theory, Lambert Pub-
lishing, 2005.
[6] Sahoo P. K., Riedel T., Mean Value Theorems And Functional Equations, World Scientific,
1998.

58
Hội thảo khoa học, Ninh Bình 15-16/09/2018

ĐƯỜNG TRÒN APOLLONIUS VÀ MỘT BÀI


TOÁN IMO

Nguyễn Bá Đang
Hội Toán học Hà Nội

Tóm tắt nội dung

Trước năm 2000 trong chương trình môn toán Trung học phổ thông có đưa bài toán
quỹ tích Đường tròn Apllonius vào sách giáo khoa lớp 10. Hiện nay, do giảm tải nên chỉ
còn các lớp Chuyên toán được học bài toán này.
Những năm gần đây, trong các kỳ thi Olympiad ở rất nhiều nước đã đưa bài toán này
với tính chất áp dụng. Kỳ thì IMO lần thứ 59 vừa qua có bài số 6 liên quan đến vấn đề
này. Bài này được đánh giá là bài khó, hầu hết các thí sinh đều không giải được, chỉ có
trên mười em làm trọn vẹn.
Từ khâu vẽ hình tới giải bài toán đều dùng tới Đường tròn Apllonius. Chính vì thế tôi
muốn nhắc lại kiến thức và các bài toán liên quan đến Đường tròn Apllonius trong Hội
thảo này.

1 Đường tròn Apollonius


Bài toán 1. Cho đoạn thẳng AB = a, k là số cho trước (0 < k < 1), M là điểm chuyển
MA
động trong mặt phẳng sao cho = k. Tìm quỹ tích của điểm M.
MB
Lời giải.
\ = 90◦ , Theo tính chất
Phần thuận: Dựng MD, ME là phân giác của ∆MAB nên DME
DA EA MA DA k
đường phân giác và giả thiết thì = = = k nên = suy ra
DB EB MB DA + DB k+1
ak ak
DA = , tương tự AE = . (1)
k+1 k−1
\ = 90◦ ,
suy ra D, E cố định, DME
suy ra M thuộc đường tròn đường kính DE.

59
Hội thảo khoa học, Ninh Bình 15-16/09/2018

Phần đảo: Lấy M’ trên đường tròn đường kính DE qua B kẻ đường vuông góc DM’
cắt M’D và MA tại H và K suy ra BK k M’E suy ra

BK AB k − 1 BH DB ak ak 2ak
0 = = , 0 = ; DE = AE − AD = − = 2 .Từ (1)
EM AE k EM DE k−1 k+1 k −1
a
suy ra DB = ,
k+1
BH DB a k2 − 1 k−1 BH BK
suy ra = = = suy ra 2 = ,
EM DE k + 1 2ak 2k EM EM
suy ra BK = 2BH suy ra HB = HK suy ra ∆MBK là tam giác cân suy ra M’D là phân
\ 0 \ 0
giác AM B suy ra M’E là phân giác ngoài AM B.
Đây là quỹ tích cơ bản được gọi là Đường tròn Apollonius, mang tên nhà toán học
Apolonius.

Bài toán 2. Cho tam giác ABC không cân. Điểm M thay đổi trong tam giác thỏa mãn điều

kiện ∠ AMC − ∠ ABC = ∠ AMB − ∠ ACB.

60
Hội thảo khoa học, Ninh Bình 15-16/09/2018

Chứng minh rằng M thuộc đường tròn cố định.

Lời giải. Dựng ra phía ngoài tam giác ABC tam giác ADC đồng dạng với ∆AMB nên
AD AC
= và ∠ BAC = ∠ MAD suy ra ∆AMD và ∆ABC đồng dạng (c.g.c), suy ra
AM AB
∠ AMD = ∠ ABC suy ra ∠ AMC − ∠ ABC = ∠ AMB − ∠ ACB = ∠CMD
Mặt khác ∠ ADC = ∠ AMB và ∠ ADM = ∠ ACB, suy ra ∠ AMB − ∠ ACB = ∠ ADC −
MB CD CM
∠ ADM = ∠ MDC suy ra ∠CMD = ∠CDM suy ra CD = CM suy ra = =
AB AC AC
BM AB
suy ra = không đổi suy ra M thuộc đường tròn Apollonius dựng trên cạnh BC
CM AC
AB
tỷ số .
AC
Bài toán 3 (Iran 1997). Cho tam giác ABC nội tiếp đường tròn (O). Điểm M thay đổi trên
cung BC (không chứa A) của đường tròn (O). Gọi I, J là tâm đường tròn nội tiếp tam giác
ABM, CAN. Chứng minh rằng đường tròn ngoại tiếp tam giác MIJ luôn đi qua điểm cố
định.

Lời giải. Gọi N là giao điểm của đường tròn ngoại tiếp ∆MI J, MI cắt đường tròn (O) tại
E, MJ cắt đường tròn (O) tại D suy ra EA = EB, DA = DC, mặt khác EA = EB = EI

DA = DC = DJ.
Xét ∆N IE và ∆N JD, ta có: ∠ NEI = ∠ NDJ (chắn cung MN) ∠EI N = ∠ DJN (cùng
NE EI AE
bù với ∠ N I M = ∠ N J M) suy ra hai tam giác đồng dạng suy ra = = suy ra
ND DJ AD
AE
N thuộc đường tròn Apollonius dựng trên đoạn ED với tỷ số suy ra N là giao điểm
AD
của đường tròn (O) và đường tròn Apollonius.

Bài toán 4. Cho bốn điểm A, B, C, D thẳng hàng theo thứ tự đó, AB 6= CD. Điểm M thay
đổi sao cho ∠ AMB = ∠CMD (M không thuộc AB). Chứng minh rằng M thuộc đường
tròn cố định.

Lời giải. Gọi P và Q là giao điểm của MA, MD với đường tròn ngoại tiếp ∆MBC, theo
giả thiết ∠ AMB = ∠CMD, suy ra PB = QC suy ra PQk BC, theo định lý Thales

61
Hội thảo khoa học, Ninh Bình 15-16/09/2018

suy ra
AP DQ
= (1)
AM DM
Theo hệ thức đường tròn, AP.AM = AB.AC suy ra
AP
AM2 = AB.AC. (2)
AM
Từ DQ.DM = DC.DB, suy ra
DQ
DM2 = DC.DB. (3)
DM
Kết hợp (1), (2), (3) suy ra
AM2 AB.AC AB.AC AM
2
= , A, B, C, D cố định suy ra không đổi suy ra =
r DM DC.DB DC.DBr DM
AB.AC AB.AC
suy ra M thuộc đường tròn Apollonius có tỷ số trên đoạn AD.
DC.DB DC.DB
Bài toán 5 (IMO 2018). Cho tứ giác ABCD thỏa mãn AB.CD = BC.DA, X là điểm trong tứ
giác sao cho ∠XAB = ∠XCD, ∠XBC = ∠XDA. Chứng minh ∠ BXA + ∠ DXC = 180◦ .

AB CB
Lời giải. Theo giả thiết AB.CD = BC.DA suy ra = theo tính chất đường phân
AD CD
giác ta suy ra đường phân giác góc ∠ BAD và góc ∠ BCD cắt nhau tại một điểm trên BD.

62
Hội thảo khoa học, Ninh Bình 15-16/09/2018

Gọi E là giao điểm của AC và BD, đường thẳng đối xứng với AE qua phân giác góc
∠ BAD cắt BD tại điểm P (AE và AP là hai đường đẳng giác của góc ∠ BAD ) từ đó CE và
CP cũng là hai đường đẳng giác của góc ∠ BCD ;
Theo tính chất của đường đẳng giác ta có đẳng thức:
PB EB AB2 CB2
= 2
= và ∠ BCA = ∠ DCP.
PD ED AD CD2
Tiếp tuyến tại của đường tròn (DAB) cắt BD tại Q suy ra theo tính chất của đường đối
QB AB2 CB2
trung ngoài (hoặc tam giác ADQ và tam giác BAQ đồng dạng) ta có = =
QD AD2 CD2
suy ra
∠QCB = ∠CDQ suy ra QC là tiếp tuyến của đường tròn (BCD) suy ra QA2 =
QB.QD = QC2 suy ra QA = QC ;
∠ BAC + ∠ BDA = ∠ BAC + ∠ BAQ = ∠QAC = ∠QCA = ∠QCB + ∠ BCA =
∠CDQ + ∠ BCA
∠ APB = ∠ PAD + ∠ PDA = ∠ BAC + ∠ BDA = ∠CDQ + ∠ BCA = ∠ PCD +
∠ PDC = ∠ BPC từ đó suy ra BP là phân giác góc ∠ APC.
Gọi M là giao của đường tròn (APB) và (DPC) suy ra

∠ MCD = ∠ MPB = ∠ MAP, (1)

∠ MBC = ∠ ABC − ∠ ABM = 180◦ − ∠ BAC − ∠ BCA − (180◦ − ∠ APM)

= ∠ APC − ∠ MPC − ∠ PAD − ∠ PCD = ∠ ADC − ∠ MCD = ∠ MDA. (2)

Từ (1) và (2), kết hợp giả thiết ∠XAB = ∠XCD, ∠XBC = ∠XDA suy ra M ≡ X suy
ra ∠ BMC = ∠ BPC = 180◦ − ∠ BPC = 180◦ − ∠ BPA = 180◦ − ∠ BMA suy ra ∠ BMC +
∠ BMA = 180◦ suy ra ∠ BXA + ∠ DXC = 180◦ .
BA DA
Cách dựng điểm M: Tứ giác ABCD thỏa mãn AB.CD = BC.DA nên = suy
BC DC
ra B và C năm trên đường tròn Apollonius bằng cách:
Dựng đường phân giác trong và ngoài của góc B Gọi hai I, J chân đường đường
phân giác trong và ngoài góc ∠ ABC. Đường tròn đường kính IJ có tên là đường tròn

Apollonius.
E là giao điểm AB và CD, F là giao điểm BC và AD. Giao điểm của hai đường tròn
(ACE) và đường tròn (BDF) là M tứ giác AMCE và BMDF là các tứ giác nội tiếp ta có:
∠ BAM = ∠ MCE = ∠ MCD và ∠ MBC = ∠ MDF = ∠ MDA suy ra M là điểm cần dựng
đóng vai trò điểm X.

63
Hội thảo khoa học, Ninh Bình 15-16/09/2018

Cách 2. Giả sử X là điểm trong tứ giác ABCD thỏa mãn ∠XAB = ∠XCD, ∠XBC =

∠XDA.
Gọi M là giao điểm của đường tròn (XAB) và (XCD) từ đó suy ra tứ ABXM và DMXC
là các tứ giác nội tiếp suy ra ∠XMB = ∠XAB = ∠XCD = 180◦ − ∠XMD suy ra
∠XMB + ∠XMD = 180◦ suy ra D, M, B thẳng hàng.
AB CB
Theo giả thiết AB.CD = BC.DA nên = suy ra A và C nằm trên đường
AD CD
tròn Apollonius có tâm O nằm trên BD và OA là tiếp tuyến của đường tròn (ABD),
OC là tiếp tuyến của đường tròn (BCD). Áp dụng góc ngoài tam giác ta có ∠ AOC =
∠ ABC − ∠ BAO − ∠ BCO = ∠ ABC − ∠ ADO − ∠CDO = ∠ ABC − ∠ ADC = = ∠ ABX +
∠XBC − ∠ ADC = 180◦ − ∠ AMX + ∠XDA − ∠ ADC = 180◦ − ∠ AMX − ∠XDC =
= 180◦ − ∠ AMX − ∠XMC = 180◦ − ∠ AMC từ đó ∠ AOC + ∠ AMC = 180◦ suy ra tứ
giác AOCM nội tiếp, do OA = OC suy ra ∠ AMB = ∠ BMC suy ra AXB = ∠ AMB =
180◦ − ∠ AMD = 180◦ − ∠CMD = 180◦ − ∠CXD suy ra AXB + ∠CXD = 180◦ .

Tài liệu
[1] Tạp chí Toán học và Tuổi trẻ, Tuyển tập.
[2] Các bài thi Olympic toán Trung học phổ thông Việt Nam, Nhà xuất bản Giáo dục,
Hà Nội, 2006.

64
Hội thảo khoa học, Ninh Bình 15-16/09/2018

PHƯƠNG PHÁP TỌA ĐỘ GIẢI BÀI TOÁN


CỰC TRỊ CỦA MODUL SỐ PHỨC

Nguyễn Thị Thu Hằng


Trường THPT Lê Quý Đôn-Đống Đa, Hà Nội

Tóm tắt nội dung

Trong số các bài toán về số phức trong kì thi THPT Quốc gia gần đây bài toán: Tìm
giá trị lớn nhất và giá trị nhỏ nhất của modul số phức là một dạng toán khó và xuất hiện
thường xuyên. Chẳng hạn, trong đề thi minh họa√năm 2018 của Bộ giáo dục Đào tạo:
”Cho số phức z = a + bi thỏa mãn |z − 4 − 3i | = 5. Tính P = a + b khi |z + 1 − 3i | +
|z − 1 + i | đạt giá trị lớn nhất.” đã làm học sinh và giáo viên khá đau đầu.
Có rất nhiều phương pháp để giải quyết dạng bài tập này, trong số các phương pháp
ấy, phương pháp "Sử dụng kiến thức hình học tọa độ để giải bài toán tìm cực trị modul
số phức " được tôi lựa chọn để trình bày trong sáng kiến kinh nghiệm của mình.

1 Biểu diễn hình học của số phức


Mỗi số phức z = x + iy( x, y ∈ R) được biểu diễn trong mặt phẳng Oxy là một điểm
M ( x; y). Khi đó
−−→
a) modul của số phức z bằng |OM|
b) Hai điểm biểu diễn số phức z và số phức liên hợp z của số phức z đối xứng nhau
qua trục thực.
c) Hai điểm biểu diễn số phức z và số phức đối −z của số phức z đối xứng nhau qua
gốc O.

65
Hội thảo khoa học, Ninh Bình 15-16/09/2018

Nhận xét 1 (Ý nghĩa hình học của phép cộng, phép trừ hai số phức). q Cho z1 = x1 + y1 i
−−→
là số phức có điểm biểu diễn hình học là M1 ( x1 ; y1 ) với |OM1 | = x12 + y21 .
−−→
q Cho z2 = x2 + y2 i là số phức có điểm biểu diễn hình học là M2 ( x2 ; y2 ) với |OM2 | =
x22 + y22 .
Khi đó
−−→ −−→ −→
Tổng hai số phức z1 + z2 =OM1 + OM2 = OQ thì điểm Q là điểm biểu diễn số phức
z1 + z2 và |z1 + z2 | = |OQ|.
−−→ −−→ −−−→ −−−→
Hiệu hai số phức z1 + z2 =OM1 − OM2 = M2 M1 thì M2 M1 biểu diễn số phức z1 − z2
và |z1 − z2 | = | M1 M2 |.
−−→ −−→
Nếu hai vecto OM1 và OM2 không cùng phương thì đỉnh Q là đỉnh của hình bình
hành OM1 QM2 và |z1 − z2 | và |z1 + z2 | lần lượt là độ dài hai đường chéo M1 M2 và OQ
của hình bình hành đó.

Nhận xét 2 (Một số kiến thức bổ sung).

a. Phương trình đường thẳng ax + by + c = 0.

b.Phương trình đường tròn ( x − a)2 + (y − b)2 = R2 .


x2 y2
c.Phương trình Elip 2 + 2 = 1.
a b
d. Phương trình Parabol y = ax2 .

2 Một số áp dụng
Để giải quyết các bài tập về tìm cực trị của modul số phức, cần thành thục kỹ năng
về tìm tập hơp các điểm biểu diễn số phức cho trước.

2.1 Tìm tập hơp các điểm biểu diễn số phức z


Giả sử M, A, B lần lượt là các điểm biểu diễn của các số phức z, a, b.
1) |z − a| = |z − b| ⇔ MA = MB. Khi đó tập hợp M là đường thẳng trung trực của
đoạn AB.
2) |z − a| = R ⇔ | MA| = R. Khi đó tập hợp M thuộc đường tròn tâm A, bán kính R.
3) |z − a| + |z − b| = k ⇔ MA + MB = k (k > 0, k ∈ R, | a − b| < k). Khi đó tập hợp
M là đường Elip nhận A, B là hai tiêu điểm và có độ dài trục lớn bằng k.

66
Hội thảo khoa học, Ninh Bình 15-16/09/2018

Bài toán 1 (Đề thi minh họa THPT Quốc gia 2017). Cho số phức z thỏa mãn |z| = 4. Biết
tập hợp các điểm biểu diễn cho số phức z = (3 + 4i )z + i là đường tròn tâm I bán kính
R. Khi đó √
A. I (0; 1); R = 2 5 B. I (1; 0); R = 10
C. I (0; 1); R = 20 D. I (1; −2); R = 22
Lời giải.


Từ giả thiết z = (3 + 4i )z + i ⇔ |z − i | = |(3 + 4i )z| ⇔ |z − i | = 32 + 42 |z|
⇔ |z − i | = 5 × 4 ⇔ |z − i | = 20.
Giả sử z = x + yi thì ta có x2 + (y − 1)2 = 20. √
Tập hợp điểm M trong mặt phẳng là đường tròn tâm I (0; 1) bán kính R = 20. Đáp
án đúng là (A)
Bài toán 2 (Tạp chí Toán học tuổi trẻ số 490 (năm 2018)). Cho số phức z thỏa mãn |z| = 2.
Tập hợp điểm biểu diễn số phức w = (1 − i )z + 2i là
A. Một Parabol B. Một đường tròn C.Một Elip D. Một đường thẳng.
Lời giải.
w − 2i |w − 2i |
Từ giả thiết ta có z = ⇔ |z| =
1−i |1 − i |
|w − 2i | √
⇔ |z| = √ ⇔ |w − 2i | = 2 2.
2

√ biểu diễn số phức w trong mặt phẳng phức là đường tròn tâm I (0; −2)
Tập hợp điểm
bán kính R = 2 2.
Bài toán 3 (Sở Giáo Dục và Đào tạo Hưng Yên - Đề thi thử THPT Quốc gia 2018). Cho số
phức z thỏa mãn |z − i | = 5. Biết tập hợp các điểm biểu diễn cho số phức w = iz + 1 − i
là đường tròn . Tính bán kính đường tròn đó.
A. R = 20 B. R = 5 C. R = 22 D. R = 4.

67
Hội thảo khoa học, Ninh Bình 15-16/09/2018

Lời giải.
Từ giả thiết z = iz − i2 − i ⇔ w = i (z − i ) − i ⇔ |w + i | = |i (z − i )| ⇔ |w + i | = 5.
Giả sử z = x + yi thì ta có x2 + (y + 1)2 = 25. Bán kính đường tròn cần tìm R = 5.

Bài toán 4 (Đề thi thử THPT Quốc


gia 2018 lần 1 của trường THPT Trần Phú - Quảng
2z − z + 3i
Ninh). Cho số phức z thỏa mãn = 3. Tập hợp các điểm biểu diễn trong mặt
z+i
phẳng phức là
A. Một Parabol B. Một đường tròn C.Một Elip D. Một đường thẳng.

Lời giải.
Giả sử z = x + yi ( x, y ∈ R) suy ra z = x − yi.
x + (3 − 3y)i
Từ giả thiết ta có | | = 3 ⇔ x2 + (3 − 3y)2 = 9( x2 + (y + 1)2 )
x + ( y + 1) i
2
⇔ −8x2 = 36y ⇔ y = − x2 .
9

Tập hợp các điểm biểu diễn trong mặt phẳng phức là một Parabol. Đáp án đúng (A).

Bài toán 5 (Toán học tuổi trẻ, 478 (2017)). Cho tập hợp các điểm biểu diễn số phức z thỏa
mãn |z + 2| + |z − 2| = 5 trên mặt phẳng tọa độ là
A. Một Parabol B.Một đường tròn C.Một Elip D. Một đường thẳng.

Lời giải.
Gọi số phức z = x + yi ( x, y ∈ R),
từ giả thiết ta có |( x + 2) + yi | + |( x − 2) + yi | = 5 ⇔ | MA| + | MB| = 5
vớiF1 (−2; 0); F2 (2; 0).

Tập hợp các điểm biểu diễn trong mặt phẳng phức là một Elip. Đáp án đúng là (C).

68
Hội thảo khoa học, Ninh Bình 15-16/09/2018

Bài toán 6. . Xác định tập hợp các điểm trong mặt phẳng phức biểu diễn các số phức z
thỏa mãn |z| = |z − 3 + 4i | là đường thẳng có phương trình
25
A. 3x + 4y − =0 B. 3x + 4y − 25 = 0
2
25
C. 3x − 4y − =0 D. 3x − 4y − 25 = 0.
2
Lời giải.

Vì |z| = |z| nên |z − 3 + 4i | = |z − 3 + 4i | = |z − 3 − 4i | suy ra |z| = |z − 3 − 4i |


Tập hợp điểm M là đường trung trực của đoạn thẳng OA, với O(0; 0) và A(3; 4).
25
Đường trung trực đó có phương trình là: 3x + 4y − = 0. Chọn đáp án đúng là (A).
2
1
Bài toán 7. . Điểm M biểu diễn số phức z(z 6= 0) và điểm M0 biểu diễn số phức z−1 = .
√ z
Nếu điểm M di động trên đường tròn tâm A(−1; 1) bán kính R = 2 thì M0 di động
trên đường nào?
A. x2 + y2 + 2x − 2y = 0 B. 2x + 2y + 1 = 0
C. 2x − 2y + 1 = 0 D. 2x + 2y − 1 = 0.

Lời giải.
1 z
Ta có z−1 = = 2 .
z |z|

0 x
 x = 2

x + y2
Do đó 0 y
 y = 2

x + y2


Vì M di động trên đường tròn tâm A(−1; 1) bán kính R = 2 nên tập hợp M thuộc
( x + 1)2 + (y − 1)2 = 2 ⇔ x2 + y2 + 2x − 2y = 0

69
Hội thảo khoa học, Ninh Bình 15-16/09/2018

2x 2y 0
⇔ 1+ − 2 ⇔ 2x − 2y0 + 1 = 0.
x2+y 2 x +y 2
0
Do đó điểm M chạy trên đường thẳng 2x − 2y + 1 = 0. Đáp án được chọn là C.

Bài toán 8. . Biết số phức z thỏa mãn điều kiện 3 ≤ |z − 3i − 1| ≤ 5. Tập hợp các điểm
biểu diễn của z tạo thành một hình phẳng. Diện tích của hình phẳng đó bằng
A. 16π B. 4π C.9π D.25π
Lời giải. q
Đặt z = x + yi ta có |z − 3i − 1| = ( x − 1)2 + ( y − 3)2
Do đó 3 ≤ |z − 3i − 1| ≤ 5 ⇔ 9 ≤ ( x − 1)2 + (y − 3)2 ≤ 25

Tập hợp các điểm biểu diễn của z là hình phẳng nằm trong đường tròn tâm I (1; 3) với
bán kính bằng r = 5 đồng thời nằm ngoài đường tròn tâm I (1; 3) với bán kính r = 3.
Diện tích của hình phẳng đó là S = 52 π − 32 π = 16π. Đáp án đúng là (A).

Bài tập tương tự


Bài 1. Cho các số phức z thỏa mãn |z| = 2. Biết rằng tập hợp các điểm biểu diễn các số
phức w = 3 − 2i + (2 − i )z là một
√ đường tròn. Tính bán kính r của đường tròn
√ đó.
A. 20 B. 20 C.7 D 7

√ z thỏa mãn |z − 1| = 2 . Biết rằng tập hợp các điểm biểu diễn các
Bài 2. Cho các số phức
số phức w = (1 + i 3)z + 2 là một đường tròn. Tính bán kính r của đường tròn đó.
A. r = 4 B. r = 5 C. r = 20 D. r = 22

Bài 3. Cho số phức z = 2 + i. Điểm nào dưới đây biểu diễn cho số phức nghịch đảo của
z
2 1 2 1
A.M ( ; − ) B. N (2; −1) C.P(− ; ) D Q(2; i )
5 5 5 5
Bài 4. Tìm tập hợp điểm biểu diễn các số phức z thỏa mãn: |z − (3 − 4i )| = 2 .
A. Đường tròn tâm I (−3; 4) và bán kính 4.
B. Đường tròn tâm I (−3; 4) bán kính 2.
C. Đường tròn tâm I (3; −4) bán kính 2.
D. Đường tròn tâm I (3; ˘4) bán kính 4.

Bài 5. Số phức z = 3 − 2i có modul


√ bằng √
A. 1 B. 5 C. 13 D. 13

70
Hội thảo khoa học, Ninh Bình 15-16/09/2018

|3 + 4i |
Bài 6. Điểm biểu diễn số phức z = có tọa độ là
i2019
A. (0; 5) B. (4; −3) C. (−4; 3) D. (5; 0)

Bài 7. Tập hợp các điểm biểu diễn số phức z trên mặt phẳng toạ độ thoả mãn điều kiện
|z − i | = 1 là
A. Đường thẳng đi qua hai điểm A(1; 1) và B(−1; 1).
B. Hai điểm A(1; 1) và B(−1; 1).
C. Đường tròn tâm I (0; 1) bán kínhR = 1.
D. Đường tròn tâm I (0; −1) bán kínhR = 1.

Bài 8. Phương trình z4 − 16 = 0 có bốn nghiệm phức phân biệt được biểu diễn hình học
bởi bốn điểm A, B, C, D. Tính diện tích S của tứ giác ABCD. √
A. S = 4 B. S = 16 C. S = 8 D. S = 8 2.

Bài 9. Cho + i, z2 = 3 − 4i. Tính modul số phức z1 + z2 √


√ hai số phức z1 = 2 √
A. 34 B. 43 C. 34 D. 5 2
3 − 5i
Bài 10. Tọa độ điểm M biễu diễn trong mặt phẳng Oxy của số phức z = + 7 − 2i
1−i
A. M (11; −3) B.M(11; 3) C.M(11; 3) D. M(3; 11)

Bài 11. Cho số phức z thỏa mãn 2 |z − 2 + 3i | = |2i − 1 − 2z|. Tập hợp điểm biểu diễn
cho số phức z là
A. Một Parabol B.Một đường tròn C.Một Elip D.Một đường thẳng.

Bài 12. Tập hợp điểm biểu diễn số phức |z − 2i | = 3 là đường tròn tâm I. Tìm tất cả các
1
giá trị m để khoảng cách t từ điểm I đến đường thẳng d :3x + 4y − m = 0 bằng
5
A. m = −7, m = 9 B. m = 8, m = −8 C. m = 7, m = 9 D.m = 8, m = 9.

Bài 13. Cho điểm A, B, C theo thứ tự là điểm biểu diễn của ba số phức phân biệt z1 , z2 , z3
thỏa mãn |z1 |√= |z2 | = |z3 | = 1và z1√
+ z2 + z3 = 0. Tính diện
√ tích S tam giác ABC. √
3 2 3 3 2 3 2 2
AS= B.S = C.S = D.S = .
4 4 4 4

2.2 Tìm giá trị lớn nhất, giá trị nhỏ nhất của modul số phức
Bước 1. Từ giả thiết ban đầu của bài toán, tìm tập những điểm M biểu diễn số phức z
trong mặt phẳng phức.
Bước 2. Sử dụng những kiến thức hình học để tìm giá trị lớn nhất, giá trị nhỏ nhất
của modul số phức.

Bài toán 9 (Câu 46 Đề thi mẫu THPTQG của√Bộ Giáo dục năm 2018). Cho số phức
z = a + bi thỏa mãn điều kiện |z − 4 − 3i | = 5. Tính giá trị biểu thức P = a + b khi
|z + 1 − 3i | + |z − 1 + i | đạt giá trị lớn nhất.
A P = 10 B.P = 4 .P = 6 D.P = 8.
Lời giải. √
Với z = a + bi ta có |z − 4 − 3i | = 5 ⇔ ( a − 4)2 + (b − 3)2 = 5
Các điểm M biểu diễn √ số phức z thỏa mãn hệ thức đã cho nằm trên đường tròn tâm
I (4; 3) và bán kính R = 5.

71
Hội thảo khoa học, Ninh Bình 15-16/09/2018

Khi đó T = |z + 1 − 3i | + |z − 1 + i |
T = |( a + 1) + (b − 3)i | + |( a − 1) + (b + 1)i |
⇔ T = MA + MB với A(−1; 3), B(1; −1) √
Xét vị trí tương đối của hai điểm A, B với đường tròn tâm I (4; 3) và bán kính R = 5.


Ta tính được I A = IB = 5 > 5 suy ra điểm A, B nằm ngoài đường tròn tâm I.
Mặt khác T 2 = ( MA + MB)2 ≤ (1 + 1)( MA2 + MB2 ) = 4DM2 + AB2 ≤ 4DK2 +
AB2 . (Với D là trung điểm đoạn AB.)
Dấu bằng xảy ra khi M ≡ √ K ⇔ D, I, K√thẳng hàng. √ √ √
Tìm tọa độ K. Ta có DI = 16 + 4 = 20, DK = DI + R = 20 + 5 = 3 5.
−→ −→
DK DI −→ DK − →
Suy ra = ⇔ DK = DI
DK √ DI DI
−→ 3 5 − → 3− →
⇔ DK = √ DI = DI.
20 ( 2
a=4
Với M( a; b) thì
b=6
Do đó giá trị lớn nhất của biểu thức T 2 là 4DK2 + AB2 khi và chỉ khi M (4; 6) Suy ra
P = 10. Đáp án đúng là (A).

Bài toán 10 (Đề thi học kì 2 Trường THPT Lê Quý Đôn- Đống Đa-Hà Nội- năm 2018).
Cho số phức z thỏa mãn điều kiện |z − 2i | = |z + 2|. Tính giá trị nhỏ nhất của biểu thức
P = |z +√2i | + |z − 5 + 9i |. √ √ √
A 3 10 B. 70 C. 74 D.4 5.
Lời giải.
Với z = x + yi ta có |z − 2i | = |z + 2| ⇔ x + y = 0
Các điểm M biểu diễn số phức z thỏa mãn hệ thức đã cho nằm trên đường thẳng
x + y = 0. p p
Khi đó P = |z + 2i | + |z − 5 + 9i | = x2 + (y + 2)2 + ( x − 5)2 + (y + 9)2
⇔ P = MA + MB với A(0; −2), B(5; −9)

72
Hội thảo khoa học, Ninh Bình 15-16/09/2018

Xét vị trí tương đối của hai điểm A, B với đường thẳng ∆ : x + y = 0 với f ( x; y) =
x + y, ta có f (0; −2). f (5; −9) = 8 > 0 suy ra A, B nằm cùng phía với đường thẳng ∆.
Gọi điểm C đối xứng với điểm A qua đường thẳng ∆ thì P = MA + MB = MC +
MB ≥ BC
Dấu bằng xảy ra khi M ≡ M1 ⇔ C, M1 , B thẳngp hàng. √
Giá trị lớn nhất của biểu thức P là P = BC = 32 + (−9)2 = 3 10. Suy ra đáp án
đúng là (A).
Bài toán 11 (Đề thi thử THPTQG của trường THPT chuyên Lào Cai- năm 2018). Cho số
phức z thỏa mãn điều kiện |z + 1 − 5i | = |z + 3 − i |. Giả sử số phức có modul nhỏ nhất
a
có dạng P = a + b. Khi đó S = bằng bao nhiêu?
b
2 1 1 3
A.S = B.S = C.S = D.S =
3 3 4 2
nLời giải.
Với z = a + bi ta có |z + 1 − 5i | = |z + 3 − i | ⇔ a + 3b − 4 = 0 hay M nằm trên đường
thẳng d : x + 3y − 4 = 0

Số phức z có modul nhỏ nhất ⇔ OM có độ dài ngắn nhất, mà OM ≥ OH (với H


là chân đường vuông góc của gốc tọa độ O trên đường thẳng d) nên OM ngắn nhất khi
M ≡ H.
Ta đi tìm tọa độ điểm H ( a; b), vì H nằm trên đường thẳng d : x + 3y − 4 = 0 hay tọa
−→ → 6
độ H (−3t + 4; t) Suy ra OH ⊥d hay OH.− u d = 0 ⇔ 9t − 12 + t = 0 ⇔ t = . Vậy tọa độ
5
2 6 1
điểm H ( ; ), suy ra S = . Đáp án đúng là (B)
5 5 3
Bài toán 12 (Đề thi thử THPTQG của trường
√ đại học Vinh khối chuyên- năm 2018). Cho
số phức z1 ; z2 thỏa mãn điều kiện |iz + 2 − i | = 1 và |z1 − z2 | = 2. Giá trị lớn nhất của
biểu thức P = |z1 | + |z2 | bằng

73
Hội thảo khoa học, Ninh Bình 15-16/09/2018
√ √
A.4 B.2 3 C.3 2 D.3
nLời giải. √ √
Từ giả thiết ta có |iz + 2 − i | = 1 ⇔ ( x − 1)2 + (y − 2)2 = 1 và M1 M2 = 2
Với các điểm M1 ; M2 biểu diễn hai số phức z1 ; z2 trên
√ mặt phẳng phức thỏa mãn hai
hệ thức trên nên M1 ; M2 nằm trên đường tròn tâm I (1; 2) bán kính R = 1 và M1 M2 đi
qua tâm I của đường tròn.

Khi đó P = |z1 | + |z2 | = OM1 + OM2 .


Theo công thức đường trung tuyến trong tam giác OM1 M2 ,
OM12 + OM22 M1 M22
ta có OI 2 = −
2 4
M M 2
1 2
⇔ OM12 + OM22 = 2OI 2 + .
2
M1 M22
Mặt khác P2 = (OM1 + OM2 )2 ≤ (1 + 1)(OM12 + OM22 ) = 2OI 2 + = 2(2 ×
2
3 + 2) = 16. Suy ra P ≤ 4. Đáp án đúng là (A)

Bài toán 13 (Toán


√ học tuổi trẻ số 491-năm 2018). Cho số phức z thỏa mãn điều kiện
|z − 1 + 2i | = 5. Khi đó số phức
√ w = z + 1 + i có modul√ lớn nhất |w|max bằng

A.20 B.2 5 C. 5 D.5 2
Lời giải. √
Từ giả thiết ta có |iz + 2 − i | = 1 ⇔ ( x − 1)2 + (y + 2)2 = 5
Điểm M √ biểu diễn số phức z trên mặt phẳng nằm trên đường tròn tâm I (1; −2) bán
kính R = 5 .

p
Khi đó |w| = | ( x + 1)2 + (y + 1)2 ⇔ |w√
| = MA với A(−1; −1)
Mà MA ≤ M1 A) nên |w|max bằng 2R = 2 5 khi M ≡ M1 . Đáp án đúng là (B)

Bài toán 14 (Toán học tuổi trẻ


√ số 491-năm 2018). Cho hai số phức z1 ; z2 đồng thời thỏa
mãn hai điều kiện |z − 1| = 34 và |z + 1 + mi | = |z + m + 2i |(m ∈ R) sao cho |z1 − z2 |

74
Hội thảo khoa học, Ninh Bình 15-16/09/2018

là lớn nhất.
√ Khi đó giá trị của √ |z1 + z2 | bằng
A. 2 B. 130 C.2 D.10.
nLời giải.
Gọi điểm M1 ; M2 biểu diễn số phức z1 ; z2 trên mặt phẳng thì |z1 − z2 | =M2 M
√ 1 , mà
theo giả thiết hai số phức z1 ; z2 đồng thời thỏa mãn hai điều kiện |z − 1| = 34 và
|z + 1 + mi | = |z + m + 2i |(m ∈ R), .

Nên tọa độ M1 ; M2 là giao điểm của dường tròn ( x − 1)2 + y2 = 34 và đường thẳng
2x + 1 + 2my + m2 = 2xm + m2 + 4y + 4 ⇔ 2(m − 1) x + (4 − 2m)y + 3 = 0.
Vây | M1 M2 | lớn nhất khi M1 M2 đi qua tâm I (1; 0). Hay |z1 + z2 | = 2OI = 2. Đáp án
đúng là (C)

Bài toán 15 (Đề√ minh họa thi THPTQG-năm 2017). Xét số phức z thỏa mãn |z + 2 − i | +
|z − 4 − 7i | = 6 2. Gọi m,M lần lượt là giá trị nhỏ nhất và giá trị lớn nhất của |z − 1 + i |.
Tính P = m + M. √ √
√ √ 5 2 + 2 73
A.P = 13 + 73 B.P =
√ 2 √
√ √ 5 2 + 73
C.P = 5 2 + 2 73 D.P = .
2
Lời giải.
Gọi M ( x; y) là điểm biểu √ A (−2; 1), B√(4, 7), C (1; −1). Ta có
√ diễn của z. Các điểm
|z + 2 − i | + |z − 4 − 7i | = 6 2 ⇔ MA + MB = 6 2, mà AB = 6 2 ⇒ MA + MB = AB
Suy ra M thuộc đoạn thẳng AB.

5
Phương trình đường thẳng AB : y = x + 3, với x ∈ [−2; 4]. CMmin = d (C; AB) = √ .
√ √ √ 2
CB = 73; CA = 13 ⇒ CM √ max = CB
√ = 73.
√ 5 2 73 + 5 2
Vậy P = 73 + √ = . Chọn đáp án B.
2 2

75
Hội thảo khoa học, Ninh Bình 15-16/09/2018

Bài tập tương tự


Bài 14. Cho các số phức z thỏa mãn |z − (1 + 5i )| = |iz − 1 + 3i |. Tìm giá trị nhỏ nhất
của |z| .√ √
A. 3 B. 2 C. 1 D. 2.

Bài 15. Biết số phức z = a + bi ( a, b ∈ RR)thỏa mãn điều kiện |z − 2 − 4i | = |z − 2i |có


modul nhỏ nhất. Tính M = a2 + b2
A. M = 10 B. M = 16 C. M = 26 D. M = 8.

Bài 16. Cho số phức z thay đổi và luôn thỏa mãn |z − 3 + 4i | = 4. Tìm giá trị lớn nhất
Pmax của biểu thức P = |z|
A. Pmax = 12 B. Pmax = 5 C. Pmax = 9 D. Pmax = 3

Bài 17. Cho số phức z thỏa mãn điều kiện |z − 2 − 4i | = |z − 2i |. Tìm số phức z có modul
nhỏ nhất
A.z = −1 + i B.z = −2 + i C.z = 2 + 2i D.z = 3 + 2i

Bài 18. Cho số phức z thỏa mãn (z + 3 − i ) (z + 1 + 3i ) là một số thực. Tìm giá trị nhỏ
nhất của z. √
√ √ 2
A. 2 B.2 2 C.z = 2 D.
2
Bài 19. Trong các số phức z thỏa mãn điều kiện |z − 2 − 4i | = |z − 2i |. Tìm số phức z có
modul bé nhất.
A.z = 2 + i B.z = 3 + i C.z = 2 + 2i D.z = 1 + 3i
i−m
Bài 20. Cho số phức z = , m ∈ R. Tìm giá trị nhỏ nhất của số thực k sao
1 − m (m − 2i )
√ để |z + 1| ≤ k.
cho tồn tại m √
5−1 5+1
A.k = B.k = 0 C.k = D.k = 1
2 2

2.3 Một số bài toán số phức liên quan đến hình học tọa độ
Bên cạnh các bài toán Tìm giá trị lớn nhất và giá trị nhỏ nhất của modul số phức có
liên quan đến hình học tọa độ còn có các bài toán khác về số phức cũng có thể đưa kiến
thức hình học tọa độ vào giải quyết chúng.Sau đây là một số bài toán về số phức có đặc
điểm như đã nêu ở trên.

Bài toán 16 (Toán học tuổi trẻ số 490-năm 2018). Cho hai số phức z1 ; z2 đồng thời thỏa
mãn hai điều kiện |z1 | = 1 và |z2 | = 2 sao cho |z1 + z2 | = 3. Khi đó giá trị của |z1 − z2 |
bằng
A. 0 B. 1 C. 2 D. Một giá trị khác
Lời giải.
Gọi điểm M1 ; M2 biểu diễn số phức z1 ; z2 trên mặt phẳng thì OM1 = 1, OM2 = 2, và
| z1 + z2 | = 3
−−→ −−→ −→
⇔ |OM1 + OM2 | = 2|OK | với K là trung điểm M1 M2
3
⇔ OK =
2

76
Hội thảo khoa học, Ninh Bình 15-16/09/2018

−−−→
|z1 − z2 | = | M2 M1 |.
OM22 + OM12 M1 M22
Xét ∆OM2 M1 có trung tuyến: OK2 = −
2 4
9 5 M1 M22
⇔ = − ⇔ M1 M2 = 1
4 2 4
suy ra |z1 − z2 | = 1. Đáp án đúng là (B.)

Bàitoán 17 (Toán học tuổi trẻ số 488-năm 2018). Tìm số phức z thỏa mãn
|z − 14 − 7i | = |z − 8 + 7i |
|z + 16i | = |z − 10 + 12i |
Lời giải.
Điểm M là điểm biểu diễn số phức z = x + yi trên mặt phẳng, suy ra tọa độ M ( x; y).
Khi đó z1 = 14 + 7i có điểm biểu diễn hình học là A(14; 7) ; z2 = 8 − 7i có điểm
biểu diễn hình học là B(8; −7) ;z3 = −16i có điểm biểu diễn hình học là C (0; −16);
z4 = 10 − 12i có điểm biểu diễn hình học là D (10; −12) .
Ta có |z − 14 − 7i | = |z − 8 + 7i | ⇔ MA = MB, suy ra M thuộc đường thẳng trung
trực d của đoạn AB. Phương trình đường thẳng d : 3x + 7y − 33 = 0.
Ta có |z + 16i | = |z − 10 + 12i | ⇔ MC = MD, suy ra M thuộc đường thẳng trung
trực ∆ của đoạn CD. Phương trình đường thẳng ∆ : 5x + 2y + 3 = 0
Tọa độ M là tọa độ giao điểm của hai đường thẳng d và ∆. Nên giải hệ ta có M(−3; 6).
Số phức cần tìm là z = −3 + 6i.

Bài toán 18 (Toán học tuổi trẻ số 488-năm 2018). Tìm số phức z thỏa mãn
|z − 13 − 6i | = |z −√3 − 12i |
|z − 7 + 4i | = 34
Lời giải.
Gọi điểm M là điểm biểu diễn số phức z = x + yi trên mặt phẳng, suy ra tọa độ
M ( x; y).
Khi đó z1 = 13 + 6i có điểm biểu diễn hình học là A(13; 6) ; z2 = 3 + 12i có điểm biểu
diễn hình học là B(3; 12) ;z3 = 7 − 4i có điểm biểu diễn hình học là I (7; −4).

77
Hội thảo khoa học, Ninh Bình 15-16/09/2018

Từ giả thiết ta suy ra M chính là giao


√ điểm của đường thẳng trung trực đoạn AB và
đường tròn tâm I (7; −4) bán kính R = 34
Phương trình đường thẳng trung trực đoạn AB 5x − 3y − 13 = 0
Nên giải hệ ta có M(2; −1). Số phức cần tìm là z = 2 − i.

Bài tập tương tự


Bài 21. Gọi A, B lần lượt là các điểm biểu diễn của các số phức z = 1 − 3i và w = −2 + i
trên mặt phẳng tọa độ. Tính độ dài của đoạn thẳng AB. √ √
A. 5 B. 3 C. 5 D. 13

Bài 22. Tính tích modul của tất cả các số phức z thỏa mãn |2z − 1| = |z + 1 + i | , đồng
thời điểm √
biểu diễn của z trên mặt phẳng tọa độ thuộc đường tròn có tâm I (1; 1) và bán
kính R = 5. √ √
A. 1 B. 3 5. C. 5 D. 3

Bài 23. Gọi (H) là hình biểu diễn tập hợp các số phức z trong mặt phẳng tọa đọ Oxy để
|2z − z| ≤ 3 số phức z có phần thực không âm. Tính diện tích hình (H)
3 3
A. 3π B. π C. π D. 6π
2 4
Bài 24. Cho hai số phức z1 =√2 − 3i, z2 = 1 + √
2i. Tính modul của√số phức z = (z1 + 2)z2
A. |z| = 15 B. |z| = 5 5 C. |z| = 65 D. |z| = 137

Bài 25. Tìm số phức z thỏa mãn hệ thức (1 + i )z + z = 1 + i


A. z = 1 + i B. z = 1 − i C. z = 2 + i D. z = 2 − i

Bài 26. Tập hợp điểm M biểu diễn cho số phức zthỏa mãn |z − i | = |(1 + i )z| là đường
tròn có phương trình
A. x2 + (y − 1)2 = 2 B. ( x − 1)2 + y2 = 2
2 2
C. x2 + (y + 1) = 2 D. ( x + 1) + y2 = 2

Bài 27. Cho điểm M biểu diễn số phức z = 3 − 4i và điểm M’ biểu diễn số phức z0 =
1+i
z. Tính diện tích tam giác OMM0 (với O là gốc tọa độ)
2
15 25 25 31
A. B. C. D.
2 4 2 4

78
Hội thảo khoa học, Ninh Bình 15-16/09/2018

Bài 28. Cho hai số phức z1 , z2 thỏa mãn |z1 | = |z2 | = |z1 − z2 | = 1. Tính giá trị của biểu
 2  2
z1 z2
thức P = +
z2 z1
A.P = 1 − i B. P = −1 − i C. P = −1 D. P = 1 + i

Bài 29. Cho số phức z thỏa mãn z − (1 + i )z = (1 − 2i√)2 . Tìm modul của số phức z
A.100 B. 10 C. 109 D. 3

Bài 30. Cho số phức z thỏa mãn (1 − i )z + (3 − i )z = 2 − 6i. Tìm phần ảo của số phức
w = 2z + 1
A.6 B. 3 C.5 D. 2

Bài 31. Cho số phức z thỏa mãn (1 − i )z + (3 − i )z = 2 − 6i. Tìm số phức w biết w =
2z + 2
A.2 + 3i B. 2 − 3i C. 6 + 6i D. 6 − 6i

 z − 1 = 1

 z−i
Bài 32. Tìm số phức z thoả mãn hệ
z − 3i

 =1
z+i

3 Kết luận
Trên đây là những kinh nghiệm của bản thân tôi đã đúc kết và tổng hợp được trong
quá trình giảng dạy bộ môn Toán tại trường trung học phổ thông.
Trên nền tảng cơ sở lí thuyết là kiến thức Sách giáo khoa, ví dụ áp dụng là bài tập
trong đề thi và tạp chí Toán học tuổi trẻ. Tôi đã sắp xếp các ví dụ từ dễ đến khó, sao cho
trò có thể tiếp cận lời giải nhanh gọn, chính xác, dễ dàng nhất. Sau mỗi ví dụ là phần tự
luyện tập với các bài tương tự học sinh có thể tự học ở nhà để rèn kỹ năng trình bày tư
duy, giải nhanh.
Dưới sự hỗ trợ của phần mềm Geogebra hình vẽ trong tài liệu được trình bày rõ ràng
và chính xác, giúp người đọc có thể nhìn ra ngay kết quả của bài toán. Nhưng do thời
gian nghiên cứu về giá trị lớn nhất và giá trị nhỏ nhất của môđun số phức chưa nhiều và
sâu nên các dạng bài tập về sử dụng hình học tọa độ trong số phức chưa được đa dạng.
Tôi cũng mong ước trong lần viết sau tôi sẽ bổ sung thêm dạng sử dụng bất đẳng thức
trong hình học để giải các bài toán về số phức.

Tài liệu
[1] Huỳnh Văn Minh (2018), Giải toán số phức bằng phương pháp tọa độ, Tạp chí toán
học tuổi trẻ.
[2] Các bài thi Olympic toán Trung học phổ thông Việt Nam, Nhà xuất bản Giáo dục,
Hà Nội, 2006.
[3] Nguyễn Văn Mậu (2006), Số phưsc và áp dụng, NXB Giáo dục.
[4] Đoàn Quỳnh, Nguyễn Huy Đoan (2009), SGK Giải tích 12, NXB Giáo dục.

79
Hội thảo khoa học, Ninh Bình 15-16/09/2018

HÀM ĐƠN ĐIỆU VÀ MỘT SỐ ỨNG DỤNG CỦA PHÉP


ĐƠN ĐIỆU HÓA HÀM SỐ

Lê Văn Hiểu
Trường THPT Yên Khánh B, Huyện Yên Khánh, Ninh Bình

Tóm tắt nội dung

Lớp các hàm số đơn điệu và lồi, lõm có vị trí rất quan trọng trong Giải tích Toán học
vì nó không những là một đối tượng nghiên cứu trọng tâm của nhiều mô hình toán học
mà còn là một công cụ đắc lực để khảo sát bất đẳng thức và các bài toán cực trị.
Trong hầu hết các kì thi học sinh giỏi toán quốc gia và Olympic toán quốc tế thì các
bài toán về hàm số thường được đề cập đến và được xem như những dạng toán rất khó
của bậc phổ thông.
Báo cáo này nhằm khảo sát sâu hơn về lớp các hàm đơn điệu, tựa đơn điệu và các
dạng toán ứng dụng liên quan cho ta hiểu sâu sắc hơn về lý thuyết cũng như các ứng
dụng liên quan đến hàm số và nêu bật vai trò quan trọng của hàm đơn điệu, tựa đơn
điệu trong các dạng toán thi HSG quốc gia và Olympic quốc tế.

1 Hàm đơn điệu


Trong chương trình giải tích, chúng ta đã biết đến các tiêu chuẩn để nhận biết khi nào
thì một hàm số khả vi cho trước trên khoảng ( a, b) là một hàm đơn điệu trên khoảng đó.
Các định lí sau đây cho ta một số đặc trưng đơn giản khác của hàm đơn điệu.

Định lý 1 (xem [2-6]). Hàm số f ( x ) xác định trên R+ là một hàm đơn điệu tăng khi và
chỉ khi với mọi cặp bộ số dương a1 , a2 , . . . , an và x1 , x2 , . . . , xn , ta đều có
n  n   n 
∑ ak f ( xk ) ≤ ∑ ak f ∑ xk . (1.1)
k =1 k =1 k =1

Định lý 2 (xem [2-6]). Để bất đẳng thức


n  n 
∑ f ( xk ) ≤ f ∑ xk , (1.2)
k =1 k =1

f (x)
được thỏa mãn với mọi bộ số dương x1 , x2 , . . . , xn , điều kiện đủ là hàm g( x ) :=
x
đơn điệu tăng trên R+ .

80
Hội thảo khoa học, Ninh Bình 15-16/09/2018

f (x)
Hệ quả 1. Giả sử g( x ) = là hàm đơn điệu tăng trong [0, +∞]. Khi đó, với mọi dãy
x
số dương và giảm x1 , x2 , . . . , xn , ta đều có
n −1  
f ( x1 − x n ) ≥ ∑ f ( x k ) − f ( x k +1 ) .
k =1

Nhận xét 1. Tương tự, ta cũng phát biểu các đặc trưng với hàm đơn điệu giảm.
Định lý 3 (xem [2-6]). Để bất đẳng thức
n  n 
∑ f ( xk ) ≥ f ∑ xk ,
k =1 k =1

f (x)
được thỏa mãn với mọi bộ số dương x1 , x2 , . . . , xn , điều kiện đủ là hàm g( x ) :=
x
đơn điệu giảm trên R+ .

Định lý 4 (xem [2-6]). Giả thiết rằng, với mọi cặp bộ số dương
a1 , a2 , . . . , an ; x1 , x2 , . . . , xn , ta đều có
n  n 
∑ ak f ( xk ) ≥ f ∑ ak xk , (1.3)
k =1 k =1

thì f ( x ) = ax, trong đó a là hằng số.


Định lý 5 (Maclaurin, Cauchy). Giả thiết rằng f ( x ) là một hàm đơn điệu giảm trên
(0, +∞). Khi đó, ta luôn có
n Z n n −1
∑ f (k) ≤
0
f ( x )dx ≤ ∑ f ( k ). (1.4)
k =1 k =0

Khi f ( x ) là hàm nghịch biến thì có dấu bất đẳng thức thực sự.
Định lý 6 (xem [2-6]). Giả thiết rằng f ( x ) là một hàm đơn điệu giảm trên (0, +∞) và
{ ak } là một dãy tăng trong (0, +∞). Khi đó, ta luôn có
n Z an n
∑ ( a k − a k −1 ) f ( a k ) ≤ a0
f ( x )dx ≤ ∑ ( a k − a k −1 ) f ( a k −1 ). (1.5)
k =1 k =1

Khi f ( x ) là hàm nghịch biến thì có dấu bất đẳng thức thực sự.
Định lý 7 (Bất đẳng thức thứ tự Chebyshev). Giả sử f ( x ) và g( x ) là hai hàm đơn điệu
tăng và ( xk ) là một dãy đơn điệu tăng
x1 ≤ x2 ≤ · · · ≤ x n .
Khi đó với mọi bộ trong ( p j )
p j ≥ 0, j = 1, 2, . . . , n; p1 + p2 + · · · + pn = 1,

ta đều có
 n  n   n 
∑ pk f ( xk ) ∑ pk g( xk ) ≤ ∑ pk f ( xk ) g( xk ) .
k =1 k =1 k =1

81
Hội thảo khoa học, Ninh Bình 15-16/09/2018

Định nghĩa 1 (xem [2]). Hàm số f ( x ) được gọi là hàm đơn điệu tuyệt đối trong khoảng
( a, b) nếu đạo hàm mọi cấp của nó đều không đổi dấu

f (k) ( x ) ≥ 0; ∀ x ∈ ( a, b), k = 0, 1, 2, . . .

Cũng vậy, ta có định nghĩa hàm đồng biến và nghịch biến tuyệt đối.

Định nghĩa 2 (xem [2]). Hàm số f ( x ) được gọi là hàm đồng biến (nghịch biến) tuyệt đối
trong khoảng ( a, b) nếu các đạo hàm mọi cấp của nó đều là hàm đồng biến (nghịch biến)
tuyệt đối trong khoảng đó.
Ví dụ về các hàm số sơ cấp đơn điệu, đồng biến (nghịch biến) tuyệt đối trong khoảng
( a, b), ( a > 0) là các hàm số sau.
Ví dụ 1. Mọi đa thức P( x ) với các hệ số đều dương là hàm đơn điệu tăng tuyệt đối trong
khoảng (0, +∞).

Ví dụ 2. Hàm số f ( x ) = e x là hàm đồng biến tuyệt đối trong khoảng (0, +∞).

Ví dụ 3. Hàm số
x−1
f (x) = − ex ,
x+1
là hàm nghịch biến tuyệt đối trong khoảng (0, +∞).

Nhận xét 2. Nếu hàm số f ( x ) là hàm đồng biến tuyệt đối trong khoảng ( a, b) thì hàm
số g( x ) := − f ( x )sẽ là hàm nghịch biến tuyệt đối trong khoảng đó và ngược lại. Vì vậy,
không mất tính tổng quát, ta chỉ trình bày các bài toán liên quan đến hàm đơn điệu tăng
và đồng biến tuyệt đối trong khoảng đã cho.

Song song với lớp hàm đơn điệu thông thường và đơn điệu tuyệt đối, nhiều lớp hàm
đơn điệu khác cũng được đưa ra và nghiên cứu các đặc trưng của chúng như đơn điệu
đầy đủ, đơn điệu có tính tuần hoàn hoàn toàn, . . .

Định nghĩa 3 (xem [2]). Hàm số f ( x ) được gọi là hàm đơn điệu có tính tuần hoàn trong
khoảng ( a, b) khi và chỉ khi các đạo hàm của chúng không triệt tiêu (có dấu không đổi)

f (k) ( x ) f (k+2) ( x ) ≤ 0; ∀ x ∈ ( a, b), k = 0, 1, 2, . . .

Ví dụ về các hàm số sơ cấp đơn điệu có tính tuần hoàn trong khoảng ( a, b), ( a > 0)
là các hàm số sau.

Ví dụ 4. Hàm số
f ( x ) = sin x,
 π
là hàm số đơn điệu có tính tuần hoàn trong khoảng 0, .
2

Nhận xét 3. Trong các dạng toán liên quan đến khảo sát hàm số, ta thương gặp lớp hàm
có các đạo hàm bậc nhất, bậc hai không đổi dấu trong khoảng cho trước. Lớp hàm này
có nhiều ứng dụng trong bất đẳng thức và các bài toán cực trị.

82
Hội thảo khoa học, Ninh Bình 15-16/09/2018

Định nghĩa 4 (xem [2-6]). Giả sử f ( x ) là hàm khả vi bậc hai trên I ( a, b). Nếu f 0 ( x ) ≥ 0
với ∀ x ∈ I ( a, b) và f 00 ( x ) ≥ 0 với ∀ x ∈ I ( a, b) thì ta nói rằng f ( x ) là hàm đơn điệu tăng
liên tiếp bậc (1 − 2) trên I ( a, b).
Chẳng hạn, f 0 ( x ) ≥ 0, f 00 ( x ) ≤ 0, ∀ x ∈ I ( a, b) thì ta nói f ( x ) là hàm tăng - giảm bậc
(1 − 2) trên I ( a, b) và f 0 ( x ) ≤ 0, f 00 ( x ) ≥ 0, ∀ x ∈ I ( a, b) thì ta nói f ( x ) là hàm giảm -
tăng bậc (1 − 2) trên I ( a, b).

Bài toán 1. Cho các hàm f 1 (t), f 2 (t), . . . , f n (t) đồng thời đồng biến (nghịch biến) liên tiếp
bậc (1 − 2) trên I ( a, b). Giả sử dãy số {vk } với vk ∈ ={ f k0 ( x )}, x ∈ I ( a, b), k = 1, 2, . . . , n.
Tìm
n
f (x )
min ∑ k k ,
k =1
υk

ứng với mọi dãy số { xk } với xk ∈ I ( a, b); k = 1, 2, . . . , n có tổng


x1 + x2 + · · · + xn = ( f 10 )−1 (v1 ) + ( f 20 )−1 (v2 ) + · · · + ( f n0 )−1 (vn ) cho trước.
  −1
Lời giải. Theo nhận xét, đặt uk = f k0 (vk ), k = 1, 2, . . . , n. Tức là

f k0 (uk ) = vk , k = 1, 2, . . . , n.

Ta thu được vk = f k0 (uk ), uk ∈ I ( a, b), k = 1, 2, . . . , n.


Vậy nên
n n
f k ( xk ) f (x )
∑ vk = ∑ f k0 (ukk ) . (1.6)
k =1 k =1 k

Mặt khác, ta có
n n
f k ( xk ) f k (uk )
∑ 0
f k (uk )
≥ ∑ f 0 (u )
. (1.7)
k =1 k =1 k k

Từ (1.6), (1.7), ta suy ra


n n
f k ( xk ) f (u )
∑ vk
≥ ∑ k0 k .
f (u )
(1.8)
k =1 k =1 k k

Dấu đẳng thức xảy ra khi

xk = uk = ( f k0 )−1 (vk ), k = 1, 2, . . . , n.

Vậy nên
n n
f k ( xk ) f (u )
min ∑ = ∑ k0 k .
k =1
υk f (u )
k =1 k k
Tương tự, ta có

Bài toán 2. Cho các hàm f 1 (t), f 2 (t), . . . , f n (t) đồng thời lồi (lõm) và có đạo hàm bậc
nhất là các hàm số âm (dương) trên khoảng I ( a, b).
Giả sử dãy số {vk } với vk ∈ ={ f k0 ( x )}; x ∈ I ( a, b), k = 1, 2, . . . , n.
Tìm
n
f (x )
max ∑ k k ,
k =1
υk

ứng với mọi dãy số { xk } với xk ∈ I ( a, b); k = 1, 2, . . . , n có tổng x1 + x2 + · · · + xn =


( f 10 )−1 (v1 ) + ( f 20 )−1 (v2 ) + · · · + ( f n0 )−1 (vn ) cho trước.

83
Hội thảo khoa học, Ninh Bình 15-16/09/2018
  −1
Lời giải. Đặt uk = f k0 (vk ), k = 1, 2, . . . , n. Khi đó

f k0 (uk ) = vk , k = 1, 2, . . . , n,

ta thu được vk = f k0 (uk ), uk ∈ I ( a, b), k = 1, 2, . . . , n.


Vậy nên
n n
f k ( xk ) f (x )
∑ vk = ∑ f k0 (ukk ) . (1.9)
k =1 k =1 k

Mặt khác, ta có
n n
f k ( xk ) f k (uk )
∑ 0
f k (uk )
≤ ∑ f 0 (u )
. (1.10)
k =1 k =1 k k

Từ (1.9), (1.10) suy ra


n n
f k ( xk ) f (u )
∑ vk
≤ ∑ k0 k .
f (u )
(1.11)
k =1 k =1 k k

Dấu đẳng thức xảy ra khi

xk = uk = ( f k0 )−1 (vk ), k = 1, 2, . . . , n.

Vậy nên
n n
f k ( xk ) f (u )
max ∑ = ∑ k0 k .
k =1
υk f (u )
k =1 k k

Ta xét trường hợp riêng khi hàm f 1 (t) = f 2 (t) = · · · = f n (t) = f ( x ).


Từ kết quả hai bài toán trên ta thu được.

Hệ quả 2. Cho hàm số f (t) đồng biến (nghịch biến) liên tiếp bậc (1 − 2) trên I ( a, b).
Giả sử dãy số {vk } với vk ∈ ={ f k0 ( x )}; x ∈ I ( a, b), k = 1, 2, . . . , n. Khi đó
n n
f ( xk ) f (u )
min ∑ =∑ 0 k ,
k =1
υk k =1
f (uk )

ứng với mọi dãy số { xk } với xk ∈ I ( a, b); k = 1, 2, . . . , n có tổng

x1 + x2 + · · · + x n = u1 + u2 + · · · + u n .

Trong đó uk = ( f 0 )−1 (vk ); k = 1, 2, . . . , n.

Hệ quả 3. Cho hàm số f (t) lồi (lõm) và có đạo hàm bậc nhất là các hàm số âm (dương)
trên I ( a, b).
Giả sử dãy số {vk } với vk ∈ ={ f k0 ( x )}; x ∈ I ( a, b), k = 1, 2, . . . , n. Khi đó
n n
f k ( xk ) f (u )
max ∑ = ∑ k0 k ,
k =1
υk f (u )
k =1 k k

ứng với mọi dãy số { xk } với xk ∈ I ( a, b); k = 1, 2, . . . , n có tổng

x1 + x2 + · · · + x n = u1 + u2 + · · · + u n .

Trong đó uk = ( f 0 )−1 (vk ); k = 1, 2, . . . , n.

84
Hội thảo khoa học, Ninh Bình 15-16/09/2018

Bài toán 3. Cho hàm số f ( x ) = ax2(k+1) + bx + c, ( a 6= 0), k ≥ 0.


Chứng minh rằng

n 2( k +1) n 2( k +1)
axk + bxk + c auk + buk + c
min ∑ +1
= ∑ 2(k + 1)au2k+1 + b , ∀ x > x0 .
k =1 2( k + 1) au2k
k +b k =1 k

Trong đó x0 là nghiệm của phương trình f 0 ( x ) = 0, ứng với mọi dãy số { xk } với xk ∈
I ( a, b); k = 1, 2, . . . , n có tổng x1 + x2 + · · · + xn ≥ u1 + u2 + · · · + un .

Lời giải. Ta có

f ( x ) = ax2(k+1) + bx + cx, ( a 6= 0);


f 0 ( x ) = 2(k + 1) ax2k+1 + b;
f 00 ( x ) = 2(k + 1)(2k + 1) ax2k .

Trường hợp 1: Nếu a > 0. Khi đó f 00 ( x ) = 2(k + 1)(2k + 1) ax2k > 0, ∀ x, suy ra f 0 ( x ) là
hàm số đồng biến trên R.
Vậy, phương trình f 0 ( x ) = 0 có nghiệm duy nhất x = x0 . Ta có f 0 ( x ) > 0, ∀ x > x0 ,
nên ∀ x > x0 hàm số f ( x ) đồng biến liên tiếp bậc (1 − 2).
Trường hợp 2: Nếu a < 0. Khi đó f 00 ( x ) = 2(k + 1)(2k + 1) ax2k < 0, ∀ x, suy ra f 0 ( x ) là
hàm số nghịch biến trên R.
Do đó, phương trình f 0 ( x ) = 0 có nghiệm duy nhất x = x0 . Ta có f 0 ( x ) < 0, ∀ x > x0 ,
nên ∀ x > x0 hàm số f ( x ) nghịch biến liên tiếp bậc (1 − 2).
Do đó, với ∀ x > x0 hàm số f ( x ) đồng biến hoặc nghịch biến liên tiếp bậc (1 − 2), nên
theo Hệ quả 2, ta có

n 2( k +1) n 2( k +1)
axk + bxk + c auk + buk + c
min ∑ +1
= ∑ 2(k + 1)au2k+1 + b , ∀ x > x0 ,
k =1 2( k + 1) au2k
k +b k =1 k

trong đó x0 là nghiệm của phương trình f 0 ( x ) = 0, ứng với mọi dãy số { xk } với xk ∈
I ( a, b); k = 1, 2, . . . , n có tổng x1 + x2 + · · · + xn ≥ u1 + u2 + · · · + un .

Bài toán 4. Cho hàm số f ( x ) = ax2(k+1) + bx + c, ( a 6= 0), k ≥ 0. Chứng minh rằng

n 2( k +1) n 2( k +1)
axk + bxk + c auk + buk + c
max ∑ +1
= ∑ 2(k + 1)au2k+1 + b , ∀ x < x0 .
k =1 2( k + 1) au2k
k +b k =1 k

Trong đó x0 là nghiệm của phương trình f 0 ( x ) = 0, ứng với mọi dãy số { xk } với xk ∈
I ( a, b); k = 1, 2, . . . , n có tổng x1 + x2 + · · · + xn ≤ u1 + u2 + · · · + un .

Lời giải. Ta có

f ( x ) = ax2(k+1) + bx + cx, ( a 6= 0);


f 0 ( x ) = 2(k + 1) ax2k+1 + b;
f 00 ( x ) = 2(k + 1)(2k + 1) ax2k .

Trường hợp 1: Nếu a > 0. Khi đó f 00 ( x ) = 2(k + 1)(2k + 1) ax2k > 0, ∀ x, suy ra f 0 ( x ) là
hàm số đồng biến trên R hay phương trình f 0 ( x ) = 0 có nghiệm duy nhất x = x0 . Ta có

85
Hội thảo khoa học, Ninh Bình 15-16/09/2018

f 0 ( x ) < 0, ∀ x < x0 , nên ∀ x < x0 hàm số f ( x ) lồi có đạo hàm bậc nhất là những số âm.
Trường hợp 2: Nếu a < 0. Khi đó f 00 ( x ) = 2(k + 1)(2k + 1) ax2k < 0, ∀ x. Suy ra f 0 ( x ) là
hàm số nghịch biến trên R nên phương trình f 0 ( x ) = 0 có nghiệm duy nhất x = x0 . Ta
có f 0 ( x ) > 0, ∀ x < x0 , nên ∀ x < x0 hàm số f ( x ) lõm có đạo hàm bậc nhất là những số
dương.
Do đó, với ∀ a 6= 0, ∀ x < x0 hàm số f ( x ) lồi (lõm) có đạo hàm bậc nhất là những số
âm (dương), nên theo Hệ quả 3 ta có

n 2( k +1) n 2( k +1)
axk + bxk + c auk + buk + c
max ∑ +1
= ∑ 2(k + 1)au2k+1 + b , ∀ x < x0 .
k =1 2( k + 1) au2k
k +b k =1 k

trong đó x0 là nghiệm của phương trình f 0 ( x ) = 0, ứng với mọi dãy số { xk } với xk ∈
I ( a, b); k = 1, 2, . . . , n có tổng x1 + x2 + · · · + xn ≤ u1 + u2 + · · · + un .

Bài toán 5. Cho hàm số f ( x ) = ax2(k+1) + bx k+2 + cx2 + dx + e, ( a 6= 0) thỏa mãn điều
kiện (k2 + 3k + 2)2 b2 − 16(2k2 + 3k + 1) ac < 0, ∀k ≥ 1.
Chứng minh rằng
2( k +1)
n axk + bxkk+2 + cxk2 + dxk + e
min ∑ 2(k + 1)au2k+1 + (k + 2)bk+1 + 2cu
k =1 k k k+d
2( k +1) k +2
n auk + buk + cu2k + duk + e
= ∑ 2( k + 1) au2k +1
+ (k + 2)bkk+1 + 2cuk + d
, ∀ x > x0 .
k =1 k

Trong đó x0 là nghiệm của phương trình f 0 ( x ) = 0, ứng với mọi dãy số { xk } với xk ∈
I ( a, b); k = 1, 2, . . . , n có tổng x1 + x2 + · · · + xn ≥ u1 + u2 + · · · + un .

Lời giải. Ta có

f ( x ) = ax2(k+1) + bx k+2 + cx2 + dx + e, ( a 6= 0);


f 0 ( x ) = 2(k + 1) ax2k+1 + (k + 2)bx k+1 + 2cx + d;
f 00 ( x ) = 2(k + 1)(2k + 1) ax2k + (k + 1)(k + 2)bx k + 2c.

Đặt x k = t (t > 0), xét hàm số

g(t) = 2(k + 1)(2k + 1) at2 + (k + 1)(k + 2)bt + 2c.

Trường hợp 1: Nếu a > 0.


Vì ∆ g = (k2 + 3k + 2)2 b2 − 16(2k2 + 3k + 1) ac < 0, ∀k ≥ 1, suy ra g(t) > 0, ∀t > 0. Suy
ra f 00 ( x ) > 0, ∀ x, suy ra f 0 ( x ) là hàm số đồng biến trên R.
Vậy, phương trình f 0 ( x ) = 0 có nghiệm duy nhất x = x0 . Ta có f 0 ( x ) > 0, ∀ x > x0 ,
nên x > x0 hàm số f ( x ) là hàm số đồng biến liên tiếp bậc (1 − 2).
Trường hợp 2: Nếu a < 0.
Vì ∆ g = (k2 + 3k + 2)2 b2 − 16(2k2 + 3k + 1) ac < 0, ∀k ≥ 1, suy ra g(t) < 0, ∀t > 0.
Suy ra f 00 ( x ) < 0, ∀ x nên f 0 ( x ) là hàm số nghịch biến trên R. Suy ra phương trình
f 0 ( x ) = 0 có nghiệm duy nhất x = x0 . Ta có f 0 ( x ) < 0, ∀ x > x0 , nên x > x0 hàm số f ( x )
là hàm số nghịch biến liên tiếp bậc (1 − 2).

86
Hội thảo khoa học, Ninh Bình 15-16/09/2018

Do đó, với ∀ a 6= 0, ∀ x > x0 . Hàm số f ( x ) đồng biến hoặc nghịch biến liên tiếp bậc
(1 − 2), nên theo Hệ quả 2, ta có
2( k +1)
n axk + bxkk+2 + cxk2 + dxk + e
min ∑ 2(k + 1)au2k+1 + (k + 2)bk+1 + 2cu
k =1 k k k+d
2( k +1)
n auk + bukk+2 + cu2k + duk + e
= ∑ 2( k + 1) au2k +1
+ (k + 2)bkk+1 + 2cuk + d
, ∀ x > x0 .
k =1 k

Trong đó, x0 là nghiệm của phương trình f 0 ( x ) = 0, ứng với mọi dãy số { xk } với xk ∈
I ( a, b); k = 1, 2, . . . , n có tổng x1 + x2 + · · · + xn ≥ u1 + u2 + · · · + un .
Bài toán 6. Cho hàm số f ( x ) = ax2(k+1) + bx k+2 + cx2 + dx + e, ( a 6= 0) thỏa mãn điều
kiện (k2 + 3k + 2)2 b2 − 16(2k2 + 3k + 1) ac < 0, ∀k ≥ 1. Chứng minh rằng
2( k +1)
n axk + bxkk+2 + cxk2 + dxk + e
max ∑ 2(k + 1)au2k+1 + (k + 2)bk+1 + 2cu
k =1 k k k+d
2( k +1) k +2
n auk + buk + cu2k + duk + e
= ∑ 2( k + 1) au2k +1
+ (k + 2)bkk+1 + 2cuk + d
, ∀ x < x0 .
k =1 k

Trong đó x0 là nghiệm của phương trình f 0 ( x ) = 0, ứng với mọi dãy số { xk } với xk ∈
I ( a, b); k = 1, 2, . . . , n có tổng x1 + x2 + · · · + xn ≤ u1 + u2 + · · · + un .
Lời giải. Ta có
f ( x ) = ax2(k+1) + bx k+2 + cx2 + dx + e, ( a 6= 0);
f 0 ( x ) = 2(k + 1) ax2k+1 + (k + 2)bx k+1 + 2cx + d;
f 00 ( x ) = 2(k + 1)(2k + 1) ax2k + (k + 1)(k + 2)bx k + 2c.
Đặt x k = t (t > 0), xét hàm số
g(t) = 2(k + 1)(2k + 1) at2 + (k + 1)(k + 2)bt + 2c.
Trường hợp 1: Nếu a > 0.
Vì ∆ g = (k2 + 3k + 2)2 b2 − 16(2k2 + 3k + 1) ac < 0, ∀k ≥ 1, suy ra g(t) > 0, ∀t > 0.
Do đó, f 00 ( x ) > 0, ∀ x, suy ra f 0 ( x ) là hàm số đồng biến trên R.
Vậy, phương trình f 0 ( x ) = 0 có nghiệm duy nhất x = x0 . Ta có f 0 ( x ) < 0, ∀ x < x0 ,
nên x < x0 hàm số f ( x ) lồi có đạo hàm bậc nhất là những số âm.
Trường hợp 2: Nếu a < 0.
Vì ∆ g = (k2 + 3k + 2)2 b2 − 16(2k2 + 3k + 1) ac < 0, ∀k ≥ 1, suy ra g(t) < 0, ∀t > 0.
Do đó, f 00 ( x ) < 0, ∀ x, suy ra f 0 ( x ) là hàm số nghịch biến trên R. Suy ra phương trình
f 0 ( x ) = 0 có nghiệm duy nhất x = x0 . Ta có f 0 ( x ) > 0, ∀ x < x0 , nên x < x0 hàm số f ( x )
lõm có đạo hàm bậc nhất là những số dương.
Do đó, với ∀ a 6= 0, ∀ x < x0 . Hàm số f ( x ) lồi (lõm)có đạo hàm bậc nhất là những số
âm (dương), nên theo Hệ quả 3 ta có
2( k +1)
n axk + bxkk+2 + cxk2 + dxk + e
max ∑ 2(k + 1)au2k+1 + (k + 2)bk+1 + 2cu
k =1 k k k+d
2( k +1)
n auk + bukk+2 + cu2k + duk + e
= ∑ 2( k + 1) au2k +1
+ (k + 2)bkk+1 + 2cuk + d
, ∀ x < x0 ,
k =1 k

87
Hội thảo khoa học, Ninh Bình 15-16/09/2018

trong đó, x0 là nghiệm của phương trình f 0 ( x ) = 0, ứng với mọi dãy số { xk } với xk ∈
I ( a, b); k = 1, 2, . . . , n có tổng x1 + x2 + · · · + xn ≤ u1 + u2 + · · · + un .

2 Sử dụng tính đơn điệu của hàm số trong chứng


minh bất đẳng thức
Có thể nói rằng các tính chất cơ bản của hàm số luôn đóng vai trò quan trọng như là
những công cụ hữu hiệu nhất để định hướng giải cũng như sáng tác bài tập mới. Những
kiến thức đầu tiên liên quan đến khái niệm đơn điệu được đề cập ở bậc tiểu học chính là
các bài toán về tỷ lệ thuận và tỷ lệ nghịch. Đặc biệt, nhiều tính chất cơ bản của phân số
số học là những kiến thức sâu sắc được sử dụng giải quyết khá nhiều bài toán khó của
các kỳ thi Olympic các quốc gia và quốc tế.

2.1 Một số bài toán áp dụng trong bất đẳng thức đại số
Ta xét ví dụ rất quen thuộc sau đây.

Bài toán 7. Chứng minh rằng với mọi bộ số dương a, b, c, ta đều có

a b c 3
+ + ≥ . (2.1)
b+c c+a a+b 2

Đây là bài toán cơ bản (có trong tất cả các sách giáo trình về bất đẳng thức) nhằm để
mô tả các ứng dụng khác nhau của các bất đẳng thức cổ điển như bất đẳng thức Cauchy,
bất đẳng thức AG, . . . để giải. Tuy nhiên, nếu viết lại (2.1) dưới dạng

a1 b1 c1 a0 b0 c0
+ + ≥ + + . (2.2)
b1 + c1 c1 + a1 a1 + b1 b0 + c0 c0 + a0 a0 + b0

với ngầm định a0 = b0 = c0 = 1, thì ta có ngay nhận xét rằng (2.2) có dáng dấp của một
hàm đồng biến
g (1) ≥ g (0),
với
at bt ct
g(t) = + + , t ∈ R.
bt + ct ct + at at + bt
Ta chứng minh rằng nhận xét vừa nêu ở trên là hoàn toàn đúng. Tuy nhiên (bạn đọc
hãy tự kiểm chứng), các kỹ thuật cơ bản của bất đẳng thức Cauchy, bất đẳng thức AG
không còn hiệu lực. Tính đồng biến của g(t) ứng với t ≥ 0 được suy ra từ nhận xét sau
đây (xem lại tính chất phân số ở bậc tiểu học).

Tính chất 1.
(i) Nếu hai phân số dương có cùng tử số dương thì phân số nào có mẫu số lớn hơn
thì bé hơn,
(ii) Nếu hai phân số âm có cùng tử số dương thì phân số nào có mẫu số lớn hơn thì
lớn hơn.

88
Hội thảo khoa học, Ninh Bình 15-16/09/2018
p
Tính chất 2. Xét phân số với q > 0. Khi đó
q
p
(i) Nếu phân số dương thì khi tăng mẫu số, phân số sẽ giảm,
q
p
(ii) Nếu phân số âm thì khi tăng mẫu số, phân số sẽ tăng.
q
Nói cách khác, ta có.
p
Bài toán 8. Cho phân số với q > 0 và số dương d. Khi đó
q
p p p
(i) Nếu phân số dương thì ≥ ,
q q q+d
p p p
(ii) Nếu phân số âm thì ≤ .
q q q+d
Từ kết quả của bài toán này, ta dễ dàng chứng minh.
Bài toán 9. Với mọi bộ số dương a, b, c cho trước, hàm số
at bt ct
g(t) = + + , t ∈ R,
bt + ct ct + at at + bt
là một hàm đồng biến trong [0, +∞).
Hệ quả 4. Cho α ≥ β ≥ 0. Chứng minh rằng với mọi bộ số dương a, b, c, ta đều có
aα bα cα aβ bβ cβ
+ + ≥ + + .
bα + cα cα + aα aα + bα bβ + cβ cβ + aβ aβ + bβ
Bài toán 10. Chứng minh rằng với mọi bộ số a1 , a2 , · · · , an ∈ R ta luôn có đa thức
n n ai a j
Q( x ) = ∑ ∑ i + j xi+ j ,
i =1 j =1

là một hàm đồng biến trong [0, +∞).


Lời giải. Thật vậy, ta có
n n
1 1 n 2
Q0 ( x ) =
x ∑ ∑ ai a j x i + j = x i∑
a i x i
.
i =1 j =1 =1

Suy ra Q0 ( x ) ≥ 0 với mọi x ≥ 0. Do đó, hàm số Q( x ) đồng biến trong [0, +∞).
Từ đây, ta thu được
Hệ quả 5 (Bất đẳng thức thứ tự Hilbert). Với mọi bộ số thực a1 , a2 , · · · , an , ta luôn có
n n ai a j
∑ ∑ i + j ≥ 0.
i =1 j =1

Chứng minh. Do đa thức


n n ai a j
Q( x ) = ∑ ∑ i + j xi+ j ,
i =1 j =1

đồng biến trong [0, +∞) nên Q(1) ≥ Q(0) = 0, hay


n n ai a j
∑ ∑ i + j = Q(1) ≥ 0.
i =1 j =1

chính là đpcm.

89
Hội thảo khoa học, Ninh Bình 15-16/09/2018

2.2 Một số bài toán áp dụng cho bất đẳng thức trong tam giác
Sau đây là một số bài toán sử dụng các kiến thức của lớp hàm đơn điệu liên tiếp trên
một đoạn, đối với tổng không đổi ta xét dấu đạo hàm các cấp một và hai trong việc biến
đổi chia hai vế.
Bài toán 11. Cho tam giác nhọn A0 B0 C0 . Chứng minh rằng với mọi tam giác nhọn ABC
ta đều có
sin A sin B sin C
+ + 6 tan A0 tan B0 tan C0 . (2.3)
cos A0 cos B0 cos C0
Lời giải. Xét hàm số  π
f ( x ) = sin x, x ∈ 0, .
2
π
Ta có f 0 ( x ) = cos x > 0 và f 00 ( x ) = − sin x < 0 trong (0, ).
2
Vậy nên f ( A) ≤ f ( A0 ) + f 0 ( A0 )( A − A0 ),
suy ra
sin A
≤ tan A0 + A − A0 . (2.4)
cos A0
Tương tự, ta cũng có
sin B
≤ tan B0 + B − B0 (2.5)
cos B0

sin C
≤ tan C0 + C − C0 . (2.6)
cos C0
Từ (2.4), (2.5), (2.6), suy ra
sin A sin B sin C
+ + ≤ tan A0 + tan B0 + tan C0 .
cos A0 cos B0 cos C0
Để ý rằng
tan A0 + tan B0 + tan C0 = tan A0 tan B0 tan C0 ,
suy ra điều phải chứng minh.
Các bài toán sau được giải quyết tương tự.
Bài toán 12. Cho tam giác A0 B0 C0 . Chứng minh rằng với mọi tam giác ABC ta đều có
A B C
cos cos cos
2 + 2 + 2 6 cot A0 + cot B0 + cot C0 . (2.7)
A0 B0 C0 2 2 2
sin sin sin
2 2 2
Bài toán 13. Cho tam giác nhọn A0 B0 C0 . Chứng minh rằng với mọi tam giác nhọn ABC
ta đều có
tan A tan B tan C
+ + > 2 sin A0 sin B0 sin C0 . (2.8)
1 + tan2 A0 1 + tan2 B0 1 + tan2 C0

Bài toán 14. Cho tam giác ABC có một góc không nhỏ hơn . Chứng minh rằng
3
A B C √
tan + tan + tan ≥ 4 − 3.
2 2 2

90
Hội thảo khoa học, Ninh Bình 15-16/09/2018

Các bài toán sau đây cho ta mở rộng vô hạn các bất đẳng thức từ một bất đẳng thức
nhờ lớp các hàm số đơn điệu.
Ta nhận thấy rằng ứng với a, b, c, α, β là các số dương, α > β thì ta luôn có
 a α  b α  c α  a β  b β  c β
+ + ≥ + +
b c a b c a
và đẳng thức xảy ra khi và chỉ khi a = b = c. Tính chất này có thể nhìn nhận như là hàm
số  a x  b x  c x
g( x ) := + +
b c a
đồng biến trong [0. + ∞).
Vậy câu hỏi tự nhiên nảy sinh là ta có thể thiết lập được hay không các hàm tương
tự đối với các bất đẳng thức dạng khác khi đã tường minh cách chứng minh cho trường
hợp đơn lẻ và cụ thể? Ta thu được bài toán nội suy bất đẳng thức, tức là từ bất đẳng thức
đã cho, ta xét hai vế của nó như là giá trị của một hàm cần tìm tại hai tọa vị cho trước.
Bài toán 15. Cho a, b, c, α, β là các số dương, α > β. Chứng minh rằng
 b + c α  c + a α  a + b α  b + c β  c + a β  a + b β
+ + ≥ + + . (2.9)
2a 2b 2c 2a 2b 2c
Lời giải. Xét hàm số:
 b + c t  c + a t  a + b t
F (t) = + + , ∀t > 0.
2a 2b 2c
Ta cần chứng minh F (t) là hàm số đồng biến trên (0, +∞) hay ∀t1 , t2 ∈ (0, +∞), t1 < t2 ,
ta cần chứng minh F (t1 ) ≤ F (t2 ) hay cần chứng minh:
 b + c  t2  c + a  t2  a + b  t2  b + c  t1  c + a  t1  a + b  t1
+ + ≥ + +
2a 2b 2c 2a 2b 2c
Ta có  b + c  t2
t2 t 2  b + c  t1
−1 ≥ +
2a t1 t1 2a
 c + a  t2 t t 2  c + a  t1
2
+ −1 ≥
2b t1 t1 2b
 a + b  t2 t t 2  a + b  t1
2
+ −1 ≥
2c t1 t1 2c
r
t h b + c t1  c + a t1  a + b t1 i  t  h (b + c)(c + a)( a + b) it1 t2
2 2 3
−1 + + ≥ −1 3 ≥ 3 − 3.
t1 2a 2b 2c t1 8abc t1
Cộng theo vế 4 bất đẳng thức trên ta thu được
 b + c  t2  c + a  t2  a + b  t2  b + c  t1  c + a  t1  a + b  t1
+ + ≥ + + , ∀t2 > t1 > 0.
2a 2b 2c 2a 2b 2c
Suy ra F (t) là hàm đồng biến trên (0, +∞).
Khi đó ∀α > β ta luôn có
 b + c α  c + a α  a + b α  b + c β  c + a β  a + b β
+ + ≥ + + .
2a 2b 2c 2a 2b 2c

91
Hội thảo khoa học, Ninh Bình 15-16/09/2018

Từ đây, ta thu được điều phải chứng minh.


Đẳng thức xảy ra khi và chỉ khi a = b = c.
 a t  b t  c t
Với hàm g(t) := + đồng biến trong [0. + ∞) cho ta vô hạn các bất
+
b c a
đẳng thức, trong đó (2.9) là một hệ quả của nó, từ đó ta mở rộng được vô hạn các bài
toán, chẳng hạn các bài toán sau.
Bài toán 16. Cho a, b, c là ba cạnh của tam giác và α > β ≥ 1.
Chứng minh rằng
 c α  a α  b α  c β  a β  b β
+ + ≥ + + .
a+b−c b+c−a c+a−b a+b−c b+c−a c+a−b
(2.10)

Bài toán 17. Cho a, b, c là ba cạnh của một tam giác và α > β > 1. Chứng minh rằng
 3a α  3b α  3c α
+ +
2b + 2c − a 2c + 2a − b 2a + 2b − c
 3a β  3b β  3c β
≥ + + .
2b + 2c − a 2c + 2a − b 2a + 2b − c

2.3 Sử dụng tính đơn điệu của hàm số trong bài toán cực trị
Bài toán 18 (Mỹ, 1977). Chứng minh rằng với các số dương tùy ý a ≤ b ≤ c ≤ d, ta có
bất đẳng thức
ab bc cd d a ≥ b a cb dc ad .
Lời giải. Ta đặt b = ax, c = ay, d = az.
Khi đó, từ điều kiện của bài toán, ta có 1 ≤ x ≤ y ≤ z, còn bất đẳng thức cần chứng
minh tương đương với

a ax ( ax ) ay ( ay) az ( az) a ≥ ( ax ) a ( ay) ax ( az) ay a az .

Sau khi giản ước hai vế cho a a a ax a ay a az và nâng lũy thừa hai vế (đều dương) lên lũy thừa
1
ta được bất đẳng thức tương đương
a
x y yz z ≥ xy x zy .

Đặt x = xs, z = xt, khi đó, 1 ≤ s ≤ t (vì x ≤ y ≤ z) và y ≥ s (vì x ≥ 1), suy ra

x xs y xt xt ≥ xy x ( xt)sx .
1
Sau khi giản ước x xs y x xt và nâng lũy thừa hai vế (đều dương) lên lũy thừa ta được bất
x
đẳng thức tương đương
1
y t −1 ≥ t s − x .
- Nếu y = 1 thì bất đẳng thức đúng vì
y
x = 1, y = = 1, yt−1 = 1 = tt−1 = 1.
x

92
Hội thảo khoa học, Ninh Bình 15-16/09/2018

- Nếu t = 1 thì bất đẳng thức đúng vì


1
yt−1 = y0 = 1 = 1s− x = 1.

Giả sử y > 1 và t > 1. Khi đó, nâng lũy thừa hai vế của bất đẳng thức lên lũy thừa
1 y
> 0, ta được bất đẳng thức tương đương
t−1y−1
y s
y y−1 ≥ t t−1 với 1 ≤ s ≤ t, s ≤ y.

Ta xét hai trường hợp:


Trường hợp 1:
α
Giả sử y ≥ t. Ta chứng minh rằng hàm số f 1 (α) = α α−1 tăng với α > 1. Ta có
 α 0  1 ln α 
f 100 (α) = e α−1 ln α = e α−1 ln α
α

α − 1 ( α − 1)2
α 1
= e α −1 (α − 1 − ln α) > 0,
( α − 1)2
1
ta có g1 (α) = α − 1 − ln α > 0 với α > 1, vì g10 (α) = α −
> 0 và ( g10 (0) = 0).
α
Suy ra trong trường hợp này bất đẳng thức thỏa mãn, vì
y t s
y y −1 = f 1 ( y ) ≥ f 1 ( t ) = t t −1 ≥ t t −1 .

Trường hợp 2:
1
Giả sử y < t. Ta chứng minh rằng hàm số f 2 (α) = α α−1 giảm với α > 1. Ta có
 α 0  1 ln α 
f 200 (α) = e α−1 ln α = e α−1 ln α
α

α ( α − 1) ( α − 1)2
1 1
= e α −1 (α − 1 − α ln α) < 0,
α ( α − 1)2
ta có g2 (α) = α − 1 − α ln α < 0 với α > 1, vì g20 (α) = 1 − ln α − 1 < 0 và ( g20 (1) = 0).
Suy ra trong trường hợp này bất đẳng thức thỏa mãn, vì
y y s
y y−1 = ( f 2 (y))y ≥ ( f 2 (t))y = t t−1 ≥ t t−1 .

Bài toán 19 (Ireland, 2000). Cho x, y ≥ 0 với x + y = 2. Chứng minh rằng

x2 y2 ( x2 + y2 ) ≤ 2.
 x + y 6
Lời giải. Do x + y = 2 nên = 1, bất đẳng thức cần chứng minh được viết
2
thành  x + y 6
2 ≥ x 2 y2 ( x 2 + y2 ),
2
hay
( x + y)6 ≥ x2 y2 32( x2 + y2 ).
Trong trường hợp xy = 0 bất đẳng thức hiển nhiên đúng.

93
Hội thảo khoa học, Ninh Bình 15-16/09/2018
√ √
Xét trường hợp xy 6= 0. Đặt k = xy. Ta có điều kiện x = y = 2 ≥ 2 xy = 2 k, suy
ra 0 ≤ k ≤ 1. Ta có thể chứng minh bất đẳng thức khi k = 1, nghĩa là xy = 1 lúc đó bất
đẳng thức trở thành
 1 6  1
x+ ≥ 32 x2 + 2 .
x x
 1  2
Đặt p = x + ≥ 4. Ta cần tìm giá trị nhỏ nhất của F ( p) = p3 − 32( p − 2) trên
x
[4, +∞].
Vì F 0 ( p) = 3p2 − 32 ≥ 0 (do p ≥ 4), nên F ( p) là hàm đơn điệu tăng trên [4, +∞].
Vậy F ( p) ≥ F (4) = 0 với ∀ p ≥ 4 (đpcm).

Ta sử dụng định lý và hệ quả sau cho các bài toán dưới đây.
Định lý 8 (xem [2-6]). Cho hàm số y = f ( x ) liên tục và nghịch biến trên [0, b],
∀ a ∈ [0, b]. Khi đó, ta luôn có
Z a Z b
b f ( x )dx ≥ a f ( x )dx. (2.11)
0 0

Tương tự, với f ( x ) liên tục và đồng biến trên [0, b], ∀ a ∈ [0, b] thì
Z a Z b
b f ( x )dx ≤ a f ( x )dx.
0 0

Hệ quả 6. Nếu b = 1 và f ( x ) liên tục và nghịch biến trên [0, 1] thì ∀ a ∈ [0, 1], ta đều có
Z a Z 1
f ( x )dx ≥ a f ( x )dx.
0 0

Nếu b = 1, f ( x ) liên tục và đồng biến trên [0, 1] thì ∀ a ∈ [0, 1], ta đều có
Z a Z 1
f ( x )dx ≤ a f ( x )dx.
0 0

Bài toán 20. Tìm giá trị lớn nhất của hàm số

π 2  π2  π h πi
h( x ) = x − + 1 x − cos x, x ∈ 0, .
4 8 2 2
h πi
Lời giải. Ta có f (t) = t + sin t là hàm liên tục và đồng biến trên 0, . Khi đó, theo
h πi 2
Định lý 8, ∀ x ∈ 0, , thì
2
Z x Z π
π 2
(t + sin t)dt ≤ x (t + sin t)dt,
2 0 0

hay
π  t2  x  t2  π2
− cos t ≤ x
− cos t .
2 2 0 2 0
Vậy nên
π h x2 i  π2 
− cos x + 1 ≤ x +1 ,
2 2 8

94
Hội thảo khoa học, Ninh Bình 15-16/09/2018

hay
πx2 π π xπ 2
− cos x + ≤ + x.
4 2 2 8
Suy ra
πx2 π  π2  π
− cos x − x +1 ≤ − .
4 2 8 2
π π
Vậy, giá trị lớn nhất của hàm số h( x ) bằng − khi x = 0 hoặc x = .
2 2
Bài toán 21. Tìm giá trị nhỏ nhất của hàm số
√ i
√ h 2
f ( x ) = πx − 2 2 arcsin x, x ∈ 0, .
2
1
Lược đồ giải. Tương tự áp dụng Định lý 8 cho hàm g(t) = √ là hàm liên tục và
√ 1 − t2
h 2i
đồng biến trên 0, ta được giá trị nhỏ nhất của hàm số f ( x ) bằng 0 khi x = 0 hoặc
√ 2
2
x= .
2
Bài toán 22. Tìm giá trị lớn nhất của hàm số
√ √  √ 
f (x) = 3x3 − 3 3 arctan x + x π − 3 3

trên đoạn [0, 3].

1
Lược đồ giải. Tương tự áp dụng Định lý 8 cho hàm g(t) = t2 − là hàm liên tục
√ 1 + t2
và đồng
√ biến trên [0, 3] ta được giá trị lớn nhất của hàm số f ( x ) bằng 0 khi x = 0 hoặc
x = 3.

Bài toán 23. Tìm giá trị nhỏ nhất của hàm số
p 2 √ 1
f ( x ) = x arccos x − 1 − x2 − x x − x, x ∈ [0, 1].
3 3

Lược đồ giải.Tương tự áp dụng Định lý 8 cho hàm g(t) = t − arccos t là hàm liên tục
và đồng biến trên [0, 1] ta được giá trị nhỏ nhất của hàm số f ( x ) bằng −1 khi x = 0 hoặc
x = 1.

Bài toán 24. Tìm giá trị lớn nhất của biểu thức

1
f ( x, y) = x cos y − y cos x + ( x − y)( xy − 1), 0 ≤ x ≤ y.
2

Lược đồ giải. Tương tự áp dụng Định lý 8 cho hàm g(t) = sin t + t là hàm số liên tục và
đồng biến với mọi t ∈ [0, y] ta được giá trị lớn nhất của hàm số f ( x, y) bằng 0 khi x = 0
hoặc x = y.

95
Hội thảo khoa học, Ninh Bình 15-16/09/2018

Bài toán 25. Tìm giá trị lớn nhất của biểu thức

1 1 1 1 1 1  πi
A= + + − − − ; x, y, z ∈ 0, .
sin2 x sin2 y sin2 z x2 y2 z2 2

π
Lời giải. Từ bất đẳng thức cos t ≤ 1, ∀t ∈ R và cos t < 1 với 0 < t ≤ , ta có
2
Z x Z x
cos tdt < dt ⇒ sin x < x. (2.12)
0 0

Tiếp theo, từ (2.12), khi x > 0, thì

x2
Z x Z x
sin tdt < tdt ⇒ cos x > 1 − . (2.13)
0 0 2

Tiếp theo, từ (2.13), khi x > 0, thì

t2  x3
Z x Z x
cos tdt > 1− dt ⇒ sin x > x − . (2.14)
0 0 2 3!

Tiếp theo, từ (2.14), khi x > 0, thì

t3  x2 x4
Z x Z x
sin tdt > t− dt ⇒ cos x < 1 − + .
0 0 3! 2! 4!
Mặt khác, từ (2.14), ta có
 sin x 3  x 2 3 x2 x4 x6
> 1− = 1− + − ,
x 6 2 12 216

x2 x4 x6 x2 x4
1− + − > 1− + ,
2 12 216 2 24
nên  sin x 3
> cos x,
x
hay
1 cos x  πi
> , x ∈ 0, .
x3 sin3 x 2
Do vậy
π π
1 cos t
Z Z
2 2
dt ≥ dt.
x t3 x sin3 t
Suy ra
1 1 4  πi
− ≤ 1 − , x ∈ 0, .
sin2 x x2 π2 2
Từ đây, ta có
1 1 1 1 1 1 12
2
+ 2
+ 2
− 2 − 2 − 2 ≤ 3 − 2.
sin x sin y sin z x y z π
12 π
Vậy, giá trị lớn nhất của A bằng 3 − khi x = y = z = .
π2 2

96
Hội thảo khoa học, Ninh Bình 15-16/09/2018

Bài toán 26. Cho hàm số f ( x ) khả vi hai lần liên tiếp trên I ( a, b), g( x ) = ln f ( x ). Chứng
minh rằng
n
f 0 (uk ) ln f ( xk ) n
f 0 (uk ) ln f (uk )
min ∑ =∑ ⇔ f 0 ( x )[ f ( x ) f 00 ( x ) − f 0 ( x )2 ] ≥ 0.
k =1
f (uk ) k =1
f ( u k )

Với ∀ x ∈ I ( a, b), ứng với mỗi dãy số { xk } với xk ∈ I ( a, b); k = 1, 2, . . . , n có tổng

x1 + x2 + · · · + x n ≥ u1 + u2 + · · · + u n .

Chứng minh. Ta có

g( x ) = ln f ( x );
f 0 (x)
g0 ( x ) = ;
f (x)
f 00 ( x ). f ( x ) − f 0 ( x )2
g00 ( x ) = ,
f 0 ( x )2

f ( x ) > 0, ∀ x ∈ I ( a, b), suy ra g0 ( x ) > 0 ⇔ f 0 ( x ) > 0. Khi đó

g00 ( x ) > 0 ⇒ f 00 ( x ). f ( x ) − f 0 ( x )2 > 0.

Vậy hàm số g( x ) đồng biến liên tiếp bậc (1 − 2) khi

f 0 ( x )[ f ( x ) f 00 ( x ) − f 0 ( x )2 ] > 0.

Nên theo Hệ quả 2, ta có


n
f 0 (uk ) ln f ( xk ) n
f 0 (uk ) ln f (uk )
min ∑ =∑ ⇔ f 0 ( x )[ f ( x ) f 00 ( x ) − f 0 ( x )2 ] ≥ 0.
k =1
f (uk ) k =1
f ( u k )

Với ∀ x ∈ I ( a, b), ứng với mỗi dãy số { xk } với xk ∈ I ( a, b); k = 1, 2, . . . , n có tổng

x1 + x2 + · · · + x n ≥ u1 + u2 + · · · + u n .

Bài toán 27. Cho f ( x ) = ax3 + bx2 + cx + d, ( a 6= 0), thỏa mãn điều kiện b2 − 3ac > 0.
Chứng minh rằng
n axk3 + bxk2 + cxk + d n au3 + bu2 + cu + d
min ∑ ∑ 3au k k k
= ,
k =1 3au2k + 2buk + c k =1
2 + 2bu + c
k k

 b
∀ x ∈ x1 , − ∪ ( x2 , +∞), trong đó x1 , x2 là nghiệm của phương trình f 0 ( x ) = 0, ( x1 <
3a
x2 ) ứng với mọi dãy số { xk } với xk ∈ I ( a, b); k = 1, 2, . . . , n có tổng

x1 + x2 + · · · + x n ≥ u1 + u2 + · · · + u n .

97
Hội thảo khoa học, Ninh Bình 15-16/09/2018

Lời giải. Ta có
f ( x ) = ax3 + bx2 + cx + d, ( a 6= 0);
f 0 ( x ) = 3ax2 + 2bx + c;
f 00 ( x ) = 6ax + 2b.
b
Vì ∆0f 0 = b2 − 3ac > 0 nên phương trình f 0 ( x ) = 0 có hai nghiệm x1 , x2 ( x1 < − < x2 ).
3a
Trường hợp 1: Nếu a > 0. Ta có f 0 ( x ) > 0 ⇔ x < x1 hoặc x > x2 , suy ra
−b
f 00 ( x ) > 0 ⇔ x > ,
3a
nên
f 0 ( x ) > 0 và f 00 ( x ) > 0, ∀ x > x2 , suy ra f ( x ) đồng biến liên tiếp bậc (1 − 2).
Vì f 0 ( x ) < 0 ⇔ x1 < x < x2 , nên
b
f 00 ( x ) < 0 ⇔ x < − ,
3a
b
do đó f 0 ( x ) < 0 và f 00 ( x ) < 0, ∀ x1 < x < − , suy ra f ( x ) nghịch biến liên tiếp bậc
3a
(1 − 2).
Trường hợp 2: Nếu a < 0. Ta có f 0 ( x ) > 0 ⇔ x1 < x < x2 nên
−b
f 00 ( x ) > 0 ⇔ x < ,
3a
−b
do đó f 0 ( x ) > 0 và f 00 ( x ) > 0, ∀ x, x1 < x < , suy ra f ( x ) đồng biến liên tiếp bậc
3a
(1 − 2). Vì f 0 ( x ) < 0 ⇔ x < x1 hoặc x > x2 nên
−b
f 00 ( x ) < 0 ⇔ x > ,
3a
do đó f 0 ( x ) < 0 và f 00 ( x ) < 0, ∀ x > x2 , suy ra f ( x ) nghịch biến liên tiếp bậc (1 − 2).
 b
Do đó, với ∀ a 6= 0, ∀ x ∈ x1 , − ∪ ( x2 , +∞) hàm số f ( x ) đồng biến hoặc nghịch
3a
biến liên tiếp bậc (1 − 2), nên theo Hệ quả 2, ta có
n axk3 + bxk2 + cxk + d n au3 + bu2 + cu + d
min ∑ ∑ 3au k k k
=
k =1 3au2k + 2buk + c k =1
2 + 2bu + c
k k
 b
∀ x ∈ x1 , − ∪ ( x2 , +∞), trong đó x1 , x2 là nghiệm của phương trình f 0 ( x ) = 0 ( x1 <
3a
x2 ). ứng với mọi dãy số { xk } với xk ∈ I ( a, b); k = 1, 2, . . . , n có tổng x1 + x2 + · · · + xn ≥
u1 + u2 + · · · + u n .
Bài toán 28. Cho f ( x ) = ax3 + bx2 + cx + d, ( a 6= 0), thỏa mãn điều kiện b2 − 3ac > 0.
Chứng minh rằng
n axk3 + bxk2 + cxk + d n au3 + bu2 + cu + d
max ∑ ∑ 3au k k k
2 + 2bu + c
= 2 + 2bu + c
,
k =1 3au k k k =1 k k
 b 
∀ x ∈ (−∞, x1 ) ∪ − , x2 , trong đó x1 , x2 là nghiệm của phương trình f 0 ( x ) =
3a
0, ( x1 < x2 ), ứng với mọi dãy số { xk } với xk ∈ I ( a, b); k = 1, 2, . . . , n có tổng

98
Hội thảo khoa học, Ninh Bình 15-16/09/2018

x1 + x2 + · · · + x n ≤ u1 + u2 + · · · + u n .

Lời giải. Ta có

f ( x ) = ax3 + bx2 + cx + d, ( a 6= 0);


f 0 ( x ) = 3ax2 + 2bx + c;
f 00 ( x ) = 6ax + 2b.

b
Vì ∆0f 0 = b2 − 3ac > 0 nên phương trình f 0 ( x ) = 0 có hai nghiệm x1 , x2 ( x1 < − < x2 ).
3a
Trường hợp 1: Nếu a > 0. Ta có f 0 ( x ) > 0 ⇔ x < x1 hoặc x > x2 thì

−b
f 00 ( x ) < 0 ⇔ x < ,
3a
suy ra f 0 ( x ) > 0 và f 00 ( x ) < 0, ∀ x < x1 , suy ra f ( x ) lõm, có đạo hàm bậc nhất là số
b
dương. Vì f 0 ( x ) < 0 ⇔ x1 < x < x2 , suy ra f 00 ( x ) > 0 ⇔ x > − suy ra f 0 ( x ) < 0 và
3a
b
f 00 ( x ) > 0, ∀ x, − < x < x2 , suy ra f ( x ) lồi có đạo hàm bậc nhất là những số dương.
3a
Trường hợp 2: Nếu a < 0. Ta có f 0 ( x ) > 0 ⇔ x1 < x < x2 nên

−b
f 00 ( x ) < 0 ⇔ x > ,
3a
suy ra f 0 ( x ) > 0 và
−b
f 00 ( x ) < 0, ∀ x, < x < x2 ,
3a
suy ra f ( x ) lõm có đạo hàm bậc nhất là những số dương.
−b
Vì f 0 ( x ) < 0 ⇔ x < x1 hoặc x > x2 , suy ra f 00 ( x ) > 0 ⇔ x < , nên f 0 ( x ) < 0 và
3a
f 00 ( x ) > 0, ∀ x < x1 , suy ra f ( x ) lồi và có đạo hàm bậc nhất là những số âm.
 b 
Do đó, với ∀ a 6= 0, ∀ x ∈ (−∞, x1 ) ∪ − , x2 hàm số f ( x ) lồi (lõm) có đạo hàm
3a
bậc nhất là những số âm (dương), nên theo Hệ quả 3, ta có
n axk3 + bxk2 + cxk + d n au3 + bu2 + cu + d
max ∑ ∑ 3au k k k
= ,
k =1 3au2k + 2buk + c k =1
2 + 2bu + c
k k

 b 
∀ x ∈ (−∞, x1 ) ∪ − , x2 . Trong đó x1 , x2 là nghiệm của phương trình f 0 ( x ) = 0 ( x1 <
3a
x2 ), ứng với mọi dãy số { xk } với xk ∈ I ( a, b); k = 1, 2, . . . , n có tổng

x1 + x2 + · · · + x n ≤ u1 + u2 + · · · + u n .
b
Bài toán 29. Cho hàm số f ( x ) = ln( ax + b), ( a < 0), ∀ x < − . Chứng minh rằng
a
n n
( auk + b) ln( axk + b) ( auk + b) ln( auk + b)
min ∑ =∑ ,
k =1
a k =1
a

ứng với mọi dãy số { xk } với xk ∈ I ( a, b); k = 1, 2, . . . , n có tổng

99
Hội thảo khoa học, Ninh Bình 15-16/09/2018

x1 + x2 + · · · + x n ≥ u1 + u2 + · · · + u n .

Lời giải. Ta có

f ( x ) = ln( ax + b);
a
f 0 (x) = ;
ax + b
a2
f 00 ( x ) = − .
( ax + b)2
Do a < 0 nên
a b
f 0 (x) = < 0, ∀ x < − ,
ax + b a

a2 b
f 00 ( x ) = − 2
< 0, ∀ x < − .
( ax + b) a
Vậy nên, hàm f ( x ) nghịch biến liên tiếp bậc (1 − 2), theo Hệ quả 2, ta có
n n
( auk + b) ln( axk + b) ( auk + b) ln( auk + b)
min ∑ =∑ .
k =1
a k =1
a

ứng với mọi dãy số { xk } với xk ∈ I ( a, b); k = 1, 2, . . . , n có tổng

x1 + x2 + · · · + x n ≥ u1 + u2 + · · · + u n .

Nhận xét rằng, nếu hàm số đã cho là hàm đơn điệu và khả vi thì ta nhận được nhiều
áp dụng trong những lớp hàm khác nhau như hàm lượng giác, hàm đa thức, hàm phân
thức, hàm mũ và hàm logarit,...

Tài liệu
[1] Lê Hải Châu, Các bài thi Olympic Toán trung học phổ thông Việt Nam (1990-2006), NXB
Giáo dục, 2006.
[2] Nguyễn Văn Mậu, Bất đẳng thức, định lí và áp dụng, NXB Giáo dục, 2006.
[3] Nguyễn Văn Mậu, Đa thức đại số và phân thức hữu tỷ, NXB Giáo dục, 2007.
[4] Nguyễn Văn Mậu, Phạm Thị Bạch Ngọc, Một số bài toán chọn lọc về lượng giác, NXB
Giáo dục, 2003.
[5] Nguyễn Văn Mậu (Chủ biên), Trịnh Đào Chiến, Trần Nam Dũng, Nguyễn Đăng
Phất, Chuyên đề chọn lọc về đa thức và áp dụng, NXB Giáo dục, 2008.
[6] Nguyễn Văn Mậu (Chủ biên), Nguyễn Văn Tiến, Một số chuyên đề giải tích bồi dưỡng
học sinh giỏi trung học phổ thông, NXB Giáo dục Việt Nam, 2010.
[7] Paulo Ney de Sausa, Jorge- Nume Silva (1998), Berkeley Problems in Mathematics,
Springer.
[8] T-L.T. Radulescu, V.D. Radulescu, T.Andreescu (2009), Problems in Real Analysis: Ad-
vanced Calculus on the real axis, Springer Sciences+Business Media.

100
Hội thảo khoa học, Ninh Bình 15-16/09/2018

ỨNG DỤNG PHÉP NGHỊCH ĐẢO ĐỂ GIẢI TOÁN


Phạm Văn Khanh
Trường THPT Chuyên Lương Văn Tụy, Ninh Bình

Tóm tắt nội dung

Phép nghịch đảo là một công cụ giúp giải quyết tốt một lớp bài toán hình học, cho lời
giải hay và thú vị. Bài viết về ứng dụng của một phép biến hình là tích của phép đối xứng
trục và phép nghịch đảo áp dụng trong tam giác, vận dụng kiến thức cơ bản về phép dời
hình, đồng dạng trong chương trình THPT và phép nghịch đảo ở lớp 10 chuyên.

1 Mô hình áp dụng

Cho 4 ABC độ dài cạnh là a, b, c nội tiếp đường tròn (O), xét các phân giác la , lb , lc
và ánh xạ tích của √
phép đối xứng trục Dla và phép nghịch đảo cực A, tỷ số nghịch đảo

r = bc√kí hiệu NA bc , với điểm X thuộc mặt phẳng tam giác thì X 7→ √
X 0 = Dl a ( X ) 7 →
X 00 = NA bc ( X 0 ). Phép biến hình tích như trên ta kí hiệu là f A = NA bc · Dla , f A ( X ) =
X 00 , ∀ X 6= A. Trong mf ( P) mở rộng đã bổ sung điểm vô cực ∞ và cho A 7→ ∞,
∞ 7→ A, f A xác định như trên là√một song ánh của √mf ( P). Dễ thấy tích ánh xạ này
giao hoán được. Ta gọi f A = NA bc · Dla = Dla · NA bc ngắn gọn là phép biến hình

bc-nghịch đảo cực A.

ac
Định nghĩa tương tự cho các phép biến hình tích: f B = NB · Dl b ,

f C = NC ab · Dlc .

2 Nội dung

Tính chất 1. Gọi đường tròn ngoại tiếp 4 ABC là ω, xét phép biến hình f A = NA bc · Dla
trong mặt phẳng chứa tam giác, ta có:
a) B 7→ C, C 7→ B;
b) ω 7→ BC, BC 7→ ω;

101
Hội thảo khoa học, Ninh Bình 15-16/09/2018

c) Với X 6= A, X 7→ X 0 , X 0 7→ X thì các đường thẳng AX và AX 0 là các đường đẳng


giác.

Chứng minh. a) Ta có Dla ( B) = B1 ∈ AC, AB1 = AB = c, NA bc ( B1 ) = B0 : AB1 ·
AB0 = bc, AB0 = c, B ≡ C, B 7→ C.
Tương tự ảnh của C là B.

X
B C

X0
la

b) Do B, C là ảnh của nhau và đường tròn ( ABC ) có ảnh là đường thẳng qua BC hay
chính là đt( BC ), và ngược lại ảnh của đường thẳng BC là đường tròn ( ABC ).
c) Điểm X thuộc đường thẳng BC và giả sử XAB [ = α qua phép đối xứng trục la thì
AB → AC, AX → At, XAB [ → tAC, d X 7→ X 0 ∈ AT : X \0 AC 7 → XAB
[ = α; hay các
đường thẳng AX, AX 0 là các đường đẳng giác tại đỉnh A.

Tính chất 2. Tam giác ABC có AA1 , BB1 , CC1 đồng quy tại điểm P trong tam giác. Chứng
minh 3 đường đẳng giác tương ứng là AA2 , BB2 , CC2 cũng đồng quy tại điểm Q.
(Hai điểm P, Q gọi là 2 điểm liên hợp đẳng giác).

Chứng minh. Ta có AA1 , BB1 , CC1 đồng quy tại điểm P trong tam giác nên theo Định
lý Xêva dạng lượng giác ta có

sin CAA
\1 sin \ ABB1 sin \ BCC1
· · = 1. (1)
sin A
\ 1 AB sin B
[1 BC sin C
\1 CA

Gọi Q là giao điểm của [ d2 . Nối CQ cắt AB tại C 0 , ta chứng minh CC2 ≡ CC1 .
AA2 và BB 2
Đặt \
BCC1 = γ = C 0 CA = γ0 , A = = α. Suy ra A
CA, C AB A AC 1 AC = A2 AB =
\2
\ 1
\ 2
\ \
2
B − α và B[1 BC = A2 BA = β. Do đó ABB1 = B2 BC = B − β. Do đó kết hợp với (1) ta
\ \ [

102
Hội thảo khoa học, Ninh Bình 15-16/09/2018


sin A
\ 2 AB sin CBB2
[ sin γ
· · = 1. (2)
sin A
\ 2 AC sin B
\2 BA sin ( C − γ )
Do AA2 , BB2 , CC20 đồng quy tại điểm Q nên theo Định lý Xêva dạng lượng giác ta có

sin A
\ 2 AB sin CBB2
[ sin γ0
· · 0)
= 1. (3)
sin A
\ 2 AC sin B
\2 BA sin ( C − γ

Từ (2) và (3) suy ra


sin γ sin γ0
= ,
sin (C − γ) sin (C − γ0 )
hay sin γ · sin (C − γ0 ) = sin γ0 · sin (C − γ), do đó sin (γ − γ0 ) = 0, nên γ = γ0 . Suy
ra C20 CA = γ. Do đó CC20 ≡ CC2 .

Dưới đây xét tam giác ABC. Gọi ω là đường tròn ngoại tiếp tam giác; ωa là đường
tròn Mixtilinear nội tiếp của tam giác ABC tương ứng với góc A, đường tròn này tiếp
xúc trong với cạnh AB, AC và đường tròn ngoại tiếp tam giác ABC. Định nghĩa tương
tự các đường tròn ωb , ωc . Gọi ωa0 , ωb0 , ωc0 tương ứng là đường tròn bàng tiếp các góc
A, B, C của tam giác ABC.

Bài toán 1. Cho tam giác ABC có ω và ωa tiếp xúc nhau tại T, ωa0 và BC tiếp xúc
nhau tại D. Chứng minh rằng BAT
[ = DAC.
[

Lời giải.

D C
B
T

103
Hội thảo khoa học, Ninh Bình 15-16/09/2018

Xét phép biến hình f A là bc-nghịch đảo cực A. Ta có B 7→ C, C 7→ B, AC 7→ AB,
AB 7→ AC, ω 7→ BC. Theo giả thiết ta có ωa tiếp xúc với AB, AC và ω nên ảnh của
ωa tiếp xúc với AC, AB và BC. Vậy ảnh của ωa là ωa0 là đường tròn bàng tiếp góc A,
[ −f→
ωa 7→ ωa0 . Do đó T 7→ D với D là tiếp điểm của BC và ωa0 . Ta có BAT
A
[ Suy
DAC.
ra BAT
[ = DAC. [ Do đó AT và AD là hai đường đẳng giác.

Bài toán 2. Cho tam giác ABC, các đường tròn Mixtilinear nội tiếp ωa , ωb , ωc tiếp
xúc với ω tại A1 , B1 , C1 tương ứng. Chứng minh rằng các đường thẳng AA1 , BB1 , CC1
đồng quy tại điểm M.


Lời giải. Xét phép biến hình bc-nghịch đảo cực A. Ta có B 7→ C, C 7→ B, BC 7→ ω,
ω 7→ BC, AB 7→ AC, AC 7→ AB, ωa 7→ ωa0 , A1 = ω ∩ ωa 7→ A1∗ = BC ∩ ωa0 . (Đường
tròn ωa tiếp xúc AB, AC, tiếp xúc trong với ω tại A1 có ảnh qua phép biến hình là ωa0 ,
tiếp xúc ngoài với cạnh AC, AB và tiếp xúc với BC là ảnh đường tròn ( ABC ). Vậy đó
là đường tròn bàng tiếp góc A.) Ta có

A1∗ B = p − c, A1∗ C = p − b.
A
B1
B2
C1
M

A1∗
B C

A1
A2

Tương tự với các điểm B1∗ , C1∗ ta có

B1∗ C = p − a, B1∗ A = p − c;

C1∗ A = p − b, C1∗ B = p − a.
Theo Thí dụ 1 ta có AA1∗ và AA1 , BB1∗ và BB1 , CC1∗ và CC1 là các đường đẳng giác
tương ứng. Mặt khác,
A1∗ B B1∗ C C1∗ A p−c p−a p−b
     
· · = − · − · − = −1
A1∗ C B1∗ A C1∗ B p−b p−c p−a

104
Hội thảo khoa học, Ninh Bình 15-16/09/2018

suy ra các đường AA1∗ , BB1∗ , CC1∗ đồng quy tại một điểm N (điểm Nagen).
Theo Tính chất 2 ta có AA1 , BB1 , CC1 là các đường đẳng giác tương ứng cũng đồng
quy tại một điểm M (gọi là điểm Mixtilinear của tam giác).

Bài toán 3. Gọi X là điểm tiếp xúc của đường tròn ωa và đường tròn ω của tam giác
ABC. Chứng minh các đường tròn Mixtilinear nội tiếp ứng với góc A của hai tam
giác ABX và ACX tiếp xúc nhau.


Lời giải. Xét phép biến hình bc-nghịch đảo cực A. Gọi D là tiếp điểm của đường
tròn bàng tiếp góc A với cạnh BC.
Ta có ωa 7→ ωa0 , X = ω ∩ ωa 7→ D = BC ∩ ωa0 .
Ta có đường thẳng AX 7→ AD, AB 7→ AC, AC 7→ AB.
Chú ý là các tam giác ABC, ABX, ACX có chung đường tròn ngoại tiếp.

T D
B C

X
J0

Đường tròn ảnh của đường tròn Mixtilinear của tam giác ABX qua phép biến hình là
đường tròn bàng tiếp góc A của tam giác ACD, nó tiếp xúc với đường thẳng AD tại
J; đường tròn ảnh của đường tròn Mixtilinear của tam giác ACD là đường tròn bàng
tiếp góc A của tam giác ABD, nó tiếp xúc với AD tại J 0 .
Ta có D chia chu vi tam giác ABC thành 2 phần bằng nhau nên
p
AB + BD = AC + CD = ,
2
do đó
AB + BD + AD AC + CD + AD
= ,
2 2

105
Hội thảo khoa học, Ninh Bình 15-16/09/2018

suy ra
AB + BD + AD AC + CD + AD
AJ 0 = = = AJ,
2 2
từ đó ta có J 0 ≡ J.
Hai đường tròn ảnh tiếp xúc nhau tại J suy ra hai đường tròn Mixtilinear của hai tam
giác ABX và ACX tạo ảnh tiếp xúc nhau tại T.

Bài toán 4 (Serbia - 2006). Cho 4 ABC, D, E thuộc AB sao cho AD = AC, BE = BC,
điểm D thuộc tia đối của tia AB, điểm E thuộc tia đối của tia BA. Các phân giác
trong của các góc A, B cắt BC, AC tại P, Q và cắt đường tròn ngoại tiếp 4 ABC tại
M, N tương ứng. Gọi O1 , O2 là tâm các đường tròn ( BME), ( AND ), X = AO1 ∩ BO2 .
Chứng minh CX ⊥ PQ.

QA c AQ c bc
Lời giải. Ta có = , suy ra = , AQ = , AE = c + a,
√ QC a AC c+a c+a
AE · AQ = (√ bc)2 .
Vậy f A = NA bc · Dla : E 7→ Q, ω = ( ABC ) 7→ BC. Do đó M 7→ P, P 7→ M, B 7→ C.

C
M
O
N Q P O1
O2
X
D E
A B


ac
Tương tự, f b = NB · Dlb : D 7→ P, Q 7→ N, đường tròn ( BME) 7→ (CPQ), gọi O là
tâm (CPQ) ta có tia AO1 7→ AO; do đó AO1 và AO đối xứng nhau qua phân giác
[ (hai đường đẳng giác). Tương tự, BO2 và BO là 2 đường đẳng giác góc CBA.
BAC [
Ta có O = BO ∩ AO, X = AO1 ∩ BO2 , do đó O và X là hai điểm đẳng giác liên hợp
nhau trong tam giác ABC. Suy ra CO, CX là hai đường đẳng giác góc C của tam giác
ABC và cũng là trong tam giác CPQ. Vì O là tâm đường tròn ngoại tiếp 4CPQ nên
đường CX đi qua trực tâm 4CPQ (*). Suy ra CX là (chứa) đường cao 4CPQ, do
đó CX ⊥ PQ.

106
Hội thảo khoa học, Ninh Bình 15-16/09/2018

A
K
H
O

B I C

Chứng minh. Gọi AI là đường đẳng giác với AO, I ∈ BC, ta có OAC [ = BAI. d Có
[ = OAC,
OCA [ OCD [ = CDO [ = CDA [ = CBA.[ Có OCA [ + OCD [ = 90◦ , suy ra
[ + ABI
OAC d = 90◦ , hay OAC
[ + BAI d = 90◦ . Suy ra AI ⊥ BC (1). Tương tự, gọi BK
là đường đẳng giác với BO nên ta có BK ⊥ AC (2). Từ (1) và (2) suy ra H = BK ∩ AI
là trực tâm 4 ABC và H, O là hai điểm đẳng giác liên hợp, do đó CH ⊥ AB (đpcn).

Bài toán 5 (KaZakhstan - 2012). Cho K, L nằm trên cạnh BC của 4 ABC sao cho BAK
[=
[ < 1 A.
CAL b Giả sử ω1 là đường tròn tiếp xúc với AB, AL; ω2 là đường tròn tiếp xúc với
2
PAC = QAB.
AC, AK; ω1 và ω2 cắt nhau tại P, Q. Chứng minh rằng [ [
Lời giải.

P
T2
T1

K
B L C
Q

Giả sử AP < AQ. Theo giả thiết thì AK, AL là 2 đường đẳng giác góc A và T1 , T2 là các

tiếp điểm của ω1 và ω2 với AB, AC. Xét phép nghịch đảo AT1 · AT2 -nghịch đảo tâm A, tỉ
số r và r2 = AT1 · AT2 , nên T1 7→ T2 , T2 7→ T1 , AL 7→ AK, ω1 7→ ω2 , ω2 7→ ω1 . Do đó
P 7→ Q, AP, AQ đẳng giác tại A, suy ra [ PAC = QAB.
[

Bài toán 6. Cho 4 ABC không đều, phân giác trong tại đỉnh A cắt cạnh BC tại D và đường
tròn ngoại tiếp ( ABC ) ở E, đường tròn ω 0 đường kính DE cắt lại ( ABC ) ở F. Chứng minh
AF là đường đối trung của 4 ABC.

Lời giải.

107
Hội thảo khoa học, Ninh Bình 15-16/09/2018

O
D M
B C

F E

Gọi M- giao của đường tròn ω 0 và BC, M 6= D, DME


\ = 90◦ , suy ra M là trung điểm BC,

do đó AM là trung tuyến 4 ABC. Xét phép biến hình bc-nghịch đảo cực A. Ta có B 7→ C,
C 7→ B, E 7→ D, D 7→ E, E, D là 2 đầu đường kính nên ω 0 bất biến, BC 7→ ω, ω 0 7→ ω 0 .

Suy ra M = BC ∩ ω 0 7→ ω ∩ ω 0 = F. Theo tính chất của bc-nghịch đảo cực A thì AF và
AM là 2 đường đẳng giác góc A. Do AM là đường trung tuyến và AD là đường phân giác
nên AF là đường đối trung.

Bài toán 7 (Baltic Way 2006). Cho 4 ABC, K-trung điểm AB, L-trung điểm AC,
P-giao điểm thứ hai của hai đường tròn ( ABL) và ( ACK ), Q là giao điểm thứ hai
của AP với ( AKL). Chứng minh rằng 2AP = 3AQ.

Lời giải.

K L
Q
C0 B0
P0
B C
P
L0 Q0 K0


Ta quan tâm tới điểm A và phép
r bc-nghịch đảo cực A, nhưng với bán kính
1
nghịch đảo điều chỉnh đôi chút r = bc, qua phép biến hình này ta có K 7→ K 0 = C,
2
C 7→ C 0 = K, L 7→ L0 = B, B 7→ B0 = L. Do đó đường tròn ω = ( ABL) 7→ L0 B0 = BL,
ω1 = ( AKC ) = K 0 C 0 = KC,
L0 B0 , K 0 C 0 -trung tuyến tam giác AL0 K 0 , L0 B0 ∩ K 0 C 0 = P0 , P0 là trọng tâm tam
giác AL0 K 0 , do Q = AP ∩ ( AKL) có ảnh Q0 = AP0 ∩ K 0 L0 , do vậy AQ0 -

108
Hội thảo khoa học, Ninh Bình 15-16/09/2018

r2
trung tuyến tam giác AK 0 L0 , suy ra 3AP0 = 2AQ0 (1). Mà AP0 = ,
AP
r2
AQ0 = (2). Từ (1) và (2) ta có 3AQ = 2AP.
AQ
Bài toán 8 (IMO - 2010). Cho 4 ABC và đường tròn nội tiếp tâm I, đường tròn ngoại
tiếp ω tâm O. Giả sử AI cắt ω tại D, và E là điểm trên cung BCD, F là điểm trên dây
cung BC sao cho BAF
[ = EAC [ < 1 BAC, [ gọi G là trung điểm của IF. Chứng minh
2
rằng đường thẳng EI và DG cắt nhau trên ω.

Lời giải. Theo giả thiết ta có E, F thuộc các đường đẳng giác BAC
[ và
[ = EAC
BAF [ < 1 BAC [ (1).
2 √
Xét phép biến hình bc-nghịch đảo cực A ta có ω = ( ABC ) 7→ CB, BC 7→ ω,
E ∈ ω 7→ F ∈ BC. Do đó AE · AF = bc (*).

I
O
G
B C
F

D E

Ia

Gọi Ia là tâm đường tròn bàng tiếp góc A. Giả sử đường tròn ( BIC ) cắt AB tại
C 0 . Ta có AC 0 = AC và PA/( BIC) = AI · AIa = AB · AC 0 = AB · AC. Vậy AI · AIa =
bc (**).
Từ (1) ta suy ra FAI
[a = Id AE (2).
AE AI
Từ (*) và (**) ta có AE · AF = AI · AIa . Do đó = (3).
AIa AF
Từ (2) và (3) ta có 4 FAIa ∼ 4 I AE. Suy ra AEI d = AI [ a F. (4).
Vẽ tam giác Ia Ib Ic , trong đó Ia , Ib , Ic là tâm các đường tròn bàng tiếp; dễ có I là trực tâm

109
Hội thảo khoa học, Ninh Bình 15-16/09/2018

4 Ia Ib Ic có A, B, C là chân các đường cao. Vậy ( ABC ) là đường tròn Ơle của 4 Ia Ib Ic
nên nó cắt I Ia tại trung điểm D của I Ia (tính chất đường tròn 9 điểm của Ơle).

Ib
A
Ic

B C

Ia

Khi đó 4 IFIa có GD là đường trung bình: GD k FI, suy ra FI


d [ hay AI
a I = GDI, [ aF =
[ (5).
ADG
Từ (4) và (5) suy ra ADG d Do đó EI cắt DG trên ω.
[ = AEI.

Bài toán 9 (Balkan - MO2009). Cho 4 ABC, M ∈ AB, N ∈ AC sao cho MN k BC,
BN ∩ CM = P, ( BMP) ∩ (CNP) = { P, Q}. Chứng minh rằng BCQ
[ = CAP.
[

Lời giải.

M N

B C
Q

Áp dụng tính chất của tứ giác toàn phần vào tứ giác AMPN ta có các đường tròn
( ANB), ( AMC ), ( MPB), ( NPC ) cùng đi qua một điểm Q-điểm Miquel của tứ giác.
Xét mô hình đường tròn ngoại tiếp tam giác AMC, tam giác ANB với phép biến hình

f A = NA AM· AC · Dla : M 7→ C, C 7→ M, ( AMC ) 7→ CM,

110
Hội thảo khoa học, Ninh Bình 15-16/09/2018

g A = NA AN · AB · Dla : N 7→ B, B 7→ N, ( ANB) 7→ BN.
AM AN
Do MN k BC nên = , do đó AM · AC = AN · AB = r2 ,
AB AC
h A = NAr · D : ( AMC ) 7 → CM, ( ANB ) 7 → BN và Q = ( AMC ) ∩ ( ANB ) 7 → P =
la
CM ∩ BN, P 7→ Q.
Vậy AP, AQ là hai đường đẳng giác (theo tính chất 1). Suy ra BCQ
[ = CAP.
[

Bài toán 10 (TST Việt Nam 2007). Cho tam giác nhọn ABC và ( I ) là đường trọn nội
tiếp tam giác. Gọi (Ka ) là đường tròn đi qua A, có tâm nằm trên đường cao hạ từ
đỉnh A và tiếp xúc trong với đường tròn ( I ) tại A1 . Tương tự, xác định các đường
tròn (Kb ), (Kc ) và các tiếp điểm B1 , C1 của chúng với đường tròn ( I ).
1) Chứng minh rằng các đường thẳng AA1 , BB1 , CC1 đồng quy tại một điểm P.
2) Kí hiệu ( Ja ), ( Jb ), ( Jc ) là các đường tròn đối xứng với các đường tròn bàng tiếp
trong góc A, B, C qua trung điểm các cạnh BC, CA, AB tương ứng. Chứng minh rằng
P là tâm đẳng phương của các đường tròn ( Ja ), ( Jb ), ( Jc ).

J
Lời giải.
A

E
F
I
B D A1 C

D∗
F∗

E∗
Ia

da
A1∗


(Ta chứng minh ý 1) bằng phép biến hình bc-nghịch đảo)
1) Gọi D, E, F là tiếp điểm của đường tròn ( I ) với các cạnh BC, CA, AB.
Ta có A1 , B1 , C1 là các tiếp điểm của các đường tròn (Ka ), (Kb ), (Kc ) với ( I ).

111
Hội thảo khoa học, Ninh Bình 15-16/09/2018

Xét phép biến hình f A = NA bc · Dla , ta có ( I ) có ảnh là đường tròn tâm Ia thuộc phân
giác la , ( Ia ) là đường tròn Mixtilinear bàng tiếp góc A tiếp xúc với AB tại E∗ là ảnh của
E, tiếp xúc với AC tại F ∗ là ảnh của F, tiếp xúc ngoài với đường tròn ngoại tiếp ( ABC )
tại D ∗ , ( ABC ) 7→ BC, B 7→ C, C 7→ B, BC 7→ ( ABC ), D là tiếp điểm của ( I ) và BC
nên ảnh của D là tiếp điểm của ảnh của ( I ) và ảnh của BC, do đó D ∗ = ( ABC ) ∩ ( Ia ).
Tâm của (Ka ) nằm trên đường cao AH của tam giác nên ảnh của (Ka ) là đường thẳng
d a k BC; (Ka ) tiếp xúc với ( I ) tại A1 nên các ảnh d a , ( Ia ) tiếp xúc nhau tại A1∗ là ảnh
của A1 .
r Ia
r rI AIa IE
Mặt khác, xét phép vị tự VA I tỉ số a = = ∗ ta có ( I ) 7→ ( Ia ) và do ID ⊥ BC,
rI AI IF
r Ia
rI
Ia A1∗⊥ d a , d a k BC, r1 = ID, rla = Ia A1∗ ,
ID k Ia A1∗
vì vậy VA ( D ) = A1∗ . Do đó
A, D, A1∗ thẳng hàng.
Ta có f A ( AA1 ) = AA1∗ -đi qua D. Tương tự có BB1∗ đi qua E, CC1∗ qua F. Ta có

p−c p−a p−b


     
DB EC FA
· · = − · − · − = −1,
DC EA FB p−b p−c p−a

theo Định lý Xeva suy ra AD, BE, CF đồng quy, hay AA1∗ , BB1∗ , CC1∗ đồng quy. Do đó
theo Tính chất 1 và Tính chất 2 suy ra AA1 , BB1 , CC1 đồng quy.
Qua các ví dụ trên ta thấy ứng dụng thú vị của phép nghịch đảo thông qua một

phép biến hình, cụ thể bc-nghịch đảo cực A. Sau đây là một số bài tập tương tự.

3 Bài tập

Bài 1. Cho (O), ( I ) là đường tròn ngoại tiếp, nội tiếp tam giác ABC, M là điểm
Mixtilinear của tam giác. AM, BM, CM cắt (O) tại A1 , B1 , C1 tương ứng; AI, BI, CI cắt
(O) tại A2 , B2 , C2 tương ứng; A3 , B3 , C3 là giao điểm thứ hai của đường tròn đường
kính AI, BI, CI với (O) tương ứng. Chứng minh rằng
1) Các đường thẳng A2 A3 , B2 B3 , C2 C3 đi qua M.
2) M là tâm vị tự của ( I ), (O).
3) A1 A3 , B1 B3 , C1 C3 đồng quy tại P và 4 điểm O, I, M, P thẳng hàng.

Bài 2. Cho 4 ABC, M ∈ AB, N ∈ AC, P ∈ BC sao cho MN k BC, NP k AB,


PM k AC, BN ∩ CM = G, ( BMG ) ∩ (CNG ) = { G, Q}, ( AMG ) ∩ (CPG ) = { G, R},
( ANG ) ∩ ( BPG ) = { G, T }. Chứng minh rằng AQ, BR, CT đồng quy tại một điểm.

112
Hội thảo khoa học, Ninh Bình 15-16/09/2018

Bài 3. Cho 4 ABC không đều, phân giác trong tại đỉnh A, B, C cắt cạnh BC, CA, AB
tương ứng tại D, E, F và cắt đường tròn ngoại tiếp ( ABC ) ở M, N, P. Đường tròn
đường kính DM, EN, FP tương ứng cắt lại ( ABC ) tại R, S, T. Chứng minh rằng
AR, BS, CT đồng quy tại một điểm.

Bài 4. Đường tròn mixtilinear bàng tiếp góc A của tam giác ABC là đường tròn
tiếp xúc ngoài với các cạnh AB, AC và đường tròn ngoại tiếp ( ABC ) kí hiệu là ω1 .
Tương tự các đường tròn Mixtilinear ngoại tiếp ứng với góc B, C của tam giác kí
hiệu là ω2 , ω3 . Gọi các tiếp điểm của các đường tròn Mixtilinear ngoại tiếp ω1 , ω2 , ω3
với đường tròn ngoại tiếp ( ABC ) là A1 , B1 , C1 . Chứng minh rằng các đường thẳng
AA1 , BB1 , CC1 đồng quy tại điểm N.

Tài liệu

[1] Titu Andreescu-Michal Rolinek-Josef Tkadlec, 107 Geometry Problems, From the
AwesomeMath Year –Round Program.

[2] Nguyễn Văn Mậu (chủ biên), 4/2012, Các chuyên đề toán học bồi dưỡng học sinh
giỏi, Kỉ yếu hội nghị khoa học.

[3] Lê Anh Vinh (chủ biên), Nhà XBGD Việt Nam,1/2018; Định hướng bồi dưỡng
học sinh năng khiếu toán.

[4] Praxolop, Hình học phẳng.

[5] Đỗ Thanh Sơn, Nhà xuất bản Giáo dục 2004, Phép biến hình trong mặt phẳng.

113
Hội thảo khoa học, Ninh Bình 15-16/09/2018

BẤT ĐẲNG THỨC VÀ CÁC BÀI TOÁN CỰC TRỊ


TRONG ĐẠI SỐ TỔ HỢP

Bùi Thị Lợi


Trường THPT Yên Khánh A, Ninh Bình

Tóm tắt nội dung

Tổ hợp có vị trí rất quan trọng trong Toán học vì nó không những là một đối tượng
nghiên cứu trọng tâm của Đại số và Giải tích mà còn là một công cụ đắc lực của toán rời
rạc và lý thuyết trò chơi.
Ngoài ra, tổ hợp còn được sử dụng nhiều trong tính toán và ứng dụng. Trong các kì
thi học sinh giỏi toán quốc gia và Olympic toán quốc tế thì các bài toán về tổ hợp cũng
thường được đề cập đến và được xem như những bài toán rất khó của bậc phổ thông.
Việc khảo sát sâu hơn về các vấn đề tính toán tổ hợp và các dạng toán liên quan cho
ta hiểu sâu sắc về lý thuyết cũng như các ứng dụng liên quan đến tổ hợp và toán rời rạc.
Báo cáo này nhằm trình bày một số vấn đề liên quan tính chất và các dạng toán ứng
dụng liên quan đến tổ hợp nhằm thể hiện rõ vai trò quan trọng của tổ hợp trong các dạng
toán thi HSG và Olympic quốc gia và quốc tế.

1 Một số đẳng thức tổ hợp


Trong phần này, để tính tổng liên quan đến tổ hợp hoặc chứng minh một đẳng thức
tổ hợp, ta phải quan sát các số hạng trong tổng để tìm nhị thức cần khai triến, kết hợp
với các phép toán đạo hàm, tích phân hoặc các phép toán về số phức để giải bài toán.

1.1 Tính chia hết của biểu thức tổ hợp


Bài toán 1 (Hungarian MO 2001). Cho m và n là các số nguyên. Chứng minh rằng m là
m −1
một ước của n ∑ (−1)k Cnk .
k =0

114
Hội thảo khoa học, Ninh Bình 15-16/09/2018

Lời giải. Ta có thể viết lại biểu thức đã cho như sau:
m −1 m −1
n ∑ (−1)k Cnk = n ∑ (−1)k (Cnk −1 + Cnk− 1
−1 )
k =0 k =0
m −1 m −1
=n ∑ (−1)k Cnk −1 + n ∑ (−1)k Cnk−
−1
1

k =0 k =0
m −1 m −2
=n ∑ (−1)k Cnk −1 − n ∑ (−1)k Cnk −1
k =0 k =0
m −1
= n(−1) Cnm−−11
= m(−1)m−1 Cnm .
m −1
Vậy m|n ∑ (−1)k Cnk .
k =0

Bài toán 2 (Chinese MO 1998). Xác định tất cả các số nguyên dương n ≥ 3 sao cho 22000
chia hết cho 1 + Cn1 + Cn2 + Cn3 .
Lời giải. Vì 2 là số nguyên tố nên
Cn1 + Cn2 + Cn3 = 2k , với 0 ≤ k ≤ 2000.
Ta có
(n + 1)(n2 − n + 6)
1 + Cn1 + Cn2 + Cn3 = .
6
Khi đó
(n + 1)(n2 − n + 6) = 3.2k+1 .
Đặt m = n + 1, thì m ≥ 4 và m(m2 − 3m + 8) = 3.2k+1 . Xét các trường hợp sau:
+) Nếu m = 2s .
Từ m ≥ 4, s ≥ 2 ta có 22s − 3.2s + 8 = m2 − 3m + 8 = 3.2t với mọi số nguyên dương t.
.
Nếu s ≥ 4 thì 8 − 3.2t ..16. Suy ra
2t = 8 ⇒ m2 − 3m + 8 = 24 ⇒ m(m − 3) = 16 (điều này không thể xảy ra).
Như vậy hoặc s = 3, m = 8, t = 4, n = 7 hoặc s = 2, m = 4, t = 2, n = 3 .
+) Nếu m = 3.2u .
Từ m ≥ 4 và u ≥ 1 ta có 9.22u − 9.2u + 8 = m2 − 3m + 8 = 2v với mọi số nguyên
dương v.
Dễ dàng kiểm tra được rằng không có nghiệm nào của v khi u = 1; 2.
.
Nếu u ≥ 4 ta có 8 − 2v ..16, suy ra v = 3 và m(m − 3) = 0, điều này không thể xảy ra.
Vì vậy u = 3, m = 3.23 = 24, v = 9, n = 23.
Tóm lại, n = 3; 7; 23.
Bài toán 3 (Gặp gỡ Toán học 2014). Cho p ≥ 5 là số nguyên tố và m, k ∈ Z+ . Chứng
p −1
minh rằng pk+2 |Cmpk −1 − 1.
Lời giải. Để giải quyết tốt bài toán này chúng ta cần sử dụng những kiến thức hỗ trợ:
Định lý 1 (Định lí Lagrange (về đa thức đồng dư)). Cho P( x ) là một đa thức hệ số nguyên bậc
n và các hệ số của P( x ) không đồng thời chia hết cho p. Khi đó phương trình đồng dư P( x ) ≡ 0
(mod p) có min{n, p} nghiệm.

115
Hội thảo khoa học, Ninh Bình 15-16/09/2018

Bổ đề 1. Nếu phương trình P( x ) ≡ 0 (mod p) có số nghiệm nhiều hơn bậc của P( x ) thì tất cả
các hệ số của P( x ) đều chia hết cho p.
Quay trở lại bài toán
Xét đa thức P( x ) = ( x − 1)( x − 2) . . . ( x − ( p − 1)) − ( x p−1 + ( p − 1)!).
Đa thức P( x ) có thể được viết lại như sau:

P( x ) = S1 x p−2 + · · · + S p−2 x.

Tập nghiệm của đa thức P( x ) gồm p − 1 phần tử là tập hợp {1; 2; ...; p − 1} , mà deg P =
.
p − 2 cho nên S ..p, ∀i = 1, p − 2. Mặt khác, ta có
i

P( p) = − p p−1 ⇒ p3 | P( p) ⇒ p3 | pS p−2 ,

hay p2 |S p−2 . Để ý rằng

p −1 (mpk − 1)(mpk − 2) . . . (mpk − ( p − 1))


Cmpk −1 = ,
( p − 1) !

gcd(( p − 1)!, p) = 1.
Nên bài toán quy về việc chứng minh
p −1
pk+2 |(Cmpk −1 − 1)( p − 1)!.

Suy ra

pk+2 |(mpk − 1) . . . (mpk − ( p − 1)) − ( p − 1)!


p −1
⇔ pk+2 | P(mpk ) + (mpk )
p −2 p −1
⇔ pk+2 |S1 (mpk ) + · · · + S p−2 mpk + (mpk ) . (1.1)

Kết quả (1.1) hoàn toàn đúng do p2 |S p−2 và p|Si , ∀i = 1, p − 3.

1.2 Quan hệ đồng dư giữa các biểu thức tổ hợp


 
2p
Bài toán 4 (Putnam 1996). Cho số nguyên tố p ≥ 5 và k = . Chứng minh rằng
3
k
∑ Cip ≡ 1 (mod p2 ).
i =0

Lời giải. Ta có
k k
p i −1
∑ Cip ≡ 1 (mod p2 ) ⇔ ∑ C
i p −1
≡0 (mod p2 ),
i =0 i =1

hay
k
1 k
(−1)i−1
∑ i Cip−−11 ≡ 0 (mod p) ⇔ ∑ i ≡ 0 (mod p).
i =1 i =1

116
Hội thảo khoa học, Ninh Bình 15-16/09/2018

Do p ≥ 5 là số nguyên tố nên p có dạng 6m ± 1.


• Xét p = 6m − 1, k = 4m − 1 ta có
k
(−1)i−1 4m−1 1 2m−1
1
∑ i = ∑ i − 2 ∑ 2j
i =1 i =1 j =1
4m−1
1 2m−1 1
= ∑ i
− ∑
j
i =1 i =1
4m−1
1
= ∑ i
.
i =2m

Lại có
4m−1 2m  
1 1 1
2 ∑ i
=∑ +
2m − 1 + i 4m − i
i =2m i =1
2m
6m − 1
= ∑ (2m − 1 + i)(4m − i)
i =1
2m
p
= ∑ (2m − 1 + i)(4m − i) ≡ 0 (mod p).
i =1

Vì gcd( p, (2m − 1 + i )(4m − i )) = 1, ∀i = 1, 2m nên


4m−1
1
∑ i
≡0 (mod p).
i =2m

• Xét p = 6m + 1, k = 4m ta có
k
(−1)i−1 4m
1 2m
1 4m
1 2m 1 4m
1
∑ i = ∑i − 2 ∑ 2j ∑ i ∑ j
= − = ∑ i.
i =1 i =1 j =1 i =1 j =1 i =2m+1

Lại có
4m 2m  
1 1 1
2 ∑ =∑ +
i =2m+1
i i =1
2m + i 4m + 1 − i
2m
6m + 1
= ∑ (2m + i)(4m + 1 − i)
i =1
2m
p
= ∑ (2m + i)(4m + 1 − i) ≡ 0 (mod p).
i =1

Vì gcd( p, (2m + i )(4m + 1 − i )) = 1, ∀i = 1, 2m nên


4m
1
∑ i
≡0 (mod p).
i =2m+1

p
j j
Bài toán 5 (Putnam 1991). Cho p là số nguyên tố lẻ. Chứng minh rằng ∑ C p C p+ j ≡ 2 p + 1
j =0
(mod p2 ).

117
Hội thảo khoa học, Ninh Bình 15-16/09/2018

Lời giải. Sử dụng đẳng thức Vandermonde ta có


p
∑ Cp Cp+ j = ∑ Cp ∑ Cjh Cp
j j j p−h

j =0 j ≥0 h ≥0
p! ( p − h)!
∑ Cp ∑ ( p − j)!( j − h)!
p−h
=
h ≥0
h!( p − h)! j ≥0
2
= ∑ (C hp ) 2 p−h
h ≥0
p
≡ 2 + 1 (mod p2 ).

(Do số nguyên tố p là ước của C hp với 0 < h < p).

Bài toán 6 (Thi chọn đội tuyển VMO 2012, tỉnh Nghệ An). Cho p > 3 là số nguyên tố và
tập hợp M = {1; 2; . . . ; p}. Với mỗi số nguyên k thỏa mãn 1 ≤ k ≤ p ta đặt

Ek = {A ⊂ M : | A| = k },


xk = ∑ (min A + max A).
A∈ Ek

p −1
Chứng minh rằng ∑ xk C kp ≡ 0 (mod p3 ).
k =1

Lời giải. Giả sử A = {a1 ; a2 ; . . . ; ak} ∈ Ek . Khi ấy B = {p + 1 − a1 ; . . . ; p + 1 − ak} ∈ Ek .


• Nếu a1 = min A thì p + 1 − a1 = max B.
• Nếu ak = max A thì p + 1 − ak = min B.
Bây giờ, ta có

2xk = ∑ ( a1 + a k + p + 1 + p + 1 − a k − a1 ) = ∑ 2( p + 1) = 2( p + 1)C kp ,
A∈ Ek A∈ Ek

hay
xk = ( p + 1)C kp .
Ta sẽ chứng minh
p −1
2
( p + 1) ∑ (Ckp ) ≡ 0 (mod p3 ),
k =1

hay
p −1
2
∑ (Ckp ) ≡ 0 (mod p3 ). (1.2)
k =1

Thật vậy,
. 1 ( p − 1) !
C kp ..p ⇒ C kp = .
p k!( p − k)!
Do đó
p −1  2
( p − 1) !
(1.2) ⇔ ∑ k!( p − k )!
≡ 0 (mod p).
k =1

118
Hội thảo khoa học, Ninh Bình 15-16/09/2018

Đặt

( p − 1) !
αk =
k!( p − k )!
⇒ k!αk = ( p − 1)( p − 2) . . . ( p − k + 1) ≡ (−1)k−1 (k − 1)! (mod p)
⇒ kαk ≡ (−1)k−1 (mod p).

Lại đặt

( p − 1) !
βk =
k
⇒ kβ k = ( p − 1)! ≡ −1 (mod p),
⇒ αk ≡ (−1)k β k (mod p).

Ta có mệnh đề sau:
“∀k ∈ {1; 2; . . . ; p − 1}, ∃ duy nhất j ∈ {1; 2; . . . ; p − 1} sao cho jk ≡ 0 (mod p)”.
Khi đó
p −1 p −1 p −1 p −1
( p − 1)(2p − 1) p
∑ ∑ β2k (kj)2 ∑ ( bk k ) ∑ j2 ≡
2 2
β2k ≡ ≡ j ≡ (mod p).
k =1 k =1 k =1 j =1
6

Mặt khác, do p > 3 nên

.
( p − 1)..2 và ( p − 1)(2p − 1) = 2p2 + 1 − 3p ≡ 2.1 + 1 ≡ 0 (mod 3).

Suy ra
.
( p − 1)(2p − 1)..6.
Cuối cùng ta đi đến
p −1 p −1
∑ α2k ≡ ∑ β2k ≡ 0 (mod p).
k =1 k =1

Vậy ta hoàn tất việc chứng minh


p −1
∑ xk Ckp ≡ 0 (mod p3 ).
k =1

Bài toán 7 (VMO 2017). Chứng minh rằng


1008
a) ∑ kC2017
k
≡0 (mod 20172 ).
k =1
504  
b) ∑ (−1)k C2017
k
≡3 22016 − 1 (mod 20172 ).
k =1

119
Hội thảo khoa học, Ninh Bình 15-16/09/2018

Lời giải. a) Ta có
1008 1008
k.2017!
∑ k
kC2017 ≡ ∑ k!(2017 − k )!
k =1 k =1
1008
2016!
≡ 2017 ∑ (k − 1)!(2017 − k)!
k =1
1007
2016!
≡ 2017 ∑ k!(2016 − k )!
k =0
1007
≡ 2017 ∑ C2016
k

k =0
1007
≡ 2017 ∑ C2016
k

k =0
!
2016
2017

2 ∑ k
C2016 1008
− C2016
k =0
2017 2016 1008
≡ (2 − C2016 ) (mod 20172 ).
2
Do 2017 là số nguyên tố nên áp dụng định lí Fermat nhỏ ta có

22016 ≡ 1 (mod 2017).

Mặt khác, ta có

1008 1009.1010 . . . 2016 − 1008!


C2016 −1 =
1008!
(2017 − 1)(2017 − 2) . . . (2017 − 1008) − 1008!
= .
1008!
Tử số chia hết cho 2017 và gcd(1008!, 2017) = 1 nên
1008
C2016 ≡ 1 ≡ 22016 (mod 2017).

Do đó
1008
∑ kC2017
k
≡0 (mod 20172 ).
k =1
x z
b) Ký hiệu ≡ ( modm) với x, y, z, t ∈ Z, y 6= 0, t 6= 0 và m ∈ Z, m > 1 nếu dạng tối
y t
xt − zy
giản của phân số chia hết cho m.
yt
Nhận xét 1. Với mọi số nguyên tố p thì
k −1 p
C kp ≡ (−1) (mod p2 ).
k
Thật vậy, ta có

p! p ( p − k + 1)( p − k + 2) . . . ( p − 1)
C kp = = . .
k!( p − k )! k ( k − 1) !

120
Hội thảo khoa học, Ninh Bình 15-16/09/2018

Suy ra

p p ( p − k + 1)( p − k + 2) . . . ( p − 1) − (−1)k−1 (k − 1)!


C kp − (−1)k−1 = . .
k k ( k − 1) !

Để ý rằng thừa số thứ nhì trong tích này chia hết cho p2 cho nên khẳng định trên được
chứng minh.
Đến đây, ta có
1 k (−1)k−1
Cp ≡ (mod p).
p k
Khi đó
504 504
k −1 2017
∑ (−1) C2017 ∑ (−1) (−1)
k k k

k =1 k =1
k
504
2k−1 2017
≡ ∑ (−1) k
k =1
504
1
≡ −2017 ∑ (mod 20172 ).
k =1
k

Bài toán quy về


504
1
−2017 ∑ ≡ 3(22016 − 1) (mod 20172 ),
k =1
k

hay
504
1 3(22016 − 1)
−∑ ≡ (mod 2017).
k =1
k 2017

Đặt
1 1 1 1
Sn = + + + · · · + , n ∈ Z+ .
1 2 3 n
Khi đó
1 1 1
− +···− = S2n − Sn .
1 2 2n
Ta có
1008
(−1)k−1
S504 = S1008 − ∑ k
k =1
2016
(−1)k−1 1008 (−1)k−1
= S2016 − ∑ k
−∑
k
.
k =1 k =1

Do
1008  
1 1
S2016 = ∑ +
k 2017 − k
≡ 0 (mod 2017).
k =1

121
Hội thảo khoa học, Ninh Bình 15-16/09/2018

nên ta cần chứng minh


2016
(−1)k−1 1008 (−1)k−1 3(22016 − 1)
∑ k
+∑
k

2017
(mod 2017).
k =1 k =1

Sử dụng nhận xét trên một lần nữa, ta đưa quan hệ đồng dư trên về
!
2016 1008
1 3(22016 − 1)
2017 k=1 ∑ C2017 + ∑ C2017 ≡
k k
2017
(mod 2017).
k =1

Điều này là hợp lý bởi vì


2016 1008 2016
1 2016
∑ C2017
k
+ ∑ C2017
k
≡ ∑ C2017
k
+ ∑ C2017
2
k

k =1 k =1 k =1 k =1
!
2017
3

2 ∑ C2017
k
−2
k =0
3(22017 − 2)
≡ ≡ 3(22016 − 1) (mod 2017).
2

2 Bất đẳng thức trong tính toán tổ hợp


2.1 Một số bài toán về bất đẳng thức trong tính toán tổ hợp
Xét các phép biến đổi tương đương, phương pháp làm trội, làm giảm.
Bài toán 8. Chứng minh rằng nếu n và k là hai số tự nhiên thỏa mãn 0 ≤ k ≤ n thì
n n n 2
C2n +k .C2n−k ≤ (C2n ) .

Nhận xét 2. Để chứng minh bất đẳng thức này ta sử dụng phép biến đổi tương đương.
+k .C2n−k với 0 ≤ k ≤ n, n, k ∈ N.
Chứng minh. Đặt uk = C2nn n

Ta chứng minh dãy (uk ) giảm. Ta có

n n (2n + k)! (2n − k)!


uk = C2n +k .C2n−k = . ,
n!(n + k )! n!(n − k )!
n n (2n + k + 1)! (2n − k − 1)!
u k +1 = C2n +k+1 .C2n−k = . .
n!(n + k + 1)! n!(n − k − 1)!
Do đó
u k +1 (2n + k + 1)(n − k)
= .
uk (n + k + 1)(2n − k) ≤ 1
Thật vậy
u k +1
≤ 1 ⇔ (2n + k + 1)(n − k) ≤ (n + k + 1)(2n − k)
uk
⇔ 2nk + n ≥ 0 (hiển nhiên đúng) vì 0 ≤ k ≤ n.
Do đó dãy (uk ) giảm. Từ đó suy ra với k ≥ 0 thì
n n n 2
uk ≤ u0 ⇔ C2n +k .C2n−k ≤ (C2n ) .

122
Hội thảo khoa học, Ninh Bình 15-16/09/2018

Bài toán 9. Chứng minh rằng

Cn2 + 2Cn3 + · · · + (n − 1)Cnn > (n − 2)2n−1 .

Nhận xét 3. Để chứng minh bất đẳng thức này ta phải tính tổng ở vế trái để làm gọn bất
đẳng thức cần chứng minh.
Chứng minh. Ta có
n
(1 + x ) n = ∑ Cnk .xk . (2.1)
k =0

Đạo hàm hai vế của đẳng thức (2.1), ta có


n
n (1 + x ) n −1 = ∑ Cnk .kxk−1 . (2.2)
k =1

Thay x = 2 vào bất đẳng thức (2.2), ta được


n
n2n−1 = ∑ kCnk . (2.3)
k =1
Thay x = 1 vào (2.1)
n
2n = ∑ Cnk . (2.4)
k =0

Trừ từng vế (2.3) cho (2.4), ta có

n2n−1 − 2n = −Cn0 + Cn2 + 2Cn3 + · · · + (n − 1)Cnn

⇔ Cn2 + 2Cn3 + · · · + (n − 1)Cnn = n2n−1 − 2n + 1 = (n − 2)2n−1 + 17(n − 2)2n−1 .


Vậy
Cn2 + 2Cn3 + · · · + (n − 1)Cnn > (n − 2)2n−1 .
Bài toán 10. Chứng minh rằng

Cn1 + 2Cn2 + 3Cn3 + · · · + nCnn


< n!, với n ∈ N, n > 3.
n
Nhận xét 4. Để chứng minh bất đẳng thức này ta phải tính tổng ở tử của vế trái để làm
gọn bất đẳng thức cần chứng minh, rồi sử dụng phương pháp qui nạp toán học để chứng
minh.
Chứng minh. Ta có
Cn1 + 2Cn2 + 3Cn3 + · · · + nCnn = n2n−1 .
Suy ra
Cn1 + 2Cn2 + 3Cn3 + · · · + nCnn
= 2n −1 .
n
Vậy ta phải chứng minh
2n−1 < n!. (2.5)
Thật vậy, ta đi chứng minh (2.5) bằng phương pháp quy nạp.
Khi n = 3, ta có 2n−1 = 22 = 4 < 3!. Vậy (2.5) đúng.
Giả sử (2.5)đúng với n = k, k ∈ N, k > 3, nghĩa là, ta có

2k−1 < k!.

123
Hội thảo khoa học, Ninh Bình 15-16/09/2018

Ta chứng minh (2.5) đúng khi n = k + 1, tức là phải chứng minh

2k < (k + 1)!. (2.6)

Ta có (2.5)⇔ 2k < 2k!.


Vì 2 < 3 ≤< k + 1 nên ta có

2k! < (k + 1)k! = (k + 1)! ⇒ 2k < (k + 1)!.

Vậy (2.6) đúng.


Theo nguyên lý qui nạp ta có 2n < n!, ∀n ∈ N, n ≥ 3.
Suy ra điều phải chứng minh.

Bài toán 11. Chứng minh rằng


 n
1
2< 1+ < 3, n ∈ Z, n > 1.
n

Nhận xét 5. Để chứng minh bất đẳng thức này, ta sử dụng công thức khai triển nhị thức
Newton ở vế trái, rồi sử dụng phương pháp làm trội, làm giảm để chứng minh.
Chứng minh. Theo công thức nhị thức Newton, ta có
 n  2  n
1 1 1 1
1+ = Cn0 + Cn1 + Cn2 + · · · + Cnn .
n n n n

1 1
Ta có Cn0 = 1; Cn1 = n. = 1. Do đó
n n
 n
1
1+ > 2. (2.7)
n

Mặt khác, ta có
 2
1 n ( n − 1) 1 1 1
Cn2 = < = = ,
n 2!n2 2! 2 1.2
 3
3 1 n(n − 1)(n − 2) 1 1
Cn = < < ,
n 3!n3 3! 2.3
. . . . . . . . . . . . .,
 p
p 1 n ( n − 1) . . . ( n − p + 1) 1 1
Cn = p
< < ,
n p!n p! ( p − 1).p
. . . . . . . . . . . . .,
 p
n 1 n! 1 1
Cn = n
< < .
n n!n n! (n − 1).n

Do đó, ta có  n
1 1 1 1
1+ < 2+ + +···+ . (2.8)
n 1.2 2.3 (n − 1).n

124
Hội thảo khoa học, Ninh Bình 15-16/09/2018

Nhưng
1 1 1
= − ,
1.2 1 2
1 1 1
= − ,
2.3 2 3
. . . . . . . . . . . . . . . .,
1 1 1
= − .
(n − 1).n n−1 n
Suy ra
1 1 1 1
+ +···+ = 1 − < 1. (2.9)
1.2 2.3 (n − 1).n n
Từ (2.8) và (2.9), suy ra  n
1
1+ < 3. (2.10)
n
Kết hợp (2.7) và (2.10), ta có điều phải chứng minh.

2.2 Sử dụng các bất đẳng thức cơ bản để chứng minh


Bài toán 12. Cho n + 1 số nguyên đôi một khác nhau x0 , x1 , . . . xn . Xét các đa thức dạng

P ( x ) = x n + a n −1 x n −1 + · · · + a 1 x + a 0 . (2.11)

Chứng minh rằng


f ( x j ) ≥ n! .

max (2.12)
j∈{0,1,...,n} 2n
Chứng minh. Không mất tính tổng quát ta có thể coi x0 < x1 < · · · < xn thì xn − x0 ≥
n, . . . , xn − xn−1 ≥ 1. Khi đó theo công thức nội suy Lagrange thì có thể viết (2.11) dưới
dạng " #
n n x − xj
P( x ) = ∑ P( xk ) ∏ .
k =0
x − xj
j6=i;j=0 i

So sánh hệ số của x n ta được


" #
n n
1
1= ∑ P( xk ) ∏ xi − x j
.
k =0 j6=i;j=0

Nếu (2.12) không đúng thì


" #
n n
1
1= ∑ | P( xk )| ∏ xi − x j
k =0 j6=i;j=0
 
n! 1 1 1 1
≤ n + +···+ +···+
2 n! (n − 1)!1! (n − k)!k! 0!n!
1  0  1
= n Cn + Cn1 + · · · + Cnk + · · · + Cnn = n 2n = 1, (mâu thuẫn).
2 2
Vậy ta có điều phải chứng minh.

125
Hội thảo khoa học, Ninh Bình 15-16/09/2018

Nhận xét 6. Để giải bài toán trên, ta đã sử dụng công thức nội suy Lagrange và công
thức tính tổng của các số tổ hợp.

Bài toán 13. Cho x1 , x2 , . . . , xn > 0 và đặt:


n n n
S1 = ∑ x i ; S2 = ∑ x i x j ; S3 = ∑ xi x j x k ; . . . ,
i =1 1≤ i ≤ j ≤ n 1≤ i < j < k ≤ n

và Sn = x1 x2 . . . xn là các hàm cơ bản của x1 , x2 , . . . , xn . Chứng minh rằng:


s s s
S1 S2 S 3 Sn
1
≥ 2
≥ 3 3 ≥ ... ≥ n n. (2.13)
Cn Cn Cn Cn

Nhận xét 7. Để chứng minh bất đẳng thức (2.13), ta sử dụng phương pháp quy nạp kết
hợp với bất đẳng thức AM-GM.
Chứng minh. Ta chứng minh bằng quy nạp:
Với n = 1, bất đẳng thức (2.13) đúng.
Giả sử bất đẳng thức (2.13) đúng với n − 1. Ta giả thiết 0 < x1 < x2 < · · · < xn .
Xét đa thức

P( x ) = ( x − x1 )( x − x2 ) . . . ( x − xn )
= x n − S1 x n−1 + S2 x n−2 + . . . , (−1)n .Sn .

Vì P( x ) có nghiệm x1 , x2 , . . . , xn nên theo định lí Lagrăng thì:


P0 ( x ) = n.x n−1 − (n − 1).S1 x n−2 + (n − 2).S2 x n−3 − (n − 3).S3 x n−4 + . . . + (−1)n−1 .Sn−1
có n − 1 nghiệm y1 , y2 , . . . , yn−1 .
Xen kẽ: x1 < y1 < x2 < y2 < . . . < xn−1 < yn−1 < xn . Suy ra
n−1 n−2 n−3 S n −1
S1 ; S2 ; S3 ; . . . ; là các hàm cơ bản của y1 , y2 , . . . , yn−1 .
n n n n
Theo giả thiết quy nạp, ta có:
s s s s
( n − 1) ( n − 2) S n −1 S S S
≥ . . . ≥ n−1 nn−1 .
2
1
S1 ≥ 2
S2 ≥ . . . ≥ n − 1 n − 1
⇒ 11 ≥ 2
nCn−1 nCn−2 nCn−1 Cn Cn Cn−1

Ta lại có
1 1 1
x1 + x2 +···+ xn 1
≥ √ AM-GM)
n n x1 x2 . . . x n
Suy ra
1 1 1
!
x1 + x2 +···+ xn
( x1 .x2 . . . xn )
n q
≥ n
( x 1 x 2 . . . x n ) n −1 .
n

Do đó: s s
n −1
S n −1 n
Sn
≥ ⇒ đpcm.
Cnn−1 Cnn

126
Hội thảo khoa học, Ninh Bình 15-16/09/2018

Đây là bất đẳng thức xen kẽ của Cauchy. Chẳng hạn với a, b, c, d > 0 :

r
a+b+c+d ab + ac + ad + bc + bd + cd

4 6
+ bcd √
r
3 abc + abd + acd 4
≥ ≥ abcd.
4

3 Các dạng toán cực trị liên quan đến tổ hợp trong
dãy số
3.1 Một số bài toán cực trị liên quan đến tổ hợp trong dãy số
Bài toán 14. Cho đa thức
P( x ) = (1 + 2x )n = a0 + a1 x + a2 x2 + · · · + an x n ,
a1 a2 an
biết a0 + + 2 + · · · + n = 4096.
2 2 2
Xét dãy số ( ak ), k = 0, 1, 2, . . . , n. Tìm số hạng lớn nhất của dãy số ( ak ).
Nhận xét 8. Để giải bài toán này trước hết ta phải tìm ra n, rồi tìm số hạng tổng quát của
dãy số ak .
Lời giải.
Ta có  
1 a a2 an
f = a0 + 1 + 2 + · · · + n
2 2 2 2
⇔ 2n = 4096 ⇔ 2n = 212 ⇔ n = 12.
Khi đó, ta có
12 12
P( x ) = ∑ Cnk (2x)k = ∑ C12k xk .
k =0 k =0
Suy ra
k k
ak = C12 2 , k = 0, 1, 2, . . . , n.
Số hạng ak lớn nhất khi và chỉ khi
(
a k ≥ a k +1
a k ≥ a k −1
12!
 12!
2k ≥ 2k +1
(
k 2k k +1 k +1 
C12 ≥ C12 2 k!(12 − k )!

( k + 1)!(11 − k )!
⇔ k 2k

k −1 k −1 12! 12!
C12 ≥ C12 2 
 2k ≥ 2k −1
k!(12 − k )! (k − 1)!(13 − k)!
1 2
 

 ≥ k ≥ 22

⇔ 212 − k 1 k + 1 ⇔ 3
26
 ≥
 k ≤
 .
k 13 − k 3
Vì k ∈ Z nên k = 8. Vậy a8 = 28 C12
8 = 126720 là số hạng lớn nhất của dãy ( a )
k

127
Hội thảo khoa học, Ninh Bình 15-16/09/2018

Bài toán 15. Cho số nguyên dương N. Tìm giá trị lớn nhất trong dãy số CN n (n =

0, 1, . . . , N ).  
n N −n n N
Lời giải. Do CN = CN nên ta chỉ cần xét dãy {CN } với n = 0, 1, . . . , . Mặt khác
   2
N n −1 n , nên max {C n } = C p , p = N .
thì với n = 0, 1, . . . , , ta có CN < CN N N
2 0≤ n ≤ N 2

Bài toán 16 (Olympic 30/4/2009). Với mỗi bộ n số nguyên dương a1 , a2 , . . . , an có tổng


bằng 2009. Đặt A là tổng tất cả các số hạng có dạng

1
,
a1 ( a1 + a2 )( a1 + a2 + a3 ) . . . ( a1 + a2 + · · · + an )

trong đó k1 , k2 , . . . , k n là một hoán vị bất kì của {1, 2, . . . , n} .


Tìm giá trị nhỏ nhất của A.
Lời giải. Xét một bộ bất kì n số nguyên dương a1 , a2 , . . . , an có tổng bằng 2009
Ta chứng minh bằng quy nạp
1
A= . (3.1)
a1 a2 . . . a n
+) Với n = 1, (3.1) đúng
+) Giả sử (3.1) đúng đến n − 1
Với mỗi n số nguyên dương a1 , a2 , . . . , an
1
Trong A, đặt làm thừa số chung, ta được
a1 + a2 + · · · + a n
1
A= ,
a1 + a2 + · · · + a n

trong đó B gồm n! số hạng.


Trong B ta nhóm các số hạng không chứa a1 thành tổng B1 gồm (n − 1)! số hạng ứng với
(n − 1)! hoán vị của {2, 3 . . . , n}
1
Do đó theo giả thiết quy nạp thì B = .
a2 a3 . . . a n
Tiếp tục như thế với a2 , a3 , . . . , an ta được

1 1 1 a + a2 · · · + a n
B= + +···+ = 1 .
a2 a3 . . . a n a1 a3 . . . a n a 1 a 2 . . . a n −1 a1 a2 . . . a n

1
Từ đó suy ra A = .
a1 a2 . . . a n
Vậy theo nguyên lí quy nạp (3.1) đúng với mọi n ≥ 1.
+) Nếu a1 = 1, loại a1 = 1, thế a2 = 1 bởi a20 = 1 + a2 thì tổng các số ai không đổi, còn
tích tăng lên.
+) Nếu ai ≥ 5 thế a1 bởi 2( a1 − 2) thì tổng các số ai không đổi, tích tăng lên vì 2( a1 − 2) =
2a1 − 4 > a1 .
+) Nếu có một số ai = 4, thế số đó bởi 2 + 2 thì tổng và tích các số ai không đổi.
+) Nếu có ba số 2, thế ba số đó bằng hai số 3 thì tổng không đổi, tích tăng lên.
Vậy để tích p của các số ai lớn nhất thì phải chọn không quá hai số 2, các số khác bằng 3
1
Do 2009 = 3.669 + 2 nên maxP = 2.3669 . Suy ra minA = .
2.3669

128
Hội thảo khoa học, Ninh Bình 15-16/09/2018

3.2 Một số bài toán cực trị liên quan qua các đề thi Olympic
Bài toán 17 (Olympic 30/4/2011). Cho hàm số
2011
F(x) = ∑ (k − 2011x)2 C2011
k
x k (1 − x )2011−k .
k =0

Tìm giá trị nhỏ nhất của hàm số F ( x ) trên [0; 1]


Nhận xét 9. Để giải bài toán này, trước hết ta phải sử dụng các tính chất của các số tổ
hợp, công thức khai triển nhị thức Newton để rút gọn hàm số F ( x ).
Lời giải.
n
∑ (k − nx) Cnk x k (1 − x )n−k
2
A=
k =0
n n n
∑ Cnk xk (1 − x) + ∑ ∑ kCnk xk (1 − x)
n−k
= (nx )2 k2 Cnk x k (1 − x )n−k − 2nx n−k

k =0 k =0 k =0

Xét
n
∑ kCnk xk (1 − x)
n−k
A1 =
k =0
n n
= n ∑ Cnk− 1 k
−1 x (1 − x )
n−k
= nx ∑ Cnk−−11 xk−1 (1 − x)n−k
k =1 k =1
n −1
= nx [ x + (1 − x )] = nx
n
n−k
A2 = ∑ k2 Cnk xk (1 − x)
k =0
n
= n ∑ kCnk− 1 k
−1 x (1 − x )
n−k

k =1
n n
= nx ∑ Cnk−−11 xk−1 (1 − x)n−k + n ∑ (k − 1)Cnk−−11 xk (1 − x)n−k
k =1 k =1
n
= nx + n(n − 1) x2 ∑ Cnk−−22 xk−2 (1 − x)n−k = nx + n(n − 1)x2 .
k =2
n
n−k
A3 = ∑ Cnk xk (1 − x) = [ x + (1 − x )]n = 1.
k =0

Vậy A = (nx )2 + nx + n(n − 1) x2 − 2(nx )2 = nx (1 − x ).


Áp dụng kết quả trên ta được F ( x ) = 2011x (1 − x ).
Do x ∈ [0; 1] nên x ≥ 0, 1 − x ≥ 0. Áp dụng bất đẳng thức AM-GM, ta có
x + (1 − x )
 
1
F ( x ) ≤ 2011 . Dấu đẳng thức xảy ra khi x = .
2 2
2011 1
Vậy max F ( x ) = ⇔x= .
[0;1] 4 2
Nhận xét 10. Ta có thể thay số 2011 trong hàm số f ( x ) bởi một số thực dương bất kì.
Xuất phát từ

129
Hội thảo khoa học, Ninh Bình 15-16/09/2018

(−1)k Cnkn p(n)


Bài toán 18 (Canada 1997). Hãy viết tổng ∑ 3 2
dưới dạng , trong
k=0 k + 9k + 26k + 24 q(n)
đó p(n), q(n) là các đa thức với hệ số nguyên, ta có thể xây dựng được bài toán cực trị
liên quan đến tổ hợp là bài toán sau:

Bài toán 19. Tìm giá trị nhỏ nhất của biểu thức
n
(−1)k C k 2n4 − 10n3 − 64n2 + 272n + 959
f (n) = ∑ k3 + 9k2 + 26kn + 24 + 2(n + 3)(n + 4)
.
k =0

Nhận xét 11. Để giải bài toán này, trước hết ta phải sử dụng công thức tính số tổ hợp,
tính chất của các số tổ hợp, công thức khai triển nhị thức Newton để rút gọn biểu thức
f ( n ).
Lời giải. Đặt
n
(−1)k Cnk
S(n) = ∑ 3 .
k =0
k + 9k2 + 26k + 24
Ta có
k3 + 9k2 + 26k + 24 = (k + 2)(k + 3)(k + 4),
nên
n
(−1)k .n!
S(n) = ∑ k!.(n − k)!(k + 2)(k + 3)(k + 4)
k =0
n 
(−1)k .(n + 4)!
 
k+1
= ∑ (k + 4)!.(n − k)! (n + 1)(n + 2)(n + 3)(n + 4)
.
k =0

Đặt
n
T (n) = (n + 1)(n + 2)(n + 3)(n + 4)S(n) = ∑ Cnk++44 (k + 1)(−1)k .
k =0

Ta có
n n
∑ (−1)i Cni = (x − 1)n = 0 ⇒ −n ∑ (−1) j Cn−1 = 0,
j

i =0 j =0


n n
i.n! 0.n!
∑ (−1)i Cni .i = ∑ (−1)i i!(n − i)! + (−1)0 0!n!
i =0 i =1
n n
n!
= ∑ (−1)i (i − 1)!(n − i)! = ∑ (−1)i nCni−−11
i =1 i =1
n n
= n ∑ (−1)i Cni−−11 = −n ∑ (−1)i−1 nCni−−11 = 0.
i =1 i =1

130
Hội thảo khoa học, Ninh Bình 15-16/09/2018

Do đó
n
T (n) = ∑ (−1)k Cnk++44 (k + 1)
k =0
n
= ∑ (−1)k+4 Cnk++44 (k + 1)
k =0
n
= ∑ (−1)k+4 Cnk+ 4 2
+4 ( k + 1) − (−3 + 2( n + 4) − Cn+4 )
k=−4
n +4  
(n + 4)(n + 3)
∑ (−1) j j
= Cn+4 ( j − 3) − 2n + 8 − 3 −
j =0
2
n +4 n +4
1
∑ (−1) j Cn+4 j − 3 ∑ (−1) j Cn+4 − 2 (4n + 10 − n2 − 7n − 12)
j j
=
j =0 j =0
1 1
= 0 + 0 + (n2 + 3n + 2) = (n + 1)(n + 2).
2 2
Suy ra

T (n) (n + 1)(n + 2)
S(n) = = .
(n + 1)(n + 2)(n + 3)(n + 4) (n + 1)(n + 2)(n + 3)(n + 4)
Vậy
1
S(n) = .
2(n + 3)(n + 4)
Do đó
1 2n4 − 10n3 − 64n2 + 272n + 959
f (n) = +
2(n + 3)(n + 4) 2(n + 3)(n + 4)
4 3 2
2n − 10n − 64n + 272n + 960
=
2(n + 3)(n + 4)
2(n + 3)(n + 4)(n2 − 12n + 40)
=
2(n + 3)(n + 4)
= n2 − 12n + 40.

Ta có

n2 − 12n + 40 = (n − 6)2 + 4 ≥ 4, ∀n ∈ N. Dấu đẳng thức xảy ra khi n = 6

Vậy f (n) đạt giá trị nhỏ nhất bằng 4 khi n = 6.

Bài toán 20. Trong một cuộc thi có 11 thí sinh tham gia giải 9 bài toán. Hai thí sinh bất kì
giải chung với nhau không quá 1 bài. Tìm k lớn nhất để mọi bài có ít nhất k thí sinh giải
được.
Lời giải. Gọi Hi là thí sinh thứ i và tập các bài toán là {b1 , b2 , . . . , b9 }
Theo đề bài ta có Hi ∩ Hj ≤ 1, ∀i 6= j.
Đặt ni là số thí sinh giải được bài bi Ta đi đếm bộ (bi , Hj , Hi ), trong đó bi ∈ Hj ∩ Hl .

131
Hội thảo khoa học, Ninh Bình 15-16/09/2018

Ta có số bộ này chính bằng ∑ Hi ∩ Hj
i< j
9
Mặt khác, số bộ này lại bằng ∑ Cn2i . Do đó ta có
i =1

9
∑ Hi ∩ Hj = ∑ Cn2 .

i
i< j i =1

Suy ra
9
∑ n2i − ni ≤ 2 ∑ Hi ∩ Hj ≤ 2.C11
2
= 110 ⇒ 9(k2 − k) ≤ 110 ⇒ k ≤ 4.

i =1 i< j

9
Với k = 4, giả sử tồn tại ni ≥ 5, suy ra ∑ d2i − di ≥ 8.12 + 20 = 116 (vô lí).

i =1
Suy ra
11
| Hi | ≥ 4 ⇒ ∑ | Hi | ≥ 36 + | H1 | + | H2 | > 36.
i =1

∑ Hi ∩ Hj = 54 = C112 − 1.

ni = 4, ∀i = 1, 9 ⇒
i< j

Hi ∩ H
Do đó tồn tại (i, j) sao cho j = ∅.
Giả sử H1 ∩ H2 = ∅ và Hi ∩ Hj = 1, ∀t ∈ {1, 2, . . . , 11} \ {i } .
Nên tồn tại một phần tử của Hi thuộc ít nhất 10
 
3 + 1 = 4 tập Ht , t 6 = i.
Suy ra sự tồn tại một phần tử thuộc nhiều hơn 5 tập Hj , vô lí.
Do đó k ≤ 3. Với k = 3 ta chỉ ra như sau:
Quy ước:
Số 1 là thí sinh giải được bài đó.
Số 0 là thí sinh không giải được bài đó.

b1 b2 b3 b4 b5 b6 b7 b8 b9
H1 1 0 0 1 0 0 1 0 0
H2 1 0 0 0 1 0 0 0 1
H3 0 1 1 0 0 1 0 1 0
H4 0 1 0 1 1 0 0 0 0
H5 0 0 0 1 0 0 0 1 1
H6 0 0 1 0 0 0 1 0 1
H7 0 0 0 0 1 1 0 0 0
H8 1 1 0 0 0 0 0 0 0
H9 0 0 1 0 0 0 0 0 0
H10 0 0 0 0 0 1 1 0 0
H11 0 0 0 0 0 0 0 1 0

Vậy k lớn nhất thỏa mãn yêu cầu bài toán bằng 3.

132
Hội thảo khoa học, Ninh Bình 15-16/09/2018

Tài liệu
[1] Nguyễn Văn Mậu (2006), Bất đẳng thức, định lí và áp dụng, NXB Giáo dục.
[2] Nguyễn Văn Mậu (2007), Đa thức đại số và phân thức hữu tỷ, NXB Giáo dục.
[3] Nguyễn Văn Mậu (2010), Số phức và áp dụng, NXB Giáo dục.
[4] Lê Hoành Phò, Nguyễn Văn Nho, Nguyễn Tài Chung (2013), Chuyên khảo đa thức,
NXB Đại học Quốc gia Hà Nội.
[5] Ban tổ chức kì thi Tổng tập đề thi Olympic 30 tháng 4 Toán học 10 (2002-2012), NXB Đại
học Sư phạm.
[6] Hà Văn Chương (2002), 342 bài toán giải tích tổ hợp, NXB Giáo dục.
[7] Tủ sách Toán học tuổi trẻ, Các bài thi Olympic Toán trung học phổ thông Việt Nam (1990-
2006), NXB Giáo dục.
[8] Luis Comlet (1974), Advanced combinatorics, D.Reidel Pub. Com.
[9] Titu Andreescu, Zuming Feng (2002), 102 combinatorial problems from the training of
the USA IMO team.

133
Hội thảo khoa học, Ninh Bình 15-16/09/2018

MỘT VÀI ỨNG DỤNG CỦA NGUYÊN LÝ


ĐIỂM BẤT ĐỘNG TRONG DÃY SỐ

Dương Trọng Luyện


Khoa Tự nhiên, Trường Đại học Hoa Lư

Tóm tắt nội dung

Lý thuyết điểm bất động và ứng dụng là lĩnh vực nghiên cứu hấp dẫn của toán học
hiện đại. Đây là lĩnh vực đã và đang thu hút được sự quan tâm của rất nhiều nhà toán
học trong và ngoài nước.
Lý thuyết điểm bất động là một công cụ quan trọng để nghiên cứu các hiện tượng phi
tuyến. Nó có nhiều ứng dụng trong nhiều lĩnh vực khác nhau của Toán học như sự tồn
tại nghiệm của các phương trình vi, tích phân, hệ phương trình tuyến tính, phương trình
hàm, quỹ đạo đóng của hệ động lực...
Hơn nữa, nó còn có nhiều ứng dụng trong các ngành khoa học khác như khoa học
máy tính, lý thuyết điều khiển, lý thuyết trò chơi, vật lý toán, sinh học, kinh tế... Sự phát
triển mạnh mẽ của lý thuyết điểm bất động có thể nói bắt nguồn từ những ứng dụng
rộng rãi của nó.
Hơn nữa việc ứng dụng điểm bất động trong giải toán sơ cấp cũng là vấn đề được
nhiều người quan tâm, đặc biệt là việc tìm giới hạn của dãy số, phương trình hàm, ....
Như ta đã biết dãy số đặc biệt quan trọng trong Toán học không chỉ là đối tượng để
nghiên cứu mà con đóng vai trò như là một công cụ đắc lực của các mô hình toán rời rạc
của giải tích trong lý thuyết phương trình, lý thuyết xấp xỉ, lý thuyết biểu diễn, ....
Trong nhiều kỳ thi học sinh giỏi quốc gia, thi Olympic toán Quốc tế, thi vô địch toán
các nước, các bài toán liên quan đến dãy số cũng hay được đề cập và thường thuộc loại
khó.
Trong bài viết này tôi xin trình bày ứng dụng của nguyên lý điểm bất động trong việc
tìm giới hạn của dãy số.

1 Nguyên lý điểm bất động


Thông thường khi một ánh xạ tác động vào một phần tử sẽ sinh ra giá trị hàm khác
với đối số. Nếu lặp ánh xạ ấy một cách đệ qui, nghĩa là lấy giá trị hàm làm đối số cho lần
lặp tiếp theo thì chúng ta thấy một cách trực quan rằng ánh xạ nhảy theo các giá trị mới.
Nếu có một phần tử mà được ánh xạ tới chính nó sẽ làm ánh xạ không di chuyển được.
Phần tử đó được gọi là điểm bất động của ánh xạ.

Định nghĩa 1. Điểm x ∗ là một điểm bất động của hàm f ( x ) nếu và chỉ nếu f ( x ∗ ) = x ∗ .

134
Hội thảo khoa học, Ninh Bình 15-16/09/2018

Chúng ta phân biệt điểm bất động với điểm cố định. Điểm cố định là điểm mà tại đó
hàm số không biến thiên, tức là f 0 ( x ) = 0. Còn điểm bất động cho thấy tại đó dù lặp ánh
xạ bao nhiêu lần đi nữa thì giá trị của hàm số tại điểm đó vẫn không thay đổi, tức là

f ( f (. . . f ( x ∗ ))) = x ∗ .

Định nghĩa 2 (Ánh xạ co). Cho D ⊂ R. Ánh xạ f : D → D được gọi là một ánh xạ co
trên D nếu và chỉ nếu tồn tại λ ∈ [0, 1) sao cho với mọi x, y ∈ D ta có

| f ( x ) − f (y)| ≤ λ | x − y| ,

λ được gọi là hằng Lipschitz.

Định lý 1 (Nguyên lý điểm bất động). Cho k là số tự nhiên, tập D ≡ R hoặc D =


k
[ ai−1 , ai ], [ ai−1 , ai ] ⊂ R, với ánh xạ co f : D → D. Khi đó f có một điểm bất động duy
S
i =1
nhất x ∗ ∈ D thỏa mãn với mọi x0 ∈ D

lim f n ( x0 ) = x ∗ ,
n→∞

trong đó f 0 ( x0 ) = x0 , f n ( x0 ) = f ( f n−1 ( x0 )).

Chứng minh. Dãy { f n ( x0 )} hội tụ tới x ∗ . Thật vậy, ta có

| x − y| ≤ | x − f ( x )| + | f ( x ) − f (y)| + | f (y) − y| .

Do | f ( x ) − f (y)| ≤ λ | x − y| nên ta có

| x − y| ≤ | x − f ( x )| + λ | x − y| + | f (y) − y|

suy ra

| f ( x ) − x | + | f (y) − y|
| x − y| ≤ . (1.1)
1−λ
Thay x ≡ f n ( x0 ), y ≡ f m ( x0 ) vào (1.1) ta có
n +1 n ( x ) + f m +1 ( x ) − f m ( x )

f ( x 0 ) − f 0 0 0
| f n ( x0 ) − f m ( x0 )| ≤ . (1.2)
1−λ

2
f ( x0 ) − f 1 ( x0 ) ≤ λ | f ( x0 ) − x0 | ,

nên bằng phương pháp quy nạp chúng ta có



n +1
f ( x0 ) − f n ( x0 ) ≤ λ n | f ( x0 ) − x0 | . (1.3)

Từ (1.3) và (1.2) chúng ta có

λn + λm
| f n ( x0 ) − f m ( x0 )| ≤ | f ( x0 ) − x0 | . (1.4)
1−λ

135
Hội thảo khoa học, Ninh Bình 15-16/09/2018
n m
Do λ < 1, nên λ1+−λ | f ( x0 ) − x0 | → 0 khi m, n → ∞, nên dãy { f ( x0 )} là dãy Cauchy, và
λ n

D là tập đóng trong R, do vậy tồn tại x ∗ ∈ D thỏa mãn

lim f n ( x0 ) = x ∗ .
n→∞

Ta chứng minh x ∗ là điểm bất động. Thật vậy, từ (1.4) chúng ta có

λn
| f n ( x0 ) − x ∗ | ≤ | f ( x0 ) − x0 | .
1−λ

Theo giả thiết ta có


λ n +1
( x0 ) − f ( x ) ≤ λ | f n ( x0 ) − x ∗ | ≤

n +1
f | f ( x0 ) − x0 | . (1.5)
1−λ
λ n +1
Ta có lim | f ( x0 ) − x0 | = 0, suy ra lim f n+1 ( x0 ) = lim f n ( x0 ) = f ( x ∗ ). Do tính giới
n → ∞ 1− λ n→∞ n→∞
hạn là duy nhất nên f ( x ∗ ) = x ∗ .
Ta chứng minh x ∗ là điểm bất động duy nhất. Thật vậy, giả sử có hai điểm bất động
x và y. Chúng ta có f ( x ) = x và f (y) = y. Suy ra | f ( x ) − x | = 0 và | f (y) − y| = 0. Áp
dụng vào bất đẳng thức (1.1) thì vế phải của (1.1) bằng 0, do đó | x − y| ≤ 0, tức là x = y.
Vậy hai điểm bất động này phải là một nên điểm bất động x ∗ là duy nhất.

Định lý 2. Giả sử ánh xạ f : [ a, b] → [ a, b] hoặc f : R → R là ánh xạ khả vi thỏa mãn


| f 0 ( x )| ≤ d < 1 với mọi x. Khi đó f là ánh xạ co.
Định lý 3. Giả sử ánh xạ f : [ a, b] → [ a, b] hoặc f : R → R là ánh xạ co, và dãy {un } xác
định bởi u0 ∈ [ a, b], un+1 = f (un ). Khi đó dãy số {un } có giới hạn u∗ là điểm bất động
của ánh xạ f .

Chứng minh. Ta có

|un+1 − un | = | f (un ) − f (un−1 )| ≤ λ |un − un−1 | .

Áp dụng liên tiếp các bất đẳng thức ta được

|un+1 − un | ≤ λn |u1 − u0 | , ∀n ≥ 0.

Do vậy

|un+k − un | = |(un+k − un+k−1 ) + (un+k−1 − un+k−2 ) + · · · + (un+1 − un )|


≤ ( λ n + k −1 + λ n + k −2 + · · · + λ n ) | u 1 − u 0 |
λn
≤ | u1 − u0 | .
1−λ
Do λ ∈ [0, 1) nên
" ε (1− λ ) #
λn ln |u
1 − u0 |
|u1 − u0 | < ε với n > .
1−λ ln λ

Vậy dãy {un } là dãy hội tụ theo tiêu chuẩn Cauchy. Nên giả sử lim un = u∗ khi đó
n→∞
lim un+1 = u∗ và do f là ánh xạ co nên liên tục do vậy lim f (un ) = f (u∗ ) = u∗ .
n→∞ n→∞

136
Hội thảo khoa học, Ninh Bình 15-16/09/2018

2 Phương pháp tìm giới hạn của dãy nhờ sử dụng


nguyên lý điểm bất động
Để giải bài toán tìm giới hạn của dãy số nhờ sử dụng phương pháp điểm bất động ta
nên tiến hành theo 3 bước sau:
Bước 1. Tìm miền giá trị của dãy số nếu có sau đó viết dãy số đã cho dưới dạng
u n +1 = f ( u n ).
Bước 2. Xác định ánh xạ co từ hàm số f trong bước này ánh xạ có thể có từ một chỉ số
nào đó của n.
- Bản thân f không phải là ánh xạ co nhưng ánh xạ g = f 2 là ánh xạ co, khi này ta xét
hai dãy chẵn và lẻ. Để chứng minh ánh xạ là co ta nên sử dụng Định lý 2.
Bước 3. Sử dụng các Định lý 1 và Định lý 3 ta có điều phải chứng minh.
Bài toán 1. Cho dãy số thực {un } xác định như sau:
1
u1 = a; un+1 = ln(1 + u2n ) − b, n ≥ 1,
2
trong đó a, b là các hằng số cho trước. Chứng minh rằng dãy số {un } là dãy hội tụ.

Chứng minh. Ta có un ∈ R với mọi n ∈ N. Xét hàm số f ( x ) = 21 ln(1 + x2 ) − b trên R.


Khi đó chúng ta có

0 x 0 x
≤ 1 , ∀ x ∈ R,
f (x) = 2
⇒ f (x) =

2
1+x 1+x 2

Nên f ( x ) là ánh xạ co trên R. Do vậy theo Định lý 3 ta có điều phải chứng minh.
Bài toán 2. Cho dãy số thực {un } xác định như sau:
p
u1 = u; un+1 = 2un + 1, n ≥ 1,

trong đó u là hằng số dương cho trước. Chứng minh rằng dãy số {un } là dãy hội tụ.

Chứng minh.
√ Từ định nghĩa của dãy {un } ta có un ∈ [0, +∞] với mọi n ∈ N. Xét hàm số
f ( x ) = 2x + 1. Khi đó chúng ta có
1
f 0 (x) = √
2x + 1

√ là ánh xạ co trên [0, +∞]. Tuy vậy f ( x ) > 0 nên f ( x ) đồng biến. Do
Nên f ( x ) chưa phải 0

vậy u2 = f (u1 ) = 2u + 1 ≥ 1 suy ra un ≥ 1 với n ≥ 1. Với x ≥ 1 ta có


1 1
f 0 (x) = √ ≤ √ < 1, ∀ x ≥ 1.
2x + 1 3
Nên f ( x ) là ánh xạ co trên [0, +∞]. Mặt khác ta có

f ( x ) = x, x ≥ 1 ⇒ x = 1 + 2.

Do vậy theo Định lý 3 ta có √


lim un = 1 + 2.
n→∞

137
Hội thảo khoa học, Ninh Bình 15-16/09/2018

k
Bài toán 3. Cho D ≡ R hoặc D = [ ai−1 , ai ], [ ai−1 , ai ] ⊂ R, ánh xạ f : D → D là ánh xạ
S
i =1
liên tục sao cho tồn tại α, β ∈ [0, 1) với α + β < 1 thỏa mãn

| x − f ( x )| |y − f (y)|
| f ( x ) − f (y)| ≤ α + β | x − y| ,
a + | x − y|

với mọi x, y ∈ D và a là hằng số dương cho trước. Lấy u0 ∈ D xét dãy số thực {un } xác
định như sau:
u1 = f (u0 ); un+1 = f (un ), n ≥ 1, n ∈ N.
Chứng minh rằng dãy số {un } là dãy hội tụ.

Chứng minh. Đặt k =


β
1− α ∈ [0, 1). Với mọi n ∈ N ta có

|un+1 − un | = | f (un ) − f (un−1 )|


|un − f (un )| |un−1 − f (un−1 )|
≤α + β | u n − u n −1 |
a + | u n − u n −1 |
| u n − u n +1 | | u n −1 − u n |
=α + β | u n − u n −1 |
a + | u n − u n −1 |
≤ α | u n − u n +1 | + β | u n − u n −1 | .

Suy ra

| u n +1 − u n | ≤ k | u n − u n −1 | ≤ k 2 | u n −1 − u n −2 | ≤ · · · ≤ k n | u 1 − u 0 | .

Do đó sử dụng chứng minh trong Định lý 3 ta có điều phải chứng minh.

Bài toán 4 (Đề thi HSG Vĩnh Phúc vòng 1, năm học 2011 -2012). Cho u là số thực dương
thỏa mãn 0 < u < 1. Lập dãy {un } như sau

u1 = u, un+1 = uun , n ≥ 1.

Chứng minh rằng dãy số {un } là dãy hội tụ.

Chứng minh. Ta thấy rằng un ∈ [0, 1]. Xét hàm số f ( x ) = u x trên [0, 1]. Khi đó ta có

f ( x ) = |u x ln u| ≤ |u ln u| = u ln 1 = u ln(1 + 1 − 1).
0
u u

Áp dụng bất đẳng thức ln(1 + t) ≤ t, với mọi t > 0, ta có

f ( x ) ≤ u( 1 − 1) = 1 − u < 1.
0
u
Nên f ( x ) là ánh xạ co trên [0, 1]. Do vậy, theo Định lý 3 ta có điều phải chứng minh.

Bài toán 5. Cho u là số thực dương thỏa mãn 0 < u < 1. Lập dãy {un } như sau
s
un + k
u1 = u, un+1 = 9 , n ≥ 1, k > 1.
kun + 1

138
Hội thảo khoa học, Ninh Bình 15-16/09/2018

Chứng minh rằng dãy số {un } là dãy hội tụ.

Chứng minh. Từ công thức xác q định dãy số, bằng phương pháp quy nạp ta có được
x +k
un ∈ [0, 1]. Xét hàm số f ( x ) = 9 kx +1 , k > 1 trên [0, 1]. Khi đó ta có

1 − k2 1
f 0 (x) = ⇒ f 0 ( x ) < .

p
9
9(kx + 1) (kx + 1)( x + k ) 8 9

Nên f ( x ) là ánh xạ co trên [0, 1]. Do vậy theo Định lý 3 ta có điều phải chứng minh.

3 Bài tập áp dụng


Bài 1 (Đề thi HSG Cần Thơ vòng 2, năm học 2011 -2012). Cho dãy số thực {un } xác định
như sau:
2011
u1 = a; un+1 = ln(u2n + 20112 ) − 20112 , n ≥ 1,
3
trong đó a là hằng số cho trước. Chứng minh rằng dãy số {un } là dãy hội tụ.

Bài 2 (Đề thi HSG Bình Dương vòng 2, năm học 2011 -2012). Cho dãy số thực {un } xác
định như sau:
1 b
u1 = a; un+1 = (un + ), n ≥ 1,
2 un
trong đó a, b là các hằng số dương cho trước. Chứng minh rằng dãy số {un } là dãy hội
tụ.

Bài 3 (Đề thi VMO 2000). Cho dãy số thực {un } xác định như sau:
q √
u1 = 0; un+1 = a − un + a, n ≥ 1,

trong đó a > 2 là hằng số cho trước. Chứng minh rằng dãy số {un } là dãy hội tụ.

Bài 4 (Đề dự bị VMO 2008). Cho dãy số thực {un } xác định như sau:

u1 = a; un+1 = ln(3 + cos un + sin un ) − 2008, n ≥ 1,

trong đó a là hằng số cho trước. Chứng minh rằng dãy số {un } là dãy hội tụ.

Bài 5 (Đề thi Olympic sinh viên 2015, Đại học Hoa Lư). Cho dãy số thực {un } xác định
như sau:
u1 = 1; un+1 = 2014 ln(u2n + 20152 ) − 20142 , n ≥ 1.
Chứng minh rằng dãy số {un } là dãy hội tụ.

Bài 6. Cho dãy số thực {un } xác định như sau:

1
u1 = 1; un+1 = , n ≥ 1.
un + 2

Chứng minh rằng dãy số {un } là dãy hội tụ.

139
Hội thảo khoa học, Ninh Bình 15-16/09/2018

Bài 7. Cho dãy số thực {un } xác định như sau:

1
u1 = u; un+1 = , n ≥ 1,
1 + un

trong đó u là hằng số không âm cho trước. Chứng minh rằng dãy số {un } là dãy hội tụ.

Bài 8. Cho dãy số thực {un } xác định như sau:

u1 = 0; un+1 = a + u2n , n ≥ 1, a > 0.

Tìm điều kiện cần và đủ để dãy số {un } là dãy hội tụ.

Tài liệu
[1] Tô Văn Ban (2005), Giải tích những bài tập ứng dụng, NXBGD.
[2] Trần Lưu Cường (2000), Toán Olympic cho sinh viên , tập 1, NXBGD.
[3] Nguyễn Văn Mậu (2007), Các bài toán về dãy số, NXBGD.
[4] IMO longlist 1996-2002, accessed March, 2003.

140
Hội thảo khoa học, Ninh Bình 15-16/09/2018

MỘT SỐ BẤT ĐẲNG THỨC ỨNG VỚI CÁC


TAM GIÁC ĐẲNG CHU

Hoàng MInh Quân


Trường THPT Ngọc Tảo, Hà Nội

Tóm tắt nội dung

Bất đẳng thức trong tam giác là một chủ đề hay, xuất hiện trong nhiều kì thi học sinh
giỏi các cấp. Bài viết sau đây trình bày về một lớp các bài toán bất đẳng thức trong tam
giác có cùng chu vi (bằng 1) được giải quyết và sáng tạo bằng cách sử dụng bất đẳng
thức Jensen mở rộng và bất đẳng thức Popoviciu mở rộng.
Hi vọng thông qua bài báo này bạn đọc có thể sáng tạo ra rất nhiều bài toán hay và đẹp
khác bằng cách xây dựng thông qua các hàm số thích hợp.

Ý tưởng cho bài viết này xuất phát từ những lần nói chuyện, trao đổi của tác giả với
PGS.TS Tạ Duy Phượng về bất đẳng thức trội và các vấn đề liên quan, và học hỏi cách
phát triển bài toán từ GS.TSKH Nguyễn Văn Mậu thông qua các lần hội thảo toán học.

1 Bất đẳng thức Jensen mở rộng cho tam giác có chu


vi bằng 1
Định lý 1. Cho tam giác ABC có chu vi bằng 1 và x, y, z là các số thực dương. Khi đó ta có

a b c 1
+ 2+ 2 ≥ . (1.1)
x 2 y z ( ax + by + cz)2
1 −2
Chứng minh. Xét hàm số f ( x ) = 2 trên khoảng (0, +∞) , ta có f 0 ( x ) = 3 , suy ra
x x
6 1
f ( x ) = 4 > 0, ∀ x ∈ (0; +∞) nên hàm số f ( x ) = 2 lồi trên khoảng (0, +∞) .
00
x x
Áp dụng bất đẳng thức Jensen, ta được
 
a f ( x ) + b f (y) + c f (z) ax + by + cz
≥ f
a+b+c a+b+c

tương đương

a f ( x ) + b f (y) + c f (z) ≥ f ( ax + by + cz) vì a+b+c = 1

141
Hội thảo khoa học, Ninh Bình 15-16/09/2018

tương đương
a b c 1
+ 2+ 2 ≥ , ∀ x, y, z > 0.
x 2 y z ( ax + by + cz)2
Vậy Định lí (1) được chứng minh. Bất đẳng thức (1.1) là bất đẳng thức có nhiều ứng
dụng, từ bất đẳng thức này chúng ta xây dựng được nhiều bất dẳng thức mới như sau

Bài toán 1. Cho tam giác ABC có chu vi bằng 1. Chứng minh rằng

a b c 1
+ 2+ 2 ≥ . (1.2)
2
ma mb mc ( am a + bmb + cmc )2

Bài toán 2. Cho tam giác ABC có chu vi bằng 1. Chứng minh rằng

a b c 1
+ 2+ 2 ≥ . (1.3)
2
la lb lc ( ala + blb + clc )2

Bài toán 3. Cho tam giác ABC có chu vi bằng 1. Chứng minh rằng

a b c 1
+ 2+ 2 ≥ . (1.4)
2
ra rb rc ( ar a + brb + crc )2

Bài toán 4. Cho tam giác ABC có chu vi bằng 1. Chứng minh rằng

a b c 1
+ 2+ 2 ≥ . (1.5)
2
ha hb hc ( ah a + bhb + chc )2

Bài toán 5. Cho tam giác ABC có chu vi bằng 1. Chứng minh rằng

2R 2R 2R 1
+ + ≥ . (1.6)
sin A sin B sin C ( a sin A + b sin B + c sin C )2

Bài toán 6. Cho tam giác nhọn ABC có chu vi bằng 1. Chứng minh rằng

a b c 1
+ + ≥ . (1.7)
2 2 2
cos A cos B cos C ( a cos A + b cos B + c cos C )2

Bài toán 7. Cho tam giác nhọn ABC có chu vi bằng 1. Chứng minh rằng

a b c 1
2
+ 2
+ 2
≥ . (1.8)
tan A tan B tan C ( a tan A + b tan B + c tan C )2

Bài toán 8. Cho tam giác ABC có chu vi bằng 1. Chứng minh rằng

a b c 1
2
+ 2
+ 2
≥ . (1.9)
(m a la ) ( m b lb ) ( m c lc ) ( am a la + bmb lb + cmc lc )2

Bài toán 9. Cho tam giác ABC có chu vi bằng 1. Chứng minh rằng

a b c 1
2
+ 2
+ 2
≥ . (1.10)
(m a h a ) (mb hb ) (mc hc ) ( am a h a + bmb hb + cmc hc )2

142
Hội thảo khoa học, Ninh Bình 15-16/09/2018

Bài toán 10. Cho tam giác ABC có chu vi bằng 1. Chứng minh rằng

a b c 1
2
+ 2
+ 2
≥ . (1.11)
(m a r a ) (mb rb ) (mc rc ) ( am a r a + bmb rb + cmc rc )2

Bài toán 11. Cho tam giác ABC có chu vi bằng 1. Chứng minh rằng

a b c 1
2
+ 2
+ 2
≥ . (1.12)
(la h a ) ( lb hb ) ( lc hc ) ( ala h a + blb hb + clc hc )2

Bài toán 12. Cho tam giác ABC có chu vi bằng 1. Chứng minh rằng

a b c 1
2
+ 2
+ 2
≥ . (1.13)
(la r a ) ( lb rb ) ( lc rc ) ( ala r a + blb rb + clc rc )2

Bài toán 13. Cho tam giác ABC có chu vi bằng 1. Chứng minh rằng

a b c 1
2
+ 2
+ 2
≥ . (1.14)
(ha ra ) ( hb rb ) ( hc rc ) ( ah a r a + bhb rb + chc rc )2

Định lý 2. Cho tam giác ABC có chu vi bằng 1 và x, y, z, α là các số thực. Khi đó

aeαx + beαy + ceαz ≥ eα(ax+by+cz) . (1.15)

Chứng minh. Xét hàm số f ( x ) = eαx có f 0 ( x ) = αeαx , f 00 ( x ) = α2 eαx > 0, ∀ x ∈ R nên hàm
số f ( x ) = eαx là hàm lồi trên R.
Áp dụng bất đẳng thức Jensen mở rộng, ta có
 
a f ( x ) + b f (y) + c f (z) ax + by + cz
≥ f
a+b+c a+b+c

tương đương

a f ( x ) + b f (y) + c f (z) ≥ f ( ax + by + cz) (Vì a+b+c=1)

tương đương
aeαx + beαy + ceαz ≥ eα(ax+by+cz) .
Vậy định lí (1.15) được chứng minh xong.

Hệ quả 1. Với α = 1, ta có bất đẳng thức

ae x + bey + cez ≥ e ax+by+cz (1.16)

Sử dụng hệ quả (1) ta chứng minh được các bất đẳng thức sau

Bài toán 14. Cho tam giác ABC có chu vi bằng 1. Chứng minh rằng
2 + b2 + c2
ae a + beb + cec ≥ e a . (1.17)

Bài toán 15. Cho tam giác ABC có chu vi bằng 1. Chứng minh rằng

aema + bemb + cemc ≥ e ama +bmb +cmc . (1.18)

143
Hội thảo khoa học, Ninh Bình 15-16/09/2018

Bài toán 16. Cho tam giác ABC có chu vi bằng 1. Chứng minh rằng

aeha + behb + cehc ≥ e aha +bhb +chc . (1.19)

Bài toán 17. Cho tam giác ABC có chu vi bằng 1. Chứng minh rằng

aela + belb + celc ≥ e ala +blb +clc . (1.20)

Bài toán 18. Cho tam giác ABC có chu vi bằng 1. Chứng minh rằng

aera + berb + cerc ≥ e ara +brb +crc . (1.21)

Bài toán 19. Cho tam giác ABC có chu vi bằng 1. Chứng minh rằng

aesin A + besin B + cesin C ≥ e a sin A+b sin B+c sin C . (1.22)

Bài toán 20. Cho tam giác ABC có chu vi bằng 1. Chứng minh rằng

aecos A + becos B + cecos C ≥ e a cos A+b cos B+c cos C . (1.23)

Bài toán 21. Cho tam giác nhọn ABC có chu vi bằng 1. Chứng minh rằng

aetan A + betan B + cetan C ≥ e a tan A+b tan B+c tan C . (1.24)

Định lý 3. Cho tam giác ABC có chu vi bằng 1 và x, y, z là các số thực dương tùy ý. Khi đó
√ √ √ p
a x + b y + c z ≤ ax + by + cz. (1.25)

√ 1 −1
Chứng minh. Xét hàm số f ( x ) = x với x > 0, ta có f 0 ( x ) = √ , f 00 ( x ) = √ <
√ 2 x 4x x
0, ∀ x > 0. Do đó f ( x ) = x là hàm lõm trên (0; +∞).
Áp dụng bất đẳng thức Jensen mở rộng, ta có
 
a f ( x ) + b f (y) + c f (z) ax + by + cz
≤ f
a+b+c a+b+c

tương đương

a f ( x ) + b f (y) + c f (z) ≤ f ( ax + by + cz) (Vì a+b+c=1)

tương đương √ √
√ p
a x + b y + c z ≤ ax + by + cz.
Vậy Định lí (3) được chứng minh.
Sử dụng Định lí (3), ta sẽ chứng minh được các bài toán sau

Bài toán 22. Cho tam giác ABC có chu vi bằng 1. Chứng minh rằng
√ √ √ √
a b + b c + c a ≤ ab + bc + ca. (1.26)

Bài toán 23. Cho tam giác ABC có chu vi bằng 1. Chứng minh rằng
√ √ √ p
a m a + b mb + c mc ≤ am a + bmb + cmc . (1.27)

144
Hội thảo khoa học, Ninh Bình 15-16/09/2018

Bài toán 24. Cho tam giác ABC có chu vi bằng 1. Chứng minh rằng
p p p √
a h a + b hb + c hc ≤ 6S. (1.28)

Bài toán 25. Cho tam giác ABC có chu vi bằng 1. Chứng minh rằng
p p p p
a la + b lb + c lc ≤ ala + blb + clc . (1.29)

Bài toán 26. Cho tam giác ABC có chu vi bằng 1. Chứng minh rằng
√ √ √ p
a r a + b rb + c rc ≤ ar a + brb + crc . (1.30)

Bài toán 27. Cho tam giác ABC có chu vi bằng 1. Chứng minh rằng
√ √ √ √
a sin A + b sin B + c sin C ≤ a sin A + b sin B + c sin C. (1.31)

Bài toán 28. Cho tam giác nhọn ABC có chu vi bằng 1. Chứng minh rằng
√ √ √ √
a cos A + b cos B + c cos C ≤ a cos A + b cos B + c cos C. (1.32)

Bài toán 29. Cho tam giác nhọn ABC có chu vi bằng 1. Chứng minh rằng
√ √ √ √
a tan A + b tan B + c tan C ≤ a tan A + b tan B + c tan C. (1.33)

Định lý 4. Cho tam giác ABC có chu vi bằng 1 và x, y, z là các số thực dương tùy ý. Khi đó

x a yb zc ≤ ax + by + cz. (1.34)
1 00 −1
Chứng minh. Xét hàm số f ( x ) = ln x trên (0; +∞), ta có f 0 ( x ) = , f (x) = 2 <
x x
0, ∀ x > 0 nên hàm số f ( x ) = ln x lõm trên khoảng (0; +∞).
Áp dụng bất đẳng thức Jensen mở rộng, ta có
 
a f ( x ) + b f (y) + c f (z) ax + by + cz
≤ f
a+b+c a+b+c
tương đương

a f ( x ) + b f (y) + c f (z) ≤ f ( ax + by + cz) (Vì a+b+c=1)

tương đương
a ln x + b ln y + c ln z ≤ ln ( ax + by + cz)
tương đương  
ln x a yb zc ≤ ln ( ax + by + cz)

tương đương
x a yb zc ≤ ax + by + cz.
Vậy Định lí (4) được chứng minh.
Áp dụng Định lí (4) ta chứng minh được các bất đẳng thức sau
Bài toán 30. Cho tam giác ABC có chu vi bằng 1. Chứng minh rằng

a a b b c c ≤ a2 + b2 + c2 . (1.35)

145
Hội thảo khoa học, Ninh Bình 15-16/09/2018

Bài toán 31. Cho tam giác ABC có chu vi bằng 1. Chứng minh rằng

ama bmb cmc ≤ am a + bmb + cmc . (1.36)

Bài toán 32. Cho tam giác ABC có chu vi bằng 1. Chứng minh rằng

aha bhb chc ≤ ah a + bhb + chc . (1.37)

Bài toán 33. Cho tam giác ABC có chu vi bằng 1. Chứng minh rằng

ala blb clc ≤ ala + blb + clc . (1.38)

Bài toán 34. Cho tam giác ABC có chu vi bằng 1. Chứng minh rằng

ara brb crc ≤ ar a + brb + crc . (1.39)

Bài toán 35. Cho tam giác ABC có chu vi bằng 1. Chứng minh rằng

asin A bsin B csin C ≤ a sin A + b sin B + c sin C. (1.40)

2 Bất đẳng thức Popoviciu mở rộng cho tam giác có


chu vi bằng 1
Định lý 5. Cho tam giác ABC có chu vi bằng 1 và x, y, z là các số thực dương. Khi đó

a b c 1 ( b + c )2 ( c + a )2 ( a + b )2
+ + + ≥ + + . (2.1)
x y z ax + by + cz by + cz cz + ax ax + by

1 −1
Chứng minh. Xét hàm số f ( x ) = trên khoảng (0; +∞) , ta có f 0 ( x ) = 2 , suy ra
x x
2 1
f ( x ) = 3 > 0, ∀ x > 0. Do đó hàm số f ( x ) = lồi trên khoảng (0; +∞) .
00
x x
Áp dụng bất đẳng thức Popoviciu mở rộng và giả thiết a + b + c = 1, ta có

a f ( x ) + b f (y) + c f (z) + f ( ax + by + cz) ≥


     
by + cz cz + ay ax + by
≥ (b + c) f + (c + a) f + ( a + b) f
b+c c+a a+b

tương đương

a b c 1 ( b + c )2 ( c + a )2 ( a + b )2
+ + + ≥ + + .
x y z ax + by + cz by + cz cz + ax ax + by

Vậy Định lí (5) được chứng minh.


Sử dụng Định lí (5) ta chứng minh được các bất đẳng thức sau

Bài toán 36. Cho tam giác ABC có chu vi bằng 1. Chứng minh rằng

a b c 1 ( b + c )2 ( c + a )2 ( a + b )2
+ + + ≥ + + . (2.2)
ma mb mc am a + bmb + cmc bmb + cmc cmc + am a am a + bmb

146
Hội thảo khoa học, Ninh Bình 15-16/09/2018

Bài toán 37. Cho tam giác ABC có chu vi bằng 1. Chứng minh rằng

a b c 1 ( b + c )2 ( c + a )2 ( a + b )2
+ + + ≥ + + . (2.3)
ha hb hc ah a + bhb + chc bhb + chc chc + ah a ah a + bhb

Bài toán 38. Cho tam giác ABC có chu vi bằng 1. Chứng minh rằng

a b c 1 ( b + c )2 ( c + a )2 ( a + b )2
+ + + ≥ + + . (2.4)
la lb lc ala + blb + clc blb + clc clc + ala ala + blb

Bài toán 39. Cho tam giác ABC có chu vi bằng 1. Chứng minh rằng

a b c 1 ( b + c )2 ( c + a )2 ( a + b )2
+ + + ≥ + + . (2.5)
ra rb rc ar a + brb + crc brb + crc crc + ar a ar a + brb

Bài toán 40. Cho tam giác ABC có chu vi bằng 1. Chứng minh rằng
a b c 1
+ + + ≥
sin A sin B sin C a sin A + b sin B + c sin C
( b + c )2 ( c + a )2 ( a + b )2
≥ + + . (2.6)
b sin B + c sin C c sin C + a sin A a sin A + b sin B
Bài toán 41. Cho tam giác ABC có chu vi bằng 1. Chứng minh rằng
a b c 1
+ + + ≥
cos A cos B cos C a cos A + b cos B + c cos C
( b + c )2 ( c + a )2 ( a + b )2
≥ + + . (2.7)
b cos B + c cos C c cos C + a cos A a cos A + b cos B
Bài toán 42. Cho tam giác nhọn ABC có chu vi bằng 1. Chứng minh rằng
a b c 1
+ + + ≥
tan A tan B tan C a tan A + b tan B + c tan C
( b + c )2 ( c + a )2 ( a + b )2
≥ + + . (2.8)
b tan B + c tan C c tan C + a tan A a tan A + b tan B
Định lý 6. Cho tam giác ABC có chu vi bằng 1 và x, y, z là các số thực dương. Khi đó

√ √ √ p
a x + b y + c z + ax + by + cz ≤
√ p √ √ √ p
≤ b + c by + cz + c + a cz + ax + a + b ax + by. (2.9)
√ 1
Chứng minh. Xét hàm số f ( x ) = x trên khoảng (0; +∞) , ta có f 0 ( x ) = √ , suy ra
2 x
−1 √
√ < 0, ∀ x > 0. Do đó hàm số f ( x ) = x lõm trên khoảng (0; +∞).
f 00 ( x ) =
4x x
Áp dụng bất đẳng thức Popoviciu mở rộng và giả thiết a + b + c = 1, ta có

a f ( x ) + b f (y) + c f (z) + f ( ax + by + cz) ≤


     
by + cz cz + ay ax + by
≤ (b + c) f + (c + a) f + ( a + b) f
b+c c+a a+b

147
Hội thảo khoa học, Ninh Bình 15-16/09/2018

tương đương
√ √ √ p
a x + b y + c z + ax + by + cz ≤
√ p √ √ √ p
≤ b + c by + cz + c + a cz + ax + a + b ax + by.

Vậy Định lí (6) được chứng minh.


Sử dụng Định lí (6) ta chứng minh được các bất đẳng thức sau

Bài toán 43. Cho tam giác ABC có chu vi bằng 1. Chứng minh rằng

√ √ √ p
a m a + b mb + c mc + am a + bmb + cmc ≤
√ p √ √ √ p
≤ b + c bmb + cmc + c + a cmc + am a + a + b am a + bmb . (2.10)

Bài toán 44. Cho tam giác ABC có chu vi bằng 1. Chứng minh rằng
p p p p
a h a + b hb + c hc + ah a + bhb + chc ≤
√ p √ p √ p
≤ b + c bhb + chc + c + a chc + ah a + a + b ah a + bhb . (2.11)

Bài toán 45. Cho tam giác ABC có chu vi bằng 1. Chứng minh rằng

p p p p
ala + b lb + c lc + ala + blb + clc ≤
√ p √ p √ p
≤ b + c blb + clc + c + a clc + ala + a + b ala + blb . (2.12)

Bài toán 46. Cho tam giác ABC có chu vi bằng 1. Chứng minh rằng
√ √ √ p
a r a + b rb + c rc + ar a + brb + crc ≤
√ p √ √ √ p
≤ b + c brb + crc + c + a crc + ar a + a + b ar a + brb . (2.13)

Bài toán 47. Cho tam giác ABC có chu vi bằng 1. Chứng minh rằng
√ √ √ √
a sin A + b sin B + c sin C + a sin A + b sin B + c sin C ≤
√ √ √ √ √ √
≤ b + c b sin B + c sin C + c + a c sin C + a sin A + a + b a sin A + b sin B.
(2.14)

Bài toán 48. Cho tam giác ABC có chu vi bằng 1. Chứng minh rằng
√ √ √ √
a cos A + b cos B + c cos C + a cos A + b cos B + c cos C ≤
√ √ √ √ √ √
≤ b + c b cos B + c cos C + c + a c cos C + a cos A + a + b a cos A + b cos B.
(2.15)

Bài toán 49. Cho tam giác nhọnABC có chu vi bằng 1. Chứng minh rằng
√ √ √ √
a tan A + b tan B + c tan C + a tan A + b tan B + c tan C ≤
√ √ √ √ √ √
≤ b + c b tan B + c tan C + c + a c tan C + a tan A + a + b a tan A + b tan B.
(2.16)

148
Hội thảo khoa học, Ninh Bình 15-16/09/2018

Định lý 7. Cho tam giác ABC có chu vi bằng 1 và x, y, z là các số thực dương. Khi đó
by+cz cz+ ax ax +by
ae x + bey + cez + e ax+by+cz ≥ (b + c) e b+c + (c + a) e c+ a + ( a + b) e a+b . (2.17)

Chứng minh. Xét hàm số f ( x ) = e x trên khoảng (0; +∞) , ta có f 0 ( x ) = e x , suy ra f 00 ( x ) =


e x > 0, ∀ x > 0. Do đó hàm số f ( x ) = e x lồi trên khoảng (0; +∞) .
Áp dụng bất đẳng thức Popoviciu mở rộng và giả thiết a + b + c = 1, ta có

a f ( x ) + b f (y) + c f (z) + f ( ax + by + cz) ≥


     
by + cz cz + ay ax + by
≥ (b + c) f + (c + a) f + ( a + b) f
b+c c+a a+b

tương đương
by+cz cz+ ax ax +by
ae x + bey + cez + e ax+by+cz ≥ (b + c) e b+c + (c + a) e c+ a + ( a + b) e a+b .

Vậy Định lí (7) được chứng minh.


Sử dụng Định lí (7) ta chứng minh được các bất đẳng thức sau

Bài toán 50. Cho tam giác ABC có chu vi bằng 1. Chứng minh rằng
2 + b2 + c2 b2 + c2 c2 + a2 a2 + b2
ae a + beb + cec + e a ≥ (b + c) e b+c + (c + a) e c+ a + ( a + b) e a+b . (2.18)

Bài toán 51. Cho tam giác ABC có chu vi bằng 1. Chứng minh rằng

aema + bemb + cemc + e ama +bmb +cmc ≥


bmb +cmc cmc + am a am a +bmb
≥ (b + c) e b+c + (c + a) e c+ a + ( a + b) e a+b . (2.19)

Bài toán 52. Cho tam giác ABC có chu vi bằng 1. Chứng minh rằng

aeha + behb + cehc + e aha +bhb +chc ≥


bhb +chc chc + ah a ah a +bhb
≥ (b + c) e b+c + (c + a) e c+ a + ( a + b) e a+b . (2.20)

Bài toán 53. Cho tam giác ABC có chu vi bằng 1. Chứng minh rằng

aela + belb + celc + e ala +blb +clc ≥


blb +clc clc + al a al a +blb
≥ (b + c) e b+c + (c + a) e c+ a + ( a + b) e a+b . (2.21)

Bài toán 54. Cho tam giác ABC có chu vi bằng 1. Chứng minh rằng

aera + berb + cerc + e ara +brb +crc ≥


brb +crc crc + ar a ar a +brb
≥ (b + c) e b+c + (c + a) e c+ a + ( a + b) e a+b . (2.22)

Bài toán 55. Cho tam giác ABC có chu vi bằng 1. Chứng minh rằng

aesin A + besin B + cesin C + e a sin A+b sin B+c sin C ≥


b sin B+c sin C c sin C + a sin A a sin A+b sin B
≥ (b + c) e b+c + (c + a) e c+ a + ( a + b) e a+b . (2.23)

149
Hội thảo khoa học, Ninh Bình 15-16/09/2018

Bài toán 56. Cho tam giác ABC có chu vi bằng 1. Chứng minh rằng
aecos A + becos B + cecos C + e a cos A+b cos B+c cos C ≥
b cos B+c cos C c cos C + a cos A a cos A+b cos B
≥ (b + c) e b+c + (c + a) e c+ a + ( a + b) e a+b . (2.24)
Bài toán 57. Cho tam giác nhọn ABC có chu vi bằng 1. Chứng minh rằng
aetan A + betan B + cetan C + e a tan A+b tan B+c cos C ≥
b tan B+c tan C c tan C + a tan A a tan A+b tan B
≥ (b + c) e b+c + (c + a) e c+ a + ( a + b) e a+b . (2.25)

3 Mở rộng cho đa giác có chu vi bằng 1


Định lý 8. Cho đa giác A1 A2 . . . An có chu vi bằng 1 và xi là độ dài cạnh Ai Ai+1 (Với đỉnh Aii
trùng đỉnh An+i .) Chứng minh rằng
n
xi
∑∏ xj
≥ n n −1 .
1 i6= j

Chứng minh. Kí hiệu Rn++


= {( x1 , . . . , xn ) : xi > 0 Với mọi i}.
Xét hàm số
x1 x2 xn
f (x) = + +···+
x2 . . . x n x3 . . . x n x 1 . . . x n −1
là hàm lồi Schur ngặt trên Rn++ . 
1 1
Vì xi > 0 và ∑ xi = 1, ta có x  ,..., . Áp dụng bất đẳng thức Karamata, ta có
n n
n
xi
∑∏ ≥ n n −1 .
1 i6= j x j

Đẳng thức xảy ra khi và chỉ khi đa giác A1 A2 . . . An là đa giác đều và x1 = x2 = · · · =


1
xn = .
n
Định lý 9. Cho đa giác A1 A2 . . . An có chu vi bằng 1 và xi là độ dài cạnh Ai Ai+1 (Với đỉnh Ai
trùng đỉnh An+i .) Chứng minh rằng
n
xi n
∑ 1 − xi ≥
n−1
.
1

Chứng minh. Kí hiệu Rn++ = {( x1 , . . . , xn ) : xi > 0 Với mọi i}.


Xét hàm số
x1 x2 xn
f (x) = + +···+
x2 + · · · + x n x3 + · · · + x n x 1 + · · · + x n −1
 
1 1
là hàm lồi Schur ngặt trên R++ . Vì xi > 0 và ∑ xi = 1, ta có x 
n ,..., . Áp dụng
n n
bất đẳng thức Karamata, ta có
n
xi n
∑ 1 − xi ≥ n − 1 .
1

Đẳng thức xảy ra khi và chỉ khi đa giác A1 A2 . . . An là đa giác đều và x1 = x2 = · · · =


1
xn = .
n

150
Hội thảo khoa học, Ninh Bình 15-16/09/2018

Tài liệu
[1] Nguyễn Văn Mậu, Bất đẳng thức và một số vấn đề liên quan, Tài liệu bồi dưỡng giáo
viên THPT chuyên, 2005.
[2] Tạ Duy Phượng, Hoàng Minh Quân, Vũ Văn Thưởng, Sử dụng bổ đề trội chứng minh
bất đẳng thức hình học, bản thảo 204 trang, 2017.
[3] Lê Hồ Quý , Bất đẳng thức Karamata và một vài ứng dụng, Hội thảo Các chuyên đề
Toán học bồi dưỡng học sinh giỏi cấp THPT tổ chức tại Phú Yên,2011.
[4] Cao Minh Quang, http://sachsangtao.com/bvct/sach-tham-khao/414/bdt-
karamata-va-mot-so-ung-dung-cao-minh-quang.html
[5] Phạm Kim Hùng, Sáng tạo bất đẳng thức, NXB Tri thức 2006.
[6] Konstantinos G. Derpanis, Jensen’s Inequality, 2015.
[7] M. V. Mihai and Flavia-Corina Mitroi-Symeonidis, New extension of Popoviciu’s inequal-
ity, 2015.

151
Hội thảo khoa học, Ninh Bình 15-16/09/2018

ĐỊNH GIÁ p-ADIC VÀ ỨNG DỤNG

Phan Ngọc Toàn


Trường THPT Số 1 An Nhơn Bình Định

Tóm tắt nội dung

Trong bài viết này, ta xem xét một số ứng dụng của định giá p-adic vào giải quyết các
bài toán số học liên quan.

1 Kiến thức cơ sở

Định nghĩa 1. Số v p (n) được ký hiệu cho số mũ của p trong phân tích tiêu chuẩn
của n và quy ước v p (n) = 0 khi p không là ước của n. Số v p (n) được gọi là định giá
p − adic của số tự nhiên n.

Tính chất 1. Cho x, y, z là các số nguyên. Khi đó

(1) v p ( xy) = v p ( x ) + v p (y) .

(2) v p ( x n ) = n.v p ( x ) .

(3) v p ( x + y) > min { v p ( x ) , v p (y) } . Dấu “=” xảy ra⇔ v p ( x ) 6= v p (y) .

(4) v p (gcd (| x | , |y| , |z|)) = min { v p ( x ) , v p (y) , v p (z) } .

(5) v p (lcm (| x | , |y| , |z|)) = max { v p ( x ) , v p (y) , v p (z) } .

(6) x = y khi và chỉ khi υ p ( x ) = υ p (y), ∀ p ∈ ℘.

Bổ đề 1. Cho x, y là các số nguyên (không nhất thiết nguyên dương) và n là một số


nguyên dương. Cho p là số nguyên tố bất kỳ (đặc biệt, có thể p = 2) sao cho p| x − y
và (n, p) = ( x, p) = (y, p) = 1. Khi đó

v p ( x n − yn ) = v p ( x − y) .

152
Hội thảo khoa học, Ninh Bình 15-16/09/2018

Bổ đề 2. Cho x, y là các số nguyên (không nhất thiết nguyên dương) và n là một số


nguyên dương lẻ. Cho p là số nguyên tố bất kỳ (đặc biệt, có thể p = 2) sao cho p| x + y
và (n, p) = ( x, p) = (y, p) = 1. Khi đó

v p ( x n + yn ) = v p ( x + y) .

Định lý 1. Cho x và y là các số nguyên (không nhất thiết phải nguyên dương), n là
một số nguyên dương và p là một số nguyên tố lẻ sao cho p| x − y và ( x, p) = (y, p) =
1. Khi đó
v p ( x n − yn ) = v p ( x − y) + v p (n) .
Định lý 2. Cho x, y là hai số nguyên, n là một số nguyên dương lẻ và p là số nguyên
tố lẻ sao cho p| x + y và ( x, p) = (y, p) = 1. Khi đó

v p ( x n + yn ) = v p ( x + y) + v p (n) .

Định lý 3. Cho x và y là hai số nguyên lẻ sao cho 4|( x − y). Khi đó

v2 ( x n − y n ) = v2 ( x − y ) + v2 ( n ) .

Định lý 4. Cho x, y là hai số nguyên lẻ và n là một số nguyên dương chẵn. Khi đó

v2 ( x n − yn ) = v2 ( x − y) + v2 ( x + y) + v2 (n) − 1.

Hệ quả 1. Cho x và n là các số nguyên dương. Khi đó

(1) Nếu p > 2 là số nguyên tố sao cho v p ( x − 1) = α ∈ N∗ thì với mọi β ∈ N, ta có

v p ( x n − 1) = α + β ⇔ v p (n) = β.

(2) Nếu n chẵn sao cho v2 x2 − 1 = α ∈ N thì với mọi β ∈ N∗ , ta có




v2 ( x n − 1) = α + β ⇔ v2 (n) = β + 1.

Định lý 5 (Legendre). Cho p nguyên tố. Khi đó, ta có công thức:


+∞   n − s p (n)
n
v p (n!) = ∑ i
= .
i =1 p p − 1
j k j k
n n
Chứng minh. Trong đẳng thức đầu tiên, ta thấy rằng có đúng pi
− p i +1
số m thỏa
s
mãn điều kiện v p (m) = i, còn trong đẳng thức thứ hai, ta viết lại n = ∑ ni pi thì dễ
i =0
j k s
n
thấy p j = ∑ ni p .i − j
i= j

153
Hội thảo khoa học, Ninh Bình 15-16/09/2018

Khi đó ta có !
s   s s s i −1
n
∑ pj
= ∑ ∑ ni pi − j = ∑ ni ∑ p j
j =1 j =1 i = j i =1 j =0
s
pi −1 n − s p (n)
= ∑ ni p − 1 =
p−1
.
i =1
Định lý được chứng minh.

Định lý 6. Với p là số nguyên tố và n ∈ Z+ thỏa mãn điều kiện pm ≤ n < pm+1 với
m ∈ N. Giả sử k ∈ N thỏa mãn điều kiện k ≤ n. Khi đó
m  
n−k
     
n n k
vp =∑ j
− j − j
.
k j =1 p p p

Định lý 7(Kummer).
 Cho 1 ≤ k ≤ n là 2 số nguyên dương, p là một số nguyên tố.
n
Khi đó v p k bằng số lần nhớ khi thực hiện phép cộng k và n − k trong hệ cơ số p.

Chứng minh. Ta có

s p (k) + s p (n − k) − s p (n)
 
n
vp k = v p ( n! ) − v p ( k! ) − v p (( n − k ) ! ) = .
p−1
m
Đặt s p (n) = ∑ at . Do n > n − k, n > k nên ta hoàn toàn có thể biểu diễn s p (k ) =
t =0
m m
∑ bt và s p (n − k ) = ∑ ct . Ta sẽ thể hiện các lần nhớ bằng các αi ∈ {0, 1} như sau:
t =0 t =0

b0 + c0 = a0 + α1 p, bi + ci + αi = ai + αi+1 p, bm + cm + αm = am .
 
m
Từ đó dễ thấy s p (k ) + s p (n − k ) − s p (n) = ( p − 1) ∑ αi / Suy ra
i =1
 
s p (k) + s p (n − k) − s p (n) m
n
vp k =
p−1
= ∑ αi .
i =1

Chứng minh hoàn tất.


 
n
Nhận xét 1. Dạng tương đương của định lý Kummer: v p k bằng số lần thực
hiện nhớ khi thực hiện phép trừ n trừ đi k trong hệ cơ số p.

154
Hội thảo khoa học, Ninh Bình 15-16/09/2018

2 Các ví dụ minh họa

Trước hết ta ứng dụng định giá p − adic vào giải một số ví dụ đặc trưng cho
phương pháp này.

Bài toán 1. Cho a, b, c là ba số nguyên dương thỏa mãn điều kiện

c(ca + 1)2 = (2c + b) (3c + b) .

Chứng minh rằng c là một số chính phương.

Lời giải. Điều kiện bài toán tương đương với

c(ca + 1)2 = 6c2 + 5bc + b2 .

Suy ra c|b2 . Gọi p là ước nguyên tố của c ta có


 
c|b2 ⇒ v p (c) 6 v p b2 ⇒ v p (c) 6 2v p (b) (1).

Giả sử rằng v p (c) 6 v p (b). Do gcd (ca + 1, c) = 1 nên v p (ca + 1) = 0. Suy ra

v p (c) = v p (2c + b) + v p (3c + b) > 2v p (c) ,

vô lý. Do đó v p (b) < v p (c). Điều này dẫn đến

v p (c) = v p (2c + b) + v p (3c + b) > 2v p (b) (2).

Từ (1) và (2), suy ra v p (c) = 2v p (b). Như vậy, mọi lũy thừa trong khai triển c đều là
số chẵn hay c là một số chính phương.

Bài toán 2. Giả sử a, b ∈ N∗ thỏa mãn điều kiện a b2 , b3 a4 , a5 b6 , b7 a8 , . . . Chứng


minh rằng a = b.

Lời giải. Từ giả thiết suy ra a4n+1 b4n+2 , ∀n ∈ N và b4n+3 a4n+4 , ∀n ∈ N. Ta sẽ chứng

minh
υ p ( a ) = υ p ( b ).
Từ giả thiết a4n+1 b4n+2 , ∀n ∈ N, ta thu được

4n + 2
υ p ( a) ≤ υ p (b), ∀n ∈ N,
4n + 1
155
Hội thảo khoa học, Ninh Bình 15-16/09/2018

suy ra
4n + 2
υ p ( a) ≤ lim υ p (b) ⇒ υ p ( a) ≤ υ p (b)(1)
n→+∞ 4n + 1

Tương tự, từ b4n+3 a4n+4 , ∀n ∈ N suy ra


υ p ( b ) ≤ υ p ( a ). (2)

Từ (1) và (2) suy ra υ p ( a) = υ p (b).

Tiếp theo, ta xét một số ví dụ khác thể hiện hiệu lực của phương pháp này.

Bài toán 3 (IMO 2015). Tìm tất cả các số nguyên dương a, b, c sao cho các số ab − c,
bc − a, ca − b đều là lũy thừa số mũ tự nhiên của 2.

Lời giải. Rõ ràng ta chỉ cần xét khi a, b, c ≥ 2, các trường hợp khác không thỏa mãn.
- Nếu a, b, c đều chẵn:
Đặt A = v2 ( a), B = v2 (b), C = v2 (c) và giả sử 1 ≤ A ≤ B ≤ C. Do đó 2B | ac − b
nên ac − b = 2B ≤ b. Tương tự bc − a = 2 A ≤ a.
Cộng vào ta được

( a + b)c ≤ 2( a + b) ⇒ c ≤ 2 ⇒ c = 2 ⇒ A = B = C = 1 ⇒ a = b = c = 2.

- Nếu a, b, c đều lẻ:


Nếu có 2 số bằng nhau, giả sử là a = b thì ac − b = a(c − 1) có ước lẻ, vô lý. Vậy
a, b, c đôi một phân biệt, giả sử a < b < c.
Giả sử bc − a = 2α , ca − b = 2β ⇒ α > β và 2β | a.2α − b.2β = a(bc − a) − b(ca −
b) = b2 − a2 = (b − a)(b + a).
Do a lẻ nên b − a và b + a không thể cùng chia hết cho 4 nên một trong hai số đó
phải chia hết cho 2β−1 hay một trong hai số 2(b − a) và 2(b + a) phải chia hết cho 2β .
Do đó ac − b = 2β ≤ 2( a + b) nên ( a − 1)b < ac − b < 4b ⇒ a = 3.
Suy ra c ≤ b + 2 ⇒ c = b + 2 ⇒ bc − a = (b − 1)(b + 3) ⇒ b = 5, c = 7.
- Nếu a, b, c có cả chẵn và lẻ:
Giả sử c lẻ và a ≤ b. Do ab − c lẻ nên ab − c = 1.
Nếu a = b thì c = a2 − 1 ⇒ ac − b = a( a2 − 2) nên a và a2 − 2 đều là lũy thừa của
2. Suy ra a = b = 2, c = 3.

156
Hội thảo khoa học, Ninh Bình 15-16/09/2018

Nếu a < b và giả sử bc − a = 2α , ca − b = 2β thì α > β.


Nếu β = 0 thì ac − b = ab − c = 1 ⇒ b = c = 1, vô lý. Do đó α, β nguyên dương
và a, b chẵn. Thay c = ab − 1 ta có 2α = ab2 − ( a + b), 2β = a2 b − ( a + b).
Cộng vào ta có 2α + 2β = ( ab − 2)( a + b).
Do v2 ( ab − 2) = 1 nên v2 ( a + b) = β − 1, và do đó v2 ( ab2 ) = β − 1 và v2 ( a2 b) =
β − 1.
Suy ra tồn tại γ ≥ 1 và A, B, C lẻ sao cho a = 2γ A, b = 2γ B, a + b = 23γ C, β =
3γ + 1.
Do A + B = 22γ C ≥ 4C và A2 B − C = 2 nên 8 = 4A2 B − 4C ≥ 4A2 B − A − B ≥
A2 (3B − 1).
Do A, B lẻ và A < B nên A = 1, B = 3 ⇒ C = 1, γ = 1, a = 2, b = 6, c = 11.
Vậy ( a; b; c) là (2; 2; 2), (3; 5; 7), (2; 2; 3) hoặc (2; 6; 11) và các hoán vị.

Bài toán 4 (Turkey 2015). Tìm các số nguyên dương n thỏa mãn điều kiện ứng với
mọi số nguyên dương a nguyên tố cùng nhau với n thì 2n2 | an − 1.

e e
Lời giải. Giả sử n = p11 . . . pl l với p1 , . . . , pl là các số nguyên tố.
- Với j cố định, nếu p j lẻ. Theo định lý thặng dư Trung Hoa, hệ a − 1 ≡ p j ( mod p2j )
và a ≡ 1( mod pi )∀i 6= j có nghiệm nên ta chọn được a thỏa mãn điều kiện v p j ( a −
1) = 1 và gcd( a, n) = 1.
2e j
Khi đó, v p j ( an − 1) = v p j ( a − 1) + v p j (n) = 1 + e j . Mà từ p j |2n2 | an − 1, suy ra
2e j ≤ 1 + e j nên e j = 1.
- Với j cố định, nếu p j = 2, theo định lý thặng dư Trung Hoa, hệ a − 1 ≡ 2( mod8)
và a ≡ 1( mod pi )∀i 6= j có nghiệm, nên ta chọn được a thỏa mãn điều kiện v2 ( a −
1) = 1 và gcd( a, n) = 1.
Ta có
v2 ( a n − 1) = v2 ( a − 1) + v2 ( a + 1) + v2 ( n ) − 1 = 2 + e j .
Mà từ 22e j +1 |2n2 | an − 1 suy ra 2e j + 1 ≤ 2 + e j ⇒ e j = 1.
Vậy n = 2p1 . . . pk , p1 < p2 < . . . < pk và a2p1 ...pk ≡ 1( mod8p21 . . . p2k ) với mọi a
mà gcd( a, n) = 1.
Từ đó ta có agcd(2p1 ...pk ,p1 −1) ≡ 1( mod p1 ) với mọi a mà gcd( a, n) = 1.
Chọn a là căn nguyên thủy mod p1 mà gcd( a, n) = 1. (Gọi g là căn nguyên thủy

157
Hội thảo khoa học, Ninh Bình 15-16/09/2018

bất kỳ mod p1 . Theo định lý thặng dư Trung Hoa, hệ a ≡ g( mod p1 ) và a ≡ 1( mod


pi )∀i 6= 1 có nghiệm nên ta chọn được a thỏa mãn.)
Khi đó gcd(2p1 . . . pk , p1 − 1) = p1 − 1, mà p1 − 1|2p1 . . . pk nên p1 − 1|2. Do đó
p1 = 3.
Tương tự ta có p2 = 7, p3 = 43 và không tồn tại p4 .
Thử lại ta được n = 2, 2.3, 2.3.7, 2.3.7.43 thỏa đề bài.

Bài toán 5 (IMO Shortlist 2014). Tìm số nguyên tố p và các số nguyên dương x, y
thỏa mãn điều kiện x p−1 + y và x + y p−1 đều là lũy thừa số mũ nguyên dương của p.

Lời giải. Nếu p = 2 thì mọi cặp ( x, y) mà x + y là lũy thừa của 2 đều thỏa mãn.
Xét p > 2. Nếu v p ( x ) ≥ v p (y) > 0 thì mâu thuẫn. Do đó, ta giả sử p - y và từ đó
p - x, y.
Theo định lý Fermat nhỏ ta có x p−1 ≡ 1( mod p), y p−1 ≡ 1( mod p) nên x ≡ y ≡
−1( mod p).
Nếu x = y thì x ( x p−2 + 1) là lũy thừa của p, suy ra p = 2, mâu thuẫn.
Giả sử x > y ⇔ x p−1 + y > y p−1 + x suy ra y p−1 + x | x p−1 + y hay

y p−1 + x | (−y p−1 ) p−1 + y ⇒ y p−1 + x | y p( p−2) + 1

(do gcd(y p−1 + x, y) = 1).


Ta lại có v p (y p−1 + x ) ≤ 1 + v p (y + 1). Mà y p−1 + x là lũy thừa của p nên

y p −1 + x | p ( y + 1 ) ⇒ y p −1 + x ≤ p ( y + 1 ).

Do x > y nên y p−1 + y ≤ p(y + 1) − 1. Ngoài ra y ≥ p − 1 và vì y ≡ −1( mod p) nên


y p−1 + p − 1 ≤ p(y + 1) − 1 ⇒ y p−1 ≤ py ⇒ ( p − 1) p−2 ≤ y p−2 ≤ p. Do đó p = 3 và
y = 2.
Ta còn phải tìm x > 2 thỏa mãn điều kiện x2 + 2 và x + 4 là lũy thừa của 3.
Đặt x + 4 = 3a , a ≥ 2, ta có x2 + 2 > x + 4∀ x > 2 nên

x + 4| x2 + 2 = ( x2 − 16) + 18 ⇒ x + 4|18 ⇒ 3a |18 ⇒ a = 2.

Thử lại ta được duy nhất bộ ( x, y, p) = (5, 2, 3) thỏa đề bài

Bài toán 6 (Romania TST 2016). Cho số nguyên dương n ≥ 3, n 6= 4 và


A = {2n − 1, 3n − 1, . . . , (n − 1)n − 1}. Chứng minh rằng nếu tất cả các phần

158
Hội thảo khoa học, Ninh Bình 15-16/09/2018

tử của A đều không chia hết cho n thì n không chia hết cho bình phương của một số
nguyên tố nào cả. Ngoài ra liệu n có bắt buộc phải là số nguyên tố hay không?

Lời giải. Xét hàm số f : Z+ → Z+ thỏa mãn điều kiện f (1) = 1 và nếu p nguyên tố
mà p | n thì v p ( f (n)) = 1.

Bổ đề 3. Nếu n là số nguyên dương mà có thể chia hết cho bình phương của một số
nguyên tố thì n − f (n) ≥ 2. Đẳng thức chỉ xảy ra khi n = 4.
α α
Chứng minh. Ta có với phân tích ra thừa số nguyên tố của n là n = p1 1 ...pk k thì
αj
f (n) = p1 ...pk . Do đó nếu α j ≥ 2 thì n − f (n) ≥ p j − p j ≥ p2j − p j ≥ 2. Bổ đề được
chứng minh.
Bây giờ trở lại bài toán:
Giả sử phản chứng, tồn tại số n nguyên dương mà có thể chia hết cho bình phương
của một số nguyên tố thỏa mãn điều kiện bài toán.
Từ n 6= 4 và n 6= 1 theo bổ đề, ta có 2 ≤ f (n) ≤ n − 2. Chọn m = f (n) + 1. Ta
chứng minh rằng n | mn − 1. Thật vậy, lấy số nguyên tố p | n, ta có
- Nếu p lẻ thì v p (mn − 1) = v p ( f (n)) + v p (n) = 1 + v p (n) > v p (n).
- Nếu p = 2 thì

v2 (mn − 1) = v2 ( f (n)) + v2 ( f (n) + 2) + v2 (n) − 1 ≥ 2 + v2 (n) > v2 (n).

Từ đó n | mn − 1, mâu thuẫn.
Ngoài ra, nếu chọn n không chia hết cho bình phương của một số nguyên tố nào
cả và gcd(n, ϕ(n)) = 1, thì B = {0n − 1, 1n − 1, 2n − 1, ..., (n − 1)n − 1} là hệ thặng
dư đầy đủ mod n nên tất cả các phần tử của A = {2n − 1, 3n − 1, ..., (n − 1)n − 1}
đều không chia hết cho n.
Vậy n không bắt buộc phải là số nguyên tố.

Bài toán 7 (Art of Problem Solving). Cho các số hữu tỉ a, b sao cho tồn tại vô hạn
số tự nhiên n sao cho an − bn là số nguyên. Chứng minh rằng a, b đều là các số nguyên.

Lời giải. Rõ ràng nếu một trong hai số a, b nguyên thì số còn lại cũng nguyên.
Giả sử a, b đều không là số nguyên, khi đó dễ thấy a, b phải có dạng sau:
x y
a= , b = ; x, y ∈ Z; z ∈ N∗ , ( x, z) = (y, z) = 1.
z z

159
Hội thảo khoa học, Ninh Bình 15-16/09/2018

Do z > 1 nên tồn tại một số nguyên tố p là ước của z. Theo giả thiết ta có

zn ( x n − yn ) ⇒ pn ( x n − yn )

với vô hạn số tự nhiên n. Ta gọi tập các số n đó là M. Vì ( x, z) = (y, z) = 1 nên


( x, p) = (y, p) = 1. Xét các khả năng sau:

i) Khi p = 1, ta có 2n ( x n − yn ), ∀n ∈ M, ta đặt n = 2un vn với un ∈ N và vn là số


tự nhiên lẻ. Ta có
un v un v u n −1 v u n −1 v
x n − yn = x2 n − y2 n = ( x vn − yvn )( x vn + yvn ) · · · ( x2 n − y2 n )

Suy ra
u n −1  
n n
v2 ( x − y ) = v2 ( x vn vn
−y )+ ∑ v2 x 2k v n
+y 2k v n
k =1
Do x, y, vn lẻ nên v2 ( x vn − yvn ) = v2 ( x − y) và v2 ( x vn + yvn ) = v2 ( x + y) Và với
k ∈ {1, 2, . . . , un − 1} ta có
 k   k 
2 vn 2k v n 2 vn 2k v n
x +y ≡ 2 (mod 4) ⇒ v2 x +y = 1.

Suy ra
u n −1  
n n
v2 ( x − y ) = v2 ( x vn vn
−y )+ ∑ v2 x 2k v n
+y 2k v n
k =1
≤ v2 ( x − y ) + v2 ( x + y ) + ( u n − 1)

mà 2n ( x n − yn ) nên

2u n v n = n ≤ v 2 ( x n − y n ) ≤ v 2 ( x − y ) + v 2 ( x + y ) + ( u n − 1 ),

từ đó
2un ≤ v2 ( x − y) + v2 ( x + y) + (un − 1), ∀n ∈ M.
Do đó (un )n∈ M chỉ nhận hữu hạn giá trị, và khi đó (vn )n∈ M cũng nhận hữu hạn
giá trị, suy ra n cũng chỉ nhận hữu hạn giá trị, điều này là mâu thuẫn với giả
thiết.

ii) Khi p lẻ, chú ý rằng, ta có pn ( x n − yn ), ∀n ∈ M và ( x, p) = (y, p) = 1. Gọi


d là số nguyên dương nhỏ nhất thỏa mãn điều kiện pn ( x d − yd ). Ứng với mỗi

n ∈ M, ta đặt n = qd + r với q, r ∈ N; r ∈ {0, 1, . . . , d − 1}. Ta có

pn ( x n − yn ) = ( x qd+r ) − yqd+r

160
Hội thảo khoa học, Ninh Bình 15-16/09/2018

mà pn ( x d − yd ) nên

h qd+r i
− y ( x ) = x qd ( xr − yr ) ⇒ pn ( xr − yr ).
r d q

p x


Mặt khác vì r < d nên theo cách chọn d ta có r = 0 hay d n, ∀n ∈ M.
Như vậy M = {kd|k = 1, 2, . . . }. Đặt u = x d , v = yd thì ta có (u, p) = (v, p) =

1; p (u − v) 6= 0 và với mọi số nguyên dương k, ta có
pk pkd = pn ( x n − yn ) = (uk − vk ).

Suy ra
p v k ( p ) ≤ k ≤ v p ( u k − v k ) = v p ( u − v ) + v p ( k ).
Chọn k = Pα đủ lớn ta có điều vô lý. Vậy ta có u = v hay x = y nghĩa là a = b.
Bài toán 8 (IMO Shortlist 2009). Cho f là một hàm số khác hằng số, f : Z → Z thỏa
mãn điều kiện a − b chia hết f ( a) − f (b) với mọi số nguyên phân biệt a, b. Chứng
minh rằng tồn tại một tập vô hạn S gồm các số nguyên tố sao cho với mỗi p ∈ S đều
tồn tại c ∈ Z sao cho p f (c).

Lời giải. Giả sử ngược lại chỉ tồn tại hữu hạn các số nguyên tố p1 , p2 , . . . , ps sao cho
với mỗi i ∈ {1, 2, . . . , s} thì tồn tại ci ∈ Z sao cho pi f (ci ). (chú ý rằng f khác hằng số

nên s ≥ 1).
Chọn a = ( p1 p2 . . . ps )r , theo giả thiết ta có

a f ( a + 1) − f (1). (1)
Giả sử f ( a + 1) 6= f (1), khi đó tồn tại i ∈ {1, 2, . . . , s} sao cho
v p [ f ( a + 1)] 6= v p [ f (1)].
Từ đó, kết hợp với (1) ta suy ra
r = v pi ( a) ≤ v p ( f ( a + 1) − f (1)) = min{v p f ( a + 1), v p f (1)} ≤ v pi f (1).
Chọn r đủ lớn ta có điều mâu thuẫn nên f ( a + 1) = f (1). Khi đó với mỗi số nguyên
n, ta có

( a + 1) − n f ( a + 1) − f ( n ) = f (1) − f ( n ).
Chọn r đủ lớn sao cho ( a + 1) − n > | f (1) − f (n)|, ta được
f (1) − f (n) = 0 ⇒ f (n) = f (1), ∀n ∈ Z.
Mâu thuẫn, vì f là một hàm số khác hằng số. Vậy ta có điều phải chứng minh.

161
Hội thảo khoa học, Ninh Bình 15-16/09/2018

Bài toán 9 (China TST 2016). Cho số nguyên dương m = 2k .t, trong đó k là số tự
nhiên, t nguyên dương lẻ. Đặt f (m) = t1−k . Chứng minh rằng với mọi số nguyên
n
dương n và số nguyên dương lẻ a ≤ n thì ∏ f (m) chia hết cho a.
m =1

+∞
  $ %
a n
Lời giải. Do i
là số bội của pi mà không vượt quá a, nên ∑ h
là tổng các
p x
h =1 2 p i
v pi (z) trong các số z = 2u .v thỏa mãn điều kiện pi |z, z ≤ n và v2 (z) = u ≥ x,
+∞ +∞ j
$ %
n n hk
∑ 2 x p h − ∑ 2 x +1 p i
h =1 i h =1

là tổng các v pi (z) trong các số z = 2u .v thỏa mãn điều kiện pi | z, z ≤ n và v2 (z) = x.
Từ đó ta có
+∞ +∞ +∞ +∞ +∞ +∞
( $ % ) $ % $ %
n
n j n k n n
v pi ( ∏ f (m)) = ∑ (1 − x ) ∑ − ∑ x +1 p i ∑ ∑∑
j
= −
m =1 x =0 h =1 2x pih 2 pih x =1 h =1 2x pih
%h=1 h =1
+∞ k
$ % $
k
n n
= ∑ pih
− ∑∑ 2x pih
,
h =1 x =1 h =1

trong đó k tự nhiên thỏa mãn điều kiện pik ≤ n < pik+1 .


n
Để chứng minh a | ∏ f (m) với mọi 2 - a, a ≤ n, ta cần chứng minh
m =1
n
v pi ( ∏ f (m)) ≥ k với mọi pi nguyên tố lẻ không vượt quá n và pik ≤ n < pik+1 .
m =1
Muốn vậy, ta chứng minh
$ % $ %
l
n n
pih
− ∑ 2x pih
≥1
x =1
 
1
với 2l pih ≤ n < 2l +1 pih . Thật vậy, do b2ac = b ac + a + ≥ 2 b ac nên
2
$ % $ % $ % $ % $ % $ % $ %
l −1 l
n n n n n n n
h
≥2 h
≥ h
+ 2 2 h ≥ ... ≥ ∑ h
+ 2 ≥ ∑ + 1.
pi 2pi 2pi 2 pi x
x =1 2 p i 2l pih x =1 2x pih
Từ đó
+∞
($ % $ %) ($ % $ %)
n k k l
n n n n
v pi ( ∏ f (m)) = ∑ − ∑ = ∑ − ∑ ≥ k.
m =1 h =1 pih x =1 2x pih h =1 pih x =1 2x pih

Ta có điều phải chứng minh.

162
Hội thảo khoa học, Ninh Bình 15-16/09/2018

Bài toán 10 (IMO Shortlist 2015). Tìm tất cả M nguyên dương sao cho dãy số
1
a0 = M + , ak+1 = ak b ak c ∀k ≥ 0 chứa ít nhất một số nguyên.
2

Lời giải. M = 1 không thỏa mãn.


Ta chứng minh quy nạp theo v2 ( M − 1) rằng tất cả M > 1 đều thỏa mãn.
 
1
Nếu v2 ( M − 1) = 0, hay M chẵn thì a1 = M M + nguyên.
2
2M2 + M − 1 1
 
1 1
Nếu v2 ( M − 1) = k > 0 thì a1 = M M + = + = N+ .
2 2 2 2
 2
2M + M − 3
 
(2M + 3)( M − 1)

Lại có v2 ( N − 1) = v2 = v2 = k − 1.
2 2
Như vậy ta có thể thay đổi dãy số để đưa về xét v2 ( M − 1) = k − 1 và theo quy
nạp ta có điều phải chứng minh.

Bài toán 11. Chứng minh rằng không tồn tại k nguyên dương sao cho có một bộ k số
( a1 , a2 , . . . , ak ) với ai ∈ {0, 1} thỏa mãn
k  
k
∑ 5i a i i = 6 k − 1 .
i =0

Lời giải. Nếu đã biết về định lý Kummer thì việc nhìn ra hướng bài toán sẽ nhanh
hơn khá nhiều. Ta có a0 = 1 (nếu a0 = 0 thì vế trái sẽ chia hết cho 5 còn vế phải thì
không)
6k −1 − 1 k  
i −1 k
= a1 k + ∑ a i 5 i .
5 i =2
Vậy ta được
6k −1 − 1
= 6k−2 + · · · + 6 + 1 ≡ k − 1( mod5),
5
suy ra a1 = 0 (nếu a1 = 1 thì

6k −1 − 1 k  
k
≡ k + ∑ ai 5i−1 i ≡ k( mod5)),
5 i =2
k −1 −1
suy ra tiếp 5 | k − 1 (do 5 | 6 5 ). Đặt n = v5 (k − 1) và viết k = 5n m + 1, gcd(m, 5) =
1.) Xét đẳng thức
k  
k
∑ ai 5 i = 6k−1 − 1.
i
i =1

163
Hội thảo khoa học, Ninh Bình 15-16/09/2018
 
Ta lại có, v5 6k −1 − 1 = n + 1. Với i ≥ n + 2 thì
 
n +2 i k
5 | ai 5 i .
     
k k 5t+1 k
Với 1 ≤ i ≤ n + 1, khi đó nếu i = 5t, thì i = 5t = k−5t 5t + 1 , suy ra
      
k 5t + 1 k k
v5 = v5 = v5
5t k − 5t 5t + 1 5t + 1

Nếu i = 5t + 1, i > 1 thì


     
k k 5t + 2 k
i = 5t + 1 = k − 5t − 1 5t + 2 .

Khi đó do 5t ≤ n < 5n , nên v5 (5t) < n, suy ra

v5 (k − 5t − 1) = v5 (5n m − 5t) = v5 (5t) ,

suy ra tiếp
     
i k 5t+1 k
v5 5 i = v5 5 5t + 1
  
5t+1 5t + 2 k
= v5 5
k − 5t − 1 5t + 2
 
k
= 5t + 1 + v5 5t + 2 − v5 (k − 5t − 1)
 
k
= 5t + 1 + v5 5t + 2 − v5 (5t)
 
k
≥ 2 + v5 5t + 2 .

Nếu i ∈ {5t + 2, 5t + 3, 5t + 4} , i sẽ có chữ số tận cùng lớn hơn 1 khi viết trong hệ cơ
số 5. Còn k = 5n m + 1 khi viết trong hệ cơ số 5 sẽ có n chữ số tận cùng gồm n − 1 chữ
số 0 liên tiếp và chữ số 1 ở tận
cùng. Khi đó nếu ta lấy k − i trong hệ cơ số 5 thì sẽ cần
k
nhớ ít nhất là n lần, suy ra v5 i ≥ n, với mọi i ∈ {5t + 2, 5t + 3, 5t + 4} . Cộng
với các nhận xét trong các trường hợp i ∈ {5t, 5t + 1} , suy ra tiếp
  
k
v 5 5i i ≥ 2 + n, ∀i ≥ 2,

164
Hội thảo khoa học, Ninh Bình 15-16/09/2018

hay
k  ! k  !
k i k
 
v5 ∑ a i 5i i = v 5 ∑ a i 5 i ≥ n + 2 > v5 6 − 1 .
k −1
i =1 i =2

Ta có điều mâu thuẫn, vậy không tồn tại k thỏa mãn.

Tài liệu

[1] Nguyễn Văn Mậu, Trần Nam Dũng (2008), Một số vấn đề số học chọn lọc, NXBGD.

[2] Đề thi chọn đội tuyển các tỉnh, trường đông các khu vực, olympiad các nước, IMO Short-
listed, TST .

[3] Titu Andreescu (2001), Number theory, Springer.

[4] Đàm Văn Nhỉ, Lưu Bá Thắng (2015), Lý thuyết số và chuyên đề nâng cao, NXB ĐHSP
HN

165
Hội thảo khoa học, Ninh Bình 15-16/09/2018

TÂM TỈ CỰ VỚI BÀI TOÁN CHỨNG MINH


TÍNH ĐỒNG QUY VÀ THẲNG HÀNG

Đinh Bích Hảo


Khoa Tự nhiên, Trường Đại học Hoa Lư

Tóm tắt nội dung


Trong bài viết này, tác giả sử dụng định nghĩa tâm tỉ cự và các tính chất của tâm tỉ
cự để chứng minh một số bài toán về sự đồng quy, thẳng hàng trong hình học phẳng, và
bằng kỹ thuật tương tự chứng minh các kết quả được mở rộng trong không gian (nếu
có).

1 Định nghĩa tâm tỉ cự của hệ điểm

Định lý 1. Nếu P1 , P2 , . . . , Pn là các điểm của không gian afin và α1 , α2 , . . . , αn là các số có


tống khác 0 thì tồn tại duy nhất điểm G sao cho
n −→ −

∑ αi GPi = 0.
i =1

Chứng minh. Lấy một điểm O tùy ý, khi đó điểm G xác định bởi
n −→ −

∑ αi GPi = 0
i =1
n
−→ −→ −→
⇔ ∑ αi OPi − OG = 0
i =1
!
n
−→ n −→
⇔ ∑ αi OPi = ∑ αi OG
i =1 i =1
−→ n −→
1
⇔ OG = · ∑ αi OPi (∗).
∑in=1 αi i =1
n −→
1
Đặt n · ∑ αi OPi = −

u.
∑ αi i =1
i =1

166
Hội thảo khoa học, Ninh Bình 15-16/09/2018
−→
Với Pi xác định, điểm O cố định và các αi cho trước thì −

u cố định. Do đó OG = −

u
nên điểm G tồn tại và duy nhất.

Định nghĩa 1. Điểm G xác định trong định lí 1 được gọi là tâm tỉ cự của hệ điểm
{ P1 , P2 , . . . , Pn } gắn với họ hệ số α1 , α2 , . . . , , αn .
Hoặc ta nói điểm G là tâm tỉ cự của hệ chất điểm {α1 P1 , α2 P2 , . . . , αn Pn }.

Chú ý 1. Từ (∗) với O là điểm bất kì trong không gian, ta có thể quy ước không viết điểm O.
Khi đó công thức có dạng

mG = α1 P1 + α2 P2 + . . . + αn Pn
n
trong đó m = ∑ αi .
i =1

Trong các ví dụ và bài toán ở mục sau chúng tôi sử dụng các viết này.
Bài toán 1. Trung điểm I của đoạn thẳng AB là tâm tỉ cự của hệ {1A, 1B}, tức là
2I = 1A + 1B.
Bài toán 2. Trọng tâm G của tam giác ABC là tâm tỉ cự của hệ {1A, 1B, 1C }, tức là
3G = 1A + 1B + 1C.

2 Một số tính chất của tâm tỉ cự

Tính chất 1. Tâm tỉ cự của hệ điểm {α1 A1 , α2 A2 } là điểm G nằm trên đường thẳng A1 A2
thỏa mãn |α1 |d1 = |α2 |d2 , trong đó d1 , d2 là khoảng cách từ tâm tỉ cự G tới hai điểm A1 và
A2 .
Tính chất 2. Giả sử G là tâm tỉ cự của hệ chất điểm {α1 A1 ,α2 A2 ,. . ., αk Ak ,
αk+1 Ak+1 ,. . . , αn An }; G1 là tâm tỉ cự của hệ chất điểm {α1 A1 , α2 A2 , . . . , αk Ak }; G2 là
tâm tỉ cự của hệ chất điểm {αk+1 Ak+1 , . . . , αn An }. Khi đó G là tâm tỉ cự của hệ chất điểm
k n
{m1 G1 , m2 G2 }, trong đó m1 = ∑ αi và m2 = ∑ αi .
i =1 i = k +1

Tính chất 3. Nếu G là tâm tỉ cự của hệ chất điểm {α1 A1 ,α2 A2 ,. . .,αn An } thì G 0 là tâm
tỉ cự của hệ chất điểm {α1 A10 ,α2 A20 ,. . .,αn A0n }, ở đó G 0 , A10 , A20 , . . . , A0n là hình chiếu của
G, A1 , . . . , An theo phương d lên đường thẳng ∆.

167
Hội thảo khoa học, Ninh Bình 15-16/09/2018

3 Bài toán chứng minh các đường thẳng đồng quy

Bài toán 3 (Điểm Nagel của tam giác). Chứng minh rằng các đường thẳng đi qua
một đỉnh của tam giác và chia đôi chu vi tam giác đồng quy tại điểm Nagel Na của
tam giác.

Lời giải.

C0 B0

B C
A0

Giả sử tam giác ABC có độ dài các cạnh AB = c, BC = a, AC = b.


a+b+c
Đặt p = .
2
Theo giải thiết ta có AB + BA0 = BC + CB0 = CA + AC 0 = p.
suy ra BA0 = AB0 = p − c; CB0 = BC 0 = p − a; AC 0 = CA0 = p − b.
Xét hệ chất điểm {( p − b) B, ( p − c)C } có tâm tỉ cự là A0 , do đó

aA0 = ( p − b) B + ( p − c)C.

Xét hệ chất điểm {( p − a) A, ( p − b) B, ( p − c)C } có tâm tỉ cự Na . Khi đó

pNa = ( p − a) A + ( p − b) B + ( p − c)C
= ( p − a) A + aA0

Suy ra Na ∈ AA0 .
Chứng minh tương tự ta có Na ∈ BB0 và Na ∈ CC 0 .

168
Hội thảo khoa học, Ninh Bình 15-16/09/2018

Vậy AA0 , BB0 , CC 0 động quy tại Na .

Bài toán 4. Chứng minh rằng các đường thẳng đi qua một đỉnh của tam giác và tiếp
điểm của đường tròn nội tiếp với cạnh đối đồng quy tại một điểm K.
Lời giải. Giả sử tam giác ABC có các tiếp điểm A1 , B1 , C1 .
a+b+c
Đặt α = A1 B = BC1 , β = CA1 = CB1 , AC1 = AB1 = γ và p = .
2
Ta có
 
α + β = a α = p − b
β+γ = b ⇒ β = p−c
γ+α = c γ = p − a.
 

1 −−→ 1 −−→ − → 1 1
Ta có A1 B + A1 C = 0 suy ra A1 là tâm tỉ cự của hệ { B, C }.
α β α β
1 1 1 1
Tương tự ta có B1 là tâm tỉ cự của hệ { C, A}; C1 là tâm tỉ cự của hệ { A, B}.
β γ γ α
1 1 1
Gọi K là tâm tỉ cự của hệ { A, B, C }. Khi đó
γ α β
 
1 1 1 1 1 1
+ + K= A+ B+ C
γ α β γ α β
 
1 1 1
= A+ B+ C
γ α β
 
1 1 1
= A+ + A1 .
γ α β
Do đó K ∈ AA1 .
Tương tự, ta chứng minh được K ∈ BB1 và K ∈ CC1 . Vậy ba đường AA1 , BB1 , CC1
đồng quy tại K.
Bằng kĩ thuật tương tự, ta chứng minh kết quả mở rộng của ví dụ 2 trong không
gian như sau.

Bài toán 5. Cho tứ diện có tất cả các cạnh cùng tiếp xúc với mặt cầu. Chứng minh
rằng:
i) Ở mỗi mặt của tứ diện có 3 đường thẳng, mỗi đường thẳng nối một đỉnh của mặt
và tiếp điểm của mặt cầu tiếp xúc với cạnh đối đồng quy tại một điểm.
iii) Bốn đường thẳng, mỗi đường thẳng nối một đỉnh với điểm đồng quy ở i) của mặt
đối diện đồng quy tại Z.

169
Hội thảo khoa học, Ninh Bình 15-16/09/2018

Lời giải.

M P
N

Q
B D

S
R
C

Giả sử mặt cầu tiếp xúc với các cạnh AB, AC, AD, BD, CD, BC của tứ diện ABCD
lần lượt tại M, N, P, Q, R, S. Khi đó


 AM = AN = AP = a
BM = BQ = BS = b


 CN = CS = CR = c
DP = DQ = DR = d

với a, b, c, d > 0.
1 1 1
i) Xét mặt BCD. Gọi Z1 là tâm tỉ cự của hệ điểm { B, C, D }. Khi đó
b c d
 
1 1 1 1 1 1
+ + Z1 = B+ C+ D
b c d b c d
 
1 1 1
= B+ + R.
b c d
Suy ra Z1 ∈ BR.
Tương tự, ta chứng minh được Z1 ∈ DS, Z1 ∈ CQ. Vậy ở mặt BCD các đường
BR, CQ, DS đồng quy tại Z1 .
Các mặt còn lại chứng minh tương tự, ta được các điểm đồng quy Z2 , Z3 , Z4 .
1 1 1 1
ii) Gọi Z là tâm tỉ cự của hệ điểm { A, B, C, D }. Khi đó
a b c d

170
Hội thảo khoa học, Ninh Bình 15-16/09/2018

 
1 1 1 1 1 1 1 1
+ + + Z= A+ B+ C+ D
a b c d a b c d
 
1 1 1 1
= A+ B+ C+ D
a b c d
 
1 1 1 1
= A+ + + Z1 .
a b c d
Suy ra Z ∈ AZ1 .
Tương tự, ta chứng minh được Z ∈ BZ2 , Z ∈ CZ3 và DZ4 .Tức là AZ1 , BZ2 , CZ3 , DZ4
đồng quy tại Z.

Bài toán 6. Chứng minh rằng trong một tam giác bất kì ba đường trung tuyến đồng
quy tại một điểm và điểm đó chia theo tỷ lệ 2 : 1 tính từ đỉnh.
Lời giải.

C0 B0
G
B C
A0

Giả sử tam giác ABC có các trung tuyến lần lượt là AA0 , BB0 , CC 0 . Ta cần chứng
minh AA0 , BB0 , CC 0 đồng quy tại một điểm.
Theo giả thiết ta có A0 là tâm tỉ cự của hệ {1B, 1C }, tức là 2A0 = 1B + 1C.
Xét G là tâm tỉ cự của hệ {1A, 1B, 1C }.
Khi đó 3G = 1A + 1B + 1C ⇒ 3G = 1A + 2A0 . Suy ra G ∈ AA0 và G chia AA0 theo
tỉ lệ 2 : 1 tính từ A.
Tương tự ta chứng minh được G thuộc BB0 , CC 0 và chia hai trung tuyến này theo tỉ
lệ 2 : 1 tính từ đỉnh.

171
Hội thảo khoa học, Ninh Bình 15-16/09/2018

Bằng kĩ thuật tương tự chúng tôi chứng minh kết quả mở rộng của ví dụ 4 trong
không gian như sau.

Bài toán 7. Trong một tứ diện bất kì bốn trọng tuyến (các đoạn thẳng nối 1 đỉnh với
trọng tâm mặt đối diện) và ba trung tuyến kép (các đoạn thẳng nối trung điểm của
hai cạnh đối diện) đồng quy tại một điểm. Điểm đồng quy này là trung điểm của các
trung tuyến kép và nó chia các trọng tuyến theo tỉ số 3 : 1 tính từ đỉnh.

Lời giải.

M
R

P
Q
B D

S N

Giả sử cho tứ diện ABCD, gọi M, N, P, Q, R, S lần lượt là trung điểm các cạnh
AB, CD, AC, BD, AD, BC.
Gọi G1 là tâm tỉ cự của hệ 1D, 1B, 1C, tức là 3G1 = 1D + 1B + 1C.
Xét hệ {1A, 1B, 1C, 1D } có tâm tỉ cự là G. Khi đó

4G = 1A + 1B + 1C + 1D
= 1A + (1B + 1C + 1D )
= 1A + 3G1 .

Suy ra G ∈ AG1 và G chia trọng tuyến AG1 theo tỉ số 3 : 1. Tương tự ta chứng minh
được G thuộc các đường BG2 , CG3 , DG4 và chia các trọng tuyến theo tỉ số 3 : 1.

172
Hội thảo khoa học, Ninh Bình 15-16/09/2018

Ta lại có

4G = 1A + 1B + 1C + 1D
= (1A + 1B) + (1C + 1D )
= 2M + 2N.

Suy ra G ∈ MN và G chia MN theo tỉ số 1 : 1. Tương tự ta chứng minh được G thuộc


PQ, RS và chia các trung tuyến kép này theo tỉ số 1 : 1.

4 Bài toán chứng minh quan hệ thẳng hàng, đồng phẳng

Bài toán 8. Nếu ba điểm A1 , B1 , C1 theo thứ tự nằm trên các đường thẳng chứa cạnh
BC, CA, AB của tam giác ABC và thỏa mãn hệ thức

A1 B B1 C C1 A
· · = 1.
A1 C B1 A C1 B

thì chúng phải nằm trên một đường thẳng.


Lời giải.

C1

B1

B A1
C

Đặt

A1 B γ
=−
A1 C β
B1 C α
=−
B1 A γ

173
Hội thảo khoa học, Ninh Bình 15-16/09/2018

với 0 6= α, β, γ ∈ R. Khi đó

A1 B B1 C C1 A
· · =1
A1 C B1 A C1 B
   
γ α C A
⇔ − · − · 1 =1
β γ C1 B
C A β
⇔ 1 = .
C1 B α

A1 B γ
Ta có = − , tức là A1 là tâm tỉ cự của hệ { βB, γC }.
A1 C β
B C α
và 1 = − , tức là B1 là tâm tỉ cự của hệ {γC, αA}.
B1 A γ
Tương tự ta có C1 là tâm tỉ cự của hệ {αA, − βB}. Do đó ta có

(α − β)C1 = αA − βB
= (αA + γC ) − (γC + βB)
= (α + γ) B1 − (γ + β) A1

Suy ra C1 ∈ A1 B1 hay A1 , B1 , C1 thẳng hàng.

Bằng kĩ thuật tương tự chúng tôi chứng minh kết quả mở rộng của ví dụ 8 trong
không gian như sau.

Bài toán 9. Cho tứ diện ABCD, các điểm K, L, M, N lần lượt nằm trên các cạnh
AB, BC, CD, DA sao cho
KA LB MC ND
· · · = 1.
KB LC MD N A
Chứng minh rằng bốn điểm K, L, M, N đồng phẳng.
Lời giải.
Đặt

KB β
=− (1)
KB α
LB γ
=− (2)
LC β
MC δ
=− (3)
MD γ

174
Hội thảo khoa học, Ninh Bình 15-16/09/2018

với 0 6= α, β, γ, δ ∈ R. Khi đó
KA
LB MC ND
·· · =1
KB
LC
  MD   N A 
β γ δ ND
⇔ − · − · − · =1
α β γ NA
ND α
⇔ =− (4).
NA δ
Từ (1) suy ra K là tâm tỉ cự của hệ {αA, βB}.
Từ (2) suy ra L là tâm tỉ cự của hệ { βB, γC }.
Từ (3) suy ra M là tâm tỉ cự của hệ {γC, δD }.
Từ (4) suy ra N là tâm tỉ cự của hệ {δD, αA}.
Gọi Z là tâm tỉ cự của hệ {αA, βB, γC, δD }. Ta có

(α + β + γ + δ) Z = αA + βB + γC + δD
= (αA + βB) + (γC + δD )
= (α + γ)K + (γ + β) M

Suy ra Z ∈ KM. Tương tự ta chứng minh được Z ∈ LN.Hai đường thẳng KM và LN


cắt nhau tại Z suy ra K, L, M, N đồng phẳng.

Bài toán 10. Cho O là tâm của hình tứ diện đều ABCD và M là một điểm bất kỳ trên
một mặt nào đó của tứ diện. Gọi N, P, Q là hình chiếu vuông góc của M lên các mặt
còn lại và G là trọng tâm của 4 NPQ. Chứng minh rằng M, G, O thẳng hàng.

Lời giải. Giả sử M ∈ ( ABC ) và N, P, Q là hình chiếu vuông góc của M lên các mặt
( BCD ), (CDA), ( DAB) còn A0 , B0 , C 0 là hình chiếu vuông góc của A, B, C lên các mặt
đối diện.
Do tứ diện ABCD đều nên O là trọng tâm của tứ diện và AA0 , BB0 , CC 0 là các trọng
tuyến của tứ diện.

Do M ∈ ( ABC ) ⇒ ∃α, β, γ ∈ R, α + β + γ = 1 sao cho M là tâm tỉ cự của hệ chất


điểm: {αA, βB, γC } tức là: 1M = αA + βB + γC.

Theo tính chất chiếu được của tâm tỉ cự ta có:


1N = αA0 + βB + γC
1P = αA + βB0 + γC

175
Hội thảo khoa học, Ninh Bình 15-16/09/2018

1Q = αA + βB + γC 0

suy ra 1M + 1N + 1P + 1Q = (3αA + αA0 ) + (3βB + βB0 ) + (3γC + γC 0 )

Do đó ta có: 1M + 3G = α(3A + A0 ) + β(3B + B0 ) + γ(3C + C 0 )

M
O

B D
N
H

C
   
8 1 8 4
Mặt khác: α(3A + A0 ) = α A + (1A + 3A0 ) = α A+ O
3 3 3 3
Làm tương tự cho 2 biểu thức còn lại ta có:
8 4 8 4 8 4
1M + 3G = αA + αO + βB + βO + γC + γO
3 3 3 3 3 3
8 4
= (αA + βB + γC ) + (α + β + γ)O
3 3
8 4
= M+ O
3 3
5 4
suy ra 3G = M + O ⇔ 9G = 5M + 4O
3 3
suy ra G ∈ MO. Vậy G, M, O thẳng hàng.

5 Bài tập áp dụng

Bài 1. Cho tứ diện ABCD bất kỳ có G A , GB , GC , GD lần lượt là trọng tâm của các tam
giác BCD, CDA, DAB, ABC và G là trọng tâm tam giác G A GB GC . Chứng minh rằng
A, G, G A thẳng hàng.

176
Hội thảo khoa học, Ninh Bình 15-16/09/2018

Bài 2. Cho A, B, C, D là bốn điểm phân biệt nằm trên đường tròn. Chứng minh rằng
các đường thẳng mỗi đường đi qua trung điểm của đoạn thẳng nối 2 trong số 4 điểm
đó và vuông góc với đường thẳng đi qua 2 điểm còn lại là đồng quy.

Bài 3 (Mở rộng của ví dụ 10). Cho M là điểm bất kỳ trong không gian và N, P, Q, R
lần lượt là hình chiếu vuông góc của M lên các mặt của một tứ diện đều bất kỳ. Khi
đó M, G, O thẳng hàng ở đó G là trọng tâm của tứ diện NPQR và O là trọng tâm của
tứ diện đều đã cho

Bài 4 (USAMO 2001). Cho tam giác ABC, đường tròn (C ) nội tiếp tam giác. Các điểm
D1 và E1 lần lượt là các tiếp điểm của đường tròn (C ) với cạnh BC và AC. Điểm D2 và
E2 nằm trên BC và AC tương ứng sao cho CD2 = BD1 và CE2 = AE1 . Giao điểm của
đường thẳng AD2 và BE2 là P. Đường thẳng AD2 cắt đường tròn (C ) tại hai điểm,
điểm gần đỉnh A kí hiệu là Q. Chứng minh rằng AQ = D2 P.

Tài liệu

[1] Văn Như Cương, Tạ Mân (2000), Hình học afin và Hình học Ơclit, NXB ĐHQG Hà
Nội (2000).

[2] Tạp chí Toán học và Tuổi trẻ.

[3] Diễn đàn MathScope: http://forum.mathscope.org/index.php

[4] Eric W. Weisstein. Barycentric Coordinates.From MathWorld A Wolfram Web Re-


source.
http://mathworld.wolfram.com/BarycentricCoordinates.html

[5] AoPS. Practice Olympiad 4. WOOT 2011-2012.

177
Hội thảo khoa học, Ninh Bình 15-16/09/2018

MỘT SỐ DẠNG TOÁN ĐẾM TẬP SINH BỞI HÀM SỐ


QUA CÁC ĐỀ THI O LYMPIC

Tô Thị Lan
Trường THPT Chuyên Lương Văn Tụy, Ninh Bình

Tóm tắt nội dung


Báo cáo này nhằm trình bày một số dạng toán đếm tập hữu hạn sinh bởi hàm số qua
các kỳ thi Olympic những năm gần đây và xét các ứng dụng liên quan.

1 Một số dạng toán về đếm trong số học


Mục này trình bày một số dạng toán đếm tập hữu hạn số học dựa trên các nguyên lý
tính toán số nguyên.
Bài toán 1 (VMO-1995, Bảng B). Hỏi từ các số 1, 2, 3, 4, 5 có thể lập được bao nhiêu số có
10 chữ số thỏa mãn đồng thời các điều kiện sau:
1. Trong mỗi số, mỗi chữ số có mặt đúng hai lần
2. Trong mỗi số, hai chữ số giống nhau không đứng cạnh nhau
Lời giải. Gọi A là tập gồm tất cả các số có 10 chữ số, lập từ các số 1, 2, 3, 4, 5 thỏa mãn
điều kiện (1) của để bài. Với mỗi i kí hiệu Ai là tập gồm tất cả các số thuộc A mà trong
mỗi số đều có hai chữ số i đúng cạnh nhau.
Suy ra số
các số cần
tìm là:
5 5 5 5
A\ ∪ Ai = | A| − ∪ Ai = | A| − ∑ | Ai | + ∑ | Ai1 ∩ Ai2 |
i =1 i =1
i =1 i1;i2=1
5 5 5
− ∑ | Ai1 ∩ Ai2 ∩ Ai3 | + ∑ | Ai1 ∩ Ai2 ∩ Ai3 ∩ Ai4 | − ∩ Ai

i1;i2;i3=1 i1;i2;i3;i4=1 i =1
Ta có
10! (10 − k)!
| A| = ; | Ai1 ∩ Ai2 ∩ · · · ∩ Aik | = .
2 5 25−k
Suy ra kết quả là: 39480.
Bài toán 2 (VMO 1995 bảng B). Từ các chữ số 1, 2, 3, 4, 5 có thể lập được bao nhiêu số tự
nhiên có 10 chữ số thỏa mãn đồng thời các điều kiện sau:
a. Trong mỗi số, mỗi chữ số có mặt đúng hai lần.
b. Trong mỗi số, hai chữ số giống nhau không đứng cạnh nhau.

Lời giải. ĐặtA = {1; 2; 3; 4; 5}.


Gọi T = i1 i2 . . . i10 trong đó i j ∈ A, j = 1, 2, . . . , 10 và mỗi chữ số thuộc A có mặt
đúng hai lần.

178
Hội thảo khoa học, Ninh Bình 15-16/09/2018

Xét tập Ti ⊂ T mà trong i1 i2 . . . i10 ∈ Ti thì mỗi chữ số i thuộc A chiếm hai vị trí liên
tiếp.
Xét tập B ⊂ T mà trong i1 i2 . . . i10 ∈ B có i j 6= i j+1 với mọi j = 1, 2, . . . , 9
5
Như vậy B = T \
S
Ti
i =1
Theo tính
chất tập hợp, ta có:
5
S
| B| = T \ Ti = | T | −
i =1
!
5 T T T T T T T 5
∑ | Ti | − ∑ Ti Tj + ∑ Ti Tj Tk − ∑ Ti Tj Tk Tl + Ti .

i =1 1≤ i < j ≤5 1≤ i < j < k ≤5 1≤ i < j < k < l ≤5 i =1
• Với i1 i2 . . . i10 ∈ T
Xem như 10 chữ số hình thứci j khác nhau từng đôi một, ta có 10! cách chọn số
i1 i2 . . . i10 .
Tuy nhiên khi hoán vị hai chữ số giống nhau thì số được tạo thành không khác số ban
10!
đầu nên | T | = 5 .
2
5 9!
• ∑ | Ti | = C51 . 4 ;
i =1 2
T 8!
∑ Ti Tj = C52 . 3 ;
1≤ i < j ≤5 2
Ti Tj Tk = C3 7! ;
T T
∑ 5 2
1≤ i < j < k ≤5 2

T T T 6! T 5
4
∑ Ti Tj Tk Tl = C5 ; Ti = 5!

1≤ i < j < k < l ≤5 2 i =1
Đáp số: 39480.

Bài toán 3. Có bao nhiêu số tự nhiên có 4 chữ số mà tổng các chữ số của nó là bội của 4.
Lời giải. Gọi A là tập các số có 4 chữ số abcd ( a ≥ 1) sao cho a + b + c + d chia hết cho 4.
Xét b + c + d = 4k + r (0 ≤ r ≤ 3).
Nếu r ∈ {0, 1, 2} thì với mỗi giá trị của r tồn tại hai giá trị của a sao cho a + b + c + d
chia hết cho 4, đó là a = 4 − r hoặc a = 8 − r.
Nếu r = 3 thìntồn tại 3 giá trị của a sao cho a + b + c + d chia hếtocho 4, đó là a = 1; 5; 9.
Kí hiệu B = bcd/0 ≤ b, c, d ≤ 9; b + c + d = 4k + r, 0 ≤ r ≤ 2 .
n o
B = bcd/0 ≤ b, c, d ≤ 9; b + c + d = 4k + 3 .
Ta có
| A| = 2. | B| + 3. |C | = 2. (| B| + |C |) + |C | = 2.103 + |C |.
Xét tập hợp C. Giả sử c + d = 4m + s.
Nếu s ∈ {0, 1} thì tồn tại hai giá trị của b sao cho b + c + d = 4k + 3.
Nếu s ∈ {2, 3n } thì tồn tại ba giá trị của b sao cho b + c +o d = 4k + 3.
Kí hiệu D = cd/0 ≤ c, d ≤ 9; c + d = 4m + s, s = 0, 1
n o
E = cd/0 ≤ c, d ≤ 9; c + d = 4m + s, s = 2, 3 .
Ta có |C | = 2. | D | + 3. |C | = 2. (| D | + | E|) + | E| = 2.103 + | E|, với | E| = 24 + 25 = 49.
Vậy | A| = 2.103 + 2.102 + 49 = 2249.

179
Hội thảo khoa học, Ninh Bình 15-16/09/2018

Bài toán 4. Một số được gọi là số đẹp nếu nó là hợp số và không chia hết cho 2, 3, 5 (ví
dụ ba số 49, 77, 91 là ba số đẹp nhỏ nhất). Hỏi có tất cả bao nhiêu số đẹp nhỏ hơn 1000.
+ Có tất cả 168 số nguyên tố nhỏ hơn 1000.
+ Nếu n là số nguyên dương bất kì nhỏ hơn 999 và trong tập hợp {1, 2, 3, . . . , 999} có
999 999
k số chia hết cho n thì kn ≤ 999 < (k + 1)n ⇒ k ≤ < (k + 1) và k = .
n n
Lời giải. Gọi A, B, C tương ứng là tập hợp các số chia hết cho 2, 3, 5.
Số phần tử của
 mỗi tập hợp  là:   
999 999 999
| A| = = 499; | B| = = 333; |C | = = 199.
2 3 5  
999
Tập hợp các số chia hết cho 6 là A B suy ra | A B| =
T T
= 166
6 
999
Tập hợp các số chia hết cho 10 là A C suy ra | A C | =
T T
= 99
 10 
999
Tập hợp các số chia hết cho 15 là B C suy ra | B C | =
T T
= 66
15  
999
Tập hợp các số chia hết cho 30 là A B C nên | A B C | =
T T T T
= 33.
30
Tập hợp các số chia hết cho ít nhất một trong ba số 2, 3, 5 là A B C suy ra
S S

| A B C | = | A| + | B| + |C | − | A B| − | B C | − |C A| + | A B C | = 733
S S T T T T T

Do đó các số tự nhiên nhỏ hơn 1000 và không chia hết cho ba số 2, 3, 5 là


999 − 733 = 266.
Trong tập hợp 266 số trên có cả số 1 và các số nguyên tố khác 2, 3, 5. Mặt khác ta biết
rằng có tất cả 165 số nguyên tố nhỏ hơn 1000 và khác 2, 3, 5.
Vậy số số đẹp phải tìm là 266 − 165 − 1 = 100.
Bài toán 5. Cho hai số nguyên dương m và n sao cho n + 2 chia hết cho m. Hãy đếm số
các bộ ba số nguyên dương ( x; y; z) thoả mãn điều kiện: tổng x + y + z chia hết cho m,
trong đó mỗi số x, y, z đều không lớn hơn n.

Lời giải. Đặt k = (n + 2) /m ∈ N ∗ . Ta có n = km − 2. Xét các tập hợp:


D= {km − 1, km}, E = {1, 2, . . . , km}, 
.
A = ( x, y, z) /x, y, z ∈ E\ D; x + y + z..m ,
..
 
B = ( x, y, z) /x, y, z ∈ E; x + y + z.m ,
..
 
C = ( x, y, z) /x, y, z ∈ E; x ∈ D ∨ y ∈ D ∨ z ∈ D; x + y + z.m .
Dễ thấy | A| = | B| − |C |.
- Tính | B|: Có km cách chọn x ∈ E. Với mỗi cách chọn x ∈ E ta có km cách chọn y ∈ E.
.
Với mỗi cách chọn x và y như trên ta có k cách chọn z ∈ E sao cho x + y + z..m. Do đó
| B| = km.km.k = k3 m
.
2
..

- Tính |C |: Đặt X = ( x, y, z) /x ∈ D, y ∈ E, z ∈ E, x + y + z.m ,
..
 
Y = ( x, y, z) /y ∈ D, x ∈ E, z ∈ E, x + y + z.m ,
..
 
Z = ( x, y, z) /z ∈ D, x ∈ E, y ∈ E, x + y + z.m .

180
Hội thảo khoa học, Ninh Bình 15-16/09/2018

Khi đó, ta có C = X ∪ Y ∪ Z. Do đó
| C | = | X | + |Y | + | Z | − | X ∩ Y | − | X ∩ Z | − |Y ∩ Z | + | X ∩ Y ∩ Z | .
Chứng minh tương tự, ta được
| X | = |Y | = | Z | = 2k2 m,| X ∩ Y | = | X ∩ Z | = |Y ∩ Z | = 4k,
24−m ; m = 1, 2, 3, . . .
| X ∩ Y ∩ Z | = τ (m) = (∗)
1 ; m>3
Do đó |C | = 6k2 m − 12k + τ (m).
n+2 n3 + 8
Thay k = , ta được: | A| = | B| − |C | = − τ (m), τ (m) xác định bởi (∗).
m m

2 Một số dạng toán về đếm trong hình học tổ hợp


Tiếp theo, ta xét một số dạng toán về đếm xuất hiện trong hình học tổ hợp.

Bài toán 6 (VMO 2014). Cho đa giác đều có 103 cạnh. Tô màu đỏ cho 79 đỉnh của đa giác
và tô màu xanh cho các đỉnh còn lại. Gọi A là số cặp đỉnh đỏ kề nhau và B là số cặp đỉnh
xanh kề nhau.
a) Tìm tất cả các giá trị có thể nhận được của cặp (A, B).
b) Xác định số cách tô màu các đỉnh của đa giác để B = 14. Biết rằng, hai cách tô màu
được xem là như nhau nếu chúng có thể nhận được từ nhau thông qua một phép quay
quanh tâm đường tròn ngoại tiếp đa giác.

Lời giải.
a) Theo bài ra ta có số đỉnh màu xanh là 24. Nếu tất cả các đỉnh đỏ chia thành 1 cụm
thì A = 78. Nếu bị cắt thành 2 cụm thì A = 77. Cứ thế tiếp tục, tức là nếu có k cụm ( mỗi
cụm là các đỉnh cùng màu đỏ đứng sát nhau) thì A = 79 - k. Mặt khác, nếu có k cụm đỏ
thì hiển nhiên cũng có k cụm xanh nên ta có B = 24 - k. Các giá trị của k là từ 1 đến 24
nên do đó có 24 khả năng tất cả.
b) Để có B = 14 thì k = 10 (phải chia quân xanh thành 10 cụm, quân đỏ thành 10 cụm).
Áp dụng công thức chia kẹo trong bài toán chia kẹo Euler ta suy ra được số cách chia 24
điểm xanh vào 10 cụm là C23 9 .

Tiếp theo ta sẽ xem xét việc xếp các điểm xanh- đỏ như là việc có sẵn 79 điểm đỏ ở
trên đường tròn và ta bỏ 10 cụm xanh vào các khoảng trống giữa 2 điểm đỏ liên tiếp, mỗi
khoảng có tối đa 1 cụm.
Do đó số cách chọn ra 10 khoảng trống trong 79 khoảng là C79 10 . Sự trùng lập theo

phép quay là ở chỗ ta chọn 10 vị trí trong 79 vị trí theo đường tròn.
Nhờ có (79, 10) = 1 mà ta không phải lo về các cấu hình trùng lặp, và mỗi cách tô bị
10 C 9 ) /79.
lặp lại 79 lần, do đó đáp số là (C79 23

Bài toán 7 (VMO 2007). Cho một đa giác đều 2007 đỉnh. Tìm số nguyên dương k nhỏ
nhất thỏa mãn tính chất: trong mỗi cách chọn k đỉnh của đa giác luôn tồn tại 4 đỉnh tạo
thành một tứ giác lồi mà 3 trong số 4 cạnh của nó là ba cạnh của đa giác đã cho.
Lời giải. . Gọi các đỉnh của đa giác đều đã cho là A1 , A2 , . . . , A2007 .
Ta thấy tứ giác được tạo từ 4 đỉnh của đa giác đã cho có 3 cạnh là 3 cạnh liên tiếp của
đa giác khi và chỉ khi 4 đỉnh của tứ giác đó là 4 đỉnh liên tiếp của đa giác đều đã cho.
Gọi A là tập các đỉnh { A1 , A2 , A3 , A5 , . . . , A2006 } (tập các đỉnh của đa giác đã cho trừ
đi các đỉnh A4i , i = 1, . . . , 501, và A2007 ).

181
Hội thảo khoa học, Ninh Bình 15-16/09/2018

Số phần tử của tập A là | A| = 1505. Mọi tập con của A đều không chứa 4 đỉnh liên
tiếp của đa giác. Do đó k ≥ 1506.
Ta đi chứng minh cách chọn 1506 đỉnh bất kì của đa giác sẽ tồn tại 4 đỉnh liên tiếp của
đa giác trong 1506 đỉnh đó.
Thật vậy, giả sử T là tập hợp gồm 1506 đỉnh nào đó của đa giác. Phân hoạch tập các
đỉnh của đa giác thành các tập hợp
B1 = { A1 , A2 , A3 , A4 } , B2 = { A5 , A6 , A7 , A8 } , . . . , B502 = { A2005 , A2006 , A2007 } .
Giả sử T không chứa 4 đỉnh liên tiếp của đa giác. Lúc đó, với mỗi i = 1, 2, . . . , 501, tập
Bi không thuộc T, tức là tập Bi có ít nhất một đỉnh không thuộc T. Khi đó ta có
| T | ≤ 3.502 = 1506. Vì | T | = 1506 nên B502 ⊂ T và mỗi tập Bi ,i = 1, 2, . . . , 501,có
đúng 3 phần tử thuộc T.
Ta có
A2005 , A2006 , A2007 ∈ T nên A1 ∈ / T suy ra A2 , A3 , A4 ∈ T và A5 ∈ / T.
Cứ tiếp tục như vậy ta suy ra A2002 , A2003 , A2004 ∈ T. Khi đó ta có 4 đỉnh liên tiếp
A2002 , A2003 , A2004 , A2005 ∈ T, mâu thuẫn.
Vậy k = 1506.

Bài toán 8. Có bao nhiêu cách chia 1 đa giác đều (n+2) cạnh thành các tam giác bằng
cách kẻ các đường chéo không cắt nhau bên trong đa giác.

Lời giải.
+ n = 1 thì ∆ đều
+ n = 2 thì ta được hình vuông, có 2 cách chia thành tam giác.
+ n = 3 thì ta được ngủ giác đều, có 5 cách chia thành tam giác.
Bây giờ ta cần tìm công thức truy hồi để biểu diễn số Cn qua các số
C0 , C1 , .., Cn−1 , . . .
Cn = Cn−1 C0 + Cn−2 C1 + Cn−3 C2 + · · · + C1 Cn−2 + Cn−1 C0
Không có đường chéo nào đi qua đỉnh số 1: có Cn−1 C0 cách tam giác phân.
Theo chiều kim đồng hồ, đường chéo đầu tiên xuất phát từ đỉnh 1 sẽ nối nó với đỉnh
k. Số ∆ phân:
Ck−3 Cn−k+2
n +1 n −2
∑ Ck−3 Cn−k+2 = ∑ Ci Cn−1−i
k =3 i =0
Cn = Cn−1 C0 + Cn−2 C1 + Cn−3 C2 + · · · + C1 Cn−2 + Cn−1 C0 (1)
Xét hàm sinh
C ( x ) = C0 + C1 x + C2 x2 + · · · + Cn x n + . . .
C2 ( x ) = C0 + (C1 C0 + C0 C1 ) x + (C0 C2 + C1 C1 + C2 C0 ) x2 +
(C0 C3 + C1 C2 + C2 C1 + C3 C0 ) x3 + . . .
Theo (1) thì
C2 ( x ) = C0 + C2 x + C3 x2 + C4 x3 + . . . nên
xC2 ( x ) = x + C2 x2 + C3 x3 + C4 x4 + · · · + Cn x n + . . .
⇔ xC2 ( x ) = C1 x + C2 x2 + C3 x3 + C4 x4 + · · · + Cn x n + · · · ⇔ xC2 ( x ) = C ( x ) − 1.
Suy ra √
2 1 ± 1 − 4x
xC ( x ) − C ( x ) + 1 = 0 ⇔ C ( x ) = hay
2x
1
1 ± (1 − 4x ) 2
xC ( x ) = .
2

182
Hội thảo khoa học, Ninh Bình 15-16/09/2018

Sưr dụng khai triển


1 1
!
1 1 1 1 ∞ k k
xC ( x ) = − (1 − 4x ) 2 = − ∑ 2 (−4) x
2 2 2 2 k =0 k
1
!
∞ 1
k k
=− ∑ 2 (−4) x .
k =1 2 k
Do đó

1
!
1
Cn = − 2 (−4)n+1
2 n+1
    
1 1 1 1
−1 −2 ... −n
n 2 2 2 2 1.3.5 . . . (2n − 1)
= 2. (−4) . = 4n .
( n + 1) ! 2n ( n + 1 ) !
1.3.5 . . . (2n − 1) 1 2n!
= 2n . n! =
(n + 1)n!n! n + 1 n!n!
 
1 2n
=
n+1 n

3 Một số tính toán khác trên tập rời rạc


Bài toán 9 (VMO 2012). Cho một nhóm có 5 cô gái là G1 , G2 , G3 , G4 , G5 và 12 chàng trai.
Có 17 chiếc ghế được xếp thành 1 hàng ngang. Người ta xếp nhóm người đó chỉ ngồi vào
các chiếc ghế đó sao cho các điều kiện sau đồng thời thỏa mãn
1. Mỗi ghế có duy nhất 1 người ngồi.
2. Thứ tự ngồi của các cô gái, xét từ trái qua phải là G1 , G2 , G3 , G4 , G5 .
3. Giữa G1 và G2 có ít nhất 3 chàng trai.
4. Giữa G4 và G5 có ít nhất 1 chàng trai và nhiều nhất 4 chàng trai.
Hỏi có tất cả bao nhiêu cách sắp xếp như vậy?
(Hai cách xếp được coi là khác nhau nếu tồn tại một chiếc ghế mà người ngồi ở ghế
đó trong hai cách xếp là khác nhau).

Lời giải. Áp dụng bài toán chia kẹo Euler, ta đánh số thứ tự các ghế từ trái sang phải là
1, 2,. . . , 17. Gọi x1 là số chàng trai được xếp bên trái G1 , x2 là số chàng trai ở giữa G1 và
G2 , x3 là số chàng trai ở giữa G2 và G3 , x4 là số chàng trai ở giữa G3 và G4 , x5 là số chàng
trai ở giữa G4 và G5 , x6 là số chàng trai ngồi bên phải G5 . Khi đó bộ số (x1 , x2 ,. . . , x5 ) hoàn
toàn xác định vị trí các cô gái và ta có
1) x1 + x2 +. . . + x6 = 12
2) 3 ≤ x2
3) 1 ≤ x5 ≤ 4.
Đổi biến y2 = x2 - 3 và y5 = x5 - 1, ta được x1 + y2 + x3 + x4 + y5 + x6 = 8 với các ẩn
không âm và có thêm điều kiện y5 ≤3.
Tiếp theo áp dụng bài toán chia kẹo dạng x1 + y2 + x3 + x4 + x6 = 8 − y5 .
Ta được số cách phân ghế cho các cô gái là C1 24 + C1 14 + C1 04 + C94 = 1161.
Vì còn có 12 chàng trai có thể hoán đổi vị trí ở 12 ghế dành cho họ nên số cách xếp
thỏa mãn các yêu cầu bài toán là 12!1161.

183
Hội thảo khoa học, Ninh Bình 15-16/09/2018

Bài toán 10 (VMO 2005 - Bảng B). Thống kê kết quả học tập ở một lớp học, người ta thấy:
2
Hơn số học sinh đạt điểm giỏi ở môn Toán cũng đồng thời đạt điểm giỏi môn Vật
3
2
Lí; hơn số học sinh đạt điểm giỏi ở môn Vật lí cũng đồng thời đạt điểm giỏi môn Văn;
3
2
hơn số học sinh đạt điểm giỏi ở môn Văn cũng đồng thời đạt điểm giỏi môn Lịch sử;
3
2
hơn số học sinh đạt điểm giỏi ở môn Lịch sử cũng đồng thời đạt điểm giỏi môn Toán.
3
Chứng minh rằng trong lớp học nói trên có ít nhất một học sinh đạt điểm giỏi ở cả
bốn môn Toán, Vật lí, Văn và Lịch sử.
Lời giải. Gọi T, L, V, S lần lượt là tập hợp các học sinh đạt điểm giỏi ở các môn Toán, Vật
lí, Văn và Lịch sử.
Kí hiệu T1 = T ∩ L; L1 = L ∩ S; V1 = V ∩ S.
Theo giả thiết ta có
2 2 2
| T1 | > | T | ; | L1 | > | L| ; |V1 | > |V |.
3 3 3
Ta sẽ chứng minh | T ∩ L ∩ V ∩ S| > 0.
Không mất tổng quát, ta giả sử T là tập có nhiều phần tử nhất.
Ta có
| T1 ∩ L1 | = | T1 | + | L1 | − | T1 ∪ L1 |. (1)
Vì T1 ∪ L1 ⊂ L nên | T1 ∪ L1 | ≤ | L|.
Do đó từ (1) ta có
2 2 2 1 1
| T1 ∩ L1 | > | T | + | L| − | L| = | T | − | L| ≥ | T | . (2)
3 3 3 3 3
Lại có
| T1 ∩ L1 ∩ V1 | = | T1 ∩ L1 | + |V1 | − |( T1 ∩ L1 ) ∪ V1 |. (3)
Vì L1 ⊂ V nên T1 ∩ L1 ⊂ V.
Mà V1 ⊂ V nên ( T1 ∩ L1 ) ∪ V1 ⊂ V.
Do đó |( T1 ∩ L1 ) ∪ V1 | ≤ |V |.
Từ (2) và (3) ta được
1 2 1 1
| T1 ∩ L1 ∩ V1 | > | T | + |V | − |V | = | T | − |V | ≥ 0.
3 3 3 3
Hiển nhiên, T ∩ L ∩ V ∩ S = T1 ∩ L1 ∩ V1 , suy ra | T ∩ L ∩ V ∩ S| > 0 (đpcm).
Bài toán 11 (IMO 1991). Cho S = {1, 2, . . . , 280}. Tìm số tự nhiên n nhỏ nhất sao cho mọi
tập hợp con n phần tử của S đều chứa 5 số tự nhiên nguyên tố cùng nhau từng đôi một.
Lời giải. Ta sẽ chứng minh n > 217.
Gọi A1 , A2 , A3 , A4 là các tập con của S chứa các bội lần lượt của 2, 3,5,7. Ta có
| A1 | = 140, | A2 | = 93, | A3 | = 56, | A4 | = 40;
| A1 ∩ A2 | = 46, | A1 ∩ A3 | = 28, | A1 ∩ A4 | = 20;
| A2 ∩ A3 | = 18, | A2 ∩ A4 | = 13, | A3 ∩ A4 | = 8;
| A1 ∩ A2 ∩ A3 | = 9, | A1 ∩ A2 ∩ A4 | = 6, | A1 ∩ A3 ∩ A4 | = 4, | A2 ∩ A3 ∩ A4 | = 2;
| A1 ∩ A2 ∩ A3 ∩ A4 | = 1
Ta có | A1 ∪ A2 ∪ A3 ∪ A4 | = 140 + 93 + 56 + 40 − 46 − 28 − 20 − 18 − 13 − 8 + 9 +
6 + 4 + 2 − 1 = 216.
Nếu n ≤ 216thì tập A1 ∪ A2 ∪ A3 ∪ A4 chứa một tập con có n mà tập con này không
chứa 5 phần tử nào nguyên tố cùng nhau từng đôi một. vậy n ≥ 217
Ta sẽ dùng nguyên lí Dirichlet để chứng minh mọi tập con chứa 217 phần tử của S
đều có 5 phần tử nguyên tố cùng nhau từng đôi một.

184
Hội thảo khoa học, Ninh Bình 15-16/09/2018

Gọi A là tập con của S mà | A| ≥ 217, và B1 là tập con chứa số 1 và tất cả các số nguyên
tố của S, |B1 | = 60,
B2 = 22 , 32 , 52 , 72 , 112 , 132 ,

B3 = {2 × 131; 3 × 80; 5 × 33; 7 × 37; 11 × 23; 13 × 19},


B4 = {2 × 127; 3 × 87; 5 × 47; 7 × 31; 11 × 19; 13 × 17},
B5 = {2 × 113; 3 × 79; 5 × 43; 7 × 27; 11 × 17; 13 × 19},
B6 = {2 × 100; 3 × 73; 5 × 41; 7 × 23; 11 × 13}.
Các tập B1 , B2 , . . . , B6 từng đôi một không có phần tử chung và trong mỗi Bi các phần
tử nguyên tố cùng nhau từng đôi một.
Vậy | B1 ∪ B2 ∪ B3 ∪ B4 ∪ B5 ∪ B6 | = 88.
Suy ra
|S\ ( B1 ∪ B2 ∪ B3 ∪ B4 ∪ B5 ∪ B6 )| = 280 − 88 = 192.
Vì A có ít nhất 217 phần tử nên có ít nhất 217 - 192 =25, ít nhất 5 phần tử của A nằm
trong một tập Bi , tức là có ít nhất 5 phần tử nguyên tố cùng nhau từng đôi một.

Bài toán 12 (VMO 2007). Cho một đa giác đều 2007 đỉnh. Tìm số nguyên dương k nhỏ
nhất thỏa mãn tính chất: trong mỗi cách chọn k đỉnh của đa giác luôn tồn tại 4 đỉnh tạo
thành một tứ giác lồi mà 3 trong số 4 cạnh của nó là ba cạnh của đa giác đã cho.
Lời giải. . Gọi các đỉnh của đa giác đều đã cho là A1 , A2 , . . . , A2007 .
Ta thấy tứ giác được tạo từ 4 đỉnh của đa giác đã cho có 3 cạnh là 3 cạnh liên tiếp của
đa giác khi và chỉ khi 4 đỉnh của tứ giác đó là 4 đỉnh liên tiếp của đa giác đều đã cho.
Gọi A là tập các đỉnh { A1 , A2 , A3 , A5 , . . . , A2006 } (tập các đỉnh của đa giác đã cho trừ
đi các đỉnh A4i , i = 1, . . . , 501, và A2007 ).
Số phần tử của tập A là | A| = 1505. Mọi tập con của A đều không chứa 4 đỉnh liên
tiếp của đa giác. Do đó k ≥ 1506.
Ta đi chứng minh cách chọn 1506 đỉnh bất kì của đa giác sẽ tồn tại 4 đỉnh liên tiếp của
đa giác trong 1506 đỉnh đó.
Thật vậy, giả sử T là tập hợp gồm 1506 đỉnh nào đó của đa giác. Phân hoạch tập các
đỉnh của đa giác thành các tập hợp
B1 = { A1 , A2 , A3 , A4 } , B2 = { A5 , A6 , A7 , A8 } , . . . , B502 = { A2005 , A2006 , A2007 } .
Giả sử T không chứa 4 đỉnh liên tiếp của đa giác. Lúc đó, với mỗi i = 1, 2, . . . , 501, tập
Bi không thuộc T, tức là tập Bi có ít nhất một đỉnh không thuộc T. Khi đó ta có
| T | ≤ 3.502 = 1506. Vì | T | = 1506 nên B502 ⊂ T và mỗi tập Bi ,i = 1, 2, . . . , 501,có
đúng 3 phần tử thuộc T.
Ta có
A2005 , A2006 , A2007 ∈ T nên A1 ∈ / T suy ra A2 , A3 , A4 ∈ T và A5 ∈ / T.
Cứ tiếp tục như vậy ta suy ra A2002 , A2003 , A2004 ∈ T. Khi đó ta có 4 đỉnh liên tiếp
A2002 , A2003 , A2004 , A2005 ∈ T, mâu thuẫn.
Vậy k = 1506.

Bài toán 13 (VMO-1995, Bảng B). Hỏi từ các số 1, 2, 3, 4, 5 có thể lập được bao nhiêu số
có 10 chữ số thỏa mãn đồng thời các điều kiện sau:
1. Trong mỗi số, mỗi chữ số có mặt đúng hai lần
2. Trong mỗi số, hai chữ số giống nhau không đứng cạnh nhau
Lời giải.
Gọi A là tập gồm tất cả các số có 10 chữ số, lập từ các số 1, 2, 3, 4, 5 thỏa mãn điều
kiện (1) của để bài. Với mỗi i kí hiệu Ai là tập gồm tất cả các số thuộc A mà trong mỗi số
đều có hai chữ số i đúng cạnh nhau.

185
Hội thảo khoa học, Ninh Bình 15-16/09/2018

Suy ra số
các số cần
tìm là:
5 5 5 5
A\ ∪ Ai = | A| − ∪ Ai = | A| − ∑ | Ai | + ∑ | Ai1 ∩ Ai2 |
i =1 i =1
i =1 i1;i2=1
5 5 5
− ∑ | Ai1 ∩ Ai2 ∩ Ai3 | + ∑ | Ai1 ∩ Ai2 ∩ Ai3 ∩ Ai4 | − ∩ Ai
i1;i2;i3=1 i1;i2;i3;i4=1 i =1
Ta có
10! (10 − k)!
| A| = ; | Ai1 ∩ Ai2 ∩ · · · ∩ Aik | = .
25 25−k
Suy ra kết quả là: 39480.
Bài toán 14 (VMO 2004). Cho tập A = {1, 2, . . . , 16}. Hãy tìm số nguyên dương k nhỏ
nhất sao cho trong mỗi tập con gồm k của A đều tồn tại hai số phân biệt a, b sao cho
a2 + b2 là một số nguyên tố.
Lời giải. Nếu a, b chẵn thì a2 + b2 là một hợp số. Do đó nếu tập con X của A có hai phần
tử phân biệt a, b mà a2 + b2 là một số nguyên tố thì Xkhông thể chỉ chứa các số chẵn. Suy
ra k ≥ 9. Ta sẽ chưng minh k = 9 là giá trị nhỏ nhất cần tìm.
Ta chia tập A thành các cặp số phân biệt a, b mà a2 + b2 là một số nguyên tố, có tất cả
8 cặp:
(1, 4), (2, 3), (5, 8), (6, 11), (7, 10), (9, 16), (12, 13), (14, 15).
Mỗi tập con Xgồm 9 phần tử của A, theo nguyên lí Dirichlet, tồn tại ít nhất hai phần
tử a, b thuộc cùng một cặp, tức a2 + b2 là một số nguyên tố.
Bài toán 15 (Czech Republic 1988). Cho tập hợp X gồm 14 số nguyên dương
phân biệt. Chứng minh rằng có một số nguyên dương k ≤ 7 và có
hai
 tập con k phần  tử { a1 , a2 , . . . , ak } , {b1
, b2 , . . . , bk } rời nhau của X sao cho
1 1 1 1 1 1 < 1 .


a + + · · · + − + + · · · +
1 a2 ak b1 b2 bk 1000
7 = 3432 tập con 7 phần tử của X. Tổng các nghịc đảo của các tập con
Lời giải. Xét C14
này không vượt quá
1 1 1
+ + ··· + < 2, 6 nên phải thuộc vào một trong số 2600 nửa khoảng
 1 2  7   
1 1 2 2599 2600
0; , ; ,..., ; .
1000 1000 1000 1000 1000
Theo nguyên lí Dirichlet, tồn tại hai tập con khác nhau có tổng nghịch đảo các phần
tử cùng thuộc vào một nửa khoảng. Loại bỏ khỏi hai tập đó các phần tử chung (hai tập
con, mỗi tập có 7 phần t)ử thì có tối đa 6 phần tử chung), ta sẽ thu được hai tập con kphần
tử (kmột số nguyên dương  và k ≤ 7), giả sử  { a1, a2 , . . . , ak } , {b1 , b2 , .
. . , bk }, rời nhau và
1 1 1 1 1 1 < 1 .

thỏa yêu cầu bài toán + +···+ − + +···+
a1 a2 ak b1 b2 bk 1000
Bài toán 16. Cho n là số nguyên dương và cho k số nguyên dương a1 , a2 , . . . , ak đôi một
nguyên tố cùng nhau. Tìm số các số nguyên dương a, a ≤ n sao cho a không chia hết cho
ai với mọi i = 1, 2, . . . , k.
Lời giải.
.
 
Ứng với mỗi i = 1, 2, . . . , k, ta đặt Ai = a∈ N ∗ /a ≤ n, a..ai và A là tập hợp các số
thoả mãn điều kiện của
bài toán thì
k
| A| = n − Ai (1) .
S
i =1

186
Hội thảo khoa học, Ninh Bình 15-16/09/2018

Dễ thấy tập
Ai = { ai , 2ai , . . . , mi ai } với mi ∈ N∗ và thoả mãn điều kiện:
mi ai : n < ( mi +  1) ai nên 
..

n ∗

| Ai | = mi = . Ai ∩ A j = a ∈ N , a ≤ n vµ a. ai a j
 ai 
Ai ∩ A j = n (Do ( ai , a j ) = 1)).

ai a j
Hoàn toàn tương tự, ta có 

Ai ∩ Ai ∩ · · · ∩ Ai =
n
1 2 k
.
a i1 a i2 . . . a i k
Vậy
nên
k k k
S
Ai = ∑ | Ai | − k −1 T
∑ | A i 1
∩ A i 2
| + · · · + (− 1 ) A i


i =1 i =1 1≤ i1 < i2 ≤ k i =1
     
k n n n
= ∑ − ∑ + · · · + (−1)k−1
i =1 a i 1≤ i1 < i2 ≤ k a i1 a i2 a1 a2 . . . a n
 
   
k n n k n 
 
| i|
A = n − ∑ + ∑ − · · · + (− 1 )
i =1 a i 1≤ i1 < i2 ≤ k a i1 a i2
 k 
∏ ai
 
i =1

Bài toán 17. Trong mặt phẳng cho tập hợn A có n điểm ( n≥ 2). Một số cặp điểm được
nối với nhau bằng đoạn thẳng. Chứng minh rằng tập hợp A đã cho, có ít nhất hai điểm
đươc nối với cùng số lượng các điểm khác thuộc A.
Lời giải. Giả sử a ∈ A. Ta kí hiệu S(a) là số lượng các điểm của A nối với a mà S(a) = 2,
S(b) = 3, S(c) = 1, S(d) = 2, S(e) = 2.
Bài toán đã cho trở thành:
Chứng minh rằng tồn tại a1 , a2 ∈ A( a1 6= a2 ), mà S( a1 ) = S( a2 ).
Rõ ràng với mọi a ∈ A, ta có: 0 ≤ S( a) ≤ n − 1. (1)
Mặt khác, dễ thấy không tồn tại hai điểm a ∈ A, b ∈ A mà S( a) = n − 1 và
S(b) = 0. (2)
Thật vậy, nếu có (2), thì từ S( a) = n − 1, ta suy ra a nối với tất cả n - 1 điểm còn lại,
tức là a cũng nối với b. Điều đó có nghĩa là S(b) ≥ 1, vã dẫn đến mâu thuẫn với S(b) = 0
Gọi S là tập hợp các giả trị mà các đại lượng S( a) nhận, a ∈ A,
tức là: S = {m/m = S( a), a ∈ A}.
Như vậy từ (1) suy ra tập hợp S có tối đa n giả trị. Tuy nhiên từ (2) suy ra (n-1) và 0
không đồng thời thuộc S, vì thế tập S tối đa nhận (n - 1) giá trị. Theo nguyên lí Dirichlet
suy ra tồn tại ít nhất a1 ∈ A, a2 ∈ A( a1 6= a2 ), mà S( a1 ) = S( a2 ).

Bài toán 18. Cho một bảng có kích thước 2n × 2n ô vuông. Người ta đánh dấu vào 3n ô
bất kì của bảng. Chứng minh rằng có thể chọn ra n hàng và n cột của bảng sao cho các ô
được đánh dấu đều nằm trên n hàng và n cột này.
Lời giải. Chọn ra n hàng có chứa số ô được đánh dấu nhiều trên các hàng đó nhất.
Ta chứng minh rằng các ô được đánh dấu còn nhỏ hơn hoặc bằng n. Giả sử ngược lại
không phải như vậy, tức là số ô được đánh dấu lớn hoan hoặc bằng n + 1. Số các hàng
còn lại chưa chọn là n. Vậy theo nguyên lí Dirichlet sẽ có ít nhất một hàng ( tỏng số n
hàng còn lại) chứa ít nhất hai ô đã đánh dấu.

187
Hội thảo khoa học, Ninh Bình 15-16/09/2018

Chú ý rằng theo cách chọn thì n hàng đã chọn có chứa số ô được đánh dấu nhiều
trên các hàng đó nhất. Có một hàng còn lại chưa chọn có ít nhất hai ô đánh dấu, nên suy
ra mọi hàng trong số n hàng đã chọn đều có ít nhất hai ô được chọn, tức là trên n hàng
đã chọn có không ít hơn 2n ô đã được đánh dấu. Như vậy, số ô được đánh dấu lớn hơn
hoặc bằng 2n + (n +1) ≥ 3n. Vô lí vì chỉ có 3n ô được đánh dấu. Vậy nhận xét được chứng
mình.
Như vậy, sau khi đã chọn ra n hàng ( với cách chọn như trên), theo nhận xét còn lại có
không quá n ô được đánh dấu. Vì thế cùng lắm là có n cột chứa chúng. Vì lẽ đó sẽ không
thấy còn ô đánh dấu nào nằm ngoài các hàng hay cột được chọn.

Tài liệu
[1] Nguyễn Văn Mậu (2002), Chuyên đề toán rời rạc, NXB Giáo dục.
[2] Nguyễn Văn Mậu (2007), Đa thức đại số và phân thức hữu tỷ, NXB Giáo dục.
[3] Nguyễn Văn Mậu (2017), Tổ hợp và các dạng toán rời rạc liên quan, NXB Đại học Quốc
gia Hà Nội.
[4] Nguyễn Đức Nghĩa, Nguyễn Tô Thành, 2004,Toán rời rạc,NXB Giáo dục.
[5] Đặng Huy Ruận, 2002, Bẩy phương pháp giải các bài toán logic, NXB Khoa học và Kĩ
thuật.
[6] Đặng Huy Ruận, 2004, Lý thuyết đồ thị và ứng dụng, NXB Khoa học và Kỹ thuật.

188
Hội thảo khoa học, Ninh Bình 15-16/09/2018

PHƯƠNG PHÁP GIẢI PHƯƠNG TRÌNH HÀM SINH


BỞI HÀM HỢP QUA CÁC KỲ O LYMPIC

Nguyễn Thị Bích Ngọc


Trường THPT Chuyên Lương Văn Tụy, Ninh Bình

Tóm tắt nội dung

Báo cáo này nhằm trình bày một số phương pháp giải các dạng toán về phương trình
hàm sinh bởi hàm hợp qua các kỳ thi Olympic những năm gần đây và xét các ứng dụng
liên quan.

1 Một số dạng phương trình giải bằng phương pháp


thế biến
Bài toán 1 (THTT, Bài T11/434). Tìm tất cả các hàm số f : R → R thỏa mãn các điều
kiện
f ( x f (y) + 3y2 ) + f (3xy + y) = f (3y2 + x ) + 4xy − x + y, ∀ x, y ∈ R. (1)
Lời giải. Thay x = 0 vào (1), ta được f (3y2 ) + f (y) = f (3y2 ) + y, ∀y ∈ R, hay f (y) = y.
Thử lại ta thấy hàm số f ( x ) = x thỏa mãn điều kiện bài ra.
Bài toán 2 (THTT, Bài T11/451). Tìm tất cả các hàm số f : R → R thỏa mãn điều kiện
1 2
f ( x2 + f (y)) = 4y + f ( x ) với mọi x, y ∈ R. (2)
2

Lời giải. Thay x = 0 vào (2) ta thu được

1 2 1
f ( f (y)) = 4y + f (0) = 4y + a2 , ∀y ∈ R, (2.1)
2 2
với a = f (0). Vì vế phải là hàm bậc nhất theo y nên suy ra f là song ánh.
Thay y = 0 vào (2) ta được
1 2
f ( x2 + f (0)) = f ( x ) với mọi x ∈ R. (2.2)
2
Tiếp tục thay x bởi − x trong (2.2), ta được
1 2
f ( x2 + f (0)) = f (− x ) với mọi x ∈ R. (2.3)
2

189
Hội thảo khoa học, Ninh Bình 15-16/09/2018

Từ (2.2) và (2.3) suy ra


f 2 ( x ) = f 2 (− x ) với mọi x ∈ R.
Trường hợp f (− x ) = f ( x ) chỉ xảy ra khi − x = x ⇔ x = 0 (do f là song ánh). Vậy
f (− x ) = − f ( x ) với mọi x 6= 0.
Vì f là song ánh nên tồn tại duy nhất số b sao cho f (b) = 0. Nếu b 6= 0 thì f (−b) =
− f (b) và vì vậy f (b) = f (−b) = 0 nên b = 0, mâu thuẫn.
Do vậy, f (0) = 0. Vậy f (− x ) = − f ( x ) với mọi x ∈ R, tức f là hàm lẻ. Khi đó (2.1)
có dạng
f ( f ( x )) = 4x, ∀ x ∈ R (∗)
và (2.2) có dạng
1 2
f ( x2 ) = f ( x ) ≥ 0, ∀ x ∈ R,
2
tức f ( x ) ≥ 0 khi x ≥ 0.
Thế vào (1), ta thu được

1 2
f ( x2 + f (y)) = f ( f (y)) + f ( x ) với mọi x, y ∈ R
2
hay
f (u + v) = f (u) + f (v), ∀u ≥ 0, v ∈ R. (2.4)
Trong (2.4), thay u bởi −u với u < 0, v bởi −v và sử dụng tính chất f là hàm lẻ, ta được

f (−u − v) = f (−u) + f (−v), ∀u < 0, v ∈ R

⇔ − f (u + v) = − f (u) − f (v), ∀u < 0, v ∈ R


f (u + v) = f (u) + f (v), ∀u < 0, v ∈ R. (2.5)
Từ (2.4) và (2.5) suy ra

f (u + v) = f (u) + f (v), ∀u, v ∈ R. (2.6)

Tiếp theo, sử dụng tính chất f ( x ) ≥ 0 khi x ≥ 0, ta thu đươc f ( x ) là hàm đơn điệu tăng,
do vậy là hàm đồng biến (vì f là song ánh).
Thật vậy, nếu x > y thì

f ( x ) = f (y + ( x − y)) = f (y) + f ( x − y) ≥ f (y).

Ta chứng minh f ( x ) = 2x với mọi x ∈ R.


Thật vậy, nếu tồn tại x0 sao cho f ( x0 ) > 2x0 thì từ (*) suy ra

4x0 = f ( f ( x0 )) > f (2x0 ) = f ( x0 ) + f ( x0 ) > 4x0 ,

vô lý.
Tương tự, trường hợp f ( x0 ) < 2x0 không xảy ra.
Thử lại ta thấy hàm số f ( x ) = 2x thỏa mãn điều kiện của bài toán.

190
Hội thảo khoa học, Ninh Bình 15-16/09/2018

Bài toán 3 (THTT, Bài T11/476). Cho số thực a ( a 6= 0, a 6= −1). Tìm tất cả các hàm số
f : R → R thỏa mãn điều kiện

f ( f ( x ) + ay) = ( a2 + a) x + f ( f (y) + x ), ∀ x, y ∈ R. (3)

Lời giải. Ta chứng minh f là song ánh.

Trước tiên, ta chứng minh f là toàn ánh.


y − f (0) f (u)
Thật vậy, ứng với mỗi y ∈ R cho trước, đặt u = 2
,v = − và w = f (v) − u.
a +a a
Khi đó, thế x = u, y = v vào (3), ta thu được

f (w) = f ( f (v) − u) = f ( f (u) + av) − ( a2 + a)v

= f (0) − ( a2 + a)u = f (0) − ( f (0) − y) = y,


tức f là toàn ánh.
Tiếp theo, ta chứng minh f là đơn ánh.
Giả sử f ( x1 ) = f ( x2 ). Vì f là toàn ánh nên ứng với mỗi cặp x1 , x2 ∈ R luôn tồn tại
y ∈ R để f (y) = x1 + x2 ⇔ x2 = f (y) − x1 . Theo (3), ta thu được

( a2 + a ) x1 + f ( x2 ) = ( a2 + a ) x1 + f ( f ( y ) − x1 ) = f ( f ( x1 ) + ( a2 + a ) y )

= f ( f ( x2 ) + ( a2 + a ) y ) = ( a2 + a ) x2 + f ( f ( y ) − x2 ) = ( a2 + a ) x2 + f ( x1 )
hay
( a2 + a ) x1 + f ( x2 ) = ( a2 + a ) x2 + f ( x1 ) ⇔ x1 = x2 .
Vậy nên f là đơn ánh.
Do đó f là song ánh.
Thế x = 0 vào (3), ta thu được

f ( f (0) + ay) = f ( f (y)) ⇔ f (0) + ay = f (y) ⇔ f (y) = ay + f (0), ∀y ∈ R.

Vậy f (t) = at + b với b ∈ R tùy ý.


Thử lại, ta thấy hàm này thỏa mãn (3).
Kết luận: Vậy ta luôn có f (t) = at + b, b ∈ R.
Nhận xét. Đây là bài toán về phương trình hàm một biến tự do mở rộng bài toán đề nghị
trong IMO shortlist 2002 (ứng với trường hợp a = 1).

Bài toán 4 (THTT, Bài T11/475). Tìm tất cả các hàm số f : R+ → R+ thỏa mãn điều kiện
 x 
f + f ( x f (y)) = f ( x f ( x )), (4)
x−y

với mọi x > y > 0.


Lời giải. Nhận xét rằng, từ (4), ta  thu được f là đơn ánh. Thật vậy, giả sử x > y > 0 và
x 
f ( x ) = f (y). Khi đó từ (4) suy ra f = 0, vô lý.
x−y
1
Tiếp theo, ta chứng minh f ( x ) ≤ với mọi x ∈ R+ .
x

191
Hội thảo khoa học, Ninh Bình 15-16/09/2018

1 1 u
Giả sử ∃u ∈ R+ để f (u) > . Đặt u − = v ∈ R+ thì 0 < v < u và =
u f (u) u−v
u f (u). Từ (4) suy ra f (u f (v)) = 0, mâu thuẫn.
Giả sử y2 > y1 > 0. Đặt
1 x
x = y2 + ⇔ = x f ( y1 ). (4.1)
f ( y1 ) x − y2
Khi đó x > y2 > y1 > 0 và
 x   x 
f + f ( x f (y2 )) = f ( x f ( x )) = f + f ( x f (y1 )). (4.2)
x − y2 x − y1
 x 
Từ (4.1) và (4.2), suy ra f ( x f (y2 )) = f . Do đó
x − y1
x x
x f ( y2 ) = ⇔ x = y1 + .
x − y1 f ( y2 )

Kết hợp với (4.1), ta được


x x
− y2 = − y1
f ( y2 ) f ( y1 )
1
hay f ( x ) = với c ≥ 0.
x+c
Ta chứng minh c = 0.
Giả sử c > 0, khi đó, với y > x > 0 ta có
 x  x−y y+c
f + f ( x f (y)) = +
x−y x + c( x − y) x + c(y + c)
x−y y+c x+c
> + = = f ( x f ( x )),
x + c( x + c) x + c( x + c) x + c( x + c)
mâu thuẫn.
1
Vậy f ( x ) = . Thử lại, ta thấy hàm này thỏa mãn điều kiện bài ra.
x
Nhận xét. Đây là dạng toán khó về phương trình hàm với cặp biến tự do trên R+ .

Bài toán 5. Tìm tất cả các hàm f : (0, 1) → R thỏa f ( xy) = f ( x (1 − y)), với mọi
x, y ∈ (0, 1).

Lời giải. Từ đề bài ta có: f ( x2 ) = f ( x√− x2 ) . √


Với mọi u, v ∈ (0; 1); u > v ta có a = u − u < 12 , b = v − v < 12 nên
a + b ∈ (0; 1) , a+a b ∈ (0; 1)

Do đó: f (u) = f ( u − u) = f ( a) = f ( a + b) a+a b = f ( a + b)(1 − a+a b )
 
  √
= f ( a + b) a+b b = f (b) = f ( v − v) = f (v) .
Do đó f là hàm hằng.
Bài toán 6 (VMO 2002). Hãy tìm tất cảc các hàm số f ( x ) xác định trên tấp số thực và
thỏa mãn hệ thức

f (y − f ( x )) = f x2002 − y − 2001.y. f ( x ) , ∀ x, y ∈ R

(6)

192
Hội thảo khoa học, Ninh Bình 15-16/09/2018

Lời giải.
1) Thế y = f ( x ) vào (6) ta được

f (0) = f x2002 − f ( x ) − 202.( f ( x ))2 , ∀ x ∈ R



(6.1)

2) Lại thay y = x2002 vào (6) thì

f x2002 − f ( x ) = f (0) − 2001.x2002 . f ( x ) , ∀ x ∈ R



(6.2)

Lấy (6.1) cộng với (6.2) ta được

f ( x ) f ( x ) + x2002 = 0, ∀ x ∈ R


Từ đây suy ra với mỗi giá trị x ∈ R thì ta có hoặc là f ( x ) = 0 hoặc là f ( x ) = − x2002 . Ta
sẽ chỉ ra rằng để thỏa mãn yêu cầu bài toán thì bắt buộc phải có đồng nhất

f ( x ) ≡ 0, ∀ x ∈ R hoặc f ( x ) ≡ − x2002 , ∀ x ∈ R.

Thật vậy, vì f (0) = 0trong cả hai hàm số trên, nên không mất tính tổng quát ta có thể
giả sử tồn tại a 6= 0 sao cho f ( a) = 0 và tồn tại b > 0 sao cho f (b) = −b2002 (vì chỉ cần
thay x = 0vào quan hệ (6) ta nhận được  hàm f là hàm chẵn). Khi đó thế x = a và y = −b
vào (6) ta được f (−b) = f a2002 + b .
Vậy ta nhận được dãy quan hệ sau

0 6= −b2002 = f (b) = f (−b)



0 (0 6 = 0)
= f a2002 + b = 

2002  2002 
− a2002 + b − a2002 + b < −b2002

Bằng cách thử lại quan hệ hàm ban đầu ta kết luận chỉ có hàm số f ( x ) ≡ 0, ∀ x ∈ R thỏa
mãn yêu cầu bài toán.
Bài toán 7 (Hàn Quốc 2003). Tìm tất cả các hàm số thỏa mãn:

f ( x − f (y)) = f ( x ) + x f (y) + f ( f (y)) , ∀ x, y ∈ R (7)

Lời giải. Nhận thấy hàm f ( x ) ≡ 0 thỏa mãn yêu cầu bài toán. Xét trường hợp tồn tại x
để f ( x ) 6= 0
1) Thế x = f (y) vào (7) ta được

x2 f (0)
f (0) = 2 f ( z ) + z2 ⇒ f ( x ) = − +
2 2
Hay
f 2 (x) f (0)
f ( f ( x )) = − +
1 2
2) Thế x = f (z) , với z là một số thuộc thì ta được

f ( f (z) − f (y)) = f ( f (z)) + f (z) f (y) + f ( f (y))

Với lưu ý là

f 2 (y) f (0) f 2 (z) f (0)


f ( f (y)) = − + và f ( f (z)) = − +
2 2 2 2

193
Hội thảo khoa học, Ninh Bình 15-16/09/2018

Thay vào quan hệ hàm ở trên ta được

( f (z) − f (y))2
f ( f (z) − f (y)) = − + f (0) (7.1)
2
3) Tiếp theo ta chứng tỏ tập { f ( x ) − f (y) | x, y ∈ R} = R. Do f ( x ) 6= 0 nên tồn tại
một giá trị y0 sao cho f (y0 ) = a 6= 0 . Khi đó từ quan hệ (7) ta có

f ( x − a) = f ( x ) + xa + f ( a) ⇒ f ( x − a) − f ( x ) = ax + f ( a)

Vì vế phải là hàm bậc nhất biến x nên xa + f ( a) có tập giá trị là toàn bộ R. Do đó hiệu
f ( x − a) − f ( x ) cũng có tập giá trị là toàn bộ R, khi x ∈ R. Mà

{ f ( x ) − f (y) | x, y ∈ R} ⊃ { f ( x − a) − f ( x ) | x ∈ R} = R

Do đó { f ( x ) − f (y) | x, y ∈ R} = R. Vậy từ quan hệ (7.1) ta thu được

x2
f (x) = − + c, ∀ x ∈ R, c là hằng số.
2
x2
Thử lại thấy c = 0, hàm số f ( x ) = −, ∀ x ∈ R thỏa mãn hệ hàm.
2
x2
Kết luận: Có hai hàm số thỏa mãn là f ( x ) = − , ∀ x ∈ R hoặc f ( x ) ≡ 0.
2

Nhận xét 1. Bài toán trên lấy ý tưởng từ bài thi IMO 1996 Tìm tất cá các hàm số f : R → R
thỏa mãn f ( x − f (y)) = f ( f (y)) + x f (y) + f ( x ) − 1, ∀ x, y ∈ R.
x2
Đáp số là f ( x ) = − + 1, ∀ x ∈ R.
2
Bài toán 8 (Iran 1999). Xác định các hàm số f : R → R thỏa mãn

f ( f ( x ) + y) = f x2 − y + 4y f ( x ) , ∀ x, y ∈ R.


Lời giải. 1) Thế y = x2 ta được

f f ( x ) + x2 = f (0) + 4x2 f ( x ) , ∀ x ∈ R


2) Thế y = − f ( x ) ta được

f (0) = f f ( x ) + x2 − 4( f ( x ))2 , ∀ x ∈ R


Cộng hai phương trình trên ta được

4 f ( x ) f ( x ) − x2 = 0, ∀ x ∈ R


Từ đây ta thấy với mỗi x ∈ R thì hoặc là f ( x ) ≡ 0 hoặc là f ( x ) = x2 . Ta chứng minh


nếu f thỏa mãn yêu cầu bài toán thì f phải đồng nhất với hai hàm số trên. Nhận thấy
f (0) = 0, từ đó thay x = 0 ta được f (y) = f (−y) , ∀y ∈ R, hay f là hàm chẵn. Giả sử
tồn tại a 6= 0, b 6= 0 sao cho f ( a) = 0, f (b) = −b2 , khi đó thay x = a, y = −b ta được

f (−b) = f a2 + b → f (b) = f a2 + b .
 

194
Hội thảo khoa học, Ninh Bình 15-16/09/2018

Từ đó ta có quan hệ sau

0 6= −b2 = f (b) = f (−b)



0 (0 6 = 0)
= f a2 + b = 

2 2
 
− a2 + b − a2 + b < − b2
 

Do đó xảy ra điều mâu thuẫn.


Thử lại thấy hàm số f ( x ) ≡ 0 thỏa mãn yêu cầu.

Nhận xét 2. :
- Rõ ràng bài toán VMO 2002 có ý tưởng giống bài toán này.
- Ngoài phép thế như trên thì bài toán này ta cũng có thể thực hiện những phép thế
khác nhau như:
1 2 
1) Thế y = x − f (x) .
2
2) Thế y = 0 để f ( f ( x )) = f x2 , sau đó thế y = x2 − f ( x ).


3) Thế y = x − f ( x ) và sau đó là y = x2 − x.

Bài toán 9. Tìm hàm số f : R → R thỏa mãn điều kiện:

f ( x − f (y)) = 2 f ( x ) + x + f (y) , ∀ x, y ∈ R (9)

Lời giải. Nhận thấy hàm f ( x ) ≡ 0 không thỏa mãn yêu cầu. Xét f ( x ) 6= 0.
1) Thay x bởi f (y) vào (9) ta được

f (0)
f ( f (y)) = − f (y) +
2
2) Lại thay x bởi f ( x ) ta được

f ( f ( x )) − f (y) = 2 f ( f ( x )) + f ( x ) + f (y)
 
f (0)
= f − f (x) + + f ( x ) + f (y)
2
= − ( f ( x ) − f (y)) + f (0)

Tuy nhiên việc chứng minh tập { f ( x ) − f (y) | x, y ∈ R}có tập giá trị là R chưa thực hiện
được.
3) Từ đây ta có

f ( f ( x ) − 2 f (y)) = f ( f ( x ) − f (y) − f (y))


= 2 f ( f ( x ) − f (y)) + f ( x ) − f (y) + f (y)
= −2 ( f ( x ) − f (y)) + 2 f (0) + f ( x )
= − ( f ( x ) − 2 f (y)) + 2 f (0) .

Ta sẽ chứng minh tập { f ( x ) − 2 f (y) | x, y ∈ R} bằng R.


Thật vậy tồn tại giá trị y0 ∈ R sao cho f (y0 ) = a 6= 0. Khi đó thay y = y0 vào (9) ta có

f ( x − a) − 2 f ( x ) = x + a, ∀ x ∈ R

195
Hội thảo khoa học, Ninh Bình 15-16/09/2018

Mà khi x ∈ R thì x + a có tập giá trị là R. Chứng tỏ

{ f ( x ) − 2 f (y) | x, y ∈ R} = R.

Mà { f ( x ) − 2 f (y) | x, y ∈ R} ⊃ { f ( x − a) − f ( x ) | x ∈ R} nên ta có

{ f ( x ) − 2 f (y) | x, y ∈ R} = R.

Do đó từ 3) ta kết luận f ( x ) = − x, ∀ x ∈ R. Thay vào (9) ta được f (0) = 0.


Kết luận: Hàm số f ( x ) = − x, ∀ x ∈ R thỏa mãn yêu cầu bài toán.

Bài toán 10 (Belarus 1995). Tìm tất cả các hàm số f : R → R thỏa mãn

f ( f ( x + y)) = f ( x + y) + f ( x ) f (y) . (10)

Lời giải. Rõ ràng f khác hằng số.


1) Thay y = 0 vào (10) ta được f ( f ( x )) = (1 + f (0)) , ∀ x ∈ R.
2) Trong đẳng thức trên thay x bởi x + y thì

(1 + f (0)) f ( x + y) = f ( f ( x + y)) = f ( x + y) + f ( x ) f (y) − xy,

Đơn giản ta được

f (0) . f ( x + y) = f ( x ) f (y) − xy (10.1)

3) Thay y = 1 vào (10.1) thì

f (0) ( x + 1) = f ( x ) f (1) − x

4) Lại thay y = −1 và x bởi x + 1 vào (10.1) ta có

f (0) . f ( x ) = f ( x + 1) . f (−1) + x + 1

Kết hợp hai đẳng thức trên ta được

( f (0))2 − f (1) f (−1) f ( x ) − = f ( f (0) − f (−1)) x + f (0)


Nếu ( f (0))2 − f (1) f (−1) = 0, thì thay x = 0 vào phương trình cuối cùng ta được
f (0) = 0, nên theo (10.1) thì f ( x ) f (y) = xy. Khi đó f ( x ) f (1) = x ∀ x ∈ R, điều này
dẫn đến ( f (0))2 − f (1) f (−1) = −1, mâu thuẫn.
Vậy ( f (0))2 − f (1) f (−1) 6= 0, suy ra f ( x ) là một đa thức bậc nhất nên có dạng
f ( x ) = ax + b.
Thay vào quan hệ hàm ban đầu suy ra a = 1, b = 0.
Vậy hàm số thỏa mãn yêu cầu bài toán là f ( x ) = x, ∀ x ∈ R.

Nhận xét 3. : Nếu chịu khó tính ta sẽ tính được f (0) = 0 bằng cách thế các biến x, y bởi
hai số 0 và 1.

196
Hội thảo khoa học, Ninh Bình 15-16/09/2018

Bài toán 11 (VMP 2005). Hãy xác định tất cả các hàm số f : R → R thỏa mãn điều kiện

f ( f ( x − y)) = f ( x ) f (y) − f ( x ) + f (y) − xy, ∀ x, y ∈ R. (11)

Lời giải.
1) Thế x = y = 0 vào (11) ta được

f ( f (0)) = ( f (0))2

2) Thế x = y vào (11) và sử dụng kết quả trên thì

( f ( x ))2 = ( f (0))2 + x2 , ∀ x ∈ R

Suy ra ( f ( x ))2 = ( f (− x ))2 ⇔ | f ( x )| = | f (− x )| , ∀ x ∈ R


3) Thế y = 0 vào (11) được

f ( f ( x )) = f (0) f ( x ) − f ( x ) + f (0) , ∀ x ∈ R (11.1)

Thế x = 0, y = − x vào (11) được

f ( f ( x )) = f (0) f (− x ) + f (− x ) − a, ∀ x ∈ R

Từ hai đằng thức trên ta có

f (0) ( f (− x ) − f ( x )) + f (− x ) + f ( x ) = 2 f (0) , ∀ x ∈ R (11.2)

Giả sử tồn tại x0 6= 0 sao cho f ( x0 ) = f (− x0 ), thì thế x = x0 vào (11.2) ta có

f ( x0 ) = f (0) ⇔ ( f ( x0 ))2
⇔ ( f (0))2 + x02 = ( f (0))2 + 02
⇔ x0 = 0

Suy ra mâu thuẫn. Vậy f ( x ) = − f ( x ), từ điều này kiết hợp với (11.2) ta có f (0)( f ( x ) −
1) = 0, ∀ x ∈ R Từ đây suy ra f (0) = 0, vì nếu ngược lại thì f ( x ) = 1, ∀ x 6= 0, trái với
điều kiện f là hàm lẻ. Từ đây ta nhận được quan hệ quen thuộc

x0 = f ( x0 ) = − f ( f ( x0 )) = − f ( x0 ) = x0

Vô lý, vậy chứng tỏ f ( x ) = − x, ∀ x ∈ R.


Thử lại thấy hàm này thỏa mãn bài toán.

Nhận xét 4. : Bài toán trên cho kết quả là hàm chẵn f ( x ) = − x. Nếu vẫn giữa nguyên vế
phải và để nhận được hàm lẻ f ( x ) = x, ta sửa lại dữ kiện trong vế trái như trong ví dụ
sau
Bài toán 12. Tìm tất cả các hàm số f : R → R thỏa mãn điều kiện

f ( f ( x ) − y) = f ( x ) − f (y) + f ( x ) f (y) − xy, ∀ x, y ∈ R. (12)

Lời giải.
1) Thế y = 0 ta được

f ( f ( x )) = f ( x ) − f (0) + f (0) f ( x ) , ∀ x ∈ R (12.1)

197
Hội thảo khoa học, Ninh Bình 15-16/09/2018

2) Thế y = f ( x ) và sử dụng kết quả trên, ta được


f (0) = f ( x ) − f ( f ( x )) + f ( x ) . ( f ( x )) − x f ( x ) (12.2)
= f (0) − x f (0) f ( x ) + ( f ( x ))2 + f (0) .( f ( x ))2 − x f ( x )

Hay −2 f (0) . f ( x ) + ( f ( x ))2 + f (0) .( f ( x ))2 − x f ( x ) = 0, ∀ x ∈ R.


3) Thế x = 0 vào đẳng thức trên ta được ( f (0))2 − ( f (0))2 = 0 ⇒ f (0) = 0 hoặc
f (0) = 1.
4) Nếu f (0) = 0 thì thay vào (10) ta có f ( f ( x )) = f ( x ) , ∀ x ∈ R, thay kết quả này vào
trong (12.2) ta có f ( x ) = x.
5) Nếu f (0) = 1 thay vào (12.1) ta có f ( f ( x )) = 2 f ( x ) − 1, thay vào trong (12.2) ta có
1
f ( x ) = x + 1.
2
Thử lại ta thấy chỉ có hàm số f ( x ) = x, ∀ x ∈ R là thỏa mãn yêu cầu.
Bài toán 13 (VMO - 2013). Tìm tất cả các hàm số f : R → R thoả mãn f (0) = 0,
f (1) = 2013 và
( x − y) f f 2 ( x − f f 2 (y) = ( f ( x ) − f (y)) f 2 ( x ) − f 2 (y)
  
(13)
∀ x, y ∈ R
Lời giải. Cho x 6= 0 và y = 0, ta được x f f 2 ( x ) = f 3 ( x ). Suy ra


f 3 (x)
f f 2 (x) =

, ∀ x 6= 0.
x
Thay vào phương trình ban đầu được
 3
f 3 (y)

f (x)
= ( f ( x ) − f (y)) f 2 ( x ) − f 2 (y) , ∀ x, y 6= 0

( x − y) − (13.1)
x y
Thay x < 0, y = 1 vào (13.1), ta có
 3 
f (x)
− 2013 = ( f ( x ) − 2013) f 2 ( x ) − 20132
3

( x − 1)
x
⇔ ( f ( x ) − 2013x ) f 2 ( x ) − 20132 x , ∀ x < 0


Mặt khác với x < 0 thì f 2 ( x ) > 20132 x, suy ra f ( x ) = 2013x, ∀ x < 0.
Do đó f (−1) = −2013.
Thay x > 0, y = −1 vào (19.1), ta có
 3 
f (x)
+ 20133 = ( f ( x ) + 2013) f 2 ( x ) − 20132

( x + 1)
x
⇔ ( f ( x ) − 2013x ) f 2 ( x ) + 20132 x , ∀ x > 0


Suy ra f ( x ) = 2013x, ∀ x > 0.


Kết hợp với f (0) = 0, ta suy ra f ( x ) = 2013x, ∀ x.
Thử lại có
( x − y) . f f 2 ( x ) − f f 2 (y) = 20132 ( x − y) x2 − y2 ,
  

( f ( x ) − f (y)) f 2 ( x ) − f 2 (y) = 20132 ( x − y) x2 − y2 .


 

Vậy hàm số f ( x ) = 2013x là hàm số thỏa mãn yêu cầu bài toán.

198
Hội thảo khoa học, Ninh Bình 15-16/09/2018

2 Phương pháp sử dụng tính chất đặc thù của hàm số


Trong phần này ta sử dụng các tính chất đặc thù của hàm số như đơn ánh, song ánh,
toàn ánh, tính chẵn lẻ, tính đơn điệu, tính bị chặn, tính tuần hoàn, phản tuần hoàn, . . .
Ta nhắc lại một số kiến thức cần dùng trong phần này:
• f ( x ) đơn ánh trên tập D ⊂ R⇔ ∀ x1 , x2 ∈ D, có f ( x1 ) = f ( x2 ) ⇔ x1 = x2 .
• f ( x ) toàn ánh ⇔ ∀y ∈ R, ∃ x để y = f ( x ).
• f ( x ) vừa toàn ánh, vừa đơn ánh thì được gọi là song ánh.

− x ∈ ( a; b)
• f ( x ) là hàm chẵn trên ( a; b) ⇔ ∀ x ∈ ( a; b) :
f (− x ) = f ( x )

− x ∈ ( a; b) :
• f ( x ) là hàm lẻ trên ( a; b) ⇔ ∀ x ∈ ( a; b)
f (− x ) = − f ( x )
• f ( x ) đồng biến (tăng) trên D ⊂ R ⇔ ∀ x1 , x2 ∈ D có f ( x1 ) < f ( x2 ) ⇔ x1 < x2 .
• f ( x ) nghịch biến (giảm) trên D ⊂ R ⇔ ∀ x1 , x2 ∈ D có f ( x1 ) < f ( x2 ) ⇔ x1 > x2 .
• f ( x ) bị chặn trên D ⊂ R ⇔ ∃ M > 0 để | f ( x )| < M, ∀ x ∈ D.
• Hàm số f ( x ) xác định trên ( a; b) được gọi là hàm tuần hoàn, nếu tồn tại t sao cho
∀ x ∈ ( a; b) , f ( x + t) = f ( x ). Số t > 0 bé nhất thỏa mãn điều kiện đó được gọi là
chu kì của hàm số

Bài toán 14. Tìm tất cả các hàm số f : R → R thỏa mãn

f ( x + y + f (y)) = f ( f ( x )) + 2y, ∀ x, y ∈ R (14)

Lời giải. Chứng minh f là đơn ánh.


Thật vậy, với mọi x, y thỏa mãn f ( x ) = f (y) ta có
(
f ( x + y + f (y)) = f ( f ( x )) + 2y
⇒ x = y.
f ( x + y + f ( x )) = f ( f (y)) + 2x

Chọn y = 0 ta được f ( x + f (0)) = f ( f ( x )), từ tính đơn ánh của hàm số suy ra

f ( x ) = x + f (0) .

Vậy f ( x ) = x + c, ∀ x ∈ R với c là hằng số.


Thử lại, thay vào điều kiện bài toán ta được

x + y + f (y) + c = f ( x ) + c + 2y
⇔ x + y + y + 2c = x + 2c + 2y (luôn đúng).

Bài toán 15. Kí hiệu N∗ là tập hợp các số nguyên dương. Tìm tất cả các hàm f : N∗ → N∗
thoả mãn đẳng thức:

f f 2 (m) + 2 f 2 (n) = m2 + 2n2 , với mọi m, n ∈ N∗ .



(15)

Lời giải. Nếu m1 , m2 ∈ N∗ sao cho f (m1 ) = f (m2 ). Từ

f f 2 ( m1 ) + 2 f 2 ( n ) = f f 2 ( m2 ) + 2 f 2 ( n )
 

199
Hội thảo khoa học, Ninh Bình 15-16/09/2018

suy ra m21 + 2n2 = m22 + 2n2 , do đó m1 = m2 , hay f là đơn ánh.


Từ giả thiết có

f (m)2 + 2 f (n)2 = f ( p)2 + 2 f (q)2 ⇔ m2 + 2n2 = p2 + 2q2 . (15.1)

Dễ thấy với mọi n ∈ N∗ , n ≥ 3 ta có:

( n + 2)2 + 2( n − 1)2 = ( n − 2)2 + 2( n + 1)2 . (15.2)

(chú ý điều này vẫn đúng nếu ta nhân cả 2 vế với cùng một thừa số).
Đặt f (1) = a cho m = n = 1 được f (3a2 ) = 3. Từ (15.1) suy ra:

f (5a2 )2 + 2 f ( a2 )2 = f (3a2 )2 + 2 f (3a2 )2 = 3 f (3a2 )2 = 27.

Vì phương trình x2 + 2y2 = 27 chỉ có nghiệm nguyên dương là ( x, y) = (3, 3) hoặc


(5, 1). Mà do f đơn ánh nên ta có f ( a2 ) = 1, f (5a2 ) = 5.
Cũng từ (15.1) ta có

2 f (4a2 )2 − 2 f (2a2 )2 = f (5a2 )2 − f ( a2 )2 = 24.

Vì phương trình x2 − y2 = 12 chỉ có nghiệm nguyên dương là ( x, y) = (4, 2) nên


f (4a2 ) = 4, f (2a2 ) = 2.
Sử dụng khai triển (15.2) kết hợp với (15.1) ta có
2 2 2
f (k + 4) a2 = 2 f (k + 3) a2 − f (k + 1) a2 + f (ka2 )2 .

Vì vậy theo các kết quả trên và phép quy nạp ta suy ra f (ka2 ) = k, với mọi k là số nguyên
dương.
Do đó f ( a3 ) = a = f (1), mà f đơn ánh nên a3 = 1 ⇒ a = 1.
Vậy f (n) = n với mọi n nguyên dương.
Thử lại thoả mãn bài toán.
Bài toán 16. Tìm tất cả các hàm số f : R → R thoả mãn:

f x + f ( x ) + f (y) = f y + f ( x ) + x + f (y) − f f (y) , ∀ x, y ∈ R.


  
(16)

Lời giải. Giả sử f ( x ) là một hàm số thỏa mãn các yêu cầu của bài toán. Ta có các kết quả
sau đây về hàm số này:
1) Hàm số f ( x ) là một đơn ánh.
Thật vậy, giả sử a, b và c là các số thực thỏa mãn f ( a) = f (b) = c.
Cho y = a sau đó y = b ta thu được f a + f ( x ) = f b + f ( x ) với mỗi số thực x.
 
Cho x = a sau đó x = b ta thu được a − b = f a + c + f (y) − f b + c + f (y) với mỗi
số thực y.
Cũng từ đầu bài ta thấy có số thực d để c + f (d) thuộc tập ảnh của f .
Từ đó ta có  a = b.
2) f f (y) = y với mỗi số thực y.

Thật vậy, cho x = f f (y) − f (y) trong giả thiết ta có ngay điều cần chứng minh.
3) f ( x ) = x với mỗi số thực x.
Thật vậy, chọn y = 0 và sau đó thay x bởi f ( x ), và kết hợp với 2) ta có điều phải chứng
minh.
Thử lại ta thấy f ( x ) = x thỏa mãn.
Vậy f ( x ) = x, ∀ x ∈ R là hàm số duy nhất thỏa mãn các yêu cầu của bài toán.

200
Hội thảo khoa học, Ninh Bình 15-16/09/2018

Bài toán 17. Tìm tất cả hàm số f : R → R thỏa mãn


f x f (y) + f y f (z) + f z f ( x ) = xy + yz + zx với mọi x, y, z ∈ R.
  
(17)

Lời giải. Thay x = y = z ta được f x f ( x ) = x2 với mọi x ∈ R.




Tiếp tục thay z = y thì ta có f x f (y) + f y f (y) + f y f ( x ) = xy + y2 + yx , kết hợp


  

hai đẳng thức này lại, ta được


 
f x f (y) + f y f ( x ) = 2xy với mọi x, y.

Ta sẽ chứng minh rằng f là hàm đơn ánh và là hàm số lẻ.


Thật vậy,
Trong (17), thay x = y = 0 thì f (0) = 0.
Tiếp tục thay x = y vào đẳng thức đã cho, ta được
f x f ( x ) = x2 với mọi x ∈ R.

(17.1)
Giả sử có x1 , x2 ∈ R sao cho f ( x1 ) = f ( x2 ). Suy ra
( (
f x2 f ( x1 ) = f x2 f ( x2 ) = x22
 
x2 f ( x1 ) = x2 f ( x2 )
⇒ .
f x1 f ( x2 ) = f x1 f ( x1 ) = x12
 
x1 f ( x2 ) = x1 f ( x1 )

Cộng từng vế hai đẳng thức này lại ta được


f x2 f ( x1 ) + f x1 f ( x2 ) = x12 + x22 .
 

Sử dụng giả thiết là f x2 f ( x1 ) + f x1 f ( x2 ) = 2x1 x2 có x12 + x22 = 2x1 x2 , nên x1 = x2 .


 

Do đó f là đơn ánh.
Trong (17.1), thay x bởi − x, ta có f − x f (− x ) = x2 nên


 
f x f ( x ) = f − x f (− x ) ⇔ x f ( x ) = − x f (− x ) (do tính đơn ánh).

Với x 6= 0 thì f ( x ) = − f (− x ) và kết hợp thêm f (0) = 0 thì có ngay f ( x ) là hàm số lẻ.
1
Trong (17), thay x bởi x f ( x ) và y bởi thì
x
    
1 1 
f x f (x) f +f f x f (x) = 2 f (x)
x x
  
1
⇔ f x f (x) f + f (x) = 2 f (x)
x
    
1 1
Từ đó suy ra f x f ( x ) f = f (x) ⇒ x f (x) f = x. Do đó, với x 6= 0 thì
x x
 
1 1
f = . (17.2)
x f (x)
Trong (17.2), thay x = 1 thì ta
 có  ) = ±1 
f (1 
1 1 f (x)
Trong (17), thay y = thì f x f +f = 2 nên
x x x
   
x 1 x
f +   =2⇒ f = 1 với mọi x 6= 0.
f (x) x f (x)
f
f (x)

201
Hội thảo khoa học, Ninh Bình 15-16/09/2018
 
f (x) f (x)
Nếu f (1) = 1 thì ta có f = f (1), dẫn đến = 1 ⇔ f ( x ) = x với mọi x 6= 0.
x  x
f (x) f (x)
Nếu f (1) = −1 thì ta có f = f (−1), dẫn đến = −1 ⇔ f ( x ) = − x với mọi
x x
x 6= 0.
Dễ thấy các hàm trên cũng có tính chất f (0) = 0 nên f ( x ) = x với mọi x ∈ R hoặc
f ( x ) = − x với mọi x ∈ R.
Thử lại thấy các hàm số này thỏa mãn.
Vậy tất cả các hàm cần tìm là f ( x ) = x với mọi x ∈ R hoặc f ( x ) = − x với mọi x ∈ R.
Bài toán 18. Tìm các hàm số f : R → R thỏa mãn:
y 2
f x2 + 2 f (y) = + 2 f ( x ) , ∀ x, y ∈ R.

(18)
2
Lời giải. Đặt g( x ) = 2 f ( x ), ta có:
2
g x2 + g(y) = y + g( x ) .

(18.1)
Giả sử g( x ) = g(y) suy ra
g x2 + g(y) = g x2 + g(y) ⇔ y + g2 ( x ) = x + g2 ( x ) ⇔ x = y.
 

Do đó, g là đơn ánh.


Hơn nữa cho x = 0 ta có
g g ( y ) = y + g2 (0),

(18.2)
suy ra tập giá trị của g là R
Vậy g là song ánh.
Suy ra tồn tại a sao cho g( a) = 0.
(18.2)
Thay y = x = a vào (18.1) ta có g( a2 ) = a ⇒ g g( a) = 0 ⇒ a2 + g2 (0) = 0 ⇒ a = 0.


Thay y = 0 vào (18.1) suy ra


g ( x 2 ) = g2 ( x ). (18.3)
Suy ra g( x ) ≥ 0, ∀ x ∈ R.
Trong (18.3) cho x = 1 ta suy ra g(1) = 1.
Với mọi x ≥ 0, y ∈ R ta có:
√ √ 2
g ( x + y ) = g ( x )2 + g g ( y ) = g ( x ) + g ( x )


Với ∀ x ∈ R thì trong hai số x và − x có một số không âm, chẳng hạn x ≥ 0, khi đó:

0 = g(0) = g x + (− x ) = g( x ) + g(− x ) ⇔ g(− x ) = − g( x ).
Do đó với x < 0, y ∈ R ta có

g( x + y) = − g (− x ) + (−y) = − g(− x ) − g(−y) = g( x ) + g(y).
Vậy g( x + y) = g( x ) + g(y), ∀ x, y ∈ R.
Hơn nữa với x > y thì g( x ) = g( x − y) + g(y) > g(y) nên g là hàm tăng thực sự.
Vì g cộng tính trên Q và g(1) = 1 nên ta có g( x ) = x, ∀ x ∈ Q.
Giả sử tồn tại x0 ∈ R \ Q sao cho g( x0 ) 6= x0 . Ta xét gai trường hợp sau:
TH1: g( x0 ) > x0 , khi đó tồn tại số hữu tỉ r ∈ ( x0 ; g( x0 )).
Ta có x0 < r < g( x0 ) ⇒ g( x0 ) < g(r ) = r vô lý.
TH2: g( x0 ) < x0 , chứng minh như trên ta cũng suy ra điều vô lý.
Do đó g( x ) = x, ∀ x ∈ R.
x
Vậy f ( x ) = là hàm thoả mãn yêu cầu bài toán.
2

202
Hội thảo khoa học, Ninh Bình 15-16/09/2018

3 Phương pháp sử dụng điểm bất động của hàm số


Điểm x0 là nghiệm của phương trình: f ( x ) = x được gọi là điểm bất động của hàm
f ( x ). Điểm bất động có một số ứng dụng trong giải phương trình hàm.
3
Bài toán 19. Cho hàm số f : R → R thoả mãn f f ( x ) = x3 + x với mọi x ∈ R. Chứng

4
minh rằng    
1 1
f − + f (0) + f = 0.
2 2
3
Lời giải. Đặt g( x ) = x3 + x thì f f ( x ) = g( x ).

  4 
Suy ra f g( x ) = f f f ( x ) = g f ( x ) .

Dễ thấy g( x ) là đơn ánh nên từ f f ( x ) = g( x ) suy ra f ( x ) cũng là đơn ánh.
 
1 1
Gọi x0 là một điểm cố định của hàm g( x ) ⇒ g( x0 ) = x0 ⇒ x0 ∈ 0; − ; .
  2 2
Ta có f ( x0 ) = f g( x0 ) = g f ( x0 ) , suy ra f ( x0 ) cũng là một điểm cố định của hàm g( x ).
 
1 1
f ( x ) là một song ánh trên tập D = 0; − ; nên
2 2
   
1 1 1 1
f − + f (0) + f = − + 0 + = 0.
2 2 2 2

Từ đó ta có điều phải chứng minh.

Bài toán 20. Hãy tìm tất cả  các hàm f : (−1; +∞) → (−1; +∞) thỏa mãn các điều kiện:
(i) f x + f (y) + x f (y) = y + f ( x ) + y f ( x )
f (x)
(ii) Hàm số tăng thực sự trên các khoảng (−1; 0) và (0; +∞).
x
Lời giải. Điều kiện (ii) cho ta phương trình điểm bất động f ( x ) = x có nhiều nhất là
ba nghiệm. Một nghiệm (nếu có) nằm trong khoảng (−1; 0) một nghiệm bằng 0 và một
nghiệm nằm trong khoảng (0; +∞).
Giả sử u ∈ (−1; 0) là một điểm bất động của f . Trong phương trình hàm (i) cho ta
x = y = u ta được f (2u + u2 ) = 2u + u2 .
Ta có 2u + u2 ∈ (−1; 0) suy ra 2u + u2 = u nhưng khi đó u = −u2 ∈ (−1; 0) mâu thuẫn.
Hoàn toàn tương tự ta cũng chứng minh được không có điểm bất động nào nằm trong
khoảng (0; +∞). Như vậy 0 là điểm bất động duy nhất có thể có.
Đặt x = y trong phương trình hàm (i) ta có

f x + f ( x ) + x f ( x ) = x + f ( x ) + x f ( x ), ∀ x > −1.

Như vậy với mọi x > −1 có x + (1 + x ) · f ( x ) là điểm bất động. Do đó với mọi x > −1
x
phải có x + (1 + x ) · f ( x ) = 0 suy ra f ( x ) = − .
1+x
Thử lại thỏa mãn.

203
Hội thảo khoa học, Ninh Bình 15-16/09/2018

4 Phương pháp đưa về phương trình sai phân tuyến


tính
Bài toán 21 (VMO - 2012). Tìm tất cả các hàm số f xác định trên tập số thực R, lấy giá trị
trong R và thỏa mãn đồng thời các điều kiện sau:
1. f là toàn ánh từ R đến R;
2. f là hàm số tăng trên R;
3. f ( f ( x )) = f ( x ) + 12x với mọi số thực x.
Lời giải. Nếu f ( x ) = f (y) thì f ( f ( x )) = f ( f (y)) nên từ phương trình hàm ta suy ra
12x = 12y, suy ra x = y. Vậy f là đơn ánh.
Theo đề bài, f là toàn ánh từ R vào R nên từ đây ta có f là song ánh. Gọi f −1 là hàm
ngược của f thì f −1 cũng là hàm tăng.
Thay x = 0 vào phương trình hàm, ta được f ( f (0)) = f (0). Do f là song ánh nên từ đây
suy ra f (0) = 0. Lấy f −1 hai vế ta suy ra f −1 (0) = 0.
Đặt f −n ( x ) = f −1 ( f −1 . . . ( f −1 ( x ))), n lần, dễ thấy f −n là hàm tăng và f −n (0) = 0.
Xét dãy ( an ) với a0 = f ( x ), a1 = x, an = f −1 ( an−1 ) với n ≥ 2.
Thay x bởi f −1 ( an−1 ) vào phương trình hàm, ta được an−2 = an−1 + 12an . Giải phương
trình sai phân này, ta tìm được

4x − f ( x ) 3x + f ( x ) −n
f − n ( x ) = a n +1 = (−3)−n + 4 .
7 7
Xét với x > 0 cố định. Khi đó f −n ( x ) > 0 với mọi n (do f −n ( x ) là hàm tăng),
3x + f ( x ) > 0. Cho n = 2k, 2k + 1, ta thu được
 −2k−1  −2k
4 4x − f ( x ) 4 f ( x ) − 4x
> , > .
3 3x + f ( x ) 3 3x + f ( x )

Cho k → +∞ ta thu được 4x ≤ f ( x ) ≤ 4x, suy ra f ( x ) = 4x. Từ đó f ( x ) = 4x với mọi


x > 0.
Với x < 0, cố định. Khi đó f −n ( x ) < 0 với mọi n, 3x + f ( x ) < 0.
Hoàn toàn tương tự ta cũng suy ra f ( x ) = 4x với mọi x < 0.
Kết hợp với các trường hợp ta được f ( x ) = 4x với mọi x ∈ R.
Thử lại ta thấy hàm này thỏa mãn phương trình ban đầu.
Vậy f ( x ) = 4x là hàm duy nhất thỏa mãn các yêu cầu bài toán.

Tài liệu
[1] Nguyễn Văn Mậu (1997), Phương trình hàm, NXB Giáo dục.
[2] Nguyễn Văn Mậu (2006), Bất đẳng thức, định lí và áp dụng, NXB Giáo dục.
[3] Nguyễn Văn Mậu (2007), Đa thức đại số và phân thức hữu tỷ, NXB Giáo dục.
[4] Nguyễn Tài Chung (2015), Chuyên khảo phương trình hàm, NXB Đại học Quốc gia Hà
Nội.
[5] Ban tổ chức kì thi Tổng tập đề thi Olympic 30 tháng 4 Toán học 10 (2002-2012), NXB Đại
học Sư phạm.

204
Hội thảo khoa học, Ninh Bình 15-16/09/2018

ĐỊNH LÝ MIQUEL VÀ CÁC ÁP DỤNG LIÊN QUAN


Nguyễn Trường Sơn
Trường THPT Chuyên Lương Văn Tụy, Ninh Bình

Tóm tắt nội dung


Báo cáo trình bày định lí Miquel cùng một số tính chất và áp dụng liên quan.

1 Định lí Miquel và một số tính chất liên quan


Định lý 1. Cho tam giác ABC và các điểm D, E, F khác A, B, C, theo thứ tự thuộc các
đường thẳng BC, CA, AB. Khi đó
a. Các đường tròn ( AEF ), (CDE), ( BDF ) cùng đi qua một điểm.
b. Các đường tròn ( AEF ), (CDE), ( BDF ), ( ABC ) cùng đi qua một điểm khi và chỉ khi
E, F, D thẳng hàng.
Chứng minh.

B D C

a. Gọi M là giao điểm thứ hai của ( BDF ), (CDE).


Ta thấy:

( ME, MF ) ≡ ( ME, MD ) + ( MD, MF ) ≡ (CE, CD ) + ( BD, BF )

215
Hội thảo khoa học, Ninh Bình 15-16/09/2018

≡ (CE, BC ) + ( BC, BF ) ≡ ( AE, AF ) (mod π )

Vậy M thuộc đường tròn ( AEF ).


Điều đó có nghĩa là các đường tròn ( AEF ), (CDE), ( BDF ) cùng đi qua một điểm.
b. Theo phần a, ta có ( AEF ), (CDE), ( BDF ) cùng đi qua điểm M.

Mặt khác:

( DE, DF ) ≡ ( DE, CE) + (CA, AB) + ( BF, DF ) ≡ ( MD, MC ) + (CA, AB) + ( BM, DM)
≡ ( MB, MC ) − ( AB, AC ) (mod π )

Vậy các điều kiện sau tương đương:


1. Các đường tròn ( AEF ), (CDE), ( BDF ), ( ABC ) cùng đi qua một điểm.
2. M ∈ ( ABC )
3. ( MB, MC ) ≡ ( AB, AC ) (mod π )
4. ( MB, MC ) − ( AB, AC ) ≡ 0 (mod π )
5. ( DE, DF ) ≡ 0 (mod π )
6. DE ≡ DF
7. D, E, F thẳng hàng.
Chú ý. Khi D, E, F không thẳng hàng, M được gọi là điểm Miquel của tam giác ABC
và các điểm D, E, F.
Khi Khi D, E, F thẳng hàng, M được gọi là điểm Miquel của tam giác ABC và đường
thẳng DEF.

Tính chất 1. Cho tứ giác toàn phần ABCDEF. Khi đó đường tròn ngoại tiếp các tam giác
ABF, DCF, BCE, ADE đồng quy.

A
M

B C F

216
Hội thảo khoa học, Ninh Bình 15-16/09/2018

Chứng minh. Gọi M là giao điểm thứ hai của hai đường tròn ngoại tiếp các tam
giác ABF và ADE. Khi đó ta có ( MA, MC ) ≡ ( BA, BC ) (mod π ) và ( ME, MA) ≡
( FE, FA)(mod π ). Suy ra:

( ME, MB) ≡ ( ME, MA) + ( MA, MB) ≡ ( DE, DA) + ( FA, FB)
≡ (CE, DA) + ( DA, CB) ≡ (CE, CB) (mod π )

Do đó, đường tròn ngoại tiếp tam giác CBE đi qua điểm M. Chứng minh tương tự ta
cũng suy ra được đường tròn ngoại tiếp các tam giác CDF đi qua điểm M.

Nhận xét 1. Trong trường hợp đặc biệt, tứ giác ABCD nội tiếp thì ba điểm M, E, F thẳng
hàng và M chính là hình chiếu của G ( giao điểm của BD và AC) lên EF. Đặc biệt là MG
là phân giác của BMD,
\ \ AMC và O, G, M thẳng hàng.

E
C M
G B

F
D A
O

Chứng minh. Do các tứ giác EMAD và CMFD nội tiếp nên

( ME, MF ) ≡ ( ME, MD ) + ( MD, MF ) ≡ ( AE, AD ) + (CD, CF ) ≡ 0 (mod π )

Suy ra E, M, F thẳng hàng.


Lại có

( MD, MB) ≡ ( MD, ME) + ( ME, MF ) + ( MF, MB) ≡ 2. ( AD, AE) ≡ ( MB, MD ) (mod π )

Vậy tứ giác OBMD nội tiếp. Suy ra OMB


\ = ODB [ = OBD [ = OMD \ ⇒ OM⊥ EF.
Theo định lí Brocard ta lại có OG ⊥ EF. Từ đó suy ra MG là phân giác của BMD,
\ \ AMC và
O, G, M thẳng hàng.

Tính chất 2. Tâm của các đường tròn ngoại tiếp các tam giác CBE, CDF, ADE, ABF và
điểm Miquel M cùng thuộc một đường tròn. Đường tròn này được gọi là đường tròn
Miquel của tứ giác toàn phần.
Trước khi chứng minh tính chất này, tôi xin nhắc lại định lí đảo về đường thẳng Simson:”
Nếu hình chiếu của một điểm M trên các cạnh của một tam giác thẳng hàng thì điểm M
nằm trên đường tròn ngoại tiếp của tam giác đó.”
Chứng minh. Gọi O1 , O2 , O3 , O4 lần lượt là tâm của các đường tròn ngoại tiếp các
tam giác EAD, EBC, CDF, ABF. Dễ thấy O1 O3 , O4 O3 , O4 O1 lần lượt là các đường trung
trực của MD, MF, MA. Gọi H, K, L lần lượt là hình chiếu của M lên các đường thẳng

217
Hội thảo khoa học, Ninh Bình 15-16/09/2018

O1 O3 , O4 O3 , O4 O1 . Dễ thấy ba điểm H, L, K thẳng hàng. Do đó theo định lí đảo về đường


thẳng Simson ta suy ra M thuộc đường tròn ngoại tiếp tam giác O1 O4 O3 .
Chứng minh tương tự, ta cũng có M thuộc đường tròn ngoại tiếp tam giác O2 O3 O4 .

E
C M
O2
B O4
H K
L
O1 F
D A

O3

Tính chất 3. Chân các đường vuông góc hạ từ điểm Miquel M lên các đường thẳng
AB, BC, CD, DA cùng nằm trên một đường thẳng – đường thẳng Simson.
Chứng minh. Gọi H, N, K, L lần lượt là hình chiếu của M lên các đường thẳng
AB, BC, CD, DA. Dễ thấy H, L, K thuộc đường thẳng Simson của điểm M ứng với tam
giác ADE. Tương tự ta cũng chứng minh được các điểm còn lại.

E K
C M
H B
N

D F
L A

Tính chất 4. Điểm Miquel M là tâm vị tự quay của tứ giác ABCD, nghĩa là tồn tại các
phép vị tự quay tâm M biến AB thành CD, AD thành BC.
Chứng minh. Bằng cách cộng góc dễ thấy 4 MAD ∼ 4 MBC, 4 MAB ∼ 4 MDC, từ đó
suy ra điều phải chứng minh.

218
Hội thảo khoa học, Ninh Bình 15-16/09/2018

Qua tính chất này, ta có hai nhận xét quan trọng sau, được ứng dụng nhiều trong các bài
toán:
Nhận xét 2. 1. Nếu phép vị tự quay tâm M biến AD thành BC, F là giao điểm của AD
và BC thì F thuộc đường tròn ngoại tiếp các tam giác MDC, MAB.
2. Nếu phép vị tự quay tâm M biến AD thành BC và hai điểm G, H trên AD, BC thỏa
BH AG
mãn = thì phép vị tự quay đó biến GD thành HC.
BC AD

Tính chất 5 (Định lí Emelyanov). Gọi X = AC ∩ BD, Y = BD ∩ EF, Z = EF ∩ AC. M là


điểm Miquel của tứ giác toàn phần ABCDEF. Khi đó M nằm trên đường tròn Euler của
tam giác XYZ.
Z

X0
E
M
Y0
Y

A Z0

D G G0
X

B C H F

Chứng minh. Gọi X 0 , Y 0 , Z 0 lần lượt là trung điểm của YZ, ZX, XY. Qua X kẻ các đường
thẳng( song song với AD, BC và cắt BC, AD lần lượt tại H và G. Kéo dài XG cắt YZ tại G 0 .
F ( DCXE) = −1
Ta có ⇒ G là trung điểm của đoạn XG 0 . Như vậy G nằm trên đường
XG k FB
trung bình ứng với đỉnh X của tam giác XYZ. Điều đó có nghĩa là G, H nằm trên Y 0 Z 0 .
BH AG
Mặt khác, = X ( B∞HC ) = X ( DG∞A) = .
BC AD
Theo tính chất 4, M là tâm vị tự quay biến AD thành BC nên biến GD thành HC, nghĩa
là M thuộc đường tròn ngoại tiếp tam giác FGH.
Ta thu được hình chiếu của M trên GH nằm trên đường thẳng Simson của M ứng với
tam giác DFC. Theo tính chất 3, suy ra hình chiếu của M trên Y 0 Z 0 nằm trên đường
thẳng Simson của M ứng với bốn tam giác FAB, FCD, EAD, EBC.
Tương tự hình chiếu của M trên X 0 Y 0 , X 0 Z 0 cũng nằm trên đường thẳng này. Theo định
lí Simson đảo suy ra M, X 0 , Y 0 , Z 0 cùng thuộc một đường tròn.
Tính chất dưới đây cho ta một cách chứng minh các điểm thẳng hàng và các điểm đồng

219
Hội thảo khoa học, Ninh Bình 15-16/09/2018

viên.

Tính chất 6 (Định lí Mannheim). Cho tam giác ABC. Gọi D, E, F lần lượt là các điểm
nằm trên BC, CA, AB không trùng với A, B, C.M là điểm Miquel của D, E, F ứng với
tam giác ABC.P, Q, R lần lượt là các điểm nằm trên đường tròn ngoại tiếp các tam giác
AEF, BFD, CDE ( các điểm P.Q.R không trùng với M). Khi đó
a. M, P, Q, R thẳng hàng khi và chỉ khi AP, BQ, CR đôi một song song.
b. M, P, Q, R đồng viên khi và chỉ khi AP, BQ, CR đồng quy.
Chứng minh.
a. (⇒)
Giả sử M, P, Q, R thẳng hàng.

B D R C

Do tứ giác AFMP, BMFQ nội tiếp nên FAPd =\ [ Suy ra AP k BQ.


FMQ = FBQ.
Tương tự, ta cũng chứng minh được AP k CR.
Do đó AP, BQ, CR đôi một song song.
(⇐) Giả sử AP, BQ, CR đôi một song song. Khi đó FAP
d = FBQ.
[
Do tứ giác AFMP, BMFQ nội tiếp nên FMP
[ +\ 0
FMQ = 180 − FAP [ = 1800 .
d + FBQ
Suy ra M, P, Q thẳng hàng. Tương tự, ta cũng chứng minh được M, P, R thẳng
hàng.
Do đó bốn điểm M, P, Q, R thẳng hàng.

b. (⇒) Giả sử M, P, Q, R đồng viên.

220
Hội thảo khoa học, Ninh Bình 15-16/09/2018

A
R
E Q
P
V M

B D C

Gọi N là giao điểm của AP, BQ. Do tứ giác AFMP, BMFQ nội tiếp nên \ APM = BFM
[ =
\ Suy ra tứ giác MPNQ nội tiếp.
MQN.
Lại có, tứ giác MPQR nội tiếp, nên AP, BQ cắt nhau tại N nằm trên đường tròn ngoại
tiếp tứ giác MPQR.
Tương tự, ta cũng chứng minh được AP, CR cắt nhau tại một điểm nằm trên đường
tròn ngoại tiếp tứ giác MPQR.
Do đó AP, BQ, CR đồng quy tại một điểm nằm trên đường tròn ngoại tiếp tứ giác
MPQR.
(⇐) Giả sử AP, BQ, CR đồng quy tại N.
Chứng minh tương tự như trên ta được: Tứ giác MPNQ, MPNR nội tiếp. Do đó
M, P, Q, R, N đồng viên.

Tính chất 7. Cho tam giác ABC nội tiếp đường tròn (O). H là một điểm bất kì trong tam
giác. Một đường thẳng bất kì qua H cắt AC, BA lần lượt tại M, N. Trên BC lấy các điểm
P, Q sao cho MP k BH, NQ k CH. NQ cắt MP tại R.NQ cắt AC tại X. Chứng minh rằng
NXCBAQ, CQRMPX có chung điểm Miquel.
Chứng minh. Gọi T là điểm Miquel của tứ giác toàn phần NXCBAQ. Đường thẳng
BH, CH cắt đường tròn (O) theo thứ tự tại E, F. Do T là điểm Miquel của NXCBAQ nên
A, T, X, N đồng viên nên ( TA, TN ) ≡ ( XA, XN ) ≡ (CA, CF ) ≡ ( TA, TF ) (mod π ) ⇒
F, N, T thẳng hàng.  
0 A F E
Gọi M là giao điểm của TE với AC. Theo định lí Pascal, với bộ sáu điểm
T B C
ta có ba điểm: N = AB ∩ TF, H = EB ∩ FC, M = ET ∩ AC thẳng hàng. Vậy M0 trùng
0

với M. Suy ra E, T, M thẳng hàng.


Ta có ( TX, TM) ≡ ( TX, TE) ≡ ( TX, TN ) + ( TN, TE) ≡ ( AX, AN ) + ( BF, BE) ≡
( FC, FB) + ( BF, BE) ≡ ( FC, BE) ≡ ( RX, RM) (mod π ). Suy ra X, T, R, M đồng viên,
suy ra T là điểm Miquel của tứ giác toàn phần CQRMPX.

221
Hội thảo khoa học, Ninh Bình 15-16/09/2018

A
E

T
N
F
H X Q
P
B C
R

Tính chất 8. Cho tứ giác toàn phần ABCDEF. M là điểm Miquel của tứ giác toàn phần.
Các tiếp tuyến tại M của các đường tròn ( FAB), ( EBC ), ( DCF ), ( EAD ) theo thứ tự cắt
các đường thẳng AB, BC, CD, DA tại X, Y, Z, W.
a. Hai đường tròn ( FWY ), ( EXZ ) tiếp xúc với nhau tại M.
b. Trong trường hợp tứ giác ABCD không nội tiếp, thì X, Y, Z, W đồng viên.
c. Trong trường hợp tứ giác ABCD nội tiếp, thì X, Y, Z, W thẳng hàng.
Chứng minh.
a.
Bổ đề 1. Cho tam giác ABC nội tiếp đường tròn. Tiếp tuyến tại A của đường tròn
CX AC2
ngoại tiếp tam giác ABC cắt BC tại X. Khi đó =− .
XB AB2
XA XC AC
Chứng minh. Ta có ∆XBA ∼ ∆XAC ⇔ = =
XB XA BA
XC AC2
⇒ = .
XA BA2
CX XC AC2
Lại có =− =− . Đây chính là điều phải chứng minh.
XB XA AB2

222
Hội thảo khoa học, Ninh Bình 15-16/09/2018

X B C

Dựa vào bổ đề, ta chứng minh bài toán như sau:

E
X M

A
Z
D

R
W

B C F Y

MA MB AB
Theo tính chất 4, ta có ∆MAB ∼ ∆MDC ⇔ = =
MD MC DC
BX MB2 CZ MC2 BX CZ
Áp dụng bổ đề trên ta có = − 2
, = − 2
⇒ = ⇒
XA MA ZD MD XA ZD
∆MAX ∼ ∆MDZ ⇒ ( XA, XM) ≡ ( ZD, ZM) (mod π ).
Suy ra bốn điểm E, X, Z, M đồng viên.
Tương tự ta cũng chứng minh được bốn điểm F, Y, W, M đồng viên.
Gọi t1 , t2 theo thứ tự là tiếp tuyến của hai đường tròn EXZ, FYW tại M. Ta sẽ chứng
minh t1 ≡ t2 .

(t1 , t2 ) ≡ (t1 , EM) + ( EM, FM) + ( FM, t2 )


≡ ( XM, XE) + ( EM, FM) − (YM, YF )

223
Hội thảo khoa học, Ninh Bình 15-16/09/2018

≡ ( XM, BM) + ( BM, AB) + ( EM, FM) − [(YM, MB) + ( MB, BC )]


≡ ( FM, FB) + ( BM, AB) + ( EM, FM) − [( EM, EB) + ( MB, BC )]
≡ ( FM, FB) + ( BM, AB) + ( EM, FM) + ( EB, EM) + ( BC, MB)
≡ 0 (mod π )
Suy ra t1 ≡ t2 .
Vậy hai đường tròn ( FWY ), ( EXZ ) tiếp xúc với nhau tại M.
b. Trước hết, ta chứng minh XZ, WY cắt nhau tại một điểm thuộc tiếp tuyến chung
tại M của hai đường tròn ( FWY ), ( EXZ ).
Giả sử gọi R là giao điểm của tiếp tuyến chung tại M của hai đường tròn
( FWY ), ( EXZ ) với đường thẳng XZ .
MC MB BC
Ta có ∆MXB ∼ ∆MZC ⇔ ∆MCB ∼ ∆MZX ⇒ = = .
MZ MX ZX
CY MC 2 ZR MZ 2
Áp dụng bổ đề trên ta có =− 2
và =− .
YB MB RX MX 2
CY ZR
Do đó =
YB RX
Suy ra: ∆MXR ∼ ∆MBY ⇔ ∆MXB ∼ ∆MRY ⇒ (YM, YR) ≡ ( BM, BX )
(mod π ).
Tương tự ta chứng minh được ∆MAB ∼ ∆MWY ⇒ (YW, YM ) ≡ ( BA, BM)
(mod π ).
Vì vậy, ta có (YW, YR) ≡ (YW, YM) + (YM, YR) ≡ ( BA, BM) + ( BM, BX ) ≡
( BA, BX ) ≡ 0 (mod π ). Suy ra R, Y, W thẳng hàng.
Ta có RX.RZ = RM2 = RW.RY, suy ra bốn điểm X, Z, W, Y đồng viên.
c.

Y
B
M
W

X A

E Z D C

Do dai đường tròn ( FWY ), ( EXZ ) tiếp xúc với nhau tại M nên 0 ≡ ( ME, MF ) ≡
( ME, MR) + ( MR, MF ) ≡ −( XM, XE) + (YM, YF ) (mod π )( XM, XB) ≡ (YM, YB)

224
Hội thảo khoa học, Ninh Bình 15-16/09/2018

(mod π )
Suy ra bốn điểm M, X, Y, B đồng viên.
Chứng minh tương tự bốn điểm M, X, W, A đồng viên.
Ta có ( XM, XY ) ≡ ( BM, BY ) ≡ ( BM, BC ) ≡ ( AM, AD ) ≡ ( AM, AW ) ≡ ( XM, XW )
(mod π ).
Suy ra: W, X, Y thẳng hàng. Tương tự ta chứng minh được X, Y, Z thẳng hàng.
Vậy X, Y, Z, W thẳng hàng (điều phải chứng minh).

2 Một số bài toán liên quan tới điểm Miquel


Bài toán 1 (China Team Selection 2018, P1, Test 2). Cho tam giác ABC có điểm D di
chuyển trên cạnh BC. Dựng E, F lần lượt trên AC và AB sao cho BF = CD, CE = BD. P
là giao điểm thứ hai của hai đường tròn ngoại tiếp các tam giác BDF, CDE. Chứng minh
rằng tồn tại điểm Q cố định sao cho độ dài PQ là hằng số.

Lời giải. Gọi I là tâm đường tròn nội tiếp tam giác ABC. G là điểm trên BC sao cho
CG = BD.
Nhận thấy: G, E đối xứng với nhau qua IC suy ra IE = IG = IF. Tam giác AEF có I là
giao phân giác góc EAF
[ và trung trực đoạn EF. Vậy I nằm trên đường tròn ngoại tiếp
tam giác AEF.
Theo định lí Miquel ta có đường tròn ngoại tiếp tam giác AEF đi qua điểm P.
Gọi Y là giao điểm thứ hai của đường thẳng BI với đường tròn ngoại tiếp tam giác
BDF, Z là giao điểm thứ hai của đường thẳng CI với đường tròn ngoại tiếp tam giác
CDE.
Ta có
YF = YD, FB = DC, YFBd = YDC.
[

E
Y
I Q
Z P

D G
B M C

Suy ra ∆YFB = ∆YDC ⇒ YB = YC. Chứng minh tương tự ta có ZB = ZC . Vậy Y, Z là


giao của trung trực đoạn BC với IB, IC. Suy ra Y, Z cố định.
Áp dụng tính chất 6 ta có I, Y, Z, P đồng viên. Vậy P nằm trên đường tròn ngoại tiếp tam

225
Hội thảo khoa học, Ninh Bình 15-16/09/2018

giác IYZ cố định. Gọi Q là tâm của đường tròn này, ta có PQ cố định, không phụ thuôc
vào vị trí của D.
Bài toán 2 (Iran TST vòng 1, ngày 2, năm 2017). Cho tam giác ABC. Gọi P.Q là các điểm
bất kì trên cạnh BC sao cho BP = CQ và P nằm giữa B, Q. Đường tròn ngoại tiếp tam
giác APQ cắt các đoạn AB, AC lần lượt tại E, F.EP, FQ cắt nhau tại T. Hai đường thẳng
đi qua trung điểm của BC và song song với AB, AC lần lượt cắt EP, FQ tại X, Y. Chứng
minh rằng đường tròn ngoại tiếp tam giác TXY tiếp xúc với đường tròn ngoại tiếp tam
giác APQ.

Lời giải. Gọi M là trung điểm của đoạn BC, S là giao điểm thứ hai của đường thẳng AM
với đường tròn ngoại tiếp tam giác APQ.
Do MX k AB nên

MXP = \
\ MXE = BEP
d = PSA
[ = PSM.
[

Suy ra tứ giác MSXP nội tiếp. Do đó S thuộc đường tròn ngoại tiếp tam giác PMX.
Tương tự do MY k AC nên

MYQ = MYF
\ [ = QFC
[ = ASQ
[ = MSQ.
[

F
M
B P Q C
S Y

T
Suy ra tứ giác MSYQ nội tiếp. Do đó S thuộc đường tròn ngoại tiếp tam giác QMY.
Từ đây suy ra S thuộc cả hai đường tròn ngoại tiếp các tam giác PMX, QMY, nên theo
định lí Miquel thì S thuộc đường tròn ngoại tiếp tam giác TXY.
Do MX k BE, MY k FC, nên suy ra
MX MP MQ MY MX BE
= = = ⇒ = (1)
BE BP CQ FC MY CF
BE AC
và BE.BA = BP.BQ = CQ.CP = CF.CA ⇒ = . (2)
CF AB
MX AC
Từ (1) và (2) suy ra
= .
MY AB
Lại có XMY
\ = XMS
[ + SMY
[ = BAS[ + CAS
[ = A.
b

226
Hội thảo khoa học, Ninh Bình 15-16/09/2018

Do đó

∆ABC ∼ ∆MYX (c.g.c) ⇒ ACB


[ = QMY
\ = QSY
[=\ MXY = MXS
[ + SXY
[
= QPS
[ + SXY
[
⇒ QSY
[ = QPS
[ + SXY.
[

Từ đây suy ra đường tròn ngoại tiếp tam giác TXY tiếp xúc với đường tròn ngoại tiếp
tam giác APQ.
Nhận xét 3. Bài toán này khá thú vị, được bạn Lê Viết Ân mở rộng như sau: Cho tam giác ABC,
gọi P, Q là các điểm trên đoạn BC. Các đường tròn ( ABP) và ( ACQ) cắt nhau tại điểm thứ
hai G.M là giao điểm của AG với BC. Đường tròn ( APQ) cắt AB, AC theo thứ tự tại E, F.EP
cắt FQ tại T. Hai đường thẳng qua M song song với AB, AC theo thứ tự cắt EP, FQ tại X, Y.
Chứng minh rằng ( TXY ) tiếp xúc với ( APQ) .
Bài toán 3 (Iran TST vòng 1, ngày 1, năm 2017). Cho tam giác ABC, Ia là tâm đường tròn
bàng tiếp góc A. Gọi v là đường tròn bất kì qua A, Ia cắt các đường thẳng AB, AC theo
thứ tự tại X, Y. Gọi S, T là các điểm trên Ia B, Ia C sao cho AXI
[a = BTI [a .
[a = CSI
[a , AYI
Các đường thẳng BT, CS cắt nhau tại K, KIa , TS cắt nhau tại Z. Chứng minh rằng X, Y, Z
thẳng hàng.

Lời giải. Trên BC lấy điểm L sao cho BL = BX.


Ta có ∆BXIa = ∆BLIa (c.g.c) ⇒ LIa = XIa = Ia Y và XI
[ aB = [ LIa B.
0
180 − BAC
[
Dễ thấy CI
[ a B = CIa Y + BI
[ [ aX = . Do đó CI
[ a Y = CIa L.
[
2

B L C

S
K
X
Z

T
Y

Ia

Suy ra ∆CIa Y = ∆CIa L(c.g.c) ⇒ TY = TL.


Từ đây ta có SX, SL đối xứng với nhau qua Ia B . TY, TL đối xứng với nhau qua Ia C.

227
Hội thảo khoa học, Ninh Bình 15-16/09/2018

Biến đổi góc ta có XST


[ = STY d ⇒ XS k TY.
Dễ thấy SKTIa , BLTIa , CLSIa là các tứ giác nội tiếp, nên L là điểm Miquel của tứ giác toàn
phần nội tiếp SKTIa CB, nên LS là phân giác trong góc SLT. d
ZS LS XS
Khi đó = = . Vậy X, Y, Z thẳng hàng.
ZT LT TY
Bài toán 4 (Trường hè Bắc Trung Bộ 2015). Cho tứ giác ABCD nội tiếp đường tròn (O)
và ABCD không phải là hình thang. Gọi E là giao điểm của AC và BD, P là một điểm
thuộc đường thẳng OE. Đường tròn ngoại tiếp tam giác PAD và PBC cắt nhau tại điểm
thứ hai Q.
a. Chứng minh rằng Q thuộc một đường tròn cố định khi P di chuyển.
b. Đường tròn ngoại tiếp tam giác PAB, PCD cắt nhau tại điểm thứ hai R. Chứng minh
rằng đường thẳng QR luôn đi qua một điểm cố định khi P di chuyển.
Lời giải. a. Gọi M là giao điểm của AD và BC, N là giao điểm của AB và CD. Gọi F là
giao điểm của OE và MN. Theo nhận xét trên ta có OF ⊥ MN và F là điểm Miquel của tứ
giác toàn phần ABCDMN. Từ đó ta có tứ giác FDAN nội tiếp.
Theo tính chất phương tích ta có P, Q, M thẳng hàng.
Ta có MF.MN = MA.MD = MQ.MP, suy ra tứ giác PQFN nội tiếp. Do đó
[ = NFP
NQP [ = 900 . Vậy Q thuộc đường tròn đường kính MNcố định.

C
C F

E D
Q

O P
R
B
A N

b. Tương tự câu a, ta có MR⊥ NP tại R. Xét tam giác PMN có các đường cao MR, NQ, PF
đồng quy.
Gọi L là giao điểm của QR và MN.
Ta có ( LF, MN ) = −1 , mà M, N, F cố định nên L cố định.
Vậy QR luôn đi qua điểm L cố định.

228
Hội thảo khoa học, Ninh Bình 15-16/09/2018

Bài toán 5 (Trần Quang Hùng). Cho tam giác ABC với E, F là hai điểm lần lượt nằm trên
cạnh CA, AB sao cho AE = AF, EF cắt BC tại D. K, L lần lượt là tâm ngoại tiếp tam giác
DBF, DCE. G là điểm đối xứng của D qua KL. R nằm trên đường tròn ngoại tiếp tam
giác AEF sao cho AR k BC. Gọi BE cắt CF tại H. AH cắt BC tại S. Lấy điểm T thuộc GR
sao cho ST ⊥ BC, M là trung điểm của đoạn ST. Chứng minh rằng GM luôn đi qua một
điểm cố định khi E, F thay đổi.

Lời giải.

A Q
T R

M
K O
D E
F H

B S
C

P
L

Gọi (O) là đường tròn ngoại tiếp tam giác ABC. Dễ thấy G là giao điểm thứ hai của
hai đường tròn ngoại tiếp tam giác BDF và DCE nên theo định lí Miquel G cũng nằm
trên đường tròn (O) và đường tròn ngoại tiếp tam giác AEF.
Từ đây ta suy ra hai tam giác GFB và GEC đồng dạng.
Theo định lí Ceva cho tam giác ABC với các đường AS, BE, CF đồng quy tại H và chú ý
SB BF GB
AE = AF ta có = = . Vậy GS là phân giác nên GS đi qua trung điểm P của
SC CE GC
cung BC không chứa G.
Ta lại có ARG
[ = AFG [ = GDB [ mà AR k BC nên D, G, R thẳng hàng.
Theo định lí Menelaus cho tam giác ABC với D, E, F thẳng hàng và chú ý AE = AF ta có
DB BF GB
= = nên GD là phân giác ngoài tam giác GBC.
DC CE GC
Từ đó GD đi qua trung điểm Q của cung BC chứa G. Như vậy O là trung điểm của PQ,
mà trong tam giác GPQ có ST k PQ nên G, M, O thẳng hàng hay GM đi qua điểm O cố
định.

Nhận xét 4. Bài toán này đã giấu đi sự xuất hiện của điểm cố định là tâm ngoại tiếp O bằng
cách sử dụng định lí Miquel quen thuộc.

Bài toán 6. Cho tứ giác ABCD nội tiếp đường tròn (O), AC cắt BD tại E. Đường tròn
ngoại tiếp tam giác EAB và ECD cắt nhau tại F. Chứng minh rằng OF ⊥ FE .

229
Hội thảo khoa học, Ninh Bình 15-16/09/2018

Lời giải. Theo tính chất trục đẳng phương EF, AB, CD đồng quy tại G.
Theo định lí Miquel dễ thấy F nằm trên các đường tròn ngoại tiếp tam giác GAC, GBD.
Từ đó gọi R là bán kính của đường tròn (O) thì:

OE2 − OG2 = (OE2 − R2 ) − (OG2 − R2 ) = EA.EC − GA.GB = EF.EG − GE.GF


  
= EG EF + GF = EF − GF EF + GF
= FE2 − FG2

Từ đó ta suy ra OF ⊥ FE .

D C

E F
G
O
A
B

Nhận xét 5. Bài toán trên tuy đơn giản, song có rất nhiều ứng dụng. Sau đây là một bài toán
ứng dụng của nó và tất nhiên là có sử dụng tới định lí Miquel.
Bài toán 7. Cho tam giác ABC có đường đối trung AD, trung tuyến AM. P là điểm nằm
trong tam giác ABC sao cho PBA [ là hình chiếu của P lên AD. Chứng minh
[ = PCA.K
rằng đường tròn ngoại tiếp tam giác KDM tiếp xúc với đường tròn ngoại tiếp tam giác
PBC.

Lời giải. Gọi PB, PC theo thứ tự cắt CA, AB tại E, F. Gọi S là giao điểm thứ hai của hai
đường trong ngoại tiếp tam giác PEF và PBC.
Do PBA
[ = PCA [ nên tứ giác BCEF nội tiếp đường tròn ( L).
Gọi G là giao điểm của EF và BC. Theo định lí Brocard ta có PG ⊥ AL. Theo bài toán trên
ta có PG ⊥SL.
Mặt khác theo tính chất trục đẳng phương thì S, P, G thẳng hàng.
Từ đó suy ra PG vuông góc với AL tại S.
Các tứ giác GSLM, ASKP nội tiếp và góc có cạnh tương ứng vuông góc ta có \ SMD =
SLG = SPA = SKA, suy ra S nằm trên đường tròn ngoại tiếp tam giác KDM.
d d [
Kẻ tiếp tuyến tại S của đường tròn ngoại tiếp tam giác SPB cắt BC tại T.
Do ∆SBE ∼ ∆SCF và tính chất đường đối trung ta có

TB SB2 BE2 AB2 DB


= 2
= 2
= 2
= .
TC SC CF AC DC

230
Hội thảo khoa học, Ninh Bình 15-16/09/2018

Suy ra ( BC, DT ) = −1 ⇒ TS2 = TB.TC = TD.TM hay TS tiếp xúc với đường tròn
ngoại tiếp tam giác KDM.
Vậy đường tròn ngoại tiếp tam giác KDM và PBC tiếp xúc với nhau tại S.

F S
P K
L

T G C
B DM

Nhận xét 6. Đây là một bài toán tiếp xúc thú vị, điểm S thực chất là một trong các điểm
Miquel của tứ giác BCEF.

Bài toán 8 (Trần Quang Hùng). Cho tứ giác ABCD nội tiếp có AC và BD cắt nhau tại E.
Đường tròn ngoại tiếp các tam giác EAD và SBC cắt nhau tại F khác E. Trung trực các
đoạn thẳng DB, AC lần lượt cắt FB, FA theo thứ tự tại K, L. Chứng minh rằng KL chia
đôi AB

Lời giải. Gọi O là đường tròn ngoại tiếp tứ giác ABCD.AB cắt CD tại T.AD cắt BC tại
S. OT cắt SA, SB theo thứ tự tại P, Q. Theo các bài toán quen thuộc về phương tích và
điểm Miquel thì SE vuông góc với OT tại F và tứ giác ABOF nội tiếp.
Gọi BF cắt (O) tại giao điểm thứ hai R.
Do KD = KB nên DKR [ = 2 BDR[ = DOR.[ Suy ra tứ giác DKOR nội tiếp.
Lại có

FOR [ − FRO
[ = BFO [ − FBO
[ = BAO [
[ − FBO
= ABO [=[ ABF = 1800 − PDR.
[

Từ đó suy ra tứ giác DPOR nội tiếp.


Từ đó suy ra tứ giác DPKR nội tiếp hay APK [ = DKR [ Do đó PK k AB.
[ = SAB.
Lại có ( PQ, FT ) = S( PQ, ST ) = F ( AB, ST ) = −1.
Từ đó suy ra: K ( AB, PS) = B( AK, PQ) = ( TF, PQ) = −1.
Kết hợp với PK k AB nên KS đi qua M. Chứng minh tương tự ta có LS đi qua M.
Vậy KL đi qua M.

231
Hội thảo khoa học, Ninh Bình 15-16/09/2018

A M
E
K
Q
P F
O
T
D C
L
R

Bài toán 9. Cho tam giác ABC nội tiếp (O). Các tiếp tuyến tại B, C cắt nhau tại T. Đường
thẳng qua A vuông góc với AT cắt BC tại S. D, E trên ST trong đó T nằm giữa D và E, D
nằm giữa S và T sao cho DT = BT = ET. Chứng minh rằng tam giác ABC đồng dạng
với tam giác ADE.

Lời giải. Gọi M là trung điểm của đoạn BC. Khi đó T, M, A, S đồng viên.
Ta cũng có D, E, B, C đồng viên.
Gọi K là giao điểm của DB và CE.
K
A

S C
B M

T
E

Khi đó
[ = 1800 − KBC
BKC [ = 1800 − KBC
[ − BCK [ − DBT
[ = TBC
[ = BAC.
[

Suy ra K thuộc đường tròn ( ABC ).


Từ đây suy ra A thuộc giao hai đường tròn (KBC ) và (SMT ).

232
Hội thảo khoa học, Ninh Bình 15-16/09/2018

Gọi J là giao điểm của CD và BE. Theo định lí Brocard ta có T J ⊥SK tại A0 .
Áp dụng định lí Pascal đảo cho 6 điểm BBKCCJ ta suy ra J thuộc đường tròn (KBC ).
Lại có A0 , S, T, M đồng viên.
Suy ra: A0 thuộc vào giao của hai đường tròn (SBC ) và (SMT ). Do đó A ≡ A0 .
Từ đó suy ra B là điểm Miquel của tam giác KSC nên tứ giác ASDB nội tiếp.
Suy ra A là điểm Miquel của tứ giác BCEDSK. Do đó hai tam giác ABC và ADE đồng
dạng.

3 Bài tập tự luyện.


Bài 1 (Arab Saudi IMO Training Test 2017). Cho tam giác ABC nội tiếp đường tròn (O),
trực tâm H. Trung tuyến AM cắt (O) lần thứ hai tại N.AH cắt (O) tại K. Các đường
thẳng KN, BC và đường thẳng qua H vuông góc với AN cắt nhau tạo thành tam giác
XYZ. Chứng minh rằng ( XYZ ) tiếp xúc với (O).
Bài 2. Cho tam giác ABC. Một đường tròn Oa qua B, C cắt AC, AB lần lượt tại E, F.BE
giao CF tại P. Gọi M là trung điểm của BC, L đối xứng với K qua M. Các đường thẳng
PK, QL, BC cắt nhau tạo thành tam giác XYZ. Chứng minh rằng ( XYZ ) tiếp xúc với
( ABC ).
Bài 3. Cho tam giác ABC nội tiếp đường tròn (O). Gọi A0 là điểm đối xứng của A qua
O. Trung tuyến AM của tam giác ABC cắt BA0 , CA0 lần lượt tại L và K. Các đường thẳng
qua L vuông góc với BA0 , qua K và vuông góc với CA0 và đường thẳng OM cắt nhau tạo
thành tam giác XYZ. Gọi P là giao của hai tiếp tuyến tại B và C của (O). Chứng minh
rằng ( AMP) tiếp xúc với ( XYZ ).

Bài 4. Cho tam giác ABC. Một đường tròn bất kì qua B, C cắt AC, AB lần lượt tại E, F. BE
cắt CF tại P. Một đường thẳng d bất kì qua A cắt BE, CF lần lượt tại L, K. Đường thẳng
d0 đẳng giác với d trong góc BAC
[ cắt BE, CF lần lượt tại M, N. MK cắt LN tại X. MK, LN
cắt BC lần lượt tại Z, Y. Chứng minh rằng đường tròn ngoại tiếp tam giác BPC và XYZ
tiếp xúc với nhau.

Bài 5. Cho tam giác ABC nội tiếp đường tròn (O). Hai đường cao BE, CF cắt nhau tại
trực tâm H.EF cắt BC tại K. Qua K kẻ đường thẳng vuông góc với BC cắt CH, BH lần
lượt tại P, Q. Đường thẳng AH cắt (O) tại T khác A.
a. Chứng minh rằng P, Q, H, T đồng viên.
b. Qua H kẻ đường thẳng vuông góc với EF cắt BC tại L. Kẻ đường kính AA0 của (O).
Đường thẳng qua L vuông góc với BC cắt A0 B, A0 C lần lượt tại Y, Z. Chứng minh rằng
( A0 YZ ) tiếp xúc với ( HPQ).

Bài 6. Cho tam giác ABC với đường tròn ( Ib ), ( Ic ) bàng tiếp góc B và C. Gọi Ja , Jb , Jc lần
lượt là tiếp điểm của ( Ic ) với BC, CA, AB. Gọi L a , Lb , Lc lần lượt là tiếp điểm của ( IB ) với
BC, CA, AB.Ja Jb cắt Ib Ic tại K, L a Lc cắt Ib Ic tại Q.KLb cắt QJc tại R, K Jc cắt QLb tại P.
a. Chứng minh rằng tứ giác Jc RLb P nội tiếp.
b. Chứng minh rằng AP = r a với r a là bán kính đường tròn bàng tiếp góc A của tam giác
ABC.

233
Hội thảo khoa học, Ninh Bình 15-16/09/2018

Bài 7. Cho tứ giác lưỡng tâm ABCD có tâm đường tròn ngoại tiếp là O. Gọi E, F lần lượt
là giao điểm của AB và CD, AD và BC. Chứng minh rằng tồn tại môt đường tròn tâm O
tiếp xúc với bốn đường tròn ngoại tiếp các tam giác EAD, EBC, FAB, FCD.

Bài 8. Cho tam giác ABC nội tiếp đường tròn (O) với trực tâm H. Hai đường thẳng d1
và d2 bất kì vuông góc với nhau và đi qua H.d1 cắt BC, CA, AB lần lượt tại X1 , Y1 , Z1 .
GọiA1 B1 C1 là tam giác tạo bởi các đường thẳng qua X1 và vuông góc với BC, qua Y1
và vuông góc CA, qua Z1 và vuông góc với AB. Tương tự ta xác định được tam giác
A2 B2 C2 . Chứng minh rằng đường tròn ngoại tiếp các tam giác A1 B1 C1 và A2 B2 C2 tiếp
xúc với nhau tại một điểm trên (O).

4 Kết luận
Chuyên đề dù đã cố gắng biên soạn khá kĩ lưỡng song không thể tránh khỏi sai sót.
Rất mong được các thầy cô giáo và các em học sinh góp ý và phê bình chân thành để có
dịp tôi sửa chữa chuyên đề này hoàn thiện hơn.
Qua chuyên đề, xin được gửi lời cảm ơn tới các thầy Nguyễn Minh Hà, Trần Quang
Hùng, Nguyễn Văn Linh và Lê Phúc Lữ đã giúp tôi hoàn thành chuyên đề này. Đồng
thời, tôi cũng xin cảm ơn các thầy cô trong bộ môn Toán và Latex như các thầy Hồ Hà
Đặng, Vũ Nguyễn Hoàng Anh, Vũ Văn Trường đã giúp tôi tiếp cận và sử dụng Latex.

5 Tài liệu tham khảo


Chuyên đề có sử dụng các bài toán, tính chất trong các sách sau:
• Nguyễn Minh Hà, Hình học phẳng định hướng, NXB Dân trí, 2015.
• Trần Quang Hùng, Mỗi tuần một bài toán hình học, NXB ĐHQG Hà Nội, 2017.
• Toán học tuổi trẻ.
• Nguyễn Văn Linh,108 bài toán hình học sơ cấp,NXB ĐHQG Hà Nội, 2018.
• Yufei Zhao, Cyclic Quadrilaterals — The Big Picture.
• BALKAN MATHEMATICAL OLYMPIAD, 2009.
• Yufei Zhao,Three Lemmas in Geometry
• Các bài toán trên các diễn đàn toán như https://artofproblemsolving.com, diễn
đàn toán học, .....

234
Hội thảo khoa học, Ninh Bình 15-16/09/2018

ƯỚC NGUYÊN TỐ CỦA MỘT LỚP CÁC DÃY


SỐ NGUYÊN

Nguyễn Song Minh


Hội toán học Hà Nội

Tóm tắt nội dung


Rất nhiều vấn đề trong Số Học liên quan đến sự tồn tại vô hạn các số nguyên tố trong một
dãy nguyên. Ví dụ như định lý Dirichlet, các số nguyên tố Fermat hay các số nguyên tố Mersene.
Một vấn đề đơn giản hơn, đó là nói đến các ước nguyên tố của phần tử trong dãy. Bài viết này
bàn về khái niệm ước nguyên tố của một dãy số nguyên, và tập các ước nguyên tố đó. Phạm vi bài
viết là ở mức độ sơ cấp, mặc dù vấn đề nói đến trong bài vẫn được nghiên cứu ở lý thuyết Số cao
cấp.

1 Các quy ước và ký hiệu

Suốt bài viết này, tôi sử dụng các ký hiệu với ý nghĩa được quy ước thống nhất
như sau:

• gcd( a, b): Ước số chung lớn nhất của a; b ∈ Z.

• m - a: Số nguyên a không chia hết cho số nguyên m (với m 6= 0).

• b x c: Số nguyên lớn nhất không vượt quá số thực x (phần nguyên của x).

• ϕ(m): Phi hàm Euler.

• v p (m): Hàm định giá p-adic.

• P: Tập hợp chứa tất cả các số nguyên tố.

2 Các kiến thức cơ sở

Dãy số nguyên về bản chất, là hàm số với tập nguồn là Z+ và tập đích là Z. Do giá
trị của các phần tử trong dãy là các số nguyên, vì thế chúng ta có được các tính chất

235
Hội thảo khoa học, Ninh Bình 15-16/09/2018

Số Học của chúng. Một vấn đề rất tự nhiên được đặt ra, đó là tìm hiểu các số nguyên
tố là ước của ít nhất một trong những phần tử trong dãy. Bởi vậy, chúng ta quan tâm
đến khái niệm sau.

Định nghĩa 1. Cho dãy số nguyên { an }n∈Z+ , số nguyên tố p được gọi là ước nguyên tố của
dãy khi và chỉ khi tồn tại chỉ số m sao cho p | am .

Trong suốt bài viết này, với { an }n∈Z+ là một dãy số nguyên thì tập hợp các ước
nguyên tố của dãy { an }n∈Z+ được ký hiệu là P( an ). Còn với một tập con S của Z thì
P(S) được hiểu là tập các số nguyên tố là ước của một phần tử nào đó trong S.

Để minh họa cho định nghĩa nêu trên, ta xét một số ví dụ sau.

Ví dụ 1. Với dãy các luỹ thừa của 2 là an = 2n−1 , khi đó chỉ có 2 là ước nguyên tố của dãy.
Nói khác đi P 2n−1 = {2}.


Ví dụ 2. Với dãy các luỹ thừa của 2 là an = 2n − 1. Khi đó mọi ước nguyên tố của dãy là các
số nguyên tố lẻ, nói khác đi P (2n − 1) = P \ {2}. Lý do là bởi vì 2 - (2n − 1) ∀ n ∈ Z+
và nếu p là số nguyên tố lẻ thì theo định lý Fermat bé ta có
 
p −1
p| 2 −1 .

Ví dụ 3. Xét dãy số nguyên sau đây

an = 2n + 3n + 6n − 1.

Dễ kiểm tra rằng 6 | a2 , do vậy 2, 3 đều là các phần tử của P ( an ), thêm nữa với p ∈
P \ {2, 3} thì theo định lý Fermat bé ta có

6a p−2 = 3.2 p−1 + 2.3 p−1 + 6 p−1 − 6 ≡ 3 + 2 + 1 − 6 ≡ 0 (mod p).

Điều đó cho thấy rằng


P ( an ) = P.
Bạn đọc hẳn nhận ra vấn đề với dãy số ở ví dụ này, chính là bài toán IMO 2005.

Ngoài khái niệm về ước nguyên tố của dãy nguyên và tập ước nguyên tố, để thuận
tiện cho việc theo dõi bài viết. Tôi nhắc lại một vài định lý và bổ đề Số Học hay sử
dụng ở đây.

236
Hội thảo khoa học, Ninh Bình 15-16/09/2018

Định lý 1 (Euler). Với ϕ(m) là số các số nguyên dương không vượt quá m và nguyên tố
cùng nhau với m, khi đó
a ϕm ≡ 1 (mod m).

Từ định lý Euler, ta có một hệ quả đáng chú ý sau đây.

Hệ quả 1. Cho k số nguyên khác 0 là a1 , a2 , . . . , ak và một số nguyên dương m > 1, khi đó


tồn tại N đủ lớn để
N +nϕ(m)
ai ≡ aiN (mod m) ∀ i = 1, k, n ∈ Z+ .

Chứng minh. Giả sử ta có phân tích ra thừa số nguyên tố của m là


t
kj
m= ∏ pj , p j ∈ P, p1 < p2 < . . . < pt , k j ∈ Z+ .
j =1

Giả sử N = max k j : j = 1, t , khi đó.

kj kj
 
1. Nếu p j - ai thì từ | m, vì phi hàm Euler có tính chất nhân tính nên ϕ
pj pj |
ϕ(m) sử dụng định lý Euler ta có
ϕ(m) k
ai ≡ 1 (mod p j j ).

Từ đó mà có được
N +nϕ(m) k
ai ≡ aiN (mod p j j ).

2. Nếu p j | ai thì do N ≥ k j nên

N +nϕ(m) k
ai ≡ aiN ≡ 0 (mod p j j ).

kj
   
N +nϕ(m) N +nϕ(m)
Từ p j | ai − aiN ∀ j = 1, t để có m | ai − aiN ∀ i = 1, k.

Với số nguyên tố p bất kỳ, ta có ϕ( p) = p − 1. Vì vậy, trong trường hợp m = p thì


định lý Euler trở thành định lý sau đây.

Định lý 2 (Fermat bé). Với p là số nguyên tố và p - a, khi đó

a p −1 ≡ 1 (mod p)

237
Hội thảo khoa học, Ninh Bình 15-16/09/2018

Định lý 3 (Định lý Trung Hoa về số dư). Với m1 , m2 , . . . , mn là các số nguyên dương


đôi một nguyên tố cùng nhau còn r1 , r2 , . . . , rn là các số nguyên bất kỳ, khi đó tồn tại duy
nhất r ∈ {0, 1, . . . , m1 m2 . . . mk − 1} thỏa mãn hệ đồng dư sau

r ≡ rk mod mk ∀ k = 1, n.

Những kiến thức cơ bản nêu trên là cần thiết để theo dõi các vấn đề ở mục tiếp
theo, ngoài ra bạn đọc cần có những trang bị về định giá p-adic, bổ đề nâng bậc hay
những kiến thức về cấp, căn theo modulo và thặng dư bậc hai để xử lý các bài tập đề
nghị.

3 Một số vấn đề cơ bản

Bài toán 1 (Định lý Schur). Cho đa thức hệ số nguyên f ( x ) có bậc dương, khi đó tập các
ước nguyên tố của dãy an = f (n) là vô hạn.

Lời giải. Giả sử ngược lại là tập tất cả các ước nguyên tố của dãy an = f (n) chỉ là tập
hữu hạn sau đây
P ( f n ) = { p1 , p2 , . . . , p t } .
Do f ∈ Z[ x ] nên f ( x ) = a0 + a1 x + . . . + an x n , với ai ∈ Z ∀ i = 1, n, ta xét hai trường
hợp sau.

1. Nếu a0 = 0, khi đó p | f ( p) ∀ p ∈ P cho nên dãy an nhận mọi số nguyên tố làm


ước nguyên tố. Điều này, mâu thuẫn với giả sử ở trên.

2. Nếu a0 6= 0 với mỗi số nguyên dương k đặt kp1 p2 . . . pt = mk , ta có

f ( a0 mk ) = a0 + a1 a0 mk + . . . + an a0n mnk
 
2 n −1 n
= a0 . 1 + a1 m k + a2 a0 m k + . . . + a n a0 m k .

Đặt g( x ) = 1 + a0 a1 x + . . . + a0n−1 an x n vì deg( g) = deg( f ) = n ∈ Z+ cho nên


phương trình | g( x )| = 1 chỉ có hữu hạn nghiệm, tức là tồn tại k ∈ Z+ sao cho
| g (mk )| > 1. Để ý rằng g( x ) ∈ Z[ x ] nên giá trị g (mk ) là một số nguyên và vì thế
nó có ước nguyên tố p nào đó. Rõ ràng g (mk ) ≡ 1 (mod mk ), cho nên

gcd ( g (mk ) , mk ) = 1.

Điều này cũng dẫn đến gcd (mk , p) = 1 cho nên p ∈


/ P ( f n ), dẫn đến điều mâu
thuẫn.

238
Hội thảo khoa học, Ninh Bình 15-16/09/2018

Dựa vào định lý Schur, chúng ta có thể giải quyết bài toán sau.

Bài toán 2. Cho các số nguyên dương a, b với a > b, tìm tất cả các đa thức hệ số
nguyên P( x ) thỏa mãn
P ( n ) | ( a n − b n ) ∀ n ∈ Z+ .

Lời giải. Ta xét hai trường hợp.

1. Nếu deg( P) > 0, khi đó theo định lý Schur sẽ tồn tại vô số ước nguyên tố p của
dãy P(n). Giả sử p là một ước nguyên tố bất kỳ như thế (với p > a) và p | P(m)
với m ∈ Z+ , ta có
p | ( am − bm ) .
Mặt khác do P( x ) ∈ Z[ x ] nên

P(m + p) ≡ P(m) ≡ 0 (mod p).

Từ đây ta lại có
p | am+ p − bm+ p .


Vì p - a, p - b và am ≡ bm (mod p) nên kết hợp định lý Fermat bé ta có

am+ p − bm+ p ≡ am ( a p − b p ) ≡ am ( a − b ) (mod p).

Điều này kéo theo p | ( a − b), nhưng do a − b là một số nguyên dương cố định
cho nên nó không thể có vô số ước nguyên tố. Vậy nên trường hợp này không
thể xảy ra.

2. Nếu deg( P) ≤ 0, tức P( x ) là đa thức hằng thế thì P( x ) = d với d = P(1) và


là ước của a − b. Lúc đó do ( a − b) luôn là ước của an − bn với mọi số nguyên
dương n do đó trường hợp này thỏa.

Vậy, kết quả của bài toán là P( x ) = d ∀ x ∈ R với d là một ước số bất kỳ của a − b.


Kết quả của Schur ứng với lớp dãy số đặc biệt, đó là các dãy sinh ra từ các đa thức
hệ số nguyên. Ở bài toán tiếp theo, chúng ta sẽ xử lý một bài toán về mục đích thì
tương tự nhưng dãy số được cho phức tạp hơn chút xíu.

239
Hội thảo khoa học, Ninh Bình 15-16/09/2018

Bài toán 3. Chứng minh rằng P ( an ) là vô hạn, với

an = blog (2017n + 1)c ∀ n ∈ Z+ .

Lời giải. Giả sử P ( an ) chỉ là hữu hạn. Ta lấy ra 5 số nguyên tố pi ∈


/ P ( an ), theo CRT
sẽ tồn tại 5 số nguyên dương liên tiếp N + 1, N + 2, . . . , N + 5 sao cho

pk | ( N + k ) ∀ k = 1, 5.

Giờ gọi m là chỉ số lớn nhất sao cho am ≤ N + 1, khi đó am+1 > N + 1 và vì
j  k
m +1
1 < am+1 − am = log 2017 + 1 − blog (2017m + 1)c < 5.

Cho nên ta có được

{ am ; am+1 } ∩ { N + 1, N + 2, . . . , N + 5} 6= ∅.

/ P ( an ) thoả p | am am+1 , và điều đó mâu thuẫn


Từ đây thấy sẽ tồn tại số nguyên tố p ∈
với giả sử ban đầu của chúng ta.


Nhận xét 1. Bài toán trên, chỉ là hệ quả của bài toán sau đây (ứng với d = 5). Và lời
giải là hoàn toàn tương tự.

Bài toán 4. Cho d là một số nguyên dương lớn hơn 1, dãy số nguyên { an }n∈Z+ tăng ngặt
và thỏa mãn
a n +1 − a n < d ∀ n ∈ Z+ .
Chứng minh rằng tập các ước nguyên tố của dãy là vô hạn.

Bài toán vừa nêu này tôi không đưa ra lời giải chi tiết, vì nó còn một phiên bản
nâng cấp mạnh hơn nhiều. Ở bài toán tiếp theo, thì cách xử lý như hai bài trước tỏ ra
thiếu hiệu lực.

Bài toán 5. Cho dãy số nguyên { an }n∈Z+ tăng ngặt và giả sử tồn tại một đa thức hệ số thực
P( x ) thỏa mãn
a n +1 − a n < P ( n ) ∀ n ∈ Z+ .
Chứng minh rằng tập các ước nguyên tố của dãy { an }n∈Z+ là vô hạn.

240
Hội thảo khoa học, Ninh Bình 15-16/09/2018

Lời giải. Đặt 1 + deg( P) = k, khi đó tồn tại N0 đủ lớn sao cho

a n +1 < a n + P ( n ) < a n + n k ∀ n ∈ Z+ , n > N0 .

Giả sử P ( an ) là tập hữu hạn, cụ thể

P ( a n ) = { p1 , p2 , . . . , p t } .

Với mỗi số nguyên dương n thỏa n > M = max N0 , a N0 , ta xét hai tập hợp sau
n o
k k
Pn = p11 p2k2 . . . pkt t : k i ∈ N, p11 p2k2 . . . pkt t < n , An = { ai : ai < n} .

k
Lấy e ∈ Pn , thế thì e = p11 p2k2 . . . pkt t với
k
2ki ≤ pi i ≤ e < n.

Như vậy 0 ≤ k i < log2 n, nên theo quy tắc nhân ta có đánh giá sau

|Pn | < (1 + log2 n)t ∀ n > M.

Mặt khác ta lại có n > M thì n > N0 nên với am = max An ta có

a N0 ≤ am < am−1 + mk < . . . < a N0 + ∑ jk < a N0 + mk+1 .


N0 < j<m
 √ 
Từ đó nếu chọn m0 = k +1 n − a N0 thì am0 ∈ An , vì dãy tăng ngặt thế nên
p
|An | ≥ m 0 > k +1 n − a N0 − 1 ∀ n > M.

Với giả thiết phản chứng ở trên thì An ⊂ Pn ∀ n > M, tức là có



k +1 n − a
N0 − 1 |An |
< < 1 ∀ n > M.
(1 + log2 n) t |Pn |

Lấy giới hạn ở vô cùng, cho ta điều vô lý. Vậy, P ( an ) phải là tập vô hạn.


Bây giờ là một bài toán xét kỹ thì cùng bản chất, mặc dù đọc qua yêu cầu thì cảm
thấy khác biệt với các bài toán trên.

Bài toán 6 (China TST, 2006). Cho các số nguyên dương m và n, chứng minh rằng luôn tồn
tại số nguyên dương k sao cho 2k − m có ít nhất n ước nguyên tố phân biệt.

241
Hội thảo khoa học, Ninh Bình 15-16/09/2018

Lời giải. Giả sử với mọi số nguyên dương k số ước nguyên tố của 2k − m đều nhỏ
hơn n, ta giả sử K là số thỏa 2K − m có nhiều ước nguyên tố nhất. Và phân tích ra thừa
số nguyên tố của 2K − m là
k
2K − m = p11 p2k2 . . . pkt t .
1+ k 1 1+ k 2
Đặt M = p1 p2 . . . p1t +kt , thế thì với k đủ lớn ta có được

2K +kϕ( M) − m ≡ 2K − m (mod M).

Tức là 2K +kϕ( M) − m chia hết cho 2K − m, nên 2K +kϕ( M) − m nhận mọi ước nguyên tố
của 2K − m làm ước nguyên tố. Nhưng do vai trò của K nên 2K +kϕ( M) − m cũng chỉ
được phép có đúng bấy nhiêu ước nguyên tố, đồng thời do v pi ( M ) = v pi 2K − m +


1 ∀ i = 1, t nên
   
K +kϕ( M) K
v pi 2 − m = v pi 2 − m ∀ i = 1, t.

Từ đó dẫn đến điều vô lý là

2K +kϕ( M) − m = 2K − m ∀ k ∈ Z+ .

Bằng kỹ thuật tương tự, ta dễ dàng có được lời giải cho bài toán sau.

Bài toán 7. Cho a, b, c là các số nguyên thỏa ac 6= 0 và b > 1. Xét dãy số f n = abn + c ∀ n ∈
Z+ . Chứng minh rằng P ( f n ) là tập vô hạn.

Tôi không trình bày lời giải chi tiết cho bài toán này, thay cho việc đó tôi nêu ra và
xử lý kết quả khá mạnh sau của Polya.

Bài toán 8 (Polya). Giả sử a1 , a2 , . . . , am là các số nguyên dương phân biệt, 0 < a1 <
a2 < . . . < am và P1 ( x ), P2 ( x ), . . . , Pm ( x ) là các đa thức hệ số nguyên khác đa thức 0. Khi
đó dãy số nguyên cho bởi công thức dưới đây, sẽ có vô hạn ước nguyên tố

f n = a1n P1 (n) + a2n P2 (n) + . . . + anm Pm (n) ∀ n ∈ Z+ .

Lời giải. Không mất tính tổng quát, ta giả sử hệ số bậc cao nhất của Pm ( x ) là dương.
Giả sử rằng P ( f n ) là tập hữu hạn, cụ thể

P ( f n ) = { p1 , p2 , . . . , p t } .

242
Hội thảo khoa học, Ninh Bình 15-16/09/2018

Rõ ràng là lim f n = +∞, cho nên tồn tại N0 đủ lớn sao cho f N0 > 1 và đồng thời với
số nguyên dương m cho trước ta sẽ có
N0 +nϕ(m)
ai ≡ aiN0 (mod m).
Giả sử ta có phân tích ra thừa số nguyên tố của f N0 là
k
f N0 = p11 p2k2 . . . pkt t ; k i ∈ N.
1+ k 1 1+ k 2 2+ k
Đặt M = p1 p2 . . . p1t +kt và M0 = p1 1 p22+k2 . . . p2t +kt , thế thì

ϕ M 0 = M ∏ ( p i − 1) = p1 p2 . . . p t ϕ ( M ).

1≤ i ≤ t

Điều đó kết hợp với Pi ( x ) ∈ Z[ x ] để thấy với mọi số nguyên dương n ta có


N +nϕ( M0 )
f N0 +nϕ( M0 ) = ∑ a j 0 Pj N0 + nϕ M0 ≡ ∑ a j 0 Pj ( N0 ) ≡ f N0
 N
(mod M).
1≤ j ≤ m 1≤ j ≤ m

Do v pi ( M ) = 1 + v pi f N0 ∀ i = 1, t, nên từ đó có được
 
∀ i = 1, t, ∀ n ∈ Z+ .

v pi f N0 +nϕ( M0 ) = v pi f N0

Điều đó cho thấy rằng


lim f N0 +nϕ( M0 ) = f N0 .
Kết quả vừa rút ra, trái với nhận xét từ ban đầu của chúng ta là lim f n = +∞.

Vậy, P ( f n ) phải là tập vô hạn.




Ta quan tâm đến một vấn đề khác liên quan đến kết quả của Polya nêu trên, đó là
với dãy f n xác định như thế thì liệu số ước nguyên tố của một phần tử của dãy có bị
giới hạn hay không? Cụ thể là nếu ta cố định một số nguyên dương k, thì có tồn tại
hay không một chỉ số m để f m có ít nhất k ước nguyên tố phân biệt.
Bài toán 9. Giả sử a1 , a2 , . . . , am là các sô nguyên dương phân biệt, và
P1 ( x ), P2 ( x ), . . . , Pm ( x ) là các đa thức hệ số nguyên khác đa thức 0. Xét dãy số nguyên
cho bởi công thức

f n = a1n P1 (n) + a2n P2 (n) + . . . + anm Pm (n) ∀ n ∈ Z+ .


Chứng minh rằng, với k là số nguyên dương cho trước, luôn tồn tại m ∈ Z+ sao cho f m có ít
nhất k ước nguyên tố phân biệt.

243
Hội thảo khoa học, Ninh Bình 15-16/09/2018

Lời giải. Nếu không tồn tại m ∈ Z+ sao cho f m có ít nhất k ước nguyên tố phân biệt,
thì mọi phần tử của dãy f n đều có hữu hạn ước nguyên tố. Ta giả sử K là số thỏa f K
có nhiều ước nguyên tố nhất. Và phân tích ra thừa số nguyên tố của f K là
k
f K = p11 p2k2 . . . pkt t .
1+ k 1 1+ k 2 2+ k 1 2+ k 2
Lại đặt M = p1 p2 . . . p1t +kt và M0 = p1 p2 . . . p2t +kt , để với n đủ lớn ta có

f K +nϕ( M0 ) ≡ f K (mod M).

Từ đây có f K | f K +nϕ( M0 ) và bởi vậy f K +nϕ( M0 ) cũng có t ước nguyên tố là các ước
nguyên tố của f K . Nhưng do vai trò của K nên f K +nϕ( M0 ) cũng chỉ có bấy nhiêu ước
nguyên tố. Đồng thời do v pi ( M ) = 1 + v pi ( f K ) và f K +nϕ( M0 ) ≡ f K (mod M ) nên với
mọi n đủ lớn ta sẽ có
 
v pi f K +nϕ( M0 ) = v pi ( f K ) ∀ i = 1, t.
n o
Điều này kéo theo dãy con f K +nϕ( M0 ) là dãy dừng, mâu thuẫn với việc
+ n ∈Z
lim f n = +∞.


Bây giờ, ta xét đến một bài toán olympic ứng dụng trực tiếp kết quả của Polya ở
trên.

Bài toán 10 (China MO, 2011). Cho m, n là các số nguyên  dương. Chứng
 minh rằng tồn
tại vô hạn các cặp số nguyên dương ( a; b) sao cho ( a + b) | am a + bnb và gcd( a, b) = 1.

Lời giải. Nếu m = n = 1 thì bài toán cực kỳ là tầm thường, bởi vì nếu điều đó xảy ra
ta có thể chọn b = 1 còn a tùy ý. Vì thế, ta có thể giả sử n > 1 và xét dãy số xác định
như sau
ak = (mn)k − n ∀ k ∈ Z+ .
Theo bài toán trên thì P ( ak ) là tập vô hạn, như vậy ta có thể lấy ra vô số các số nguyên
tố p từ P ( ak ) thỏa mãn p > m + n đồng thời khi đó tồn tại k p sao cho p | ak p , tức là

(mn)k p ≡ n (mod p).

Bởi vì 0 < n − 1 < p nên từ định lý Fermat bé ( p − 1) - k p , ta chọn a là số dư của k p


khi chia p − 1 còn b = p − a, để có a, b ∈ Z+ và

gcd( a, b) = gcd( a, a + b) = gcd( a, p) = 1.

244
Hội thảo khoa học, Ninh Bình 15-16/09/2018

Lại có p - mn nên theo định lý Fermat bé, ta có

n ≡ (mn)k p ≡ (mn) a (mod p).

Thêm một lần nữa sử dụng định lý Fermat bé, để có


 
n am + bn = a(mn) a + ( p − a) n p ≡ an + ( p − a) n ≡ 0
a a b
(mod p).

Bây giờ để ý rằng p = a + b và p - nb , ta có điều cần chứng minh.




Nhận xét 2. Nếu bỏ đi yêu cầu gcd( a, b) = 1 thì bài toán sẽ trở nên tầm thường, bởi
vì khi đó với số nguyên tố p bất kỳ ta chỉ cần chọn
 
2
( a, b) = p − 1, ( p − 1) .

Từ bài toán của Polya nêu trên, ta thấy rằng với cặp số nguyên dương phân biệt
a, b bất kỳ thì dãy số f n = an + bn sẽ có vô hạn ước nguyên tố. Có một vấn đề đặt ra
ở đây là nếu ta lấy ra một dãy con nào đó của f n , thì dãy con đó còn có vô hạn ước
nguyên tố hay không? Ta có bài toán sau đây.

Bài toán 11. Cho a và b là các số nguyên dương nguyên tố cùng nhau, với a + b > 3. Khi
đó, nếu p là một số nguyên tố lẻ thì tồn tại ước nguyên tố lẻ khác p của a p + b p , và đồng thời
ước nguyên tố đó không là ước của a + b.

ap + bp
Lời giải. Ta có Φ p = là một số nguyên dương lớn hơn 1, bởi vậy Φ p luôn có
a+b
ước nguyên tố q nào đó. Mặt khác do p lẻ, nên ta có

v2 ( a p + b p ) = v2 ( a + b ).

Từ đó sẽ có Φ p là số lẻ, kéo theo q lẻ.

1. Nếu Φ p chỉ có ước nguyên tố là p, thế thì từ ( a + b) | ( a p + b p ) và

( a p + b p ) ≡ ( a + b) (mod p),

ta có a + b cũng có và chỉ có ước nguyên tố lẻ là p, đồng thời

v p ( a p + b p ) = v p ( a + b) + 1.

245
Hội thảo khoa học, Ninh Bình 15-16/09/2018

Tức là ta có biểu diễn sau

a p + b p = 2k p l +1 = p ( a + b ).

Tuy nhiên, ta lại có đánh giá sau


 
p p 3 3
a + b ≥ a + b = ( a + b) a − ab + b 2 2
> ( a + b )2 .

Nhưng p | ( a + b) do đó từ a + b ≥ p mà có điều mâu thuẫn là

a p + b p > p ( a + b ).

2. Nếu Φ p có ước nguyên tố q 6= p, thế thì hễ q | ( a + b) ta có điều vô lý là

vq Φ p = vq ( a p + b p ) − vq ( a + b) = 0.


Như vậy, Φ p luôn phải có một ước nguyên tố q lẻ và q 6= p, để ý rằng Φ p | ( a p + b p )


mà ta có điều cần chứng minh.


Từ lời giải bài toán trên, ta thấy từ dãy f n = an + bn có thể trích ra một dãy con f pn
có vô hạn ước nguyên tố. Cũng cần nói thêm rằng, bài toán trên là một phiên bản của
định lý Zsigmondy nổi tiếng. Nhưng chuyện đó bàn sau, giờ ta đến với một bài toán
khác như sau.
Bài toán 12 (Romania TST, 2014). Cho m và k là các số nguyên dương cho trước với m lẻ,
chứng minh rằng luôn tồn tại n sao cho mn + nm có ít nhất k ước nguyên tố phân biệt.
k
Lời giải. Lấy ra một số nguyên tố p lớn hơn m + 2 bất kỳ và xét n = mp2.3 , ta có
 ! 
k
p2.3 −1 k
  k   m .3
m p2.3 −1 k 3k k
nm + mn = mm m + p3 .2m = mm m + p3 .2m  .
 

Giờ để ý rằng do p > 3 nên 3 | p2 − 1 và vì thế



 k

v3 p2.3 − 1 = v3 ( p2 − 1) + k > k.
!
k
p2.3 −1
p 2.3k −1 m
3k
Tức là ∈ Z+ , nên nếu đặt m = a, p2m = b thì a, b ∈ Z+ thỏa
3k
gcd( a, b) = 1 tất nhiên a + b > 3, đồng thời
 k k

n m + m n = m m a3 + b3 .

246
Hội thảo khoa học, Ninh Bình 15-16/09/2018

Theo bài toán phía trên thì tồn tại k số nguyên tố phân biệt p1 , p2 , . . . , pk sao cho
 j   i 
3 3j 3 3i
pj | a + b , pj - a + b ∀ j = 1, k, ∀ i = 0, j − 1.

Rõ ràng nm + mn có k ước nguyên tố phân biệt là p1 , p2 , . . . , pk .




Nhận xét 3. Bài toán này hoàn toàn có thể xử lý như ở các bài China TST 2006 (bài
toán 6) và kết quả của Polya (bài toán 8). Tôi xử lý bằng cách trên, để muốn nhấn
mạnh ý nghĩa của bài toán 10, đó là một kết quả khá hay và nó có vai trò trong nhiều
bài toán khó.

Để kết thúc phần này, tôi xin đưa ra kết quả cực mạnh sau của Hiroshi Kobayashi.
Một kết quả, mà theo tôi biết là chưa có chứng minh sơ cấp.

Định lý 4. Cho S là một tập vô hạn các số nguyên dương thỏa mãn P(S) là hữu hạn, khi đó
với mọi hằng số nguyên c khác 0 ta có P(S + c) là tập vô hạn, với

S + c = { s + c : s ∈ S }.

Ở phần tiếp sau đây, là các bài toán luyện tập.

4 Bài tập

Bài 1. Tìm tất cả các đa thức hệ số nguyên P( x ) sao cho

gcd ( P (n) , P (2017n )) = 1 ∀ n ∈ Z+ .

Bài 2. Cho các số nguyên dương k, m và đa thức hệ số nguyên P( x ) có bậc dương,


chứng minh rằng tồn tại n ∈ Z+ sao cho mỗi giá trị P(n), P(n + 1), . . . , P(n + m)
đều có ít nhất k ước số nguyên tố phân biệt.

Bài 3. Cho P( x ) là đa thức hệ số nguyên bất khả quy trên Q[ x ] có bậc dương, chứng
minh rằng tồn tại vô số số nguyên tố p sao cho với mỗi p như thế sẽ tồn tại n ∈ Z+
thỏa mãn
v p ( P(n)) = 1.

247
Hội thảo khoa học, Ninh Bình 15-16/09/2018

Bài 4. Cho f ( x ) là một đa thức hệ số thực và số nguyên dương k, biết rằng với số
nguyên dương n bất kỳ đều tồn tại m ∈ N sao cho f (n) = mk . Chứng minh rằng tồn
tại đa thức hệ số hữu tỷ P( x ) sao cho

f ( x ) = P k ( x ).

Bài 5. Cho trước số nguyên dương k và đa thức hệ số nguyên P( x ) có bậc dương,


chứng minh rằng tồn tại vô số số nguyên tố p sao cho phương trình đồng dư P( x ) ≡ 0
(mod pk ) có nghiệm.

Bài 6. Cho trước số nguyên dương a không là số chính phương, chứng minh rằng tồn
tại vô số số nguyên tố p lẻ thỏa mãn
p −1
a 2 ≡ −1 (mod p).

Bài 7. Cho các số nguyên dương a và b với a > 1, xét dãy số

f n = a ϕ(n) + b.

1. Chứng minh rằng P ( f n ) là tập vô hạn.

2. Chứng minh rằng P \ P ( f n ) cũng là tập vô hạn.

Bài 8. Chứng minh rằng P ( f n ) là tập vô hạn, với


j log n k
f n = 32 ∀ n ∈ Z+ .
n
Bài 9 (Iran TST, 2009). Cho trước số nguyên dương a, xét dãy f n = 22 + a. Chứng
minh rằng P ( f n ) là tập vô hạn.

Bài 10. Với mỗi số nguyên dương m, ký hiệu p(m) là ước nguyên tố lớn nhất của

3
m2 + 2018. Chứng minh rằng tồn tại vô số số nguyên dương n sao cho p(n) < 2 n2 .

Bài 11. Cho số nguyên dương a (với a > 2), xét dãy số f n = an+1 + 1. Chứng minh
rằng với mỗi số nguyên dương m, tồn tại số nguyên tố p là ước của f m và p > 2m.

Bài 12. Chứng minh rằng, tồn tại vô số số nguyên dương n sao cho n2 + 1 không có
ước chính phương lớn hơn 1.

Bài 13. Tìm tất cả các đa thức hệ số nguyên P( x ) và hằng số k sao cho với mọi số
nguyên dương n thì 2n P(n) + k luôn là số chính phương.

248
Hội thảo khoa học, Ninh Bình 15-16/09/2018

Bài 14. Cho m là một số nguyên dương, chứng minh rằng tồn tại vô số số nguyên
dương n sao cho
7m | (3n + 5n + 6n ) .

Bài 15 (IMO SL, 2014). Cho số nguyên dương c, xét dãy an được xác định bởi công
thức truy hồi như sau a1 = c và

an+1 = a3n − 4ca2n + 5c2 an + c ∀ n ∈ Z+ .

Chứng minh rằng với mọi n > 1 sẽ tồn tại số nguyên tố p sao cho p | an và p - ai ∀ i <
n.

Bài 16. Tìm các số nguyên dương a và b, sao cho tồn tại vô số số nguyên dương n
thỏa mãn
n2 | ( a n + b n ) .

Bài 17. Cho P( x, y) là một đa thức hai biến số có các hệ số là các số nguyên, dãy số
nguyên { an }n∈Z+ thỏa mãn

a n +3 = a n + P ( a n +1 , a n +2 ) ∀ n ∈ Z+ .

Chứng minh rằng P ( an ) là tập vô hạn.

5 Nguồn tham khảo

[1] G. Pólya, Gabor Szegö: "Problems and Theorems in Analysis"


[2] P. Morton: "Musings on the Prime Divisors of Arithmetic Sequences."
[3] H. Kobayshi: "On Existence of Infinitely Many Prime Divisors in a Given Set."
[4] Diễn đàn Mathscope
[5] Diễn đàn Mathlinks
[6] Diễn đàn Mathoverflow

249

You might also like